Saturday, November 28, 2009

Best Mind work Puzzles

Three friends divided some bullets equally. After all of them shot 4 bullets the total number of bullets remaining is equal to the bullets each had after division. Find the original number divided.
Answer


18

Assume that initial there were 3*X bullets.

So they got X bullets each after division.

All of them shot 4 bullets. So now they have (X - 4) bullets each.

But it is given that, after they shot 4 bullets each, total number of bullets remaining is equal to the bullets each had after division i.e. X

Therefore, the equation is
3 * (X - 4) = X
3 * X - 12 = X
2 * X = 12
X = 6

Therefore the total bullets before division is = 3 * X = 18


Find sum of digits of D.

Let
A= 19991999
B = sum of digits of A
C = sum of digits of B
D = sum of digits of C

(HINT: A = B = C = D (mod 9))
Answer

the sum of the digits od D is 1.

Let E = sum of digits of D.

It follows from the hint that A = E (mod 9)
Consider,

A = 19991999

< 20002000

= 22000 * 10002000

= 1024200 * 106000

< 10800 * 106000

= 106800



i.e. A < 106800

i.e. B < 6800 * 9 = 61200

i.e. C < 5 * 9 = 45

i.e. D < 2 * 9 = 18

i.e. E <= 9

i.e. E is a single digit number.



Also,

1999 = 1 (mod 9)

so 19991999 = 1 (mod 9)


Therefore we conclude that E=1.
There is a 50m long army platoon marching ahead. The last person in the platoon wants to give a letter to the first person leading the platoon. So while the platoon is marching he runs ahead, reaches the first person and hands over the letter to him and without stopping he runs and comes back to his original position.

In the mean time the whole platoon has moved ahead by 50m.

The question is how much distance did the last person cover in that time. Assuming that he ran the whole distance with uniform speed.
Submitted
Answer

The last person covered 120.71 meters.

It is given that the platoon and the last person moved with uniform speed. Also, they both moved for the identical amount of time. Hence, the ratio of the distance they covered - while person moving forward and backword - are equal.

Let's assume that when the last person reached the first person, the platoon moved X meters forward.

Thus, while moving forward the last person moved (50+X) meters whereas the platoon moved X meters.

Similarly, while moving back the last person moved [50-(50-X)] X meters whereas the platoon moved (50-X) meters.

Now, as the ratios are equal,
(50+X)/X = X/(50-X)
(50+X)*(50-X) = X*X

Solving, X=35.355 meters

Thus, total distance covered by the last person
= (50+X) + X
= 2*X + 50
= 2*(35.355) + 50
= 120.71 meters


Note that at first glance, one might think that the total distance covered by the last person is 100 meters, as he ran the total lenght of the platoon (50 meters) twice. TRUE, but that's the relative distance covered by the last person i.e. assuming that the platoon is stationary.

If you take a marker & start from a corner on a cube, what is the maximum number of edges you can trace across if you never trace across the same edge twice, never remove the marker from the cube, & never trace anywhere on the cube, except for the corners & edges?
Answer

9

To verify this, you can make a drawing of a cube, & number each of its 12 edges. Then, always starting from 1 corner & 1 edge, you can determine all of the possible combinations for tracing along the edges of a cube.

There is no need to start from other corners or edges of the cube, as you will only be repeating the same combinations. The process is a little more involved than this, but is useful for solving many types of spatial puzzles.
One of Mr. Bajaj, his wife, their son and Mr. Bajaj's mother is an Engineer and another is a Doctor.
• If the Doctor is a male, then the Engineer is a male.
• If the Engineer is younger than the Doctor, then the Engineer and the Doctor are not blood relatives.
• If the Engineer is a female, then she and the Doctor are blood relatives.
Can you tell who is the Doctor and the Engineer?
Answer

Mr. Bajaj is the Engineer and either his wife or his son is the Doctor.

Mr. Bajaj's wife and mother are not blood relatives. So from 3, if the Engineer is a female, the Doctor is a male. But from 1, if the Doctor is a male, then the Engineer is a male. Thus, there is a contradiction, if the Engineer is a female. Hence, either Mr. Bajaj or his son is the Engineer.

Mr. Bajaj's son is the youngest of all four and is blood relative of each of them. So from 2, Mr. Bajaj's son is not the Engineer. Hence, Mr. Bajaj is the Engineer.

Now from 2, Mr. Bajaj's mother can not be the Doctor. So the Doctor is either his wife or his son . It is not possible to determine anything further.

Three men - Sam, Cam and Laurie - are married to Carrie, Billy and Tina, but not necessarily in the same order.

Sam's wife and Billy's Husband play Carrie and Tina's husband at bridge. No wife partners her husband and Cam does not play bridge.

Who is married to Cam?

Answer

Carrie is married to Cam.

"Sam's wife and Billy's Husband play Carrie and Tina's husband at bridge."

It means that Sam is not married to either Billy or Carrie. Thus, Sam is married to Tina.

As Cam does not play bridge, Billy's husband must be Laurie.

Hence, Carrie is married to Cam.

There are 3 persons X, Y and Z. On some day, X lent tractors to Y and Z as many as they had. After a month Y gave as many tractors to X and Z as many as they have. After a month Z did the same thing. At the end of this transaction each one of them had 24.

Find the tractors each originally had?
Answer

One way to solve it is by making 3 equations and solve them simultaneously. But there is rather easier way to solve it using Backtracing.

It's given that at the end, each had 24 tractors (24, 24, 24) i.e. after Z gave tractors to X & Y as many as they had. It means that after getting tractors from Z their tractors got doubled. So before Z gave them tractors, they had 12 tractors each and Z had 48 tractors. (12, 12, 48)

Similarly, before Y gave tractors to X & Z, they had 6 & 24 tractors respectively and Y had 42 tractors i.e. (6, 42, 24)

Again, before X gave tractors to Y & Z, they had 21 & 12 tractors respectively and X had 39 tractors i.e. (39, 21, 12)

Hence, initially X had 39 tractors, Y had 21 tractors and Z had 12 tractors.
A certain street has 1000 buildings. A sign-maker is contracted to number the houses from 1 to 1000. How many zeroes will he need?
Answer

The sign-maker will need 192 zeroes.

Divide 1000 building numbers into groups of 100 each as follow:
(1..100), (101..200), (201..300), ....... (901..1000)

For the first group, sign-maker will need 11 zeroes.
For group numbers 2 to 9, he will require 20 zeroes each.
And for group number 10, he will require 21 zeroes.

The total numbers of zeroes required are
= 11 + 8*20 + 21
= 11 + 160 + 21
= 192

There are 9 coins. Out of which one is odd one i.e weight is less or more. How many iterations of weighing are required to find odd coin?
Answer

It is always possible to find odd coin in 3 weighings and to tell whether the odd coin is heavier or lighter.
1. Take 8 coins and weigh 4 against 4.
o If both are not equal, goto step 2
o If both are equal, goto step 3

2. One of these 8 coins is the odd one. Name the coins on heavier side of the scale as H1, H2, H3 and H4. Similarly, name the coins on the lighter side of the scale as L1, L2, L3 and L4. Either one of H's is heavier or one of L's is lighter. Weigh (H1, H2, L1) against (H3, H4, X) where X is one coin remaining in intial weighing.
o If both are equal, one of L2, L3, L4 is lighter. Weigh L2 against L3.
 If both are equal, L4 is the odd coin and is lighter.
 If L2 is light, L2 is the odd coin and is lighter.
 If L3 is light, L3 is the odd coin and is lighter.

o If (H1, H2, L1) is heavier side on the scale, either H1 or H2 is heavier. Weight H1 against H2
 If both are equal, there is some error.
 If H1 is heavy, H1 is the odd coin and is heavier.
 If H2 is heavy, H2 is the odd coin and is heavier.

o If (H3, H4, X) is heavier side on the scale, either H3 or H4 is heavier or L1 is lighter. Weight H3 against H4
 If both are equal, L1 is the odd coin and is lighter.
 If H3 is heavy, H3 is the odd coin and is heavier.
 If H4 is heavy, H4 is the odd coin and is heavier.

3. The remaining coin X is the odd one. Weigh X against the anyone coin used in initial weighing.
o If both are equal, there is some error.
o If X is heavy, X is the odd coin and is heavier.
o If X is light, X is the odd coin and is lighter.
In a sports contest there were m medals awarded on n successive days (n > 1).
1. On the first day 1 medal and 1/7 of the remaining m - 1 medals were awarded.
2. On the second day 2 medals and 1/7 of the now remaining medals was awarded; and so on.
3. On the nth and last day, the remaining n medals were awarded.
How many days did the contest last, and how many medals were awarded altogether?
Answer

Total 36 medals were awarded and the contest was for 6 days.

On day 1: Medals awarded = (1 + 35/7) = 6 : Remaining 30 medals
On day 2: Medals awarded = (2 + 28/7) = 6 : Remaining 24 medals
On day 3: Medals awarded = (3 + 21/7) = 6 : Remaining 18 medals
On day 4: Medals awarded = (4 + 14/7) = 6 : Remaining 12 medals
On day 5: Medals awarded = (5 +7/7) = 6 : Remaining 6 medals
On day 6: Medals awarded 6

I got this answer by writing small program. If anyone know any other simpler method, do submit it.
A number of 9 digits has the following properties:
• The number comprising the leftmost two digits is divisible by 2, that comprising the leftmost three digits is divisible by 3, the leftmost four by 4, the leftmost five by 5, and so on for the nine digits of the number i.e. the number formed from the first n digits is divisible by n, 2<=n<=9.
• Each digit in the number is different i.e. no digits are repeated.
• The digit 0 does not occur in the number i.e. it is comprised only of the digits 1-9 in some order.
Find the number.

Answer

The answer is 381654729

One way to solve it is Trial-&-Error. You can make it bit easier as odd positions will always occupy ODD numbers and even positions will always occupy EVEN numbers. Further 5th position will contain 5 as 0 does not occur.

The other way to solve this problem is by writing a computer program that systematically tries all possibilities

1/3 rd of the contents of a container evaporated on the 1st day. 3/4th of the remaining contents of the container evaporated on the second day.

What part of the contents of the container is left at the end of the second day?


Answer

Assume that contents of the container is X

On the first day 1/3rd is evaporated.
(1 - 1/3) of X is remaining i.e. (2/3)X

On the Second day 3/4th is evaporated. Hence,
(1- 3/4) of (2/3)X is remaining
i.e. (1/4)(2/3)X = (1/6) X

Hence 1/6th of the contents of the container is remaining

Vipul was studying for his examinations and the lights went off. It was around 1:00 AM. He lighted two uniform candles of equal length but one thicker than the other. The thick candle is supposed to last six hours and the thin one two hours less. When he finally went to sleep, the thick candle was twice as long as the thin one.

For how long did Vipul study in candle light?
Answer

Vipul studied for 3 hours in candle light.

Assume that the initial lenght of both the candle was L and Vipul studied for X hours.

In X hours, total thick candle burnt = XL/6
In X hours, total thin candle burnt = XL/4

After X hours, total thick candle remaining = L - XL/6
After X hours, total thin candle remaining = L - XL/4

Also, it is given that the thick candle was twice as long as the thin one when he finally went to sleep.
(L - XL/6) = 2(L - XL/4)
(6 - X)/6 = (4 - X)/2
(6 - X) = 3*(4 - X)
6 - X = 12 - 3X
2X = 6
X = 3

Hence, Vipul studied for 3 hours i.e. 180 minutes in candle light.

If you started a business in which you earned Rs.1 on the first day, Rs.3 on the second day, Rs.5 on the third day, Rs.7 on the fourth day, & so on.

How much would you have earned with this business after 50 years (assuming there are exactly 365 days in every year)?
Answer

Rs.333,062,500

To begin with, you want to know the total number of days: 365 x 50 = 18250.

By experimentation, the following formula can be discovered, & used to determine the amount earned for any particular day: 1 + 2(x-1), with x being the number of the day. Take half of the 18250 days, & pair them up with the other half in the following way: day 1 with day 18250, day 2 with day 18249, & so on, & you will see that if you add these pairs together, they always equal Rs.36500.

Multiply this number by the total number of pairs (9125), & you have the amount you would have earned in 50 years.

Math gurus may use series formula to solve it.(series: 1,3,5,7,9,11.....upto 18250 terms)
A worker earns a 5% raise. A year later, the worker receives a 2.5% cut in pay, & now his salary is Rs. 22702.68

What was his salary to begin with?
Answer

Rs.22176

Assume his salary was Rs. X

He earns 5% raise. So his salary is (105*X)/100

A year later he receives 2.5% cut. So his salary is ((105*X)/100)*(97.5/100) which is Rs. 22702.68

Hence, solving equation ((105*X)/100)*(97.5/100) = 22702.68
X = 22176
At 6'o a clock ticks 6 times. The time between first and last ticks is 30 seconds. How long does it tick at 12'o.


Answer

66 seconds

It is given that the time between first and last ticks at 6'o is 30 seconds.
Total time gaps between first and last ticks at 6'o = 5
(i.e. between 1 & 2, 2 & 3, 3 & 4, 4 & 5 and 5 & 6)

So time gap between two ticks = 30/5 = 6 seconds.


Now, total time gaps between first and last ticks at 12'o = 11
Therefore time taken for 12 ticks = 11 * 6 = 66 seconds (and not 60 seconds)


500 men are arranged in an array of 10 rows and 50 columns according to their heights.

Tallest among each row of all are asked to come out. And the shortest among them is A.

Similarly after resuming them to their original positions, the shortest among each column are asked to come out. And the tallest among them is B.

Now who is taller A or B ?
Answer


No one is taller, both are same as A and B are the same person.

As it is mentioned that 500 men are arranged in an array of 10 rows and 50 columns according to their heights. Let's assume that position numbers represent their heights. Hence, the shortest among the 50, 100, 150, ... 450, 500 is person with height 50 i.e. A. Similarly the tallest among 1, 2, 3, 4, 5, ..... 48, 48, 50 is person with height 50 i.e. B

Now, both A and B are the person with height 50. Hence both are same.

In Mr. Mehta's family, there are one grandfather, one grandmother, two fathers, two mothers, one father-in-law, one mother-in-law, four children, three grandchildren, one brother, two sisters, two sons, two daughters and one daughter-in-law.

How many members are there in Mr. Mehta's family? Give minimal possible answer.
Answer

There are 7 members in Mr. Mehta's family. Mother & Father of Mr. Mehta, Mr. & Mrs. Mehta, his son and two daughters.

Mother & Father of Mr. Mehta
|
|
Mr. & Mrs. Mehta
|
|
One Son & Two Daughters
When Alexander the Great attacked the forces of Porus, an Indian soldier was captured by the Greeks. He had displayed such bravery in battle, however, that the enemy offered to let him choose how he wanted to be killed. They told him, "If you tell a lie, you will put to the sword, and if you tell the truth you will be hanged."

The soldier could make only one statement. He made that statement and went free. What did he say?

Answer

The soldier said, "You will put me to the sword."

The soldier has to say a Paradox to save himself. If his statement is true, he will be hanged, which is not the sword and hence false. If his statement is false, he will be put to the sword, which will make it true. A Paradox !!!

A person wanted to withdraw X rupees and Y paise from the bank. But cashier made a mistake and gave him Y rupees and X paise. Neither the person nor the cashier noticed that.

After spending 20 paise, the person counts the money. And to his surprise, he has double the amount he wanted to withdraw.

Find X and Y. (1 Rupee = 100 Paise)













Answer

As given, the person wanted to withdraw 100X + Y paise.

But he got 100Y + X paise.

After spending 20 paise, he has double the amount he wanted to withdraw. Hence, the equation is

2 * (100X + Y) = 100Y + X - 20

200X + 2Y = 100Y +X - 20

199X - 98Y = -20

98Y - 199X = 20


Now, we got one equation; but there are 2 variables. We have to apply little bit of logic over here. We know that if we interchange X & Y, amount gets double. So Y should be twice of X or one more than twice of X i.e. Y = 2X or Y = 2X+1

Case I : Y=2X
Solving two equations simultaneously
98Y - 199X = 20
Y - 2X = 0
We get X = - 20/3 & Y = - 40/2

Case II : Y=2X+1
Solving two equations simultaneously
98Y - 199X = 20
Y - 2X = 1
We get X = 26 & Y = 53

Now, its obvious that he wanted to withdraw Rs. 26.53




The game of Tic-Tac-Toe is being played between two players. Only the last mark to be placed in the game as shown.

Who will win the game, O or X? Can you tell which was the sixth mark and at which position? Do explain your answer.

Assume that both the players are intelligent enough.










Answer

O will win the game. The sixth mark was X in square 9.

The 7th mark must be placed in square 5 which is the win situation for both X and O. Hence, the 6th mark must be placed in a line already containing two of the opponents marks. There are two such possibilities - the 6th mark would have been either O in square 7 or X in square 9.

As we know both the players are intelligent enough, the 6th mark could not be O in square 7. Instead, he would have placed O in square 5 and would have won.

Hence, the sixth mark must be X placed in square 9. And the seventh mark will be O. Thus O will win the game.


At the Party:
1. There were 9 men and children.
2. There were 2 more women than children.
3. The number of different man-woman couples possible was 24. Note that if there were 7 men and 5 women, then there would have been 35 man-woman couples possible.

Also, of the three groups - men, women and children - at the party:
4. There were 4 of one group.
5. There were 6 of one group.
6. There were 8 of one group.
Exactly one of the above 6 statements is false.

Can you tell which one is false? Also, how many men, women and children are there at the party?



Answer

Statement (4) is false. There are 3 men, 8 women and 6 children.

Assume that Statements (4), (5) and (6) are all true. Then, Statement (1) is false. But then Statement (2) and (3) both can not be true. Thus, contradictory to the fact that exactly one statement is false.

So Statement (4) or Statement (5) or Statement (6) is false. Also, Statements (1), (2) and (3) all are true.

From (1) and (2), there are 11 men and women. Then from (3), there are 2 possible cases - either there are 8 men and 3 women or there are 3 men and 8 women.

If there are 8 men and 3 women, then there is 1 child. Then Statements (4) and (5) both are false, which is not possible.

Hence, there are 3 men, 8 women and 6 children. Statement (4) is false.

There is a shortage of tubelights, bulbs and fans in a village - Kharghar. It is found that
• All houses do not have either tubelight or bulb or fan.
• exactly 19% of houses do not have just one of these.
• atleast 67% of houses do not have tubelights.
• atleast 83% of houses do not have bulbs.
• atleast 73% of houses do not have fans.
What percentage of houses do not have tubelight, bulb and fan?
Answer

42% houses do not have tubelight, bulb and fan.

Let's assume that there are 100 houses. Hence, there should be total 300 items i.e. 100 tubelights, 100 bulbs and 100 fans.

From the given data, we know that there is shortage of atleast (67+83+73) 223 items in every 100 houses.

Also, exactly 19 houses do not have just one item. It means that remaining 81 houses should account for the shortage of remaining (223-19) 204 items. If those remaining 81 houses do not have 2 items each, there would be a shortage of 162 items. But total of 204 items are short. Hence, atleast (204-162) 42 houses do not have all 3 items - tubelight, bulb and fan.

Thus, 42% houses do not have tubelight, bulb and fan.
Mr. Subramaniam rents a private car for Andheri-Colaba-Andheri trip. It costs him Rs. 300 everyday.

One day the car driver informed Mr. Subramaniam that there were two students from Bandra who wished to go from Bandra to Colaba and back to Bandra. Bandra is halfway between Andheri and Colaba. Mr. Subramaniam asked the driver to let the students travel with him.

On the first day when they came, Mr. Subramaniam said, "If you tell me the mathematically correct price you should pay individually for your portion of the trip, I will let you travel for free."

How much should the individual student pay for their journey?
Answer

The individual student should pay Rs. 50 for their journey.

Note that 3 persons are travelling between Bandra and Colaba.

The entire trip costs Rs. 300 to Mr. Subramanian. Hence, half of the trip costs Rs. 150.

For Andheri-Bandra-Andheri, only one person i.e. Mr. Subramaniam is travelling. Hence, he would pay Rs. 150.

For Bandra-Colaba-Bandra, three persons i.e Mr. Subramaniam and two students, are travelling. Hence, each student would pay Rs. 50.

Substitute digits for the letters to make the following Division true
O U T

-------------

S T E M | D E M I S E

| D M O C

-------------

T U I S

S T E M

----------

Z Z Z E

Z U M M

--------

I S T
Note that the leftmost letter can't be zero in any word. Also, there must be a one-to-one mapping between digits and letters. e.g. if you substitute 3 for the letter M, no other letter can be 3 and all other M in the puzzle must be
Answer

C=0, U=1, S=2, T=3, O=4, M=5, I=6, Z=7, E=8, D=9

It is obvious that U=1 (as U*STEM=STEM) and C=0 (as I-C=I).

S*O is a single digit and also S*T is a single digit. Hence, their values (O, S, T) must be 2, 3 or 4 (as they can not be 0 or 1 or greater than 4).

Consider, STEM*O=DMOC, where C=0. It means that M must be 5. Now, its simple. O=4, S=2, T=3, E=8, Z=7, I=6 and D=9.
O U T 4 1 3

------------- -------------

S T E M | D E M I S E 2 3 8 5 | 9 8 5 6 2 8

| D M O C | 9 5 4 0

------------- -------------

T U I S 3 1 6 2

S T E M 2 3 8 5

---------- ----------

Z Z Z E 7 7 7 8

Z U M M 7 1 5 5

-------- --------

I S T 6 2 3
Also, when arranged from 0 to 9, it spells CUSTOMIZED.
At what time after 4.00 p.m. is the minutes hand of a clock exactly aligned with the hour hand?

Answer

4:21:49.5

Assume that X minutes after 4.00 PM minute hand exactly aligns with and hour hand.

For every minute, minute hand travels 6 degrees.
Hence, for X minutes it will travel 6 * X degrees.

For every minute, hour hand travels 1/2 degrees.
Hence, for X minutes it will travel X/2 degrees.

At 4.00 PM, the angle between minute hand and hour hand is 120 degrees. Also, after X minutes, minute hand and hour hand are exactly aligned. So the angle with respect to 12 i.e. Vertical Plane will be same. Therefore,

6 * X = 120 + X/2
12 * X = 240 + X
11 * X = 240
X = 21.8182
X = 21 minutes 49.5 seconds

Hence, at 4:21:49.5 minute hand is exactly aligned with the hour hand.

A soldier looses his way in a thick jungle. At random he walks from his camp but mathematically in an interesting fashion.

First he walks one mile East then half mile to North. Then 1/4 mile to West, then 1/8 mile to South and so on making a loop.

Finally how far he is from his camp and in which direction?

Answer

The soldier is 0.8944 miles away from his camp towards East-North.

It is obvious that he is in East-North direction.

Distance travelled in North and South directions
= 1/2 - 1/8 + 1/32 - 1/128 + 1/512 - 1/2048 + and so on... (a geometric series with r = (-1/4) )

(1/2) * ( 1 - (-1/4)n )
= ---------------------------
( 1 - (-1/4) )

= 1 / ( 2 * ( 1 - (-1/4) ) )
= 2/5

Similarly in East and West directions
= 1 - 1/4 + 1/16 - 1/64 + 1/256 - and so on... (a geometric series with r = (-1/4) )

(1) * ( 1 - (-1/4)n )
= ---------------------------
( 1 - (-1/4) )

= 1 / ( ( 1- (-1/4) )
= 4/5

So the soldier is 4/5 miles away towards East and 2/5 miles away towards North. So using right angled triangle, soldier is 0.8944 miles away from his camp.

Raj has a jewel chest containing Rings, Pins and Ear-rings. The chest contains 26 pieces. Raj has 2 1/2 times as many rings as pins, and the number of pairs of earrings is 4 less than the number of rings.

How many earrings does Raj have?

Answer

12 earrings

Assume that there are R rings, P pins and E pair of ear-rings.

It is given that, he has 2 1/2 times as many rings as pins.
R = (5/2) * P or P = (2*R)/5

And, the number of pairs of earrings is 4 less than the number of rings.
E = R - 4 or R = E + 4

Also, there are total 26 pieces.
R + P + 2*E = 26
R + (2*R)/5 + 2*E = 26
5*R + 2*R + 10*E = 130
7*R + 10*E = 130
7*(E + 4) + 10*E = 130
7*E + 28 + 10*E = 130
17*E = 102
E = 6

Hence, there are 6 pairs of Ear-rings i.e. total 12 Ear-rings
How many ways are there of arranging the sixteen black or white pieces of a standard international chess set on the first two rows of the board?

Given that each pawn is identical and each rook, knight and bishop is identical to its pair.
Submitted

Answer

6,48,64,800 ways

There are total 16 pieces which can be arranged on 16 places in 16P16 = 16! ways.
(16! = 16 * 15 * 14 * 13 * 12 * ..... * 3 * 2 * 1)

But, there are some duplicate combinations because of identical pieces.
• There are 8 identical pawn, which can be arranged in 8P8 = 8! ways.
• Similarly there are 2 identical rooks, 2 identical knights and 2 identical bishops. Each can be arranged in 2P2 = 2! ways.
Hence, the require answer is
= (16!) / (8! * 2! * 2! * 2!)
= 6,48,64,800


A person with some money spends 1/3 for cloths, 1/5 of the remaining for food and 1/4 of the remaining for travel. He is left with Rs 100/-

How much did he have with him in the begining?

Answer

Rs. 250/-

Assume that initially he had Rs. X
He spent 1/3 for cloths =. (1/3) * X
Remaining money = (2/3) * X

He spent 1/5 of remaining money for food = (1/5) * (2/3) * X = (2/15) * X
Remaining money = (2/3) * X - (2/15) * X = (8/15) * X

Again, he spent 1/4 of remaining maoney for travel = (1/4) * (8/15) * X = (2/15) * X
Remaining money = (8/15) * X - (2/15) * X = (6/15) * X

But after spending for travel he is left with Rs. 100/- So
(6/15) * X = 100
X = 250

Grass in lawn grows equally thick and in a uniform rate. It takes 24 days for 70 cows and 60 days for 30 cows to eat the whole of the grass.

How many cows are needed to eat the grass in 96 days?
Answer

20 cows

g - grass at the beginning
r - rate at which grass grows, per day
y - rate at which one cow eats grass, per day
n - no of cows to eat the grass in 96 days

From given data,
g + 24*r = 70 * 24 * y ---------- A
g + 60*r = 30 * 60 * y ---------- B
g + 96*r = n * 96 * y ---------- C

Solving for (B-A),
(60 * r) - (24 * r) = (30 * 60 * y) - (70 * 24 * y)
36 * r = 120 * y ---------- D

Solving for (C-B),
(96 * r) - (60 * r) = (n * 96 * y) - (30 * 60 * y)
36 * r = (n * 96 - 30 * 60) * y
120 * y = (n * 96 - 30 * 60) * y [From D]
120 = (n * 96 - 1800)
n = 20

Hence, 20 cows are needed to eat the grass in 96 days.
There is a safe with a 5 digit number as the key. The 4th digit is 4 greater than the second digit, while the 3rd digit is 3 less than the 2nd digit. The 1st digit is thrice the last digit. There are 3 pairs whose sum is 11.

Find the number.
Answer

65292

As per given conditions, there are three possible combinations for 2nd, 3rd and 4th digits. They are (3, 0, 7) or (4, 1, 8) or (5, 2, 9)

It is given that there are 3 pairs whose sum is 11. All possible pairs are (2, 9), (3, 8), (4, 7), (5, 6). Now required number is 5 digit number and it contains 3 pairs of 11. So it must not be having 0 and 1 in it. Hence, the only possible combination for 2nd, 3rd and 4th digits is (5, 2, 9)

Also, 1st digit is thrice the last digit. The possible combinations are (3, 1), (6, 2) and (9, 3), out of which only (6, 2) with (5, 2, 9) gives 3 pairs of 11. Hence, the answer is 65292.
Four friends - Arjan, Bhuvan, Guran and Lakha were comparing the number of sheep that they owned.

It was found that Guran had ten more sheep than Lakha.

If Arjan gave one-third to Bhuvan, and Bhuvan gave a quarter of what he then held to Guran, who then passed on a fifth of his holding to Lakha, they would all have an equal number of sheep.

How many sheep did each of them possess? Give the minimal possible answer

Answer


Arjan, Bhuvan, Guran and Lakha had 90, 50, 55 and 45 sheep respectively.

Assume that Arjan, Bhuvan, Guran and Lakha had A, B, G and L sheep respectively. As it is given that at the end each would have an equal number of sheep, comparing the final numbers from the above table.

Arjan's sheep = Bhuvan's sheep
2A/3 = A/4 + 3B/4
8A = 3A + 9B
5A = 9B

Arjan's sheep = Guran's sheep
2A/3 = A/15 + B/5 + 4G/5
2A/3 = A/15 + A/9 + 4G/5 (as B=5A/9)
30A = 3A + 5A + 36G
22A = 36G
11A = 18G

Arjan's sheep = Lakha's sheep
2A/3 = A/60 + B/20 + G/5 + L
2A/3 = A/60 + A/36 + 11A/90 + L (as B=5A/9 and G=11A/18)
2A/3 = A/6 + L
A/2 = L
A = 2L

Also, it is given that Guran had ten more sheep than Lakha.
G = L + 10
11A/18 = A/2 + 10
A/9 = 10
A = 90 sheep

Thus, Arjan had 90 sheep, Bhuvan had 5A/9 i.e. 50 sheep, Guran had 11A/18 i.e. 55 sheep and Lakha had A/2 i.e. 45 sheep.
Consider a number 235, where last digit is the sum of first two digits i.e. 2 + 3 = 5.

How many such 3-digit numbers are there?

Answer

There are 45 different 3-digit numbers.

The last digit can not be 0.

If the last digit is 1, the only possible number is 101. (Note that 011 is not a 3-digit number)

If the last digit is 2, the possible numbers are 202 and 112.

If the last digit is 3, the possible numbers are 303, 213 and 123.

If the last digit is 4, the possible numbers are 404, 314, 224 and 134.

If the last digit is 5, the possible numbers are 505, 415, 325, 235 and 145.

Note the pattern here - If the last digit is 1, there is only one number. If the last digit is 2, there are two numbers. If the last digit is 3, there are three numbers. If the last digit is 4, there are four numbers. If the last digit is 5, there are five numbers. And so on.....

Thus, total numbers are
1 + 2 + 3 + 4 + 5 + 6 + 7 + 8 + 9 = 45

Altogether then, there are 45 different 3-digit numbers, where last digit is the sum of first two digits.

Find the smallest number such that if its rightmost digit is placed at its left end, the new number so formed is precisely 50% larger than the original number.
Answer

The answer is 285714.

If its rightmost digit is placed at its left end, then new number is 428571 which is 50% larger than the original number 285714.

The simplest way is to write a small program. And the other way is trial and error !!!
Two identical pack of cards A and B are shuffled throughly. One card is picked from A and shuffled with B. The top card from pack A is turned up. If this is the Queen of Hearts, what are the chances that the top card in B will be the King of Hearts?

Answer

52 / 2703

There are two cases to be considered.

CASE 1 : King of Hearts is drawn from Pack A and shuffled with Pack B

Probability of drawing King of Hearts from Pack A = 1/51 (as Queen of Hearts is not to be drawn)
Probability of having King of Hearts on the top of the Pack B = 2/53

So total probability of case 1 = (1/51) * (2/53) = 2 / (51 * 53)

CASE 2 : King of Hearts is not drawn from Pack A

Probability of not drawing King of Hearts from Pack A = 50/51 (as Queen of Hearts is not to be drawn)
Probability of having King of Hearts on the top of the Pack B = 1/53

So total probability of case 2 = (50/51) * (1/53) = 50 / (51 * 53)
Now adding both the probability, the required probability is
= 2 / (51 * 53) + 50 / (51 * 53)
= 52 / (51 * 53)
= 52 / 2703
= 0.0192378
There are 3 ants at 3 corners of a triangle, they randomly start moving towards another corner.

What is the probability that they don't collide?

Answer

Let's mark the corners of the triangle as A,B,C. There are total 8 ways in which ants can move.
1. A->B, B->C, C->A
2. A->B, B->C, C->B
3. A->B, B->A, C->A
4. A->B, B->A, C->B
5. A->C, C->B, B->A
6. A->C, C->B, B->C
7. A->C, C->A, B->A
8. A->C, C->A, B->C
Out of which, there are only two cases under which the ants won't collide :
• A->B, B->C, C->A
• A->C, C->B, B->A
Answer

There are total 8 such series:
1. Sum of 2000 numbers starting from -999 i.e. summation of numbers from -999 to 1000.
(-999) + (-998) + (-997) + ..... + (-1) + 0 + 1 + 2 + ..... + 997 + 998 + 999 + 1000 = 1000
2. Sum of 400 numbers starting from -197 i.e. summation of numbers from -197 to 202.
(-197) + (-196) + (-195) + ..... + (-1) + 0 + 1 + 2 + ..... + 199 + 200 + 201 + 202 = 1000
3. Sum of 125 numbers starting from -54 i.e. summation of numbers from -54 to 70.
(-54) + (-53) + (-52) + ..... + (-1) + 0 + 1 + 2 + ..... + 68 + 69 + 70 = 1000
4. Sum of 80 numbers starting from -27 i.e. summation of numbers from -27 to 52.
(-27) + (-26) + (-25) + ..... + (-1) + 0 + 1 + 2 + ..... + 50 + 51 + 52 = 1000
5. Sum of 25 numbers starting from 28 i.e. summation of numbers from 28 to 52.
28 + 29 + 30 + 31 + 32 + 33 + 34 + 35 + 36 + 37 + 38 + 39 + 40 + 41 + 42 + 43 + 44 + 45 + 46 + 47 + 48 + 49 + 50 + 51 + 52 = 1000
6. Sum of 16 numbers starting from 55 i.e. summation of numbers from 55 to 70.
55 + 56 + 57 + 58 + 59 +60 + 61 + 62 + 63 + 64 + 65 + 66 + 67 + 68 + 69 + 70 = 1000
7. Sum of 5 numbers starting from 198 i.e. summation of numbers from 198 to 202.
198 + 199 + 200 +201 + 202 = 1000
8. Sum of 1 number starting from 1000.
1000 = 1000

There is a 4-character code, with 2 of them being letters and the other 2 being numbers.

How many maximum attempts would be necessary to find the correct code? Note that the code is case-sensitive.
Answer

The maximum number of attempts required are 16,22,400

There are 52 possible letters - a to z and A to Z, and 10 possible numbers - 0 to 9. Now, 4 characters - 2 letters and 2 numbers, can be selected in 52*52*10*10 ways. These 4 characters can be arranged in 4C2 i.e. 6 different ways - the number of unique patterns that can be formed by lining up 4 objects of which 2 are distinguished one way (i.e. they must be letters) and the other 2 are distinguished another way (i.e. they must be numbers).

Consider an example : Let's assume that @ represents letter and # represents number. the 6 possible ways of arranging them are : @@##, @#@#, @##@, #@@#, #@#@, ##@@

Hence, the required answer is
= 52*52*10*10*6
= 16,22,400 attempts
= 1.6 million approx.

Thanks to Tim Sanders for opening BrainVista's brain !!!
How many possible combinations are there in a 3x3x3 rubics cube?

In other words, if you wanted to solve the rubics cube by trying different combinations, how many might it take you (worst case senerio)?

How many for a 4x4x4 cube?
Submitted

Answer

There are 4.3252 * 10^19 possible combinations for 3x3x3 Rubics and 7.4012 * 10^45 possible combinations for 4x4x4 Rubics.


Let's consider 3x3x3 Rubics first.

There are 8 corner cubes, which can be arranged in 8! ways.
Each of these 8 cubes can be turned in 3 different directions, so there are 3^8 orientations altogether. But if you get all but one of the corner cube into chosen positions and orientations, only one of 3 orientations of the final corner cube is possible. Thus, total ways corner cubes can be placed = (8!) * (3^8)/8 = (8!) * (3^7)

Similarly, 12 edge cubes can be arranged in 12! ways.
Each of these 12 cubes can be turned in 2 different directions, so there are 2^12 orientations altogether. But if you get all but one of the edge cube into chosen positions and orientations, only one of 2 orientations of the final edge cube is possible. Thus, total ways edge cubes can be placed = (12!) * (2^12)/2 = (12!) * (2^11)

Here, we have essentially pulled the cubes apart and stuck cubes back in place wherever we please. In reality, we can only move cubes around by turning the faces of the cubes. It turns out that you can't turn the faces in such a way as to switch the positions of two cubes while returning all the others to their original positions. Thus if you get all but two cubes in place, there is only one attainable choice for them (not 2!). Hence, we must divide by 2.

Total different possible combinations are
= [(8!) * (3^7)] * [(12!) * (2^11)] / 2
= (8!) * (3^7) * (12!) * (2^10)
= 4.3252 * 10^19


Similarly, for 4x4x4 Rubics total different possible combinations are
= [(8!) * (3^7)] * [(24!)] * [(24!) / (4!^6)] / 24
= 7.4011968 * 10^45

Note that there are 24 edge cubes, which you can not turn in 2 orientations (hence no 2^24 / 2). Also, there are 4 center cubes per face i.e. (24!) / (4!^6). You can switch 2 cubes without affecting the rest of the combination as 4*4*4 has even dimensions (hence no division by 2). But pattern on one side is rotated in 4 directions over 6 faces, hence divide by 24.
Substitute digits for the letters to make the following relation true.
N E V E R

L E A V E

+ M E

-----------------

A L O N E
Note that the leftmost letter can't be zero in any word. Also, there must be a one-to-one mapping between digits and letters. e.g. if you substitute 3 for the letter M, no other letter can be 3 and all other M in the puzzle must be 3.

Answer

A tough one!!!

Since R + E + E = 10 + E, it is clear that R + E = 10 and neither R nor E is equal to 0 or 5. This is the only entry point to

solve it. Now use trial-n-error method.

N E V E R 2 1 4 1 9

L E A V E 3 1 5 4 1

+ M E + 6 1

----------------- -----------------

A L O N E 5 3 0 2 1


One of the four people - Mr. Clinton, his wife Monika, their son Mandy and their daughter Cindy - is a singer and another is a dancer. Mr. Clinton is older than his wife and Mady is older than his sister.
1. If the singer and the dancer are the same sex, then the dancer is older than the singer.
2. If neither the singer nor the dancer is the parent of the other, then the singer is older than the dancer.
3. If the singer is a man, then the singer and the dancer are the same age.
4. If the singer and the dancer are of opposite sex then the man is older than the woman.
5. If the dancer is a woman, then the dancer is older than the singer.
Whose occupation do you know? And what is his/her occupation?
Answer

Cindy is the Singer. Mr. Clinton or Monika is the Dancer.

From (1) and (3), the singer and the dancer, both can not be a man. From (3) and (4), if the singer is a man, then the dancer must be a man. Hence, the singer must be a woman.

CASE I : Singer is a woman and Dancer is also a woman
Then, the dancer is Monika and the singer is Cindy.

CASE II : Singer is a woman and Dancer is also a man
Then, the dancer is Mr. Clinton and the singer is Cindy.

In both the cases, we know that Cindy is the Singer. And either Mr. Clinton or Monika is the Dancer.
There are 20 people in your applicant pool, including 5 pairs of identical twins.

If you hire 5 people randomly, what are the chances you will hire at least 1 pair of identical twins? (Needless to say, this could cause trouble ;))
Submitted

Answer

The probability to hire 5 people with at least 1 pair of identical twins is 25.28%

5 people from the 20 people can be hired in 20C5 = 15504 ways.

Now, divide 20 people into two groups of 10 people each :
G1 - with all twins
G2 - with all people other than twins

Let's find out all possible ways to hire 5 people without a single pair of indentical twins.
People from G1 People from G2 No of ways to hire G1 without a single pair of indentical twins No of ways to hire G2 Total ways
0 5 10C0 10C5 252
1 4 10C1 10C4 2100
2 3 10C2 * 8/9 10C3 4800
3 2 10C3 * 8/9 * 6/8 10C2 3600
4 1 10C4 * 8/9 * 6/8 * 4/7 10C1 800
5 0 10C5 * 8/9 * 6/8 * 4/7 * 2/6 10C0 32
Total 11584

Thus, total possible ways to hire 5 people without a single pair of indentical twins = 11584 ways

So, total possible ways to hire 5 people with at least a single pair of indentical twins = 15504 - 11584 = 3920 ways

Hence, the probability to hire 5 people with at least a single pair of indentical twins
= 3920/15504
= 245/969
= 0.2528
= 25.28%
In a hotel, rooms are numbered from 101 to 550. A room is chosen at random. What is the probability that room number starts with 1, 2 or 3 and ends with 4, 5 or 6?

Answer

There are total 450 rooms.

Out of which 299 room number starts with either 1, 2 or 3. (as room number 100 is not there) Now out of those 299 rooms only 90 room numbers end with 4, 5 or 6

So the probability is 90/450 i.e. 1/5 or 0.20

Draw 9 dots on a page, in the shape of three rows of three dots to form a square. Now place your pen on the page, draw 4 straight lines and try and cover all the dots.

You're not allowed to lift your pen.

Note: Don't be confined by the dimensions of the square.
Submitted




Assume that you have enough coins of 1, 5, 10, 25 and 50 cents.

How many ways are there to make change for a dollar? Do explain your answer.



There are 292 ways to make change for a dollar using coins of 1, 5, 10, 25 and 50 cents.

Let's generalised the teaser and make a table as shown above.

If you wish to make change for 75 cents using only 1, 5, 10 and 25 cent coins, go to the .25 row and the 75 column to obtain 121 ways to do this.

The table can be created from left-to-right and top-to-bottom. Start with the top left i.e. 1 cent row. There is exactly one way to make change for every amount. Then calculate the 5 cents row by adding the number of ways to make change for the amount using 1 cent coins plus the number of ways to make change for 5 cents less using 1 and 5 cent coins.

Let's take an example:
To get change for 50 cents using 1, 5 and 10 cent coins.
* 50 cents change using 1 and 5 cent coins = 11 ways
* (50-10) 40 cents change using 1, 5 and 10 cent coins = 25 ways
* 50 cents change using 1, 5 and 10 cent coins = 11+25 = 36 ways

Let's take another example:
To get change for 75 cents using all coins up to 50 cent i.e. 1, 5, 10, 25 and 50 cents coins.
* 75 cents change using coins upto 25 cent = 121 ways
* (75-50) 25 cents change using coins upto 50 cent = 13 ways
* 75 cents change using coins upto 50 cent = 121+13 = 134 ways

For people who don't want to tease their brain and love to do computer programming, there is a simple way. Write a small multi-loop program to solve the equation: A + 5B + 10C + 25D + 50E = 100
where,
A = 0 to 100
B = 0 to 20
C = 0 to 10
D = 0 to 4
E = 0 to 2

The program should output all the possible values of A, B, C, D and E for which the equation is satisfied.

In a Road Race, one of the three bikers was doing 15km less than the first and 3km more than the third. He also finished the race 12 minutes after the first and 3 minutes before the third.

Can you find out the speed of each biker, the time taken by each biker to finish the race and the length of the course?

Assume that there were no stops in the race and also they were driving with constant speeds through out the

Answer


Let us assume that
Speed of First biker = V1 km/min
Speed of Second biker = V2 km/min
Speed of Third biker = V3 km/min
Total time take by first biker = T1 min
Total distance = S km

Now as per the data given in the teaser, at a time T min
X1 = V1 * T ----> 1

X1 - 15 = V2 * T ----> 2

X1 - 18 = V3 * T ----> 3

At a Distance S Km.
S = V1 * T1 ----> 4

S = V2 * (T1 + 12) ----> 5

S = V3 * (T1 + 15) ----> 6

Thus there are 6 equations and 7 unknown data that means it has infinite number of solutions.

By solving above 6 equations we get,
Time taken by first biker, T1 = 60 Min.
Time taken by Second biker, T2 = 72 Min.
Time taken by first biker, T3 = 75 Min.

Also, we get
Speed of first biker, V1 = 90/T km/min
Speed of second biker, V2 = (5/6)V1 = 75/T km/min
Speed of third biker, V3 = (4/5)V1 = 72/T km/min

Also, the length of the course, S = 5400/T km


Thus, for the data given, only the time taken by each biker can be found i.e. 60, 72 and 75 minutes. For other quantities, one more independent datum is required i.e. either T or V1 or V2 or V3

Thanks to Theertham Srinivas for the answer !!!
What is the four-digit number in which the first digit is 1/3 of the second, the third is the sum of the first and second, and the last is three times the second?

Answer

The 4 digit number is 1349.

It is given that the first digit is 1/3 of the second. There are 3 such possibilities.
1. 1 and 3
2. 2 and 6
3. 3 and 9
Now, the third digit is the sum of the first and second digits.
1. 1 + 3 = 4
2. 2 + 6 = 8
3. 3 + 9 = 12
It is clear that option 3 is not possible. So we are left with only two options. Also, the last digit is three times the second, which rules out the second option. Hence, the answer is 1349.

Difference between Bholu's and Molu's age is 2 years and the difference between Molu's and Kolu's age is 5 years.

What is the maximum possible value of the sum of the difference in their ages, taken two at a time?
Answer

The maximum possible value of the sum of the difference in their ages - taken two at a time - is 14 years.

It is given that -
"Difference between Bholu's and Molu's age is 2 years"
"Difference between Molu's and Kolu's age is 5 years"

Now, to get the maximum possible value, the difference between Bholu's and Kolu's age should be maximum i.e. Molu's age should be in between Bholu's and Kolu's age. Then, the difference between Bholu's and Kolu's age is 7 years.

Hence, the maximum possible value of the sum of the difference in their ages - taken two at a time - is (2 + 5 + 7) 14 years.

If it is given that:
25 - 2 = 3
100 x 2 = 20
36 / 3 = 2

What is 144 - 3 = ?
Submitted
Answer

There are 3 possible answers to it.

Answer 1 : 9
Simply replace the first number by its square root.
(25) 5 - 2 = 3
(100) 10 x 2 = 20
(36) 6 / 3 = 2
(144) 12 - 3 = 9

Answer 2 : 11
Drop the digit in the tens position from the first number.
(2) 5 - 2 = 3
1 (0) 0 x 2 = 20
(3) 6 / 3 = 2
1 (4) 4 - 3 = 11

You will get the same answer on removing left and right digit alternatively from the first number i.e remove left digit from first (2), right digit from second (0), left digit from third (3) and right digit from forth (4).
(2) 5 - 2 = 3
10 (0) x 2 = 20
(3) 6 / 3 = 2
14 (4) - 3 = 11

Answer 3 : 14
Drop left and right digit alternatively from the actual answer.
25 - 2 = (2) 3 (drop left digit i.e. 2)
100 * 2 = 20 (0) (drop right digit i.e. 0)
36 / 3 = (1) 2 (drop left digit i.e. 1)
144 - 3 = 14 (1) (drop right digit i.e. 1)


A 3 digit number is such that it's unit digit is equal to the product of the other two digits which are prime. Also, the difference between it's reverse and itself is 396.

What is the sum of the three digits?

Answer

The required number is 236 and the sum is 11.

It is given that the first two digits of the required number are prime numbers i.e. 2, 3, 5 or 7. Note that 1 is neither prime nor composite. Also, the third digit is the multiplication of the first two digits. Thus, first two digits must be either 2 or 3 i.e. 22, 23, 32 or 33 which means that there are four possible numbers - 224, 236, 326 and 339.

Now, it is also given that - the difference between it's reverse and itself is 396. Only 236 satisfies this condition. Hence, the sum of the three digits is 11.

There are 4 mugs placed upturned on the table. Each mug have the same number of marbles and a statement about the number of marbles in it. The statements are: Two or Three, One or Four, Three or One, One or Two.

Only one of the statement is correct. How many marbles are there under each mug?

Answer

A simple one.

As it is given that only one of the four statement is correct, the correct number can not appear in more than one statement. If it appears in more than one statement, then more than one statement will be correct.

Hence, there are 4 marbles under each mug.
At University of Probability, there are 375 freshmen, 293 sophomores, 187 juniors, & 126 seniors. One student will randomly be chosen to receive an award.

What percent chance is there that it will be a junior? Round to the nearest whole percent
Answer

19%

This puzzle is easy. Divide the number of juniors (187) by the total number of students (981), & then multiply the number by 100 to convert to a percentage.

Hence the answer is (187/981)*100 = 19%


If you were to dial any 7 digits on a telephone in random order, what is the probability that you will dial your own phone number?

Assume that your telephone number is 7-digits.
Answer

1 in 10,000,000

There are 10 digits i.e. 0-9. First digit can be dialed in 10 ways. Second digit can be dialed in 10 ways. Third digit can be dialed in 10 ways. And so on.....

Thus, 7-digit can be dialed in 10*10*10*10*10*10*10 (=10,000,000) ways. And, you have just one telephone number. Hence, the possibility that you will dial your own number is 1 in 10,000,000.

Note that 0123456 may not be a valid 7-digit telephone number. But while dialing in random order, that is one of the possible 7-digit number which you may dial.
An anthropologist discovers an isolated tribe whose written alphabet contains only six letters (call the letters A, B, C, D, E and F). The tribe has a taboo against using the same letter twice in the same word. It's never done.

If each different sequence of letters constitues a different word in the language, what is the maximum number of six-letter words that the language can employ?
Submitted

Answer

The language can employ maximum of 720 six-letter words.

It is a simple permutation problem of arranging 6 letters to get different six-letter words. And it can be done in in 6! ways i.e. 720 ways.

In otherwords, the first letter can be any of the given 6 letters (A through F). Then, whatever the first letter is, the second letter will always be from the remaining 5 letters (as same letter can not be used twice), and the third letter always be from the remaining 4 letters, and so on. Thus, the different possible six-letter words are 6*5*4*3*2*1 = 720

Kate, Demi, Madona, Sharon, Britney and Nicole decided to lunch together in a restaurant. The waiter led them to a round table with six chairs.

How many different ways can they seat?

Answer

There are 120 different possible seating arrangments.

Note that on a round table ABCDEF and BCDEFA is the same.

The first person can sit on any one of the seats. Now, for the second person there are 5 options, for the third person there are 4 options, for the forth person there are 3 options, for the fifth person there are 2 options and for the last person there is just one option.

Thus, total different possible seating arrangements are
= 5 * 4 * 3 * 2 * 1
= 120
3 blocks are chosen randomly on a chessboard. What is the probability that they are in the same diagonal?

Answer

There are total of 64 blocks on a chessboard. So 3 blocks can be chosen out of 64 in 64C3 ways.
So the sample space is = 41664

There are 2 diagonal on chessboard each one having 8 blocks. Consider one of them.
3 blocks out of 8 blocks in diagonal can be chosen in 8C3 ways.
But there are 2 such diagonals, hence favourables = 2 * 8C3 = 2 * 56 = 112

The require probability is
= 112 / 41664
= 1 / 372
= 0.002688


What is the area of the triangle ABC with A(e,p) B(2e,3p) and C(3e,5p)?

where p = PI (3.141592654)

Answer

A tricky ONE.

Given 3 points are colinear. Hence, it is a straight line.

Hence area of triangle is 0.



Silu and Meenu were walking on the road.

Silu said, "I weigh 51 Kgs. How much do you weigh?"

Meenu replied that she wouldn't reveal her weight directly as she is overweight. But she said, "I weigh 29 Kgs plus half of my weight."

How much does Meenu weigh?

Answer

Meenu weighs 58 Kgs.

It is given that Meenu weighs 29 Kgs plus half of her own weight. It means that 29 Kgs is the other half. So she weighs 58 Kgs.

Solving mathematically, let's assume that her weight is X Kgs.
X = 29 + X/2
2*X = 58 + X
X = 58 Kgs
Consider the sum: ABC + DEF + GHI = JJJ

If different letters represent different digits, and there are no leading zeros, what does J represent?

Answer

The value of J must be 9.

Since there are no leading zeros, J must be 7, 8, or 9. (JJJ = ABC + DEF + GHI = 14? + 25? + 36? = 7??)

Now, the remainder left after dividing any number by 9 is the same as the remainder left after dividing the sum of the digits of that number by 9. Also, note that 0 + 1 + ... + 9 has a remainder of 0 after dividing by 9 and JJJ has a remainder of 0, 3, or 6.

The number 9 is the only number from 7, 8 and 9 that leaves a remainder of 0, 3, or 6 if you remove it from the sum 0 + 1 + ... + 9. Hence, it follows that J must be 9.
A man has Ten Horses and nine stables as shown here.
[] [] [] [] [] [] [] [] []
The man wants to fit Ten Horses into nine stables. How can he fit Ten horses into nine stables?
Submitted
Answer

The answer is simple. It says the man wants to fit "Ten Horses" into nine stables. There are nine letters in the phrase "Ten Horses". So you can put one letter each in all nine stables.
[T] [E] [N] [H] [O] [R] [S] [E] [S]

A man is at a river with a 9 gallon bucket and a 4 gallon bucket. He needs exactly 6 gallons of water.

How can he use both buckets to get exactly 6 gallons of water?

Note that he cannot estimate by dumping some of the water out of the 9 gallon bucket or the 4 gallon bucket
Answer

For the sack of explanation, let's identify 4 gallon bucket as Bucket P and 9 gallon bucket as Bucket Q.

Operation 4 gallon bucket
(Bucket P) 9 gallon bucket
(Bucket Q)
Initially 0 0
Fill the bucket Q with 9 gallon water 0 9
Pour 4 gallon water from bucket Q to bucket P 4 5
Empty bucket P 0 5
Pour 4 gallon water from bucket Q to bucket P 4 1
Empty bucket P 0 1
Pour 1 gallon water from bucket Q to bucket P 1 0
Fill the bucket Q with 9 gallon water 1 9
Pour 3 gallon water from bucket Q to bucket P 4 6

9 gallon bucket contains 6 gallon of water, as required.

Each of the five characters in the word BRAIN has a different value between 0 and 9. Using the given grid, can you find out the value of each character?
B R A I N 31

B B R B A 31

N I A B B 32

N I B A I 30

I R A A A 23

37 29 25 27 29
The numbers on the extreme right represent the sum of the values represented by the characters in that row. Also, the numbers on the last raw represent the sum of the values represented by the characters in that column. e.g. B + R + A + I + N = 31 (from first row)
Answer

B=7, R=6, A=4, I=5 and N=9

Make total 10 equations - 5 for rows and 5 for columns - and sovle them.

From Row3 and Row4,
N + I + A + B + B = N + I + B + A + I + 2
B = I + 2

From Row1 and Row3,
B + R + A + I + N = N + I + A + B + B - 1
R = B - 1

From Column2,
R + B + I + I + R = 29
B + 2R + 2I = 29
B + 2(B - 1) + 2I = 29
3B + 2I = 31
3(I + 2) + 2I = 31
5I = 25
I = 5

Hence, B=7 and R=6

From Row2,
B + B + R + B + A = 31
3B + R + A = 31
3(7) + 6 + A = 31
A = 4

From Row1,
B + R + A + I + N = 31
7 + 6 + 4 + 5 + N = 31
N = 9

Thus, B=7, R=6, A=4, I=5 and N=9

Submit
Answer

Users
Answer (24)

BrainV


There are 9 coins. Out of which one is odd one i.e weight is less or more. How many iterations of weighing are required to find odd coin?
Answer

It is always possible to find odd coin in 3 weighings and to tell whether the odd coin is heavier or lighter.
1. Take 8 coins and weigh 4 against 4.
o If both are not equal, goto step 2
o If both are equal, goto step 3

2. One of these 8 coins is the odd one. Name the coins on heavier side of the scale as H1, H2, H3 and H4. Similarly, name the coins on the lighter side of the scale as L1, L2, L3 and L4. Either one of H's is heavier or one of L's is lighter. Weigh (H1, H2, L1) against (H3, H4, X) where X is one coin remaining in intial weighing.
o If both are equal, one of L2, L3, L4 is lighter. Weigh L2 against L3.
 If both are equal, L4 is the odd coin and is lighter.
 If L2 is light, L2 is the odd coin and is lighter.
 If L3 is light, L3 is the odd coin and is lighter.

o If (H1, H2, L1) is heavier side on the scale, either H1 or H2 is heavier. Weight H1 against H2
 If both are equal, there is some error.
 If H1 is heavy, H1 is the odd coin and is heavier.
 If H2 is heavy, H2 is the odd coin and is heavier.

o If (H3, H4, X) is heavier side on the scale, either H3 or H4 is heavier or L1 is lighter. Weight H3 against H4
 If both are equal, L1 is the odd coin and is lighter.
 If H3 is heavy, H3 is the odd coin and is heavier.
 If H4 is heavy, H4 is the odd coin and is heavier.

3. The remaining coin X is the odd one. Weigh X against the anyone coin used in initial weighing.
o If both are equal, there is some error.
o If X is heavy, X is the odd coin and is heavier.
o If X is light, X is the odd coin and is lighter.
In a sports contest there were m medals awarded on n successive days (n > 1).
1. On the first day 1 medal and 1/7 of the remaining m - 1 medals were awarded.
2. On the second day 2 medals and 1/7 of the now remaining medals was awarded; and so on.
3. On the nth and last day, the remaining n medals were awarded.
How many days did the contest last, and how many medals were awarded altogether?
Answer

Total 36 medals were awarded and the contest was for 6 days.

On day 1: Medals awarded = (1 + 35/7) = 6 : Remaining 30 medals
On day 2: Medals awarded = (2 + 28/7) = 6 : Remaining 24 medals
On day 3: Medals awarded = (3 + 21/7) = 6 : Remaining 18 medals
On day 4: Medals awarded = (4 + 14/7) = 6 : Remaining 12 medals
On day 5: Medals awarded = (5 +7/7) = 6 : Remaining 6 medals
On day 6: Medals awarded 6

I got this answer by writing small program. If anyone know any other simpler method, do submit it.

A number of 9 digits has the following properties:
• The number comprising the leftmost two digits is divisible by 2, that comprising the leftmost three digits is divisible by 3, the leftmost four by 4, the leftmost five by 5, and so on for the nine digits of the number i.e. the number formed from the first n digits is divisible by n, 2<=n<=9.
• Each digit in the number is different i.e. no digits are repeated.
• The digit 0 does not occur in the number i.e. it is comprised only of the digits 1-9 in some order.
Find the number.

Answer

The answer is 381654729

One way to solve it is Trial-&-Error. You can make it bit easier as odd positions will always occupy ODD numbers and even positions will always occupy EVEN numbers. Further 5th position will contain 5 as 0 does not occur.

The other way to solve this problem is by writing a computer program that systematically tries all possibilities.


1/3 rd of the contents of a container evaporated on the 1st day. 3/4th of the remaining contents of the container evaporated on the second day.

What part of the contents of the container is left at the end of the second day?
Answer

Assume that contents of the container is X

On the first day 1/3rd is evaporated.
(1 - 1/3) of X is remaining i.e. (2/3)X

On the Second day 3/4th is evaporated. Hence,
(1- 3/4) of (2/3)X is remaining
i.e. (1/4)(2/3)X = (1/6) X

Hence 1/6th of the contents of the container is remaining

There are four people in a room (not including you). Exactly two of these four always tell the truth. The other two always lie.

You have to figure out who is who IN ONLY 2 QUESTIONS. Your questions have to be YES or NO questions and can only be answered by one person. (If you ask the same question to two different people then that counts as two questions). Keep in mind that all four know each other's characteristics whether they lie or not.

What questions would you ask to figure out who is who? Remember that you can ask only 2 questions.
Submitted
You have 3 baskets, & each one contains exactly 4 balls, each of which is of the same size. Each ball is either red, black, white, or purple, & there is one of each color in each basket.

If you were blindfolded, & lightly shook each basket so that the balls would be randomly distributed, & then took 1 ball from each basket, what chance is there that you would have exactly 2 red balls?

Answer

There are 64 different possible outcomes, & in 9 of these, exactly 2 of the balls will be red. There is thus a slightly better than 14% chance [(9/64)*100] that exactly 2 balls will be red.

A much faster way to solve the problem is to look at it this way. There are 3 scenarios where exactly 3 balls are red:

1 2 3

-----------

R R X

R X R

X R R

X is any ball that is not red.
There is a 4.6875% chance that each of these situations will occur.

Take the first one, for example: 25% chance the first ball is red, multiplied by a 25% chance the second ball is red, multiplied by a 75% chance the third ball is not red.

Because there are 3 scenarios where this outcome occurs, you multiply the 4.6875% chance of any one occurring by 3, & you get 14.0625%

Consider a state lottery where you get to choose 8 numbers from 1 to 80, no repetiton allowed. The Lottery Commission chooses 11 from those 80 numbers, again no repetition. You win the lottery if atleast 7 of your numbers are there in the 11 chosen by the Lottery Commission.

What is the probablity of winning the lottery?
Answer

The probability of winning the lottery is two in one billion i.e. only two person can win from one billion !!!

Let's find out sample space first. The Lottery Commission chooses 11 numbers from the 80. Hence, the 11 numbers from the 80 can be selected in 80C11 ways which is very very high and is equal to 1.04776 * 1013

Now, you have to select 8 numbers from 80 which can be selected in 80C8 ways. But we are interested in only those numbers which are in 11 numbers selected by the Lottery Commision. There are 2 cases.
• You might select 8 numbers which all are there in 11 numbers choosen by the Lottery Commission. So there are 11C8 ways.
• Another case is you might select 7 lucky numbers and 1 non-lucky number from the remaining 69 numbers. There are ( 11C7 ) * ( 69C1 ) ways to do that.
So total lucky ways are
= ( 11C8 ) + ( 11C7 ) * ( 69C1 )
= (165) + (330) * (69)
= 165 + 22770
= 22935

Hence, the probability of the winning lottery is
= (Total lucky ways) / (Total Sample space)
= (22935) / ( 1.04776 * 1013)
= 2.1889 * 10-9
i.e. 2 in a billion.

Submit
Answer

Users
Answer (4)

BrainVista
Answer

Puzzle

To move a Safe, two cylindrical steel bars 7 inches in diameter are used as rollers.

How far will the safe have moved forward when the rollers have made one revolution?
Answer

The safe must have moved 22 inches forward.

If the rollers make one revolution, the safe will move the distance equal to the circumference of the roller. Hence, the distance covered by the safe is
= PI * Diameter (or 2 * PI * Radius)
= PI * 7
= 3.14159265 * 7
= 21.99115
= 22 inches approx.
SubmittIf a rook and a bishop of a standard chess set are randomly placed on a chessboard, what is the probability that one is attacking the other?

Note that both are different colored pieces.
SubmAnswer

The probability of either the Rook or the Bishop attacking the other is 0.3611

A Rook and a Bishop on a standard chess-board can be arranged in 64P2 = 64*63 = 4032 ways

Now, there are 2 cases - Rook attacking Bishop and Bishop attacking Rook. Note that the Rook and the Bishop never attack each other simultaneously. Let's consider both the cases one by one.

Case I - Rook attacking Bishop
The Rook can be placed in any of the given 64 positions and it always attacks 14 positions. Hence, total possible ways of the Rook attacking the Bishop = 64*14 = 896 ways

Case II - Bishop attacking Rook
View the chess-board as a 4 co-centric hollow squares with the outermost square with side 8 units and the innermost square with side 2 units.

If the bishop is in one of the outer 28 squares, then it can attack 7 positions. If the bishop is in one of the 20 squares at next inner-level, then it can attack 9 positions. Similarly if the bishop is in one of the 12 squares at next inner-level, then it can attack 11 positions. And if the bishop is in one of the 4 squares at next inner-level (the innermost level), then it can attack 13 positions.

Hence, total possible ways of the Bishop attacking the Rook
= 28*7 + 20*9 + 12*11 + 4*13
= 560 ways

Thus, the required probability is
= (896 + 560) / 4032
= 13/36
= 0.3611
itted ed
Here in England McDonald's has just launched a new advertising campaign. The poster shows 8 McDonald's products and underneath claims there are 40312 combinations of the above items.

Given that the maximum number of items allowed is 8, and you are allowed to have less than 8 items, and that the order of purchase does not matter (i.e. buying a burger and fries is the same as buying fries and a burger)

How many possible combinations are there? Are McDonald's correct in claiming there are 40312 combinations?
Answer

Total possible combinations are 12869.

It is given that you can order maximum of 8 items and you are allowed to have less than 8 items. Also, the order of purchase does not matter. Let's create a table for ordering total N items using X products.
Items
Ordered
(N) Products Used (X)
1 2 3 4 5 6 7 8
1 1 - - - - - - -
2 1 1 - - - - - -
3 1 2 1 - - - - -
4 1 3 3 1 - - - -
5 1 4 6 4 1 - - -
6 1 5 10 10 5 1 - -
7 1 6 15 20 15 6 1 -
8 1 7 21 35 35 21 7 1
Total (T) 8 28 56 70 56 28 8 1
Ways to choose
X products from
8 products (W) 8C1 8C2 8C3 8C4 8C5 8C6 8C7 8C8
Total combinations
(T*W) 64 784 3136 4900 3136 784 64 1

Thus, total possible combinations are
= 64 + 784 + 3136 + 4900 + 3136 + 784 + 64 + 1
= 12869
What are the chances that at least two out of a group of fifty people share the same birthday?
SubmittedAnswer

The probability of atleast two out of a group of 50 people share the same birthday is 97%

Probability of atleast two share the same birthday = 1 - probability of all 50 have different birthdays

Probability of all 50 have different birthday
= 365/365 * 364/365 * 363/365 * ... * 317/365 * 316/365
= (365 * 364 * 363 * 362 * ... * 317 * 316)/36550
= 0.0296264

Probability of atleast two share the same birthday
= 1 - 0.0296264
= 0.9703735
= 97% approx.

Thus, the probability of atleast two out of a group of 50 people share the same birthday is 97%

This explains why in a school/college with classrooms of 50 students, there are at least two students with a birthday on the same day of the year. Also, if there are 23 people in the room, then there are 50% chances that atleast two of them have a birthday on the same day of the year!!!

A tank can be filled by pipe A in 30 minutes and by pipe B in 24 minutes. Outlet pipe C can empty the full tank in X minutes.

If the tank is empty initially and if all the three pipes A, B and C are opened simultaneously, the tank will NEVER be full. Give the maximal possible value of X.
Answer

The maximum possible value of X is 13 minutes 20 seconds.

In one minute,
pipe A can fill 1/30 part of the tank.
pipe B can fill 1/24 part of the tank.

Thus, the net water level increase in one minute is
= 1/30 + 1/24
= 3/40 part of the tank

In order to keep the tank always empty, outlet pipe C should empty at least 3/40 part of the tank in one minute. Thus, pipe C can empty the full tank in 40/3 i.e. 13 minutes 20 seconds.
A worker earns a 5% raise. A year later, the worker receives a 2.5% cut in pay, & now his salary is Rs. 22702.68

What was his salary to begin with?
Answer

Rs.22176

Assume his salary was Rs. X

He earns 5% raise. So his salary is (105*X)/100

A year later he receives 2.5% cut. So his salary is ((105*X)/100)*(97.5/100) which is Rs. 22702.68

Hence, solving equation ((105*X)/100)*(97.5/100) = 22702.68
X = 22176

500 men are arranged in an array of 10 rows and 50 columns according to their heights.

Tallest among each row of all are asked to come out. And the shortest among them is A.

Similarly after resuming them to their original positions, the shortest among each column are asked to come out. And the tallest among them is B.

Now who is taller A or B ?
A person wanted to withdraw X rupees and Y paise from the bank. But cashier made a mistake and gave him Y rupees and X paise. Neither the person nor the cashier noticed that.

After spending 20 paise, the person counts the money. And to his surprise, he has double the amount he wanted to withdraw.

Find X and Y. (1 Rupee = 100 Paise)

As given, the person wanted to withdraw 100X + Y paise.

But he got 100Y + X paise.

After spending 20 paise, he has double the amount he wanted to withdraw. Hence, the equation is

2 * (100X + Y) = 100Y + X - 20

200X + 2Y = 100Y +X - 20

199X - 98Y = -20

98Y - 199X = 20


Now, we got one equation; but there are 2 variables. We have to apply little bit of logic over here. We know that if we interchange X & Y, amount gets double. So Y should be twice of X or one more than twice of X i.e. Y = 2X or Y = 2X+1

Case I : Y=2X
Solving two equations simultaneously
98Y - 199X = 20
Y - 2X = 0
We get X = - 20/3 & Y = - 40/2

Case II : Y=2X+1
Solving two equations simultaneously
98Y - 199X = 20
Y - 2X = 1
We get X = 26 & Y = 53

Now, its obvious that he wanted to withdraw Rs. 26.53

Submit
Answer

Users
Answer (2)

BrainVista
Answer

Puzzle


At the Party:
1. There were 9 men and children.
2. There were 2 more women than children.
3. The number of different man-woman couples possible was 24. Note that if there were 7 men and 5 women, then there would have been 35 man-woman couples possible.

Also, of the three groups - men, women and children - at the party:
4. There were 4 of one group.
5. There were 6 of one group.
6. There were 8 of one group.
Exactly one of the above 6 statements is false.

Can you tell which one is false? Also, how many men, women and children are there at the party?
Answer

Statement (4) is false. There are 3 men, 8 women and 6 children.

Assume that Statements (4), (5) and (6) are all true. Then, Statement (1) is false. But then Statement (2) and (3) both can not be true. Thus, contradictory to the fact that exactly one statement is false.

So Statement (4) or Statement (5) or Statement (6) is false. Also, Statements (1), (2) and (3) all are true.

From (1) and (2), there are 11 men and women. Then from (3), there are 2 possible cases - either there are 8 men and 3 women or there are 3 men and 8 women.

If there are 8 men and 3 women, then there is 1 child. Then Statements (4) and (5) both are false, which is not possible.

Hence, there are 3 men, 8 women and 6 children. Statement (4) is false.
Brain Teaser No : 00242

There is a shortage of tubelights, bulbs and fans in a village - Kharghar. It is found that
• All houses do not have either tubelight or bulb or fan.
• exactly 19% of houses do not have just one of these.
• atleast 67% of houses do not have tubelights.
• atleast 83% of houses do not have bulbs.
• atleast 73% of houses do not have fans.
What percentage of houses do not have tubelight, bulb and fan?

Answer

42% houses do not have tubelight, bulb and fan.

Let's assume that there are 100 houses. Hence, there should be total 300 items i.e. 100 tubelights, 100 bulbs and 100 fans.

From the given data, we know that there is shortage of atleast (67+83+73) 223 items in every 100 houses.

Also, exactly 19 houses do not have just one item. It means that remaining 81 houses should account for the shortage of remaining (223-19) 204 items. If those remaining 81 houses do not have 2 items each, there would be a shortage of 162 items. But total of 204 items are short. Hence, atleast (204-162) 42 houses do not have all 3 items - tubelight, bulb and fan.

Thus, 42% houses do not have tubelight, bulb and fan.
What is the remainder left after dividing 1! + 2! + 3! + … + 100! By 7?

Think carefully !!!
Answer

A tricky one.

7! onwards all terms are divisible by 7 as 7 is one of the factor. So there is no remainder left for those terms i.e. remainder left after dividing 7! + 8! + 9! + ... + 100! is 0.

The only part to be consider is
= 1! + 2! + 3! + 4! + 5! + 6!
= 1 + 2 + 6 + 24 + 120 + 720
= 873

The remainder left after dividing 873 by 7 is 5

Hence, the remainder is 5.

Imagine that you have 26 constants, labelled A through Z. Each constant is assigned a value in the following way: A = 1; the rest of the values equal their position in the alphabet (B corresponds to the second position so it equals 2, C = 3, etc.) raised to the power of the preceeding constant value. So, B = 2 ^ (A's value), or B = 2^1 = 2. C = 3^2 = 9. D = 4^9, etc.

Find the exact numerical value to the following equation: (X - A) * (X - B) * (X - C) * ... * (X - Y) * (X - Z)

Answer

(X - A) * (X - B) * (X - C) * ... * (X - Y) * (X - Z) equals 0 since (X - X) is zero
If three babies are born every second of the day, then how many babies will be born in the year 2001?
SuAnswer

9,46,08,000 babies

The total seconds in year 2001
= 365 days/year * 24 hours/day * 60 minutes/hours * 60 seconds/minute
= 365 * 24 * 60 * 60 seconds
= 3,15,36,000 seconds

Thus, there are 3,15,36,000 seconds in the year 2001. Also, three babies born are every second. Hence, total babies born
= 3 * 3,15,36,000 seconds
= 9,46,08,000bmitted

Replace the letters with the correct numbers.
T W O

X T W O

---------

T H R E E

Submitted by : Timmy Chan
Answer

T=1, W=3, O=8, H=9, R=2, E=4
1 3 8

x 1 3 8

------------

1 9 0 4 4
You can reduce the number of trials. T must be 1 as there is multiplication of T with T in hundred's position. Also, O can not be 0 or 1. Now, you have to find three digit number whose square satisfies above conditions and square of that has same last two digits. Hence, it must be between 102 and 139.
Brain Teaser No : 00052

Four words add up to a fifth word numerically:

mars

venus

uranus

saturn

-------- +

neptune

Each of the ten letters (m, a, r, s, v, e, n, u, t, and p) represent a unique number from the range 0 .. 9.

Furthermore, numbers 1 and 6 are being used most frequently.
Answer

The easiest way to solve this problem is by writing a computer program that systematically tries all possible mappings from the numbers onto the letters. This will give you only one solution which meets the condition that numbers 1 and 6 are most frequently used.

mars m = 4

venus a = 5

uranus r = 9

saturn s = 3

-------- + v = 2 4593

neptune e = 0 20163

n = 1 695163

u = 6 358691

t = 8 -------- +

p = 7 1078610

There are 4 army men. They have been captured by a rebel group and have been held at ransom. An army intelligent officer orders them to be burried deep in dirt up to their necks. The format of their burrial are as shown in the figure.

Conditions
• They each have hats on their heads. either black(b) or white (w) look at diagram above. There are total 2 white hats and 2 black hats.
• They only look in front of them not behind. They are not allowed to communicate by talking.
• Between army man 1 and 2, there is a wall.
• Captive man 4 can see the colour of hats on 2 and 3
• 3 can only see 2's hat
• 2 can only see a wall and 1 can see a wall too, but is on the other side
The officer speaks up, "If one of you can correctly tell me the colour of your hat, you will all go scott free back to your contries. If you are wrong, you will all be killed.

How can one of them be certain about the hat they are wearing and not risk the lives of their fellow souldiers by taking a 50/50 guess!
Submitted
Answer

Either soldier 3 or soldier 4 can save the life as soldier 1 and soldier 2 can not see colour of any hat, even not their own.. In our case soldier 3 will tell the colour of his hat.

Soldier 4 can see the hat on soldier 2 and soldier 3. If both are white, then he can be sure about colour of his hat which will be black and vice-versa. But if one of them is white and one is black, then soldier 4 can not say anything as he can have either of them. So he will keep mum.

If soldier 4 won't say anyhing for a while, then soldier 3 will know that soldier 4 is not in position to tell the colour of hat on his hat. It means that colour of soldier 3's hat is opposite of colour of soldier 2's hat. So soldier 3 can tell correctly the colour of hat on his head which is Black.

Here, we are assuming that all the soldiers are intelligent enough. Also, this solution will work for any combination of 2 Black hats and 2 White hats.
One side of the bottom layer of a triangular pyramid has 12 balls. How many are there in the whole pyramid?

Note that the pyramid is equilateral and solid.
Answer

There are total 364 balls.

As there are 12 balls along one side, it means that there are 12 layers of balls. The top most layer has 1 ball. The second layer has 3 (1+2) balls. The third layer has 6 (1+2+3) balls. The fourth layer has 10 (1+2+3+4) balls. The fifth layer has 15 (1+2+3+4+5) balls. Similarly, there are 21, 28, 36, 45, 55, 66 and 78 balls in the remaining layers.

Hence, the total number of balls are
= 1 + 3 + 6 + 10 + 15 + 21 + 28 + 36 + 45 + 55 + 66 + 78
= 364 balls


A blindfolded man is asked to sit in the front of a carrom board. The holes of the board are shut with lids in random order, i.e. any number of all the four holes can be shut or open.

Now the man is supposed to touch any two holes at a time and can do the following.
• Open the closed hole.
• Close the open hole.
• Let the hole be as it is.
After he has done it, the carrom board is rotated and again brought to some position. The man is again not aware of what are the holes which are open or closed.

How many minimum number of turns does the blindfolded man require to either open all the holes or close all the holes?

Note that whenever all the holes are either open or close, there will be an alarm so that the blindfolded man will know that he has won.
Submitted
Answer

The blindfolded man requires 5 turns.
1. Open two adjacent holes.
2. Open two diagonal holes. Now atleast 3 holes are open. If 4th hole is also open, then you are done. If not, the 4th hole is close.
3. Check two diagonal holes.
o If one is close, open it and all the holes are open.
o If both are close, open any one hole. Now, two holes are open and two are close. The diagonal holes are in the opposite status i.e. in both the diagonals, one hole is open and one is close.

4. Check any two adjacent holes.
o If both are open, close both of them. Now, all holes are close.
o If both are close, open both of them. Now, all holes are open.
o If one is open and one is close, invert them i.e. close the open hole and open the close hole. Now, the diagonal holes are in the same status i.e. two holes in one diagonal are open and in other are close.

5. Check any two diagonal holes.
o If both are open, close both of them. Now, all holes are close.
o If both are close, open both of them. Now, all holes are open.

In the middle of the confounded desert, there is the lost city of "Ash". To reach it, I will have to travel overland by foot from the coast. On a trek like this, each person can only carry enough rations for five days and the farthest we can travel in one day is 30 miles. Also, the city is 120 miles from the starting point.

What I am trying to figure out is the fewest number of persons, including myself, that I will need in our Group so that I can reach the city, stay overnight, and then return to the coast without running out of supplies.

How many persons (including myself) will I need to accomplish this mission?
Answer

Total 4 persons (including you) required.

It is given that each person can only carry enough rations for five days. And there are 4 persons. Hence, total of 20 days rations is available.
1. First Day : 4 days of rations are used up. One person goes back using one day of rations for the return trip. The rations remaining for the further trek is for 15 days.
2. Second Day : The remaining three people use up 3 days of rations. One person goes back using 2 days of rations for the return trip. The rations remaining for the further trek is for 10 days.
3. Third Day : The remaining two people use up 2 days of rations. One person goes back using 3 days of rations for the return trip. The rations remaining for the further trek is for 5 days.
4. Fourth Day : The remaining person uses up one day of rations. He stays overnight. The next day he returns to the coast using 4 days of rations.

Thus, total 4 persons, including you are required.
At what time after 4.00 p.m. is the minutes hand of a clock exactly aligned with the hour hand?
Answer

4:21:49.5

Assume that X minutes after 4.00 PM minute hand exactly aligns with and hour hand.

For every minute, minute hand travels 6 degrees.
Hence, for X minutes it will travel 6 * X degrees.

For every minute, hour hand travels 1/2 degrees.
Hence, for X minutes it will travel X/2 degrees.

At 4.00 PM, the angle between minute hand and hour hand is 120 degrees. Also, after X minutes, minute hand and hour hand are exactly aligned. So the angle with respect to 12 i.e. Vertical Plane will be same. Therefore,

6 * X = 120 + X/2
12 * X = 240 + X
11 * X = 240
X = 21.8182
X = 21 minutes 49.5 seconds

Hence, at 4:21:49.5 minute hand is exactly aligned with the hour hand.


Substitute digits for the letters to make the following Division true
O U T

-------------

S T E M | D E M I S E

| D M O C

-------------

T U I S

S T E M

----------

Z Z Z E

Z U M M

--------

I S T
Note that the leftmost letter can't be zero in any word. Also, there must be a one-to-one mapping between digits and letters. e.g. if you substitute 3 for the letter M, no other letter can be 3 and all other M in the puzzle must be 3.
Submitted by : Calon
Answer

C=0, U=1, S=2, T=3, O=4, M=5, I=6, Z=7, E=8, D=9

It is obvious that U=1 (as U*STEM=STEM) and C=0 (as I-C=I).

S*O is a single digit and also S*T is a single digit. Hence, their values (O, S, T) must be 2, 3 or 4 (as they can not be 0 or 1 or greater than 4).

Consider, STEM*O=DMOC, where C=0. It means that M must be 5. Now, its simple. O=4, S=2, T=3, E=8, Z=7, I=6 and D=9.
O U T 4 1 3

------------- -------------

S T E M | D E M I S E 2 3 8 5 | 9 8 5 6 2 8

| D M O C | 9 5 4 0

------------- -------------

T U I S 3 1 6 2

S T E M 2 3 8 5

---------- ----------

Z Z Z E 7 7 7 8

Z U M M 7 1 5 5

-------- --------

I S T 6 2 3
Also, when arranged from 0 to 9, it spells CUSTOMIZED.
Brain Teaser No : 00015

In the town called Alibaug, the following facts are true:
• No two inhabitants have exactly the same number of hairs.
• No inhabitants has exactly 2025 hairs.
• There are more inhabitants than there are hairs on the head of any one inhabitants.
What is the largest possible number of the inhabitants of Alibaug?
Answer

2025

It is given that no inhabitants have exactly 2025 hairs. Hence there are 2025 inhabitants with 0 to 2024 hairs in the head.

Suppose there are more than 2025 inhabitants. But these will violate the condition that "There are more inhabitants than there are hairs on the head of any one inhabitants." As for any number more than 2025, there will be same number of inhabitants as the maximum number of hairs on the head of any inhabitant.
There are four groups of Mangoes, Apples and Bananas as follows:
Group I : 1 Mango, 1 Apples and 1 Banana
Group II : 1 Mango, 5 Apples and 7 Bananas
Group III : 1 Mango, 7 Apples and 10 Bananas
Group IV : 9 Mango, 23 Apples and 30 Bananas

Group II costs Rs 300 and Group III costs Rs 390.

Can you tell how much does Group I and Group IV cost?
Answer

Group I costs Rs 120 and Group IV costs Rs 1710

Assume that the values of one mango, one apple and one banana are M, A and B respectively.

From Group II : M + 5A + 7B = 300
From Group III : M + 7A + 10B = 390

Subtracting above to equations : 2A + 3B = 90

For Group I :
= M + A + B
= (M + 5A + 7B) - (4A + 6B)
= (M + 5A + 7B) - 2(2A + 3B)
= 300 - 2(90)
= 300 - 180
= 120

Similarly, for Group IV :
= 9M + 23A + 30B
= 9(M + 5A + 7B) - (22A + 33B)
= 9(M + 5A + 7B) - 11(2A + 3B)
= 9(300) - 11(90)
= 2700 - 990
= 1710

Thus, Group I costs Rs 120 and Group IV costs Rs 1710.
Tic-Tac-Toe is being played. One 'X' has been placed in one of the corners. No 'O' has been placed yet.

Where does the player that is playing 'O' has to put his first 'O' so that 'X' doesn't win?

Assume that both players are very intelligent. Explain your answer
Answer

"O" should be placed in the center.

Let's number the positions as:
1 | 2 | 3

---------

4 | 5 | 6

---------

7 | 8 | 9
It is given that "X" is placed in one of the corner position. Let's assume that its at position 1.

Now, let's take each position one by one.
• If "O" is placed in position 2, "X" can always win by choosing position 4, 5 or 7.
• If "O" is placed in position 3, "X" can always win by choosing position 4, 7 or 9.
• If "O" is placed in position 4, "X" can always win by choosing position 2, 3 or 5.
• If "O" is placed in position 6, "X" can always win by choosing position 3, 5 or 7.
• If "O" is placed in position 7, "X" can always win by choosing position 2, 3 or 9.
• If "O" is placed in position 8, "X" can always win by choosing position 3, 5 or 7.
• If "O" is placed in position 9, "X" can always win by choosing position 3, or 7.
If "O" is placed in position 5 i.e. center position, "X" can't win unless "O" does something foolish ;))

Hence, "O" should be placed in the center.

Amit, Bhavin, Himanshu and Rakesh are sitting around a table.
• The Electonics Engineer is sitting to the left of the Mechanical Engineer.
• Amit is sitting opposite to Computer Engineer.
• Himanshu likes to play Computer Games.
• Bhavin is sitting to the right of the Chemical Engineer.
Can you figure out everyone's profession?
Answer

Amit is the Mechanical Engineer. Bhavin is the Computer Engineer. Himanshu and Rakesh are either Chemical Engineer or Elecronics Engineer.

Amit and Bhavin are sitting opposite to each other. Whereas Chemical Engineer and Elecronics Engineer are sitting opposite to each other.

We cannot find out who is Chemical Engineer and Elecronics Engineer as data provided is not sufficient

Five friends with surname Batliwala, Pocketwala, Talawala, Chunawala and Natakwala have their first name and middle name as follow.
1. Four of them have a first and middle name of Paresh.
2. Three of them have a first and middle name of Kamlesh.
3. Two of them have a first and middle name of Naresh.
4. One of them have a first and middle name of Elesh.
5. Pocketwala and Talawala, either both are named Kamlesh or neither is named Kamlesh.
6. Either Batliwala and Pocketwala both are named Naresh or Talawala and Chunawala both are named Naresh.
7. Chunawala and Natakwala are not both named Paresh.
Who is named Elesh?
Answer

Pocketwala is named Elesh.

From (1) and (7), it is clear that Batliwala, Pocketwala and Talawala are named Paresh.

From (6) and (5), if Pocketwala or Talawala both are named Kamlesh, then either of them will have three names i.e. Paresh, Kamlesh and Naresh. Hence, Pocketwala and Talawala both are not named Kamlesh. It means that Batliwala, Chunawala and Natakwala are named Kamlesh.

Now it is clear that Talawala and Chunawala are named Naresh. Also, Pocketwala is named Elesh.

Mr. Wagle goes to work by a bus. One day he falls asleep when the bus still has twice as far to go as it has already gone.

Halfway through the trip he wakes up as the bus bounces over some bad potholes. When he finally falls asleep again, the bus still has half the distance to go that it has already travelled. Fortunately, Mr. Wagle wakes up at the end of his trip.

What portion of the total trip did Mr. Wagle sleep?

Answer

Mr. wagle slept through half his trip.

Let's draw a timeline. Picture the bus route on a line showen below:

---------------- ________ -------- ________________

Start 1/3 1/2 2/3 End



----- shows time for which Mr. Wagle was not sleeping

_____ shows time for which Mr. Wagle was sleeping


When Mr. Wagle fell asleep the first time, the bus sill had twice as far to go as it had already gone, that marks the first third of his trip.

He wake up halfway through the trip i.e slept from 1/3 mark to the 1/2 mark. He fell sleep again when the bus still had half the distance to go that it had already traveled i.e 2/3 mark.

Adding up, all sleeping times,
= (1/2 - 1/3) + (1 - 2/3)
= 1/6 + 1/3
= 1/2

Hence, Mr. wagle slept through half his trip.
Brain Teaser No : 00068

In your sock drawer, you have a ratio of 5 pairs of blue socks, 4 pairs of brown socks, and 6 pairs of black socks.

In complete darkness, how many socks would you need to pull out to get a matching pair of the same color?
4 If you don't agree, try it yourself!
You have a bucket of jelly beans. Some are red, some are blue, and some green. With your eyes closed, pick out 2 of a like color.

How many do you have to grab to be sure you have 2 of the same?

You have a bucket of jelly beans. Some are red, some are blue, and some green. With your eyes closed, pick out 2 of a like color.

How many do you have to grab to be sure you have 2 of the same?
If you select 4 Jelly beans you are guarenteed that you will have 2 that are the same color.


There are 70 employees working with BrainVista of which 30 are females. Also,
• 30 employees are married
• 24 employees are above 25 years of age
• 19 married employees are above 25 years, of which 7 are males
• 12 males are above 25 years of age
• 15 males are married.
How many unmarried females are there and how many of them are above 25?
Answer

15 unmarried females & none are above 25 years of age.

Simply put all given information into the table structure and you will get the answer.
Married Unmarried
Below 25 Above 25 Below 25 Above 25
Female 3 12 15 0
Male 8 7 20 5

There is a safe with a 5 digit number as the key. The 4th digit is 4 greater than the second digit, while the 3rd digit is 3 less than the 2nd digit. The 1st digit is thrice the last digit. There are 3 pairs whose sum is 11.

Find the number.
Answer

65292

As per given conditions, there are three possible combinations for 2nd, 3rd and 4th digits. They are (3, 0, 7) or (4, 1, 8) or (5, 2, 9)

It is given that there are 3 pairs whose sum is 11. All possible pairs are (2, 9), (3, 8), (4, 7), (5, 6). Now required number is 5 digit number and it contains 3 pairs of 11. So it must not be having 0 and 1 in it. Hence, the only possible combination for 2nd, 3rd and 4th digits is (5, 2, 9)

Also, 1st digit is thrice the last digit. The possible combinations are (3, 1), (6, 2) and (9, 3), out of which only (6, 2) with (5, 2, 9) gives 3 pairs of 11. Hence, the answer is 65292.
My friend collects antique stamps. She purchased two, but found that she needed to raise money urgently. So she sold them for Rs. 8000 each. On one she made 20% and on the other she lost 20%.

How much did she gain or lose in the entire transaction?
Answer

She lost Rs 666.67

Consider the first stamp. She mades 20% on it after selling it for Rs 8000.

So the original price of first stamp is
= (8000 * 100) / 80
= Rs 6666.67

Similarly, consider second stamp. She lost 20% on it after selling it for Rs 8000

So the original price of second stamp is
= (8000 * 100) / 80
= Rs 10000

Total buying price of two stamps
= Rs 6666.67 + Rs 10000
= Rs 16666.67

Total selling price of two stamps
= Rs 8000 + Rs 8000
= Rs 16000

Hence, she lost Rs 666.67


Assume for a moment that the earth is a perfectly uniform sphere of radius 6400 km. Suppose a thread equal to the length of the circumference of the earth was placed along the equator, and drawn to a tight fit.

Now suppose that the length of the thread is increased by 12 cm, and that it is pulled away uniformly in all directions.

By how many cm. will the thread be separated from the earth's surface?
Answer

The cicumference of the earth is
= 2 * PI * r
= 2 * PI * 6400 km
= 2 * PI * 6400 * 1000 m
= 2 * PI * 6400 * 1000 * 100 cm
= 1280000000 * PI cm

where r = radius of the earth, PI = 3.141592654

Hence, the length of the thread is = 1280000000 * PI cm

Now length of the thread is increasd by 12 cm. So the new length is = (1280000000 * PI) + 12 cm

This thread will make one concentric circle with the earth which is slightly away from the earth. The circumfernce of that circle is nothing but (1280000000 * PI) + 12 cm

Assume that radius of the outer circle is R cm
Therefore,
2 * PI * R = (1280000000 * PI) + 12 cm

Solving above equation, R = 640000001.908 cm
Radius of the earth is r = 640000000 cm

Hence, the thread will be separatedfrom the earth by
= R - r cm
= 640000001.908 - 640000000
= 1.908 cm
Scientist decided to do a study on the population growth of rabbits. Inside a controlled environment, 1000 rabbits were placed.

Six months later, there were 1000Z rabbits. At the beginning of the 3rd year, there were roughly 2828Z rabbits, which was 4 times what the scientists placed in there at the beginning of the 1st year.

If Z is a positive variable, how many rabbits would be there at the beginning of the 11th year?
SubmiAnswer

At the beginning of the 11th year, there would be 1,024,000 rabbits.

At the beginning, there were 1000 rabbits. Also, there were 4000 rabbits at the beginning of third year which is equal to 2828Z. Thus, Z = 4000/2828 i.e. 1.414 (the square root of 2)

Note that 2828Z can be represented as 2000*Z*Z (Z=1.414), which can be further simplified as 1000*Z*Z*Z*Z

Also, it is given that at the end of 6 months, there were 1000Z rabbits.

It is clear that the population growth is 1.414 times every six months i.e. 2 times every year. After N years, the population would be 1000*(Z^(2N)) i.e. 1000*(2^N)

Thus, at the beginning of the 11th year (i.e. after 10 years), there would be 1000*(2^10) i.e. 1,024,000 rabbits.
tted
A class of 100 students. 24 of them are girls and 32 are not. Which base am I using?

Answer

Let the base be X.

Therefore
(X*X + X*0 + 0) = (2*X +4) + (3*X + 2)
X*X = 5*X + 6
X*X - 5*X -6 = 0
(X-6)(X+1) = 0

Therefore base is 6

A man is stranded on a desert island. All he has to drink is a 20oz bottle of sprite.

To conserve his drink he decides that on the first day he will drink one oz and the refill the bottle back up with water. On the 2nd day he will drink 2oz and refill the bottle. On the 3rd day he will drink 3oz and so on...

By the time all the sprite is gone, how much water has he drunk?
SubmittAnswer

The man drunk 190oz of water.

It is given that the man has 20oz bottle of sprite. Also, he will drink 1oz on the first day and refill the bottle with water, will drink 2oz on the second day and refill the bottle, will drink 3oz on the third day and refill the bottle, and so on till 20th day. Thus at the end of 20 days, he must have drunk (1 + 2 + 3 + 4 + ..... +18 + 19 + 20) = 210oz of liquid.

Out of that 210oz, 20oz is the sprite which he had initially. Hence, he must have drunk 190oz of water.ed
You have four 9's and you may use any of the (+, -, /, *) as many times as you like. I want to see a mathematical expression which uses the four 9's to = 100

How many such expressions can you make?
Submitted
Answer

There are 5 such expressions.

99 + (9/9) = 100

(99/.99) = 100

(9/.9) * (9/.9) = 100

((9*9) + 9)/.9 = 100

(99-9)/.9 = 100
Two planes take off at the same exact moment. They are flying across the Atlantic. One leaves New York and is flying to Paris at 500 miles per hour. The other leaves Paris and is flying to New York at only 450 miles per hour ( because of a strong head wind ).

Which one will be closer to Paris when they meet?
They will both be the same distance from Paris when they meet!!!

12 members were present at a board meeting. Each member shook hands with all of the other members before & after the meeting.

How many hand shakes were there?
Answer

132

Think of it this way: the first person shakes hands with 11 people, the second person also shakes hands with 11 people, but you only count 10, because the hand shake with the first person was already counted. Then add 9 for the third person, 8 for the fourth, & so on.

66 hand shakes took place before & 66 after the meeting, for a total of 132.
Arrange five planets such that 4 of them add up to 5th planet numerically. Each of the letters of the planet should represent a unique number from the range 0 - 9. You have to use all ten digits.

There is an amazing mathematical relationship exists among the names of the planet.
Answer

The tought process is initially to find planets such that the total number of alphabets in them is 10.

The only possible combination of planets is Saturn, Uranus, Venus, Mars and Neptune because for other combinations there will be more than 10 alphabets. Among these five, Neptune is the lenghtiest, so it must be the sum of the other four.

S A T U R N

U R A N U S

V E N U S

+ M A R S

--------------

N E P T U N E


Now the only possible value for N is 1. By finding the value for S, we can reach the result:

3 5 8 6 9 1

6 9 5 1 6 3

2 0 1 6 3

+ 4 5 9 3

--------------

1 0 7 8 6 1 0

You have 14 apples. Your Friend Marge takes away 3 and gives you 2. You drop 7 but pick up 4. Bret takes 4 and gives 5. You take one from Marge and give it to Bret in exchange for 3 more. You give those 3 to Marge and she gives you an apple and an orange. Frank comes and takes the apple Marge gave you and gives you a pear. You give the pear to Bret in exchange for an apple. Frank then takes an apple from Marge, gives it to Bret for an orange, gives you the orange for an apple.

How many pears do you have?
Submitted
Answer

None

Frank gave you a pear in exchange of the apple which Marge gave you. And you gave that pear to Bret in exchange for an apple. All the others exchanges involved apples and/or organges.
Four couples are going to the movie. Each row holds eight seats. Betty and Jim don't want to sit next to Alice and Tom. Alice and Tom don't want to sit next to Gertrude and Bill. On the otherhand, Sally and Bob don't want to sit next to Betty and Jim.

How can the couples arrange themselves in a row so that they all sit where they would like?
Submitted by : Tara Smith

Answer

From the given data, it can be inferred that:
(Sally & Bob) NOT (Betty & Jim) NOT (Alice & Tom) NOT (Gertrude & Bill)

(A) NOT (B) means A and B can not seat next to each other.

Now, it is obvious that (Betty & Jim) and (Alice & Tom) will occupy the corner seats as both of them can have only one neighbour. Therefore,
(Gertrude & Bill) will seat next to (Betty & Jim)
(Sally & Bob) will seat next to (Gertrude & Bill)
(Alice & Tom) will seat next to (Sally & Bob)

Thus, there are two possible arrangements - a mirror images of each other.

1. (Betty & Jim) - (Gertrude & Bill) - (Sally & Bob) - (Alice & Tom)
2. (Alice & Tom) - (Sally & Bob) - (Gertrude & Bill) - (Betty & Jim)
Substitute digits for the letters to make the following addition problem true.
W H O S E

T E E T H

A R E

+ A S

-------------------

S W O R D S
Note that the leftmost letter can't be zero in any word. Also, there must be a one-to-one mapping between digits and letters. e.g. if you substitute 3 for the letter H, no other letter can be 3 and all other H in the puzzle must be 3.
Answer

It is obvious that S=1 and T=9.

Also, (H + E) should be greater than 10 and hence, (E + H + E) must 20. Thus, there are 3 possible values for (E, H) pair: (6, 8) or (7, 6) or (8, 4). Use trial-n-error and everything will fit-in.

W H O S E 2 8 5 1 6

T E E T H 9 6 6 9 8

A R E 4 7 6

+ A S + 4 1

------------------- -------------------

S W O R D S 1 2 5 7 3 1

When Socrates was imprisoned for being a disturbing influence, he was held in high esteem by his guards. All four of them hoped that something would occur that would facilitate his escape. One evening, the guard who was on duty intentionally left the cell door open so that Socrates could leave for distant parts.

Socrates did not attempt to escape, as it was his philosophy that if you accept society's rules, you must also accept it's punishments. However, the open door was considered by the authorities to be a serious matter. It was not clear which guard was on that evening. The four guards make the following statements in their defense:

Aaron:
A) I did not leave the door open.
B) Clement was the one who did it.

Bob:
A) I was not the one who was on duty that evening.
B) Aaron was on duty.

Clement:
A) Bob was the one on duty that evening.
B) I hoped Socrates would escape.

David:
A) I did not leave the door open.
B) I was not surprised that Socrates did not attempt to escape.

Considering that, in total, three statements are true, and five statements are false, which guard is guiltyAnswer

David is the guilty.

Note that "All four of them hoped that something would occur that would facilitate his escape". It makes Clement's statement B True and David's statement B False.

Now consider each of them as a guilty, one at a time.
Aaron Bob Clement David True
Stmts
A B A B A B A B
If Aaron is guilty False False True True False True True False 4
If Bob is guilty True False False False True True True False 4
If Clement is guilty True True True False False True True False 5
If David is guilty True False True False False True False False 3

Since in total, three statements are true and five statements are false. It is clear from the above table that David is?
Brain Teaser No : 00474

Given any whole number take the sum of the digits, and the product of the digits, and multiply these together to get a new whole number.

For example, starting with 6712, the sum of the digits is (6+7+1+2) = 16, and the product of the digits is (6*7*1*2) = 84. The answer in this case is then 84 x 16 = 1344.

If we do this again starting from 1344, we get (1+3+4+4) * (1*3*4*4) = 576

And yet again (5+7+6) * (5*7*6) = 3780

At this stage we know what the next answer will be (without working it out) because, as one digit is 0, the product of the digits will be 0, and hence the answer will also be 0.

Can you find any numbers to which when we apply the above mentioned rule repeatedly, we never end up at 0?
Brain Teaser No : 00474

Given any whole number take the sum of the digits, and the product of the digits, and multiply these together to get a new whole number.

For example, starting with 6712, the sum of the digits is (6+7+1+2) = 16, and the product of the digits is (6*7*1*2) = 84. The answer in this case is then 84 x 16 = 1344.

If we do this again starting from 1344, we get (1+3+4+4) * (1*3*4*4) = 576

And yet again (5+7+6) * (5*7*6) = 3780

At this stage we know what the next answer will be (without working it out) because, as one digit is 0, the product of the digits will be 0, and hence the answer will also be 0.

Can you find any numbers to which when we apply the above mentioned rule repeatedly, we never end up at 0?

There were N stations on a railroad. After adding X stations 46 additional tickets have to be printed.

Find N and X.
Answer

Let before adding X stations, total number of tickets
t = N(N-1)

After adding X stations total number of tickets are
t + 46 = (N+X)(N+X-1)

Subtracting 1st from 2nd
46 = (N+X)(N+X-1) - N(N-1)
46 = N2 + NX - N + NX + X2 - X - N2 + N
46 = 2NX + X2 - X
46 = (2N - 1)X + X2
X2 + (2N - 1)X - 46 = 0

Now there are only two possible factors of 46. They are (46,1) and (23,2)

Case I: (46,1)
2N - 1 = 45
2N = 46
N = 23
And X = 1

Case II: (23,2)
2N - 1 = 21
2N = 22
N = 11
And X = 2

Hence, there are 2 possible answers.

An emergency vehicle travels 10 miles at a speed of 50 miles per hour.

How fast must the vehicle travel on the return trip if the round-trip travel time is to be 20 minutes?
Answer

75 miles per hour

While going to the destination, the vehicle travels 10 mils at the speed of 50 miles per hour. So the time taken to travel 10 miles is
= (60 * 10) / 50
= 12 minutes

Now it's given that round-trip travel time is 20 minutes. So the vehicle should complete its return trip of 10 miles in 8 minutes. So the speed of the vehicle must
= (60 * 10) / 8
= 75 miles per hour
All of the students at a college are majoring in psychology, business, or both. 73% of the students are psychology majors, & 62% are business majors.

If there are 200 students, how many of them are majoring in both psychology & business?
Answer

70 students are majoring in both, psychology & business

If 73% of the students are psychology majors, we know that 27% are not psychology majors. By the same reasoning, 38% are not business majors, because 62% of the students do major in business. So: 27 + 38 = 65

65% of the students are not majoring in both psychology & business, so 35% are double majors, a total of 70 students.
Two trains starting at same time, one from Bangalore to Mysore and other in opposite direction arrive at their destination 1hr and 4hrs respectively after passing each other.

Answer


The speed of Bangalore-Mysore train is TWICE the speed of Mysore-Bangalore train.

Let the distance between Bangalore and Mysore is D kms.
Also, let speed of the train from Bangalore to Mysore is X km/hr and speed of the tain from Mysore to Bangalore is Y km/hr.

Now, assume that both the trains met each other at T kms from the Bangalore (point P in figure)
Time taken by Bangalore-Mysore train to reach P = Time taken by Mysore-Bangalore train to reach P
( T / X ) = ( D - T ) / Y -----equ(I)

Also, Bangalore-Mysore train and Mysore-Bangalore train arrive destination 1 hr and 4 hrs respectively after passing each other. It means that Bangalore-Mysore train travels (D - T) kms in 1 hr at X km/hr and Mysore-Bangalore train travels T kms in 4 hrs at Y km/hr. Hence,
( D - T ) = X and
T = 4 * Y

Substituting these values in equation I, we get
( 4 * Y ) / X = X / Y
4 * Y * Y = X * X
2 * Y = X

Hence, the speed of Bangalore-Mysore train is TWICE the speed of Mysore-Bangalore train.How much faster is one train from other?

Answer

49 times

Let's assume that everyone clinked their mug with friend to his left only. It means that there are total 49 clinks. Now the right clink of each person is left clink of the person on right which is already happened. Hence, there are only 49 clinks.

Mrs. Watsherface had a garage sale. A custmer named Gina bought an old lamp and a rug. She paid a total of $5.25 for everything. The rug cost 25 cents more than the lamp.

How much did each cost?
Submitted by : Kimi
Answer

The lamp cost $ 2.50 and the rug cost $ 2.75

A simple one.

Assume that the lamp cost $ L.
Hence the rug must have cost $ (L + 0.25)
Also, total cost is $ 5.25, Hence the equation :
L + L + 0.25 = 5.25
2 * L = 5
L = 2.50

Hence, the lamp cost $ 2.50 and the rug cost $ 2.75

Brain Teaser No : 00518

Write 1111......(243 times) i.e. a 243 digit number with all 1s.

Prove that it is divisible by 243.
SubmittAnswer

Prove it using the mathematical induction.

First here are a couple of things to note:

[1] A number whose digits add up to a multiple of three is divisable by 3.
e.g. 369: 3+6+9=18: 1+8=9 which is a multiple of 3 hence 369 is divisable by 3.

[2] Whenever a number (X) is multiplied with another number (Y) then the product (X*Y) will have all the factors of X as well as all the factors of Y in its set of factors.
e.g. if X has factors of (1,P,Q,X) and Y has factors of (1,Q,R,Y) then X*Y has factors of (1,P,Q,Q,R,X,Y).


Let
N = any series of digits (e.g. N=369)
D = the number of digits in N (e.g. if N=369 then D=3)
P = is a number constructed in the following way : a 1, followed by (D-1) 0s, followed by another 1, followed by (D-1) 0s, followed by another 1. (e.g. if N=369 then D=3 and P would be 1001001) Note that P will always be divisible by 3.

Also, if we multiply N with P we are essentially repeating N for (D-1) times.
e.g. if N=369 then D=3, P=1001001 and N*P=369369369

Let's start with N=111. It is clear that N is divisible by 3. (From [1])
Also, D=3 and P=1001001
N*P=111111111 (9 times)
The resulting number 111111111 must be divisible by 9 as N and P both are divisible by 3.


Now, let's start with N=111111111. It is clear that N is divisible by 9.
Also, D=9 and P=1000000001000000001
N*P=111111111... (27 times)
The resulting number 1111111... (27 times) must be divisible by 27 as N is divisible by 9 and P is divisible by 3.

Repeat the same procedure for N=1111111... (27 times) The resulting number 1111111... (81 times) must be divisible by 81 as N is divisible by 27 and P is divisible by 3.

Similarly, for N=1111111... (81 times) The resulting number 1111111... (243 times) must be divisible by 243 as N is divisible by 81 and P is divisible by 3.

Thus, 1111111... (243 times) is divisible by 243.

Thanks to Ryan Hutcherson for solution !!!
edKaran bought a little box of midget matches, each one inch in length. He found that he could arrange them all in the form of a triangle whose area was just as many square inches as there were matches.

He then used up six of the matches, and found that with the remainder he could again construct another triangle whose area was just as many square inches as there were matches.

And using another six matches he could again do precisely the same.

How many matches were there in the box originally?

Note that the match-box can hold maximum of 50 matches.
Answer

Initially, there were 42 or 36 matches in the match-box.

There are 42 matches in the box with which he could form a triangle 20, 15, 7, with an area of 42 square inches. After 6 matches had been used, the remaining 36 matches would form a triangle 17, 10, 9, with an area of 36 square inches. After using another 6 matches, the remaining 30 matches would form a triangle 13, 12, 5, with an area of 30 square inches. After using another 6, the 24 remaining would form a triangle 10, 8, 6, with an area of 24 square inches.

Thus, there are two possible answers. There were either 42 or 36 matches in the match-box.


Also it is interesting to know that there are just 5 such triangles for which the perimeter and the area is the same (assuming all sides are integers) and they are :
1. 24 (10, 8, 6)
2. 30 (13, 12, 5)
3. 36 (17, 10, 9)
4. 42 (20, 15, 7)
5. 60 (29, 25, 6)

Find the values of each of the alphabets.

N O O N

S O O N

+ M O O N

----------

J U N E
Answer

Using trial and error. There are 2 solutions to it and may be more.

2 4 4 2

1 4 4 2

+ 5 4 4 2

----------

9 3 2 6


4 1 1 4

5 1 1 4

+ 0 1 1 4

----------

9 3 4 2

We have to fill number from 1 to 12 at the intersection point of two or more lines. We have to construct a star using two triangle. The sum of all number lying in straight lines should be same. This can be easilty understood by the fig. and hence solved.
Submitted by : Vaibhav Gupta



We have one answer where sum of all the numbers lying in straight lines is 26.

If you have others, do submit them.
Brain Teaser No : 00355

Montu, Bantu, Chantu and Pintu have pets.

Montu says, "If Pintu and I each have a dog, then exactly one of Bantu and Chantu has a dog."

Bantu says, "If Chantu and I each have a cat, then exactly one of Montu and Pintu has a dog."

Chantu says, "If Montu and I each have a dog, then exactly one of Bantu and Pintu has a cat."

Pintu says, "If Bantu and I each have a cat, then exactly one of Bantu and I has a dog."

Only one of the four is telling the truth. Who is telling the truth?

Answer

Bantu is telling the truth.

For a IF-THEN statement to be false, IF part has to be true and THEN part has to be false.

Since only one statement is true and remaining three are false, IF part of three statements are true & THEN part of one statement is true. Let's put the given information in table. The pet-name in the normal text represents the IF part and the pet-name in round brackets represents the THEN part.
Montu Bantu Chantu Pintu
Montu says Dog (Dog) (Dog) Dog
Bantu says (Dog) Cat Cat (Dog)
Chantu says Dog (Cat) Dog (Cat)
Pintu says Cat
(Dog) Cat
(Dog)

It is clear that the IF part of the statements made by Montu, Chantu and Pintu are true as they do not contradict each other. And the IF part of the statement made by Bantu is false.

Thus, Bantu is telling the truth.

Montu have a Dog and may or may not have a Cat.
Bantu have a Cat.
Chantu have a Dog.
Pintu have a Dog and a Cat.
Brain Teaser No : 00520

Somebody marked the six faces of a die with the numbers 1, 2 and 3 - each number twice. The die was put on a table. Four people - Abu, Babu, Calu and Dabu - sat around the table so that each one was able to see only three sides of the die at a glance.
• Abu sees the number 1 and two even numbers.
• Babu and Calu can see three different numbers each.
• Dabu sees number 2 twice and he can't remember the third number.
What number is face down on the table?
Answer

Number 3 is face down on the table.

If Abu can see two even numbers i.e. number 2 twice, and if Dabu can see number 2 twice, then number 2 must be facing up.

Now everything else is simple. (see the following diagram)

Dabu Abu



1



3 2 2



1



Calu Babu
Thus, the number hidden from the view is number 3 and hence the answer.
Two identical pack of cards A and B are shuffled throughly. One card is picked from A and shuffled with B. The top card from pack A is turned up. If this is the Queen of Hearts, what are the chances that the top card in B will be the King of Hearts?
Answer

52 / 2703

There are two cases to be considered.

CASE 1 : King of Hearts is drawn from Pack A and shuffled with Pack B

Probability of drawing King of Hearts from Pack A = 1/51 (as Queen of Hearts is not to be drawn)
Probability of having King of Hearts on the top of the Pack B = 2/53

So total probability of case 1 = (1/51) * (2/53) = 2 / (51 * 53)

CASE 2 : King of Hearts is not drawn from Pack A

Probability of not drawing King of Hearts from Pack A = 50/51 (as Queen of Hearts is not to be drawn)
Probability of having King of Hearts on the top of the Pack B = 1/53

So total probability of case 2 = (50/51) * (1/53) = 50 / (51 * 53)

Now adding both the probability, the required probability is
= 2 / (51 * 53) + 50 / (51 * 53)
= 52 / (51 * 53)
= 52 / 2703
= 0.0192378
How many possible combinations are there in a 3x3x3 rubics cube?

In other words, if you wanted to solve the rubics cube by trying different combinations, how many might it take you (worst case senerio)?

How many for a 4x4x4 cube?
Submitted
Answer

There are 4.3252 * 10^19 possible combinations for 3x3x3 Rubics and 7.4012 * 10^45 possible combinations for 4x4x4 Rubics.


Let's consider 3x3x3 Rubics first.

There are 8 corner cubes, which can be arranged in 8! ways.
Each of these 8 cubes can be turned in 3 different directions, so there are 3^8 orientations altogether. But if you get all but one of the corner cube into chosen positions and orientations, only one of 3 orientations of the final corner cube is possible. Thus, total ways corner cubes can be placed = (8!) * (3^8)/8 = (8!) * (3^7)

Similarly, 12 edge cubes can be arranged in 12! ways.
Each of these 12 cubes can be turned in 2 different directions, so there are 2^12 orientations altogether. But if you get all but one of the edge cube into chosen positions and orientations, only one of 2 orientations of the final edge cube is possible. Thus, total ways edge cubes can be placed = (12!) * (2^12)/2 = (12!) * (2^11)

Here, we have essentially pulled the cubes apart and stuck cubes back in place wherever we please. In reality, we can only move cubes around by turning the faces of the cubes. It turns out that you can't turn the faces in such a way as to switch the positions of two cubes while returning all the others to their original positions. Thus if you get all but two cubes in place, there is only one attainable choice for them (not 2!). Hence, we must divide by 2.

Total different possible combinations are
= [(8!) * (3^7)] * [(12!) * (2^11)] / 2
= (8!) * (3^7) * (12!) * (2^10)
= 4.3252 * 10^19


Similarly, for 4x4x4 Rubics total different possible combinations are
= [(8!) * (3^7)] * [(24!)] * [(24!) / (4!^6)] / 24
= 7.4011968 * 10^45

Note that there are 24 edge cubes, which you can not turn in 2 orientations (hence no 2^24 / 2). Also, there are 4 center cubes per face i.e. (24!) / (4!^6). You can switch 2 cubes without affecting the rest of the combination as 4*4*4 has even dimensions (hence no division by 2). But pattern on one side is rotated in 4 directions over 6 faces, hence divide by 24.

Brain Teaser No : 00528

Substitute digits for the letters to make the following relation true.
N E V E R

L E A V E

+ M E

-----------------

A L O N E
Note that the leftmost letter can't be zero in any word. Also, there must be a one-to-one mapping between digits and letters. e.g. if you substitute 3 for the letter M, no other letter can be 3 and all other M in the puzzle must be 3.
Answer

A tough one!!!

Since R + E + E = 10 + E, it is clear that R + E = 10 and neither R nor E is equal to 0 or 5. This is the only entry point to

solve it. Now use trial-n-error method.

N E V E R 2 1 4 1 9

L E A V E 3 1 5 4 1

+ M E + 6 1

----------------- -----------------

A L O N E 5 3 0 2 1
There are 20 people in your applicant pool, including 5 pairs of identical twins.

If you hire 5 people randomly, what are the chances you will hire at least 1 pair of identical twins? (Needless to say, this could cause trouble ;))
SubmAnswer

The probability to hire 5 people with at least 1 pair of identical twins is 25.28%

5 people from the 20 people can be hired in 20C5 = 15504 ways.

Now, divide 20 people into two groups of 10 people each :
G1 - with all twins
G2 - with all people other than twins

Let's find out all possible ways to hire 5 people without a single pair of indentical twins.
People from G1 People from G2 No of ways to hire G1 without a single pair of indentical twins No of ways to hire G2 Total ways
0 5 10C0 10C5 252
1 4 10C1 10C4 2100
2 3 10C2 * 8/9 10C3 4800
3 2 10C3 * 8/9 * 6/8 10C2 3600
4 1 10C4 * 8/9 * 6/8 * 4/7 10C1 800
5 0 10C5 * 8/9 * 6/8 * 4/7 * 2/6 10C0 32
Total 11584

Thus, total possible ways to hire 5 people without a single pair of indentical twins = 11584 ways

So, total possible ways to hire 5 people with at least a single pair of indentical twins = 15504 - 11584 = 3920 ways

Hence, the probability to hire 5 people with at least a single pair of indentical twins
= 3920/15504
= 245/969
= 0.2528
= 25.28%itted
Veeru says to Jay, "Can you figure out how many Eggs I have in my bucket?" He gives 3 clues to Jay: If the number of Eggs I have
1. is a multiple of 5, it is a number between 1 and 19
2. is not a multiple of 8, it is a number between 20 and 29
3. is not a multiple of 10, it is a number between 30 and 39
How many Eggs does Veeru have in his bucket?
Answer

32 eggs

Let's apply all 3 condition separately and put all possible numbers together.

First condition says that if multiple of 5, then the number is between 1 and 19. Hence, the possible numbers are (5, 10, 15, 21, 22, 23, 24, 25, 26, 27, 28, 29, 30, 31, 32, 33, 34, 35, 36, 37, 38, 39)

Second condition says that if not a multiple of 8, then the number is between 20 and 29. Hence, the possible numbers are (8, 16, 20, 21, 22, 23, 25, 26, 27, 28, 29, 32)

Third condition says that if not a multiple of 10, then the number is between 30 and 39. Hence, the possible numbers are (10, 20, 31, 32, 33, 34, 35, 36, 37, 38, 39)

Only number 32 is there in all 3 result sets. That means that only number 32 satisfies all three conditions. Hence, Veeru have 32 eggs in his bucket.

Submit
Answer

Users
Answer (14)

BrainV


Mr. Black, Mr. White and Mr. Grey were chatting in the Yahoo conference. They were wearing a black suit, a white suit and a grey suit, not necessarily in the same order.

Mr. Grey sent message, "We all are wearing suit that are of the same color as our names but none of us is wearing a suit that is the same color as his name."

On that a person wearing the white suit replied, "What difference does that make?"

Can you tell what color suit each of the three persons had on?
Answer

Mr. Grey is wearing Black suit.
Mr. White is wearing Grey suit.
Mr. Black is wearing White suit.

Mr. Grey must not be wearing grey suit as that is the same colour as his name. Also, he was not wearing white suit as the person wearing white suit responded to his comment. So Mr Grey must be wearing a black suit.

Similarly, Mr. White must be wearing either black suit or grey suit. But Mr. Grey is wearing a black suit. Hence, Mr. White must be wearing a grey suit.

And, Mr. Black must be wearing white suit.
Substitute numbers for the letters so that the following mathematical expressions are correct.
ABC DEF GHI

--- = IE --- = IE --- = IE

3 6 9
Note that the same number must be used for the same letter whenever it appears.
Answer

A=2, B=1, C=9, D=4, E=3, F=8, G=6, H=5, I=7

Let's start with GHI = 9 * IE. Note that I appears on both the side. Also, after multiplying IE by 9 the answer should have I at the unit's place. The possible values of IE are 19, 28, 37, 46, 55, 64, 73, 82 and 91; out of which only 64, 73 and 82 satisfies the condition. (as all alphabet should represent different digits)

Now, consider DEF = 6 * IE. Out of three short-listed values, only 73 satisfies the equation. Also, ABC = 3 * IE is satisfied by 73.

Hence, A=2, B=1, C=9, D=4, E=3, F=8, G=6, H=5, I=7
219 438 657

--- = 73 --- = 73 --- = 73

3 6 9

Brain Teaser No : 00374

A, B, C and D are related to each other.
• One of the four is the opposite sex from each of the other three.
• D is A's brother or only daughter.
• A or B is C's only son.
• B or C is D's sister.
Answer

A, B & D are males; C is female. B is C's only son. A & D are C's brothers.
A(male) --- C(female) --- D(male)

|

|

B(male)
Work out which relation can hold and discard the contradictory options.

From (2) and (4), D can not be a only daughter and have a sister (B or C). Hence, D is A's brother i.e. D is a Male.

From (4), let's say that B is D's sister i.e. B is Female.
From (3), A is C's only son i.e. A is Male.
But D is A's brother which means that A is not C's only son. Hence, our assumption was wrong.

Thus, C is D's sister i.e. C is Female. And B must be C's only son.

Now it is clear that D & B are Males and C is Female. A must be a Male as only one of them is of opposite sex from each of the other three. And he is C & D's brother.How are they related to each other?


Dr. DoLittle always goes walking to the clinic and takes the same time while going and while coming back. One day he noticed something.

When he left the home, the hour hand and the minute hand were exactly opposite to each other and when he reached the clinic, they were together.

Similarly, when he left the clinic, the hour hand and the minute hand were together and when he reached the home, they were exactly opposite to each other.

How much time does Dr. DoLittle take to reach home from the clinic? Give the minimal possible answer.
Answer

32 minutes 43.6 seconds

In twelve hours, the minute hand and the hour hand are together for 11 times. It means that after every 12/11 hours, both the hands are together.

Similarly in twelve hours, the minute hand and the hour hand are exactly opposite to each other for 11 times. It means that after every 12/11 hours, both the hands are opposite.

Now, let's take an example. We know that at 12 both the hands are together and at 6 both the hands are exactly opposite to each other.

After 6, both the hands are in opposition at [6+(12/11)] hours, [6+2*(12/11)] hours, [6+3*(12/11)] hours and so on. The sixth such time is [6+6*(12/11)] hours which is the first time after 12. Thus after 12, both the hands are opposite to each other at 12:32:43.6

Hence, Dr. DoLittle takes 32 minutes and 43.6 seconds to reach home from the clinic.


SlowRun Express runs between Bangalore and Mumbai, For the up as well as the down journey, the train leaves the starting station at 10:00 PM everyday and reaches the destination at 11:30 PM after three days.

Mr. Haani once travelled by SlowRun Express from Mumbai to Bangalore. How many SlowRun Express did he cross during his journey?
Answer

Mr. Haani crossed 7 SlowRun Expresses during his journey.

Let's say that Mr. Haani travelled by SlowRun Express on Wednesday 10:00PM from Mumbai. The first train he would have crossed is the one scheduled to arrive at Mumbai at 11:30 PM the same day i.e. the one that left Bangalore at 10:00 PM on last Sunday.

Also, he would have crossed the last train just before reaching Bangalore on Saturday.

Thus, Mr. Haani must have crossed 7 SlowRun Expresses during his journey.
Six cabins numbered 1-6 consecutively, are arranged in a row and are separated by thin dividers. These cabins must be assigned to six staff members based on following facts.
1. Miss Shalaka's work requires her to speak on the phone frequently throughout the day.
2. Miss Shudha prefers cabin number 5 as 5 is her lucky number.
3. Mr. Shaan and Mr. Sharma often talk to each other during their work and prefers to have adjacent cabins.
4. Mr. Sinha, Mr. Shaan and Mr. Solanki all smoke. Miss Shudha is allergic to smoke and must have non-smokers adjacent to her.
5. Mr. Solanki needs silence during work.
Can you tell the cabin numbers of each of them?
Answer

The cabins from left to right (1-6) are of Mr. Solanki, Mr. Sinha, Mr. Shaan, Mr. Sharma, Miss Shudha and Miss Shalaka.

From (2), cabin number 5 is assigned to Miss Shudha.

As Miss Shudha is allergic to smoke and Mr. Sinha, Mr. Shaan & Mr. Solanki all smoke, they must be in cabin numbers 1, 2 and 3 not necessarily in the same order. Also, Miss Shalaka and Mr. Sharma must be in cabin 4 and 6.

From (3), Mr. Shaan must be in cabin 3 and Mr. Sharma must be in cabin 4. Thus, Miss Shalaka is in cabin 6.

As Mr. Solanki needs silence during work and Mr. Shaan is in cabin 3 who often talks to Mr. Sharma during work, Mr. Solanki must be in cabin 1. Hence, Mr. Sinha is in cabin 2.

Thus, the cabins numbers are
1# Mr. Solanki,
2# Mr. Sinha,
3# Mr. Shaan,
4# Mr. Sharma,
5# Miss Shudha,
6# Miss Shalaka
SkyFi city is served by 6 subway lines - A, E, I, O, U and Z.
• When it snows, morning service on line E is delayed.
• When it rains or snows, service on the lines A, U and Z is delayed both morning and afternoon.
• When the temperature drops below 20 C, afternoon service is cancelled on either line A or line O, but not both.
• When the temperature rises above 40 C, afternoon service is cancelled on either line I or line Z, but not both.
• When service on line A is delayed or cancelled, service on line I is also delayed.
• When service on line Z is delayed or cancelled, service on line E is also delayed.
On February 10, it snows all day with the temperature at 18C. On how many lines service will be delayed or cancelled, including both morning and afternoon?
SkyFi city is served by 6 subway lines - A, E, I, O, U and Z.
• When it snows, morning service on line E is delayed.
• When it rains or snows, service on the lines A, U and Z is delayed both morning and afternoon.
• When the temperature drops below 20 C, afternoon service is cancelled on either line A or line O, but not both.
• When the temperature rises above 40 C, afternoon service is cancelled on either line I or line Z, but not both.
• When service on line A is delayed or cancelled, service on line I is also delayed.
• When service on line Z is delayed or cancelled, service on line E is also delayed.
On February 10, it snows all day with the temperature at 18C. On how many lines service will be delayed or cancelled, including both morning and afternoon?
In a certain game, if 2 wixsomes are worth 3 changs, and 4 changs are worth 1 plut, then 6 plutes are worth how many wixsomes?

Answer

It is given that
2 wixsomes = 3 changs
8 wixsomes = 12 changs ----- (I)

Also, given that
4 changs = 1 plut
12 changs = 3 plutes
8 wixsomes = 3 plutes ----- From (I)

Therefore,
6 plutes = 16 wixsomes

In a certain year, the number of girls who graduated from City High School was twice the number of boys. If 3/4 of the girls and 5/6 of the boys went to college immediately after graduation, what fraction of the graduates that year went to college immediately after graduation?

Answer

Assume that number of boys graduated from City High School = B
Therefore, number of girls graduated from City High School = 2*B

It is given that 3/4 of the girls and 5/6 of the boys went to college immediately after graduation.
Hence, total students went to college
= (3/4)(2*B) + (5/6)(B)
= B * (3/2 + 5/6)
= (7/3)B

Fraction of the graduates that year went to college immediately after graduation
= [(7/3)B] / [3*B]
= 7/9

Therefore, the answer is 7/9

A mule and a donkey were carrying full sacks on their backs.

The mule started complaining that his load was too heavy. The donkey said to him "Why are you complaining? If you gave me one of your sacks I'd have double what you have and if I give you one of my sacks we'd have an even amount."

How many sacks were each of them carrying? Give the minimal possible answer.
SubmittAnswer

The mule was carrying 5 sacks and the donkey was carrying 7 sacks.

Let's assume that the mule was carrying M sacks and the donkey was carrying D sacks.

As the donkey told the mule, "If you gave me one of your sacks I'd have double what you have."
D + 1 = 2 * (M-1)
D + 1 = 2M - 2
D = 2M - 3

The donkey also said, "If I give you one of my sacks we'd have an even amount."
D - 1 = M + 1
D = M + 2

Comparing both the equations,
2M - 3 = M + 2
M = 5

Substituting M=5 in any of above equation, we get D=7

Hence, the mule was carrying 5 sacks and the donkey was carrying 7 sacks.
edTwo people enter a race in whick you run to a point and back. Person A runs 20 mph to and from the point. Person B runs to the point going 10 mph and 30 mph going back.

Who came in first?
Submitted
Answer

Person A came in first.

Let's assume that the distance between start and the point is D miles.

Total time taken by Person A to finish
= (D/20) + (D/20)
= D/10
= 0.1D

Total time taken by Person B to finish
= (D/10) + (D/30)
= 2D/15
= 0.1333D

Thus, Person A is the Winner.



Alternatively (if you don't like mathematics ;)), analyse the situation as follow:

Note that initially speed of Person A (20 mph) was twice the speed of Person B (10 mph). Hence, when Person A (20 mph forward) reached the point, Person B (10 mph forward) was halfway. When Person A (20 mph back) finished, Person B (still 10 mph forward) reached the point.

Thus, Person A wins the race and by that time Person B covers only half the distance, no matter how far the point is!!!
Mark ate half of a pizza on Monday. He ate half of what was left on Tuesday and so on. He followed this pattern for one week.

How much of the pizza would he have eaten during the week?
Submitted
Answer

Mark would have ate 127/128 (99.22%) of the pizza during the week.

Mark ate half the pizza on Monday. On Tuesday, he would have ate half of the remaining pizza i.e. 1/4 of the original pizza. Similarly, he would have ate 1/8 of the original pizza on Wednesday and so on for the seven days.

Total pizza Mark ate during the week is
= 1/2 + 1/4 + 1/8 + 1/16 + 1/32 + 1/64 + 1/128
= 127/128
= 99.22% of the original pizza
In the General meeting of "Friends Club", Sameer said, "The repairs to the Club will come to a total of Rs 3120 and I propose that this amount should be met by the members, each paying an equal amount."

The proposal was immediately agreed. However, four members of the Club chose to resign, leaving the remaining members to pay an extra Rs 26 each.

How many members did the Club originally have?
Answer

The Club originally had 24 members.

Assume that there were initially N members.

As 4 members resigned and remaining members paid Rs 26 each, it means that total amount of 4 members is equal to Rs 26 each from remaining (N-4) members. Thus,

4 * (3120 / N) = 26 * (N - 4)
12480 = 26N2 - 104N
26N2 - 104N - 12480 = 0

Solving the quadratic equation we get N=24.

Hence, the Club originally had 24 members.
Brain Teaser No : 00206

A tank can be filled by pipe A in 30 minutes and by pipe B in 24 minutes. Outlet pipe C can empty the full tank in one hour and twenty minutes.

If the tank is empty initially and if all the three pipes A, B and C are opened simultaneously, in how much time will the tank be full?
Answer

The tank will be full in 16 minutes.

In one minute,
pipe A can fill 1/30 part of the tank.
pipe B can fill 1/24 part of the tank.
pipe C can empty 1/80 part of the tank.

Thus, the net water level in one minute is
= 1/30 + 1/24 - 1/80
= 15/240 part of the tank

Hence, the tank will be full in 240/15 i.e. 16 minutes.

A rich old Arab has three sons. When he died, he willed his 17 camels to the sons, to be divided as follows:

First Son to get 1/2 of the camels Second Son to get 1/3rd of the camels Third Son to get 1/9th of the camels.

The sons are sitting there trying to figure out how this can possibly be done, when a very old wise man goes riding by. They stop him and ask him to help them solve their problem. Without hesitation he divides the camels properly and continues riding on his way.

How did he do it?
Answer

The old man temporarily added his camel to the 17, making a total of 18 camels.

First son got 1/2 of it = 9

Second son got 1/3 of it = 6

Third son got 1/9 of it = 2

For a total of 17. He then takes his camel back and rides away......


There were two men standing on a street. The one says to the other, "I have 3 daughters, the product of their ages is 36. What is the age of the OLDEST daughter?"

The second guy says, "I need more information." So, the first guy says, "The sum of their ages is equal to the address of the house across the street."

The second guy looks at the address and says, "I still need more information." So, the first guy says, "My oldest daughter wears a red dress."
Answer

The answer is 9 years.

First you need to find all the possible sets of three numbers that when multiplied equals 36:

1 1 36
1 2 18
1 3 12
1 4 9
1 6 6
2 2 9
2 3 6
3 3 4

Then you add the numbers together to find the sum
1 1 36 = 38
1 2 18 = 21
1 3 12 = 16
1 4 9 = 14
1 6 6 = 13
2 2 9 = 13
2 3 6 = 11
3 3 4 = 10

Even though we don't know the address the guy knows it. For him to need more information that means that at least two of the sets of numbers has the same sum. Two of them do, 1 6 6 and 2 2 9.

When the first guy said that his OLDEST daugher wears a red dress that meant that there had to be the oldest. So 1 6 6 can't possibly be the answer. So the possible possiblity is 2 2 9 and the OLDEST daughter is 9 years old.

Therefore, the answer is 9.

Submit
Answer

User

There are 3 colored boxes - Red, Green and Blue. Each box contains 2 envelopes. Each envelope contains money - two of them contain Rs. 25000 each, two of them contain Rs. 15000 each and remaining two contain Rs. 10000 each.

There is one statement written on the cover of each box.
* Red Box: Both, a red box and a blue box contain Rs. 10000 each.
* Green Box: Both, a green box and a red box contain Rs. 25000 each.
* Blue Box: Both, a blue box and a green box contain Rs. 15000 each.

Only one of the above 3 statements is true and the corresponding box contains the maximum amount.

Can you tell which box contains the maximum amount and how much?
Answer

Blue box contains the maximum amount Rs. 40000

As it is given that only one of the given 3 statements is true; assume in turn, each statement to be true & the other 2 false and check whether the corresponding box contains the maximum amount.

Let's assume that the statement on the Blue box is true. Thus, the given 3 statements can be interpreted as
* Atmost one, a red box or a blue box contains Rs. 10000.
* Atmost one, a green box or a red box contains Rs. 25000.
* Both, a blue box and a green box contain Rs. 15000 each.

Going through all possible combinations, we can conclude that
Red Box : Rs. 10000 + Rs. 25000 = Rs. 35000
Green Box : Rs. 10000 + Rs. 15000 = Rs. 25000
Blue Box : Rs. 15000 + Rs. 25000 = Rs. 40000

You can test out for other two statements i.e. assuming Red box statement true and then Green box statement true. In both the cases, other statements will contradict the true statement.

Sachin, Dravid and Ganguly played in a Cricket match between India and England.
• None of them scored more than 99 runs.
• If you add the digits of the runs scored by Sachin to his own score, you will get the runs scored by Dravid.
• If you reverse the digits of the runs scored by Dravid, you will get the runs scored by Ganguly.
• The total runs scored by them is 240.
Can you figure out their individual scores?
Answer

Sachin, Dravid and Ganguly scored 75, 87 and 78 respectively.

Sachin's score must be less than 86, otherwise Dravid's score would be more than 99. Also, he must have scored atleast 42 - incase Dravid and Ganguly scored 99 each.

Also, as none of them scored more than 99 and the total runs scored by them is 240; their individual scores must be around 80.

Now, use trial-n-error method to solve the teaser.
Three men, including Gianni and three woman, including Sachi are in line at the BrentWood post office. Each has two different pieces of business to conduct.
1. The first person is a woman.
2. Carlos wants to send an overnight package.
3. Lau is just ahead of Pimentelli who is the same sex as Lau.
4. Gianni is two places ahead of the person who wants to buy stamps.
5. Knutson - who is the opposite sex than Rendler - isn't the person who wanted to complain about a mail carrier.
6. The six people, not necessarily in the same order are - Anthony, Donna, the person who wants to fill out a change-of-address form, the one who wants to buy a money order, the one who wants to send Airmail to Tibet and the second person in the line.
7. The four tasks of the last two people in line, not necessarily in the same order are - sending books fourth class, buying a money order, picking up a package and complaining about a mail carrier.
8. The person who wants to send books fourth class is just behind a person of the same sex.
9. Mary is just behind a person who wants to send an insured package.
10. The person who wants to send Airmail to Tibet is either two places ahead of or two places behind the one who wants to add postage to his or her meter.
11. Anthony isn't two places behind the who wants to pickup a registered letter.
12. Toriseza is two places ahead of the person who wants to pick up a package.
13. Knutson isn't just ahead of the person who wants to send an item parcel post.
Can you figure out where each customer is in the line, his or her full name (one surname is Loti) and the two things he or she wants to accomplish? Provide your answer is POSITION - FIRST NAME - LAST NAME - BUSINESS format.
Submit
Answer

Users
Answer (8)



Answer

A very TOUGH puzzle !!!
POS FIRST NAME LAST NAME BUSINESS
1 Sachi Loti • Fill Out a Change-of-Address Form
• Add Postage to Meter
2 Gianni Lau • Pick Up a Registered Letter
• Send an Item Parcel Post
3 Carlos Pimentelli • Overnight Package
• Send Airmail to Tibet
4 Donna Toriseza • Buy Stamps
• Send an Insured Package
5 Mary Knutson • Buy a Money Order
• Send Books fourth Class
6 Anthony Rendler • Complain About a Mail Carrier
• Pick Up a Package

Brain Teaser No : 00164

Substitute digits for the letters to make the following relation true.
W O R L D

+ T R A D E

-------------

C E N T E R
Note that the leftmost letter can't be zero in any word. Also, there must be a one-to-one mapping between digits and letters. e.g. if you substitute 3 for the letter W, no other letter can be 3 and all other W in the puzzle must be 3.

Answer

A tough one.

It is obvious that C=1. Also, the maximum possible value of E is 7. Now, start putting possible values of D, E and R as they occure frequently and use trial-n-error.

W O R L D 5 3 6 8 4

+ T R A D E + 7 6 0 4 2

------------ ------------

C E N T E R 1 2 9 7 2 6
Brain Teaser No : 00107

If you look at a clock and the time is 3:15.

What is the angle between the hour and the minute hands? ( The answer to this is not zero!)
Answer

7.5 degrees

At 3:15 minute hand will be perfactly horizontal pointing towards 3. Whereas hour hand will be towards 4. Also, hour hand must have covered 1/4 of angle between 3 and 4.

The angle between two adjacent digits is 360/12 = 30 degrees.

Hence 1/4 of it is 7.5 degrees.
An apple vendor has 1000 apples and 10 empty boxes. He asks his son to place all the 1000 apples in all the 10 boxes in such a manner that if he asks for any number of apples from 1 to 1000, his son should be able to pick them in terms of boxes.

How did the son place all the apples among the 10 boxes, given that any number of apples can be put in one box.
Answer

1, 2, 4, 8, 16, 32, 64, 128, 256, 489

Let's start from scratch.
• The apple vandor can ask for only 1 apple, so one box must contain 1 apple.
• He can ask for 2 apples, so one box must contain 2 apples.
He can ask for 3 apples, in that case box one and box two will add up to 3.
• He can ask for 4 apples, so one box i.e. third box must contain 4 apples.
• Now using box number one, two and three containing 1, 2 and 4 apples respectively, his son can give upto 7 apples. Hence, forth box must contain 8 apples.
• Similarly, using first four boxes containing 1, 2, 4 and 8 apples, his son can give upto 15 apples. Hence fifth box must contain 16 apples.
• You must have noticed one thing till now that each box till now contains power of 2 apples. Hence the answer is 1, 2, 4, 8, 16, 32, 64, 128, 256, 489. This is true for any number of apples, here in our case only upto 1000.

Brain Teaser No : 00261

The letters P, Q, R, S, T, U and V, not necessarily in that order represents seven consecutive integers from 22 to 33.
• U is as much less than Q as R is greater than S.
• V is greater than U.
• Q is the middle term.
• P is 3 greater than S.
Can you find the sequence of letters from the lowest value to the highest value?

Answer

The sequence of letters from the lowest value to the highest value is TUSQRPV.

From (3), Q is the middle term.
___ ___ ___ _Q_ ___ ___ ___

From (4), there must be exactly 2 numbers between P and S which gives two possible positions.

[1] ___ _S_ ___ _Q_ _P_ ___ ___

[2] ___ ___ _S_ _Q_ ___ _P_ ___


From (1), the number of letters between U and Q must be same as the number of letters between S and R. Also, the number of letters between them can be 1, 2 or 3.

Using trial and error, it can be found that there must be 2 letters between them. Also, it is possible only in option [2] above.

[2] ___ _U_ _S_ _Q_ _R_ _P_ ___

From (2) V must be the highest and the remaining T must be the lowest number.

_T_ _U_ _S_ _Q_ _R_ _P_ _V_

Thus, the sequence of letters from the lowest value to the highest value is TUSQRPV.
A contractor had employed 100 labourers for a flyover construction task. He did not allow any woman to work without her husband. Also, atleast half the men working came with their wives.

He paid five rupees per day to each man, four ruppes to each woman and one rupee to each child. He gave out 200 rupees every evening.

How many men, women and children were working with the constructor?
Answer

16 men, 12 women and 72 children were working with the constructor.

Let's assume that there were X men, Y women and Z children working with the constructor. Hence,

X + Y + Z = 100
5X + 4Y + Z = 200

Eliminating X and Y in turn from these equations, we get
X = 3Z - 200
Y = 300 - 4Z

As if woman works, her husband also works and atleast half the men working came with their wives; the value of Y lies between X and X/2. Substituting these limiting values in equations, we get

if Y = X,
300 - 4Z = 3Z - 200
7Z = 500
Z = 500/7 i.e. 71.428

if Y = X/2,
300 - 4Z = (3Z - 200)/2
600 - 8Z = 3Z - 200
11Z = 800
Z = 800/11 i.e. 72.727

But Z must be an integer, hence Z=72. Also, X=16 and Y=12

There were 16 men, 12 women and 72 children working with the constructor.

Because cigars cannot be entirely smoked, a Bobo who collects cigar butts can make a cigar to smoke out of every 3 butts that he finds.

Today, he has collected 27 cigar butts. How many cigars will he be able to smoke?
Answer

13 not 12

He makes 9 originals from the 27 butts he found, and after he smokes them he has 9 butts left for another 3 cigars. And then he has 3 butts for another cigar.

So 9+3+1=13
In a small town, there are three temples in a row and a well in front of each temple. A pilgrim came to the town with certain number of flowers.

Before entering the first temple, he washed all the flowers he had with the water of well. To his surprise, flowers doubled. He offered few flowers to the God in the first temple and moved to the second temple. Here also, before entering the temple he washed the remaining flowers with the water of well. And again his flowers doubled. He offered few flowers to the God in second temple and moved to the third temple. Here also, his flowers doubled after washing them with water. He offered few flowers to the God in third temple.

There were no flowers left when pilgrim came out of third temple and he offered same number of flowers to the God in all three temples.

What is the minimum number of flowers the pilgrim had initially? How many flower did he offer to each God?
Answer


The pilgrim had 7 flowers, initially and he offered 8 flowers to each God.

Assume that the pilgrim had X flowers initially and he offered Y flowers to each God.

From the above figure, there are (8X - 7Y) flowers when the pilgrim came out of the third temple. But it is given that there were no flowers left when he came out of third temple. It means that
(8X - 7Y) = 0
8X = 7Y

The minimum values of X and Y are 7 and 8 respectively to satisfy above equation. Hence, the pilgrim had 7 flowers and he offered 8 flowers to each God.

In general, the pilgrim had 7N flowers initially and he offered 8N flowers to each God, where N = 1, 2, 3, 4, .....

Brain Teaser No : 00432

Tanya wants to go on a date and prefers her date to be tall, dark and handsome.
1. Of the preferred traits - tall, dark and handsome - no two of Adam, Bond, Cruz and Dumbo have the same number.
2. Only Adam or Dumbo is tall and fair.
3. Only Bond or Cruz is short and handsome.
4. Adam and Cruz are either both tall or both short.
5. Bond and Dumbo are either both dark or both fair.
Who is Tanya's date?

Answer

Cruz is Tanya's date.

As no two of them have the same number of preferred traits - from (1), exactly one of them has none of the preferred traits and exactly one of them has all the preferred traits.

From (4) and (5), there are only two possibilities:
* Adam & Cruz both are tall and Bond & Dumbo both are fair.
* Adam & Cruz both are short and Bond & Dumbo both are dark.

But from (2), second possibility is impossible. So the first one is the correct possibility i.e. Adam & Cruz both are tall and Bond & Dumbo both are fair.

Then from (3), Bond is short and handsome.

Also, from (1) and (2), Adam is tall and fair. Also, Dumbo is the person without any preferred traits. Cruz is Dark. Adam and Cruz are handsome. Thus, following are the individual preferred traits:

Cruz - Tall, Dark and Handsome
Adam - Tall and Handsome
Bond - Handsome
Dumbo - None :-(

Hence, Cruz is Tanya's date.
Consider a game of Tower of Hanoi (like the one that you can play on BrainVista).

If the tower has 2 discs, the least possible moves with which you can move the entire tower to another peg is 3.

If the tower has 3 discs, the least possible moves with which you can move the entire tower to another peg is 7.

What is the least possible moves with which you can move the entire tower to another peg if the tower has N discs?
Submitted
Answer

There are number of ways to find the answer.

To move the largest disc (at level N) from one tower to the other, it requires 2(N-1) moves. Thus, to move N discs from one tower to the other, the number of moves required is
= 2(N-1) + 2(N-2) + 2(N-3) + ..... + 22 + 21 + 20
= 2N - 1


For N discs, the number of moves is one more than two times the number of moves for N-1 discs. Thus, the recursive function is
F(1) = 1
F(N) = 2*[F(N-1)] + 1
where N is the total number of discs


Also, one can arrive at the answer by finding the number of moves for smaller number of discs and then derive the pattern.
For 1 disc, number of moves = 1
For 2 discs, number of moves = 3
For 3 discs, number of moves = 7
For 4 discs, number of moves = 15
For 5 discs, number of moves = 31

Thus, the pattern is 2N – 1
A boy found that he had a 48 inch strip of paper. He could cut an inch off every second.

How long would it take for him to cut 48 pieces? He can not fold the strip and also, can not stack two or more strips and cut them together.
SubmiAnswer

47 seconds.

To get 48 pieces, the boy have to put only 47 cuts. i.e. he can cut 46 pieces in 46 seconds. After getting 46 pieces, he will have a 2 inches long piece. He can cut it into two with just a one cut in 1 second. Hence, total of 47 seconds.tted by : Kimi

The cricket match between India and Pakistan was over.
• Harbhajan scored more runs than Ganguly.
• Sachin scored more runs than Laxman but less than Dravid
• Badani scored as much runs as Agarkar but less than Dravid and more than Sachin.
• Ganguly scored more runs than either Agarkar or Dravid.
Each batsman scored 10 runs more than his immediate batsman. The lowest score was 10 runs. How much did each one of them score
Answer

A simple one. Use the given facts and put down all the players in order. The order is as follow with Harbhajan, the highest scorer and Laxman, the lowest scorer.
1. Harbhajan
2. Ganguly
3. Dravid
4. Badani, Agarkar
5. Sachin
6. Laxman
Also, as the lowest score was 10 runs. Laxman must have scored 10, Sachin 20, Badani & Agarkar 30 and so on.
1. Harbhajan - 60 runs
2. Ganguly - 50 runs
3. Dravid - 40 runs
4. Badani, Agarkar - 30 runs each
5. Sachin - 20 runs
6. Laxman - 10 runs
There are 10 statements written on a piece of paper:
1. At least one of statements 9 and 10 is true.
2. This either is the first true or the first false statement.
3. There are three consecutive statements, which are false.
4. The difference between the numbers of the last true and the first true statement divides the number, that is to be found.
5. The sum of the numbers of the true statements is the number, that is to be found.
6. This is not the last true statement.
7. The number of each true statement divides the number, that is to be found.
8. The number that is to be found is the percentage of true statements.
9. The number of divisors of the number, that is to be found, (apart from 1 and itself) is greater than the sum of the numbers of the true statements.
10. There are no three consecutive true statements.
Find the minimal possible number?
Submitted
Answer

The numebr is 420.

If statement 6 is false, it creates a paradox. Hence, Statement 6 must be true.

Consider Statement 2:
• If it is true, it must be the first true statement. Otherwise, it creates a paradox.
• If it is false, it must be the second false statement. Otherwise, it creates a paradox.
In both the cases, Statement 1 is false.

As Statement 1 is false, Statement 9 and Statement 10 both are false i.e. there are three consecutive true statements.
1 2 3 4 5 6 7 8 9 10
False - - - - True - - False False

Let\'s assume that Statement 3 is false i.e. there are no three consecutive false statements. It means that Statement 2 and Statement 8 must be true, else there will be three consecutive false statements.
1 2 3 4 5 6 7 8 9 10
False True False - - True - True False False

Also, atleast two of Statements 4, 5 and 7 must be true as there are three consecutive true statements.

According to Statement 8, the number that is to be found is the percentage of true statements. Hence, number is either 50 or 60. Now if Statement 7 is true, then the number of each true statement divides the number, that is to be found. But 7 and 8 do not divide either 50 or 60. Hence, Statement 7 is false which means that Statement 4 and 5 are true. But Statement 5 contradicts the Statement 8. Hence, our assumption that Statement 3 is false is wrong and Statement 3 is true i.e. there are 3 consecutive false statements which means that Statement 8 is false as there is no other possibilities of 3 consecutive false statements.

Also, Statement 7 is true as Statement 6 is not the last true statement.
1 2 3 4 5 6 7 8 9 10
False - True - - True True False False False

According to Statement 7, the number of each true statement divides the number, that is to be found. And according to Statement 5, the sum of the numbers of the true statements is the number, that is to be found. For all possible combinations Statement 5 is false.

There 3 consecutive true statements. Hence, Statement 2 and Statement 4 are true.
1 2 3 4 5 6 7 8 9 10
False True True True False True True False False False

Now, the conditions for the number to be found are:
1. The numebr is divisible by 5 (Statement 4)
2. The numebr is divisible by 2, 3, 4, 6, 7 (Statement 7)
3. The number of divisors of the number, that is to be found, (apart from 1 and itself) is not greater than the sum of the numbers of the true statements. (Statement 9)
The minimum possible number is 420.

The divisors of 420, apart from 1 and itself are 2, 3, 4, 5, 6, 7, 10, 12, 14, 15, 20, 21, 28, 30, 35, 42, 60, 70, 84, 105, 140, 210. There are total of 22 divisors. Also, the sum of the numbers of the true statements is 22 (2+3+4+6+7=22), which satisfies the third condition.
Ankit and Tejas divided a bag of Apples between them.

Tejas said, "It's not fair! You have 3 times as many Apples I have." Ankit said, "OK, I will give you one Apple for each year of your age." Tejas replied, "Still not fair. Now, you have twice as many Apples as I have." "Dear, that's fair enough as I am twice older than you.", said Ankit.

Ankit went to Kitchen to drink water. While Ankit was in Kitchen, Tejas took apples from Ankit's pile equal to Ankit's age.

Who have more apples now?
Answer

At the end, Ankit and Tejas, both have the same number of apples.

Let's assume that initially Tejas got N apples and his age is T years. Hence, initially Ankit got 3N apples and his age is 2T years.
Operation Ankit's Apples Tejas's Apples
Initially 3N N
Ankit gave T apples to Tejas
(equals age of Tejas) 3N - T N + T
Tejas took 2T apples from Ankit's pile
(equals age of Ankit) 3N - 3T N + 3T

It is given that after Ankit gave T apples to Tejas, Ankit had twice as many apples as Tejas had.
3N - T = 2*(N + T)
3N - T = 2N + 2T
N = 3T

From the table, at the end Ankit have (3N - 3T) apples and Tejas have (N + 3T) apples. Substituting N = 3T, we get
Ankit's apples = 3N - 3T = 9T - 3T = 6T
Tejas's apples = N + 3T = 3T + 3T = 6T

Thus, at the end Ankit and Tejas, both have the same number of apples.

On evey Sunday Amar, Akbar and Anthony lunch together at Preetam-Da-Dhaba where they order lassi based on following facts.
1. Unless neither Amar nor Akbar have lassi, Anthony must have it.
2. If Amar does not have lassi, either Akbar or Anthony or both have it.
3. Anthony has lassi only if either Amar or Akbar or both have it.
4. Akbar and Anthony never have lassi together.
Who order(s) lassi?
Answer

Amar and Anthony both have lassi whereas Akbar never does.

Fact (2) can be alternatively stated that "either Amar or Akbar or Anthony must have lassi".

From Fact (3), it can be infered that either Amar or Akbar must have lassi.

Now, from Fact (1), it is apparent that Anthony too must have lassi. But according to Fact (4), Akbar cannot have lassi when Anthony does.
Brain Teaser No : 00191

Decipher this sentence.

B R W Q H L F K W H J K Q I B W K



Q I C E D W Z B G W K K M I K E



Z B G Q H S K Z B G J K Z K W



B U U Z B G J D B H F W.

Answer

Start with ZBG and ZBGJ. It should be either "the/then" or "you/your" combination as they appear more.

B R W Q H L F K W H J K Q I B W K

o b s t a c l e s a r e t h o s e



Q I C E D W Z B G W K K M I K E

t h i n g s y o u s e e w h e n



Z B G Q H S K Z B G J K Z K W

y o u t a k e y o u r e y e s



B U U Z B G J D B H F W.

o f f y o u r g o a l s.
Brain Teaser No : 00001

At what time immediately prior to Six O'clock the hands of the clock are exactly opposite to each other. Give the exact time in hours, minutes and seconds.
Answer

It is obvious that between 5 O'clock and 6 O'clock the hands will not be exactly opposite to each other. It is also obvious that the hands will be opposite to each other just before 5 O'clock. Now to find exact time:

The hour hand moves 1 degree for every 12 degrees that the minute hand moves. Let the hour hand be X degree away from 5 O'clock. Therefore the minute hand is 12X degree away from 12 O'clock.

Therefore solving for X

Angle between minute hand and 12 O'clock + Angle between 12 O'clock and 4 O'clock + Angle between 4 O'clock and hour hand = 180
12X + 120 + (30-X) = 180
11X = 30
Hence X = 30/11 degrees
(hour hand is X degree away from 5 O'clock)

Now each degree the hour hand moves is 2 minutes.

Therefore minutes are
= 2 * 30/11
= 60/11
= 5.45 (means 5 minutes 27.16 seconds)

Therefore the exact time at which the hands are opposite to each other is
= 4 hrs. 54 min. 32.74 seconds

Ali Baba had four sons, to whom he bequeathed his 39 camels, with the proviso that the legacy be divided in the following way :

The oldest son was to receive one half the property, the next a quarter, the third an eighth and the youngest one tenth. The four brothers were at a loss as how to divide the inheritance among themselves without cutting up a camel, until a stranger appeared upon the scene.
Dismounting from his camel, he asked if he might help, for he knew just what to do. The brothers gratefully accepted his offer.

Adding his own camel to Ali Baba's 39, he divided the 40 as per the will. The oldest son received 20, the next 10, the third 5 and the youngest 4. One camel remained : this was his, which he mounted and rode away.

Scratching their heads in amazement, they started calculating. The oldest thought : is not 20 greater than the half of 39? Someone must have received less than his proper share ! But each brother discovered that he had received more than his due. How is it possible?
Answer

They took their percentages from 40 and not from 39, so they got more than their share.

The oldest son got 1/2 of 40 = 20 which is 0.5 more
The second son got 1/4 of 40 = 10 which is 0.25 more
The third son got 1/8 of 40 = 5 which is 0.125 more
The youngest son got 1/10 of 40 = 4 which is 0.1 more

And the stranger got 1/40 of 40 = 1 which is 0.025 more (As he is not supposed to get anything)

All these fractions add to = 0.5 + 0.25 + 0.125 + 0.1 + 0.025 = 1 which stranger took away.
There is a family party consisting of two fathers, two mothers, two sons, one father-in-law, one mother-in-law, one daughter-in-law, one grandfather, one grandmother and one grandson.

What is the minimum number of persons required so that this is possible?
Answer

There are total 2 couples and a son. Grandfather and Grand mother, their son and his wife and again their son. So total 5 people.

Grandfather, Grandmother
|
|
Son, wife
|
|
Son


A man went into a fast food restaurant and ate a meal costing Rs. 105, giving the accountant a Rs. 500 note. He kept the change, came back a few minutes later and had some food packed for his girl friend. He gave the accountant a Rs. 100 note and received Rs. 20 in change. Later the bank told the accountant that both the Rs. 500 and the Rs. 100 notes were counterfeit.

How much money did the restaurant lose? Ignore the profit of the food restaurant.
Answer

He lost Rs.600

First time restaurant has given food worth Rs.105 and Rs. 395 change. Similarly second time, food worth Rs.80 and Rs.20 change. Here, we are not considering food restaurant profits.

S L I D E

- D E A N

---------

3 6 5 1
Each of seven digits from 0-9 are represented by a different letter above such that the subtraction is true.

What word represents 3651?
Answer

3651 represents LENS.

Let's assign possible values to each letter and then use trial-n-error.

S must be 1.

Then D (under L) must be greater than 5. If D is 6, then L is 0. But then A must be 0 or 1 which is impossible. Hence, the possible values of D are 7, 8 or 9.

N must be E + 1. Also, D must be A + 5 as the possible values of D are 7, 8 or 9, D can not be (10+A) + 5.

Now using trial-n-error, we get S=1, I=2, L=3, A=4, N=5, E=6 and D=9

S L I D E 1 3 2 9 6

- D E A N - 9 6 4 5

-------------- --------------

3 6 5 1 L E N S
Hence, 3651 represents LENS.
Adam, Burzin, Clark and Edmund each live in an apartment. Their apartments are arranged in a row numbered 1 to 4 from left to right. Also, one of them is the landlord.
1. If Clark's apartment is not next to Burzin's apartment, then the landlord is Adam and lives in apartment 1.
2. If Adam's apartment is right of Clark's apartment, then the landlord is Edmund and lives in apartment 4.
3. If Burzin's apartment is not next to Edmund's apartment, then the landlord is Clark and lives in apartment 3.
4. If Edmund's apartment is right of Adam's apartment, then the landlord is Burzin and lives in apartment 2.
Who is the landlord?

Answer

Clark is the landlord.

Assume each statement true, one at a time and see that no other statement is contradicted.

Let's assume that Statement (1) is true. Then, Adam is the landlord and lives in apartment 1. Also, other three's apartments will be on the right of his apartment - which contradicts Statement (4) i.e. If Edmund's apartment is right of Adam's apartment, then the landlord is Burzin. Thus, Adam is not the landlord.

Let's assume that Statement (2) is true. Then, Edmund is the landlord and lives in apartment 4. Also, other three's apartments will be on the left of his apartment - which again contradicts Statement (4) i.e. If Edmund's apartment is right of Adam's apartment, then the landlord is Burzin. Thus, Edmund is not the landlord either.

Let's assume that Statement (3) is true. Then, Clark is the landlord and lives in apartment 3. It satisfies all the statements for
(1) Adam - (2) Edmund - (3) Clark - (4) Burzin

Hence, Clark is the landlord.

Similarly, you can assume Statement (4) true and find out that it also contradicts.
Brain Teaser No : 00456

B, J and P are related to each other.
1. Among the three are B's legal spouse, J's sibling and P's sister-in-law.
2. B's legal spouse and J's sibling are of the same sex.
Who is the married man?
Answer

J is the married man.

Note that a person's sister-in-law may be the wife of that person's brother or the sister of that person's spouse.

There are 2 cases:
1. If B's legal spouse is J, then J's sibling must be P and P's sister-in-law must be B.
2. If B's legal spouse is P, then P's sister-in-law must be J and J's sibling must be B.
It is given that B's legal spouse and J's sibling are of the same sex. Also, it is obvious that P's sister-in-law is female. Then, B's legal spouse and J's sibling both must be males.

B's spouse J's sibling P's sister-in-law

(male) (male) (female)

------------------------------------------------------

Case I J P B

Case II P B J

Case II is not possible as B & P are married to each other and both are male. Hence, J is the married man.
Brain Teaser No : 00041

A polygon has 1325 diagonals. How many vertices does it have?
Answer

The formula to find number of diagonals (D) given total number of vertices or sides (N) is

N * (N - 3)

D = -----------

2

Using the formula, we get
1325 * 2 = N * (N - 3)
N2 - 3N - 2650 = 0

Solving the quadratic equation, we get N = 53 or -50

It is obvious that answer is 53 as number of vertices can not be negative.

Alternatively, you can derive the formula as triange has 0 diagonals, quadrangel has 2, pentagon has 5, hexagon has 9 and so on......

Hence the series is 0, 0, 0, 2, 5, 9, 14, ........ (as diagram with 1,2 or 3 vertices will have 0 diagonals).

Using the series one can arrive to the formula given above.
Brain Teaser No : 00076

A cube is made of a white material, but the exterior is painted black.

If the cube is cut into 125 smaller cubes of exactly the same size, how many of the cubes will have atleast 2 of their sides painted black?
Answer

44

36 of the cubes have EXACTLY 2 of their sides painted black, but because a cube with 3 of its sides painted black has 2 of its sides painted black, you must also include the corner cubes. This was a trick question, but hopefully the title of the puzzle tipped you off to this.
Brain Teaser No : 00238

Imagine a triangle of coins on a table so that the first row has one coin in it and the second row has two coins in it and so on. If you can only move one coin at a time, how many moves does it take to make the triangle point the other way?

For a triangle with two row it is one, for a triangle with three rows it is two, for a triangle with four rows it is three.

For a traingle with five rows is it four?
Submitted
Answer

It takes 5 moves to make the triangle with 5 rows point the other way.

0 = a coin that has not been moved.
X = the old position of the moved coin
8 = the new position of the moved coin.

________X
_______X X
____8 0 0 0 8
_____0 0 0 0
____X 0 0 0 X
_______8 8
________8


For traingle of any number of rows, the optimal number of moves can be achieved by moving the vertically symmetrical coins i.e. by moving same number of coins from bottom left and right, and remaining coins from the top.


For a triangle with an odd number of rows, the total moves require are :
(N2/4) - (N-4) Where N = 4, 6, 8, 10, ...

For a triangle with even number of rows, the total moves require are :
((N2-1)/4) - (N-4) Where N = 5, 7, 9, 11, ...

Thanks to Alex Crosse for submitting above formulas.
Brain Teaser No : 00053

A man is going to an Antique Car auction. All purchases must be paid for in cash. He goes to the bank and draws out $25,000.

Since the man does not want to be seen carrying that much money, he places it in 15 evelopes numbered 1 through 15. Each envelope contains the least number of bills possible of any available US currency (i.e. no two tens in place of a twenty).

At the auction he makes a successful bid of $8322 for a car. He hands the auctioneer envelopes number(s) 2, 8, and 14. After opening the envelopes the auctioneer finds exactly the right amount.

How many ones did the auctioneer find in the envelopes?
Answer

Each envelope contains the money equal to the 2 raised to the envelope number minus 1. The sentence "Each envelope contains the least number of bills possible of any available US currency" is only to misguide you. This is always possible for any amount !!!

One more thing to notice here is that the man must have placed money in envelopes in such a way that if he bids for any amount less than $25000, he should be able to pick them in terms of envelopes.

First envelope contains, 20 = $1
Second envelope contains, 21 = $2
Third envelope contains, 22 = $4
Fourth envelope contains, 23 = $8 and so on...

Hence the amount in envelopes are $1, $2, $4, $8, $16, $32, $64, $128, $256, $512, $1024, $2048, $4096, $8192, $8617

Last envelope (No. 15) contains only $8617 as total amount is only $25000.

Now as he bids for $8322 and gives envelope number 2, 8 and 14 which contains $2, $128 and $8192 respectively.

Envelope No 2 conrains one $2 bill
Envelope No 8 conrains one $100 bill, one $20 bill, one $5 bill, one $2 bill and one $1 bill
Envelope No 14 conrains eighty-one $100 bill, one $50 bill, four $10 bill and one $2 bill

Hence the auctioneer will find one $1 bill in the envelopes.
Brain Teaser No : 00090

The minute and the hour hand of a watch meet every 65 minutes.

How much does the watch lose or gain time and by how much?

Answer

The minute and the hour hand meet 11 times in 12 hours in normal watch i.e. they meet after every
= (12 * 60) / 11 minutes
= 65.45 minutes
= 65 minutes 27.16 seconds

But in our case they meet after every 65 minutes means the watch is gaining 27.16 seconds.
Brain Teaser No : 00093

There is a number that is 5 times the sum of its digits. What is this number? Answer is not 0.
Answer

The number is 45, simply because
45 = 5 * (4 + 5)
How does one find this number?

Let T be the digit in the tens place and U be the digit in the units place. Then, the number is 10*T + U, and the sum of its digits is T + U.

The following equation can be readily written:
10*T + U = 5*(T + U) or
10*T + U = 5*T + 5*U or
5*T = 4*U

Thus, T / U = 4 / 5

Since T and U are digits, T must be 4 and U must be 5.
There are six boxes containing 5, 7, 14, 16, 18, 29 balls of either red or blue in colour. Some boxes contain only red balls and others contain only blue.

One sales man sold one box out of them and then he says, "I have the same number of red balls left out as that of blue."

Which box is the one he solds out?
Answer

Total no of balls = 5 + 7 + 14 + 16 + 18 + 29 = 89

Total number of balls are odd. Also, same number of red balls and blue balls are left out after selling one box. So it is obvious that the box with odd number of balls in it is sold out i.e. 5, 7 or 29.

Now using trial and error method,
(89-29) /2 = 60/2 = 30 and
14 + 16 = 5 + 7 + 18 = 30

So box with 29 balls is sold out.
Brain Teaser No : 00218

Ekta got chocolates to give her friends on her Birthday. If she gives 3 chocolates to each friend, one friend will get only 2 chocolates. Also, if she gives 2 chocolates to each friends, she will left with 15 chocolates.

How many chocolates Ekta got on her Birthday? and how many friends are there?

Answer

47 Chocolates and 16 Friends

Let's assume that there are total C chocolates and F friends.

According to first case, if she gives 3 chocolates to each friend, one friend will get only 2 chocolates.
3*(F - 1) + 2 = C

Similarly, if she gives 2 chocolates to each friends, she will left with 15 chocolates.
2*F + 15 = C

Solving above 2 equations, F = 16 and C = 47. Hence, Ekta got 47 chocolates and 16 friends
Pooja and Esha met each other after long time. In the course of their conversation, Pooja asked Esha her age. Esha replied, "If you reverse my age, you will get my husbund's age. He is of course older than me. Also, the difference between our age is 1/11th of the sum of our age."

Can you help out Pooja in finding Esha's age?
Answer

Esha's age is 45 years.

Assume that Esha's age is 10X+Y years. Hence, her hunsbands age is (10Y + X) years.

It is given that difference between their age is 1/11th of the sum of their age. Hence,
[(10Y + X) - (10X + Y)] = (1/11)[(10Y + X) + (10X + Y)]
(9Y - 9X) = (1/11)(11X + 11Y)
9Y - 9X = X + Y
8Y = 10X
4Y = 5X

Hence, the possible values are X=4, Y=5 and Esha's age is 45 years.
A fish had a tail as long as its head plus a quarter the lenght of its body. Its body was three-quarters of its total length. Its head was 4 inches long.

What was the length of the fish?
Submitted

The fish is 128 inches long.

It is obvious that the lenght of the fish is the summation of lenghts of the head, the body and the tail. Hence,
Fish (F) = Head (H) + Body (B) + Tail (T)

But it is given that the lenght of the head is 4 inches i.e. H = 4. The body is three-quarters of its total length i.e. B = (3/4)*F. And the tail is its head plus a quarter the lenght of its body i.e. T = H + B/4. Thus, the equation is
F = H + B + T
F = 4 + (3/4)*F + H + B/4
F = 4 + (3/4)*F + 4 + (1/4)*(3/4)*F
F = 8 + (15/16)*F
(1/16)*F = 8
F = 128 inches

Thus, the fish is 128 inches long.

Assume that you have just heard of a scandal and you are the first one to know. You pass it on to four person in a matter of 30 minutes. Each of these four in turn passes it to four other persons in the next 30 minutes and so on.

How long it will take for everybody in the World to get to know the scandal?

Assume that nobody hears it more than once and the population of the World is approximately 5.6 billions.
Answer

Everybody in the World will get to know the scandal in 8 hours.

You came to know of a scandal and you passed it on to 4 persons in 30 minutes. So total (1+4) 5 persons would know about it in 30 minutes.

By the end of one hour, 16 more persons would know about it. So total of (1+4+16) 21 persons would know about it in one hour.

Similarly, the other (1+4+16+64) persons would have know about it in one and a half hours. (1+4+16+64+256) persons would have know about it in two hours and so on...

It can be deduced that the terms of the above series are the power of 4 i.e. 4^0, 4^1, 4^2, 4^3 and so on upto (2N+1) terms. Also, the last term would be 4^2N where N is the number of hours.

Sum of the above mentioned series = [4^(2N+1)-1]/3

The sum of the series must be 5.6 billions. Hence, equating the sum of the series with 5.6 billions, we get N=8 hours.

Scandals travel FAST !!!
A B C

D

E F G

H

I
Each of the digits from 1 to 9 is represented by a different letter above. Also, A + B + C = C + D + E = E + F + G = G + H + I = 13

Which digit does E represent?
Answer

E represents 4.

Find out all possible groups of three different numbers that add up to 13 and arrange them according to given condition.

If one number is 9, it must go with 1 and 3.
If one number is 8, it must go with either 1 and 4 or 2 and 3.
If one number is 7, it must go with either 1 and 5 or 2 and 4.
If one number is 6, it must go with either 2 and 5 or 3 and 4.

It is clear that 9 must go with 1 and 3. Also, no digit may be used in more than two sums. Hence, there are 2 cases:
Case I: If 8 goes with 1 and 4, then 7 goes with 2 and 4, then 6 goes with 2 and 5.
Case II: If 8 goes with 2 and 3, then 7 goes with 2 and 4, then 6 goes with 3 and 4.

But in case II, 3 is used in three sums. Hence, Case I is correct. And the possible arrangements are:

9 3 1 5 6 2

8 7

4 7 2 4 8 1

6 3

5 9
Thus, E must be 4.
A, B and C are three points on a straight line, not necessarily equidistant with B being between A and C. Three semicircles are drawn on the same side of the line with AB, BC and AC as the diameters. BD is perpendicular to the line ABC, and D lies on the semicircle AC.

If the funny shaped diagram between the three semicircles has an area of 1000 square cms, find the length of BD.
Answer


The length of BD is 35.68 cms

There are 3 right-angled triangles - ABD, CBD and ADC.

From ABD, AB^2 + BD^2 = AD^2 ------ I
From CBD, CB^2 + BD^2 = CD^2 ------ II
From ADC, AD^2 + CD^2 = AC^2 ------ III

Adding I and II,
AB^2 + BC^2 + 2*BD^2 = AD^2 + CD^2 ------ IV

FROM III and IV
AB^2 + BC^2 + 2*BD^2 = AC^2
AB^2 + BC^2 + 2*BD^2 = (AB+CB)^2
2*BD^2 = 2*AB*CB
BD^2 = AB*CB
BD = SQRT(AB*CB)

Given that funny shaped diagram beween three semicircles has an area of 1000 square cms.
[PI/2 * (AC/2)^2] - [PI/2 * (AB/2)^2] - [PI/2 * (BC/2)^2] = 1000
PI/8 * [AC^2 - AB^2 - BC^2] = 1000
PI * [(AB+BC)^2 - AB^2 - BC^2] = 8000
PI * [2*AB*BC] = 8000
AB * BC = 4000/PI

Hence BD = SQRT(4000/PI) = 35.68 cms
where PI = 3.141592654

Hence, the length of BD is 35.68 cms.

Submit
Answer

Users
Answer (33)

BrainVista
Answer

P


Brain Teaser No : 00660

Gomzi has 3 timepieces in his house - a wall clock, an alarm clock and a wristwatch. The wristwatch is always accurate, whereas the wall clock gains 2 minutes everyday and the alarm clock loses 2 minutes everyday.

At exactly midnight last night, all three watches were showing the same time.

If today is 25 July 2003, then on which date all three clocks will show the same time again?

Answer

All three clocks will show the same time again on midnight between 19 July 2004 and 20 July 2004.

A clock finishes on round in 12*60 i.e. 720 minutes.

If a clock gains 2 minutes everyday, then it would be 720 minutes ahead after 360 days. Thus, after 360 days, it will show the same time again.

Similary, if a clock loses 2 minutes everyday, then it would be 720 minutes behind after 360 days. Thus, after 360 days, it will show the same time again.

Thus, after 360 days all three clocks will show the same time again i.e. midnight between 19 July 2004 and 20 July 2004.
You have 9 marbles. 8 marbles weigh 1 ounce each, & one marble weighs 1.5 ounces. You are unable to determine which is the heavier marble by looking at them. You have a weighing scale that consists of 2 pans, but the scale is only good for 2 total weighings.

How can you determine which marble is the heaviest one using the scale & in 2 weighings?
Answer

Divide 9 marbles into 3 groups of 3 marbles each.

Take any 2 groups and place them on each pan. If they balance, remove the marbles from the pans, & place any 2 of the marbles from the remaining unweighed group on the pans, 1 on each pan.

If one is heavier, it is the heavier marble, but if they balance, the remaining unweighed marble is the heavier one.

If your first weighing does not balance, remove the marbles from the lighter pan, & place 1 marble on each pan from the heavier pan. The heavier 1 is the 1.5 ounce marble, but if they balance, then the marble from the heavy pan from the first weighing that was not weighed in the second weighing is the heavy 1.
Once a week a wagon driver leaves his hut and drives his wagon to the river dock to pick up supplies for his town. At 4:05 PM, one-fifth of the way to the dock, he passes the Temple. At 4:15 PM, one-third of the way, he passes the Preetam-Da-Dhabaa.

At what time does he reached the dock?

Answer

5:05 PM

At 4:05 PM, the wagon driver passes the temple, one-fifth of the way to the dock. Also, at 4:15 PM, he passes the Preetam-Da-Dhabaa, one-third of the way. Thus, he travels 2/15 (1/3 - 1/5) of the distance in 10 minutes.

At 4:15 PM, he has already travelled 1/3 of the distance. Thus 2/3 of the way is remaining, which can be travelled in
= ( (2/3) * 10 ) / (2/15)
= 50 minutes

At 4:15, he was at Preetam-Da-Dhabaa.and remaining way will take 50 more minutes. Hence, the driver will reach at 5:05 PM to the dock.
Brain Teaser No : 00115

Four prisoners escape from a prison.
The prisoners, Mr. East, Mr. West, Mr. South, Mr. North head towards different directions after escaping.
The following information of their escape was supplied:
• The escape routes were North Road, South Road, East Road and West Road
• None of the prisoners took the road which was their namesake
• Mr. East did not take the South Road
• Mr.West did not the South Road
• The West Road was not taken by Mr. East
What road did each of the prisoners take to make their escape
Answer

Put all the given information into the table structure as follow:
North Road South Road East Road West Road
Mr. North No
Mr. South No
Mr. East No No No
Mr. West No No

Now from table, two things are obvious and they are:
• Mr.North took the South Road
• Mr.East took the North Road
Put this information into the table, Also keep in mind that the prisoners head towards different directions after escaping.
North Road South Road East Road West Road
Mr. North No YES No No
Mr. South No No
Mr. East YES No No No
Mr. West No No No

Now from the table:
• Mr.West took the East Road
• Mr.South took the West Road
So the answer is:
• Mr.North took the South Road
• Mr.South took the West Road
• Mr.East took the North Road
• Mr.West took the East Road

Shahrukh speaks truth only in the morning and lies in the afternoon, whereas Salman speaks truth only in the afternoon and lies in the morning.

A says that B is Shahrukh.

Is it morning or afternoon and who is A - Shahrukh or Salman?
Answer

It is Afternoon and A can be Salman or Shahrukh. If A is Salman, he is speaking truth. If A is Shahrukh, he is lying.

Want to confirm it? Consider following 4 possible answers and check for its truthness individually.
1. It is Morning and A is Shahrukh
2. It is Morning and A is Salman
3. It is Afternoon and A is Shahrukh
4. It is Afternoon and A is Salman

A rich man died. In his will, he has divided his gold coins among his 5 sons, 5 daughters and a manager.

According to his will: First give one coin to manager. 1/5th of the remaining to the elder son. Now give one coin to the manager and 1/5th of the remaining to second son and so on..... After giving coins to 5th son, divided the remaining coins among five daughters equally.

All should get full coins. Find the minimum number of coins he has?
Answer

We tried to find out some simple mathematical method and finally we wrote small C program to find out the answer. The answer is 3121 coins.

Here is the breakup:
First son = 624 coins
Second son = 499 coins
Third son = 399 coins
Forth son = 319 coins
Fifth son = 255 coins
Daughters = 204 each
Manager = 5 coins
There is a grid of 20 squares by 10 squares. How many different rectangles are possible?

Note that square is a rectangle.
Answer

11550

The Generic solution to this is:
Total number of rectangles = (Summation of row numbers) * (Summation of column numbers)

Here there are 20 rows and 10 columns or vice versa. Hence, total possible rectangles
= ( 20 + 19 + 18 + 17 + 16 + .... + 3 + 2 + 1 ) * ( 10 + 9 +8 + 7 + .... + 3 + 2 + 1)
= ( 210 ) * (55)
= 11550

Hence, total 11,550 different rectangles are possible.

If you don't believe it, try formula on some smaller grids like 4x2, 3x2, 3x3 etc...
If A+B=C, D-C=A and E-B=C, then what does D+F stands for? Provide your answer in letter terms as well as in number terms.
Submitted by : David
Answer

J or 10

A simple one.

Assume that each character represents the number equivalent to the position in the alphabet i.e. A = 1, B = 2, C = 3, D = 4 and so on. Now let's check our assumption.

A + B = C i.e. 1 + 2 = 3
D - C = A i.e. 4 - 3 = 1
E - B = C i.e. 5 - 2 = 3

Thus, our assumption was Correct. Hence, D + F = J i.e. 4 + 6 = 10
A woman took a certain number of eggs to the market and sold some of them.

The next day, through the industry of her hens, the number left over had been doubled, and she sold the same number as the previous day.

On the third day the new remainder was tripled, and she sold the same number as before.

On the fourth day the remainder was quadrupled, and her sales the same as before.

On the fifth day what had been left over were quintupled, yet she sold exactly the same as on all the previous occasions and so disposed of her entire stock.

What is the smallest number of eggs she could have taken to market the first day, and how many did she sell daily? Note that the answer is not zero.
Submitted
Answer

She took 103 eggs to market on the first day and sold 60 eggs everyday.

Let's assume that she had N eggs on the first day and she sold X eggs everyday. Putting down the given information in the table as follow.
Days Eggs at the start of the day Eggs Sold Eggs Remaining
Day 1 N X N-X
Day 2 2N-2X X 2N-3X
Day 3 6N-9X X 6N-10X
Day 4 24N-40X X 24N-41X
Day 5 120N-205X X 120N-206X

It is given that she disposed of her entire stock on the fifth day. But from the table above, the number of eggs remaining are (120N-206X). Hence,
120N - 206X = 0
120N = 206X
60N = 103X

The smallest value of N and X must be 103 and 60 respectively. Hence, she took 103 eggs to market on the first day and sold 60 eggs everyday.
John lives in "Friends Society" where all the houses are in a row and are numbered sequentially starting from 1. His house number is 109.

Jessy lives in the same society. All the house numbers on the left side of Jessy's house add up exactly the same as all the house numbers on the right side of her house.

What is the number of Jessy's house? Find the minimal possible answer.
Answer

There are 288 houses and Jessy's house number is 204.

Let's assume that in the "Friends Society" there are total N houses numbered from 1 to N and Jessy's house number is X.

Now it is given that all the house numbers on the left side of Jessy's house add up exactly the same as all the house numbers on the right side of her house. Hence,
1 + 2 + 3 + ..... + (X-1) = (X+1) + (X+2) + (X+3) + ..... + N

Both the sides of the above equations are in A.P. Hence, using A.P. summation formaula,

[(X-1)/2][2*(1) + (X-1-1)] = [(N-X)/2][2*(X+1) + (N-X-1)]
[X-1][(2) + (X-2)] = [N-X][(2X+2) + (N-X-1)]
(X-1)(X) = (N-X)(N+X+1)
X2 - X = N2 + NX + N - NX - X2 - X
X2 = N2 + N - X2
2X2 = N2 + N
X2 = (N2 + N)/2
X2 = N(N+1)/2

Now, using Trial and Error method to find values of N and X such that above equation is satisfied, we get
1. N = 8, X = 6
2. N = 49, X = 35
3. N = 288, X = 204
4. N = 1681, X = 1189
5. N = 9800, X = 6930
But we require minimal possible answer and it is given that John's house number is 109. It means that there are atleast 109 houses. Hence, first two are not possible. And the answer is : there are 288 houses and Jessy's house number is 204.
Makayla had $1.19 in change. None of the coins was a dollar.

Nicole ask her for change for a dollar, but Makayla could not make change.

What coins did she have?
Submitted
Answer

As it is given that Makayla had $1.19, it means she would have four pennies. Now, the remaining $1.15 in coins must not add up for exactly a dollar. Therefore she would not have 4 quarters or 2 quarters and 5 dimes. But she would have either 1 quarter or 3 quarters. Hence, there are 2 solutions.

Solution I
1 Quarter, 9 Dimes, 4 Pennies (0.25 + 0.90 + 0.04 = $1.19)

Solution II
3 Quarters, 4 Dimes, 4 Pennies (0.75 + 0.40 + 0.04 = $1.19)
A group of friends went on a holiday to a hill station. It rained for 13 days. But when it rained in the morning, the afternoon was lovely. And when it rained in the afternoon, the day was preceded by clear morning.

Altogether there were 11 very nice mornings and 12 very nice afternoons. How many days did their holiday last?
Answer

The holiday last for 18 days.

Let's assume the number of days as follows:
Rain in the morning and lovely afternoon = X days
Clear morning and rain in the afternoon = Y days
No rain in the morning and in the afternoon = Z days

Number of days with rain = X + Y = 13 days
Number of days with clear mornings = Y + Z = 11 days
Number of days with clear afternoons = X + Z = 12 days

Solving above 3 equations, we get X = 7, Y = 6 and Z = 5

Hence, total number of days on holiday = 18 days
Brain Teaser No : 00299

Substitute digits for the letters to make the following Division true
Y F Y

-----------

A Y | N E L L Y

| N L Y

----------------

P P L

P N H

----------

N L Y

N L Y

----------

0 0 0
Note that the leftmost letter can't be zero in any word. Also, there must be a one-to-one mapping between digits and letters. e.g. if you substitute 3 for the letter N, no other letter can be 3 and all other N in the puzzle must be 3.
Submitted by : Calon

Answer

See the pattern of the Y. AY * Y = NLY i.e. Y is multiplied by Y and the last digit of the answer is also Y. Thus, the value of Y would be 5 or 6.

Also,
H=0 as L - H = L
P = 2N as P - N = N
L - Y = P = 2N
E - L = p

Let's find out the minimum possible values. If N=1, then P=2, Y=5, L=7 and E=9. Note that the value of Y can not be 6 as it makes L=8 and E=10 which is not possible. Hence, Y=5, N=1, P=2, L=7, E=9, H=0

Now, using trial-n-error or rather solving F*AY=PNH, we get F=6 and A=3.

5 6 5 Y F Y

----------- -----------

3 5 | 1 9 7 7 5 A Y | N E L L Y

| 1 7 5 | N L Y

----------- -----------

2 2 7 P P L

2 1 0 P N H

----------- -----------

1 7 5 N L Y

1 7 5 N L Y

----------- -----------

0 0 0 0 0 0

Brain Teaser No : 00566

Here is the family tree of Mr. RAHUL
RAHUL

|

---------------------------------------------

| | | |

RATISH YASH OM TRILOK

| | ?

-------- ------------------

| | | | |

AMAR AMIT RAM HARSH ASHOK

| |

----- -------

| | | | |

How many children does Mr. TRILOK have?

Answer

TRILOK have 5 children.

Name of the person and number of his children are related by some pattern.

Assign each vowel following values.
A=0 E=1 I=2 O=3 U=4

The number of children to any person is the sum of the values represented by vowels in his name.
RATISH = 0(A) + 2(I) = 2
OM = 3(O) = 3
AMIT = 0(A) + 2(I) = 2
ASHOK = 0(A) + 3(O) = 3
TRILOK = 2(I) + 3(O) = 5

Hence, TRILOK have 5 children.

Major Jasbir is forming five-person Special Task Group. The group must contain one leader, two bomb-experts and two soldiers.

P, Q and R are possible bomb-experts. R, S and T are possible leaders. U, V and W are possible soldiers. Also, P and R prefers to work with each other in the same team. T prefers to work only if V works.

How many different possible Groups, Major Jasbir can make?
Answer

Major Jasbir can make 8 different possible groups.

As 2 bomb-experts to be selected from the given 3 and also P & R prefers to work together, PR must be there in all the possible Groups. Also, T prefers to work only if V works. It doesn't mean that V won't work without T.

Hence, possible groups are:
PR - S - UV
PR - S - VW
PR - S - WU

PR - T - UV
PR - T - VW

PQ - R - UV
PQ - R - VW
PQ - R - WU

Hence, there 8 different groups are possible.
The secret agent X emailed some code to his head office. They are "RADAR, LEVEL, ROTOR, REDIVIDER, MOTOR". But four of these five words have something in common and one is fake.

Can you tell which one is fake? Ignore the fact that four of the code-words are of the same length.
Answer

The fake code-word is MOTOR.

All the code-words except MOTOR are Palindromes.
Brain Teaser No : 00287

In the village called TALAJA, only three TV channels are available - Moon Plus, Mony and Mee TV.

Out of 4000 TV viewers in the village, 1500 watch Moon TV, 2000 watch Mony and 2500 watch Mee TV.

Amongst these, 500 viewers watch Moon Plus and Mony, 800 watch Moon Plus and Mee TV, and 1000 watch Mony and Mee TV.

How many viewers watch all three channels?

Answer

300 viewers watch all three channels.

Let's assume that total X viewers watch all three channels.

total viewers who watch only Moon Plus and Mony = 500-X
total viewers who watch only Moon Plus and Mee TV = 800-X
total viewers who watch only Mony and Mee TV = 1000-X

total viewers who watch only Moon Plus
= 1500 - (500-X) - (800-X) - X
= 200 + X

total viewers who watch only Mony
= 2000 - (500-X) - (1000-X) - X
= 500 + X

total viewers who watch only Mee TV
= 2500 - (1000-X) - (800-X) - X
= 700 + X

We know that total viewers are 4000. Summing up all 7 values,
X + (500 - X) + (800 - X) + (1000 - X) + (200 + X) + (500 + X) + (700 + X) = 4000
X + 3700 = 4000
X = 300

Hence, total 300 viewers watch all three channels.
A man was looking at a portrait. Someone asked him, "Whose picture are you looking at?"

He replied, pointing at the portrait: "Brothers and sisters have I none, but this man's son is my father's son."

Now whose picture is the man looking at?
Answer

The man is looking at his FATHER's portrait.

"my father's son" is the man himself as he do not have any brothers and sisters. So the statement reduces to "this man's son is myself." Now it is clear that the portrait is of his father.
Given the following facts:
1. Dinesh is younger than Farukh and older than Gurmit.
2. Jatin is younger than Chandu and older than Eshrat.
3. Amit is younger than Irfan and older than Chandu.
4. Farukh is younger than Bhavin and older than Hemant.
5. Irfan is younger than Gurmit and older than Jatin.
6. Hemant is older than Gurmit.
Who is the Youngest?
Answer

Eshrat is the youngest.

Discard whoever are older than someone.

From (1) Gurmit is younger than Dinesh and Farukh.
From (5) Jatin is younger than Irfan and Gurmit.
From (2) Eshrat is younger than Jatin and Chandu.

From above 3 deductions, Eshrat is younger than Dinesh, Farukh, Irfan, Gurmit, Jatin and Chandu.

Also,
From (3) Chandu is younger than Amit and Irfan.
From (4) Hemant is younger than Farukh and Bhavin.
From (6) Gurmit is younger than Hemant.

From above 3 deductions, Gurmit is younger than Farukh, Bhavin and Hemant. Also, Chandu is younger than Amit and Irfan. But as seen earlier, Eshrat is younger than Gurmit and Chandu.

Hence, Eshrat is the youngest.
Last Saturday Milan went for the late night show and came late. In the morning family members asked him which movie did he see. He gave different answers to everyone.
• He told to his father that he had gone to see MONEY.
• According to his mom, he saw either JOHNY or BABLU.
• His elder brother came to know that he saw BHABI.
• To his sister, he told ROBOT.
• And his grandpa heard that he saw BUNNY.
Thus, Milan gave six movie names, all five letter words. But he saw some other movie with five letter word. Moreover, each of the six movie names mentioned above has exactly two letters common with the movie he saw. (with the same positions)

Can you tell which movie did Milan see?
Answer

Milan saw BOBBY.

The six movie names are - MONEY, JOHNY, BABLU, BHABI, ROBOT and BUNNY.

Compare MONEY and JOHNY. They have O common at the second place and Y common at the fifth place. Also, they can't have two different letters each, common with the required movie as the letters in remaining three places are all different. Thus, the required movie must have either O at the second place or Y at the fifth place or both.

Similarly, comparing JOHNY and BUNNY - the required movie must have either N at the fourth place or Y at the fifth place or both. Also, comparing MONEY and BUNNY - the required movie must have either N at the third place or Y at the fifth place or both.

From the above 3 deduction, either Y is at fifth place or O is at the second place and N is at the third & fourth place. The later combination is not possible as BABLU, BHABI & ROBOT will need at least 3 other letters which makes the required movie 6 letter long. Hence, the required movie must have Y at the fifth place.

Now Y is not there in BABLU and BHABI at the fifth place and they have only B common at the first place. Hence, B must be the first letter.

As B is at the first place and Y is at the fifth place and every movie has exactly 2 letters common with the required movie. From BUNNY, the required movie do not have U at the second place and N at the third and fourth place. Now looking at JOHNY and MONEY, they must have O common at the second place.

Using the same kind of arguments for BABLU, BHABI and ROBOT, we can conclude that Milan saw BOBBY.

Jim lies a lot. He tells the truth on only one day in a week.

One day he said: "I lie on Mondays and Tuesdays."
The next day he said: "Today is either Sunday, Saturday or Thursday."
The next day he said: "I lie on Fridays and Wednesdays."

On which day of the week does Jim tell the truth?
Answer

Jim tells the truth on Tuesday.

As Jim tells truth only on one day in a week, his statement on day 1 and day 3 both can not be false. Otherwise he tells truth on more than one days in a week. Also, all three statements are mad on three consecutive days, statement made on day 1 and day 3 both can not be true. Thus, either the statement made on day 1 or day 3 is true and other is false. Also, the statement made on day 2 must be false i.e. day 1 is not Saturday, Friday or Wednesday.

Let's assume that the statement 1 is true. Then from the statement 3, day 1 must be either Friday or Wednesday. But it is already deduced that day 1 is not Saturday, Friday or Wednesday.

Hence, the statement made on day 1 is false and the last statement is true. then from the statement 1, day 3 must be either Monday or Tuesday. But it is already deduced that day 1 can not be Saturday i.e. day 3 can't be Monday. Hence, Jim tells the truth on Tuesday.

4 men can dig 4 holes in 4 days.

How many hours does it take for 1 man to dig half a hole?
Submitted
Answer

There is nothing like "HALF HOLE".
Consider a chessboard with a single Rook. A Rook can move any number of square sideways/forward, but not diagonally.

What is the minimum number of moves the Rook needs to make, in order to pass over all the squares on the chessboard and return to the original position?
Answer

16 moves

As a Rook can move any number of square sideways/forward, but not diagonally and there are 8 rows and 8 columns on the chessboard; the Rook needs minimum 16 moves to pass over all the squares and return to the original position.
A farmer needs 8 gallons of water. He has only three unmared buckets, two 6 gallon and one 11 gallon bucket.

How can he collect 8 gallons of water using three unmarked buckets? Provide solution with minimal water wastage.
Answer

Here is the solution with 10 gallon water wastage.
OPERATIONS 6 6 11
Fill 6 gallon bucket with water 6 0 0
Empty 6 gallon bucket into 11 gallon bucket 0 0 6
Fill 6 gallon bucket with water 6 0 6
Fill 11 gallon bucket to full using filled 6 gallon bucket. This will leave 1 gallon water in 6 gallon bucket 1 0 11
Empty 11 gallon bucket into second 6 gallon bucket. 1 6 5
Empty 11 gallon bucket - wastage of 5 gallon water 1 6 0
Empty second 6 gallon bucket into 11 gallon bucket 1 0 6
Fill seccond 6 gallon bucket with water 1 6 6
Fill 11 gallon bucket to full using filled second 6 gallon bucket. This will leave 1 gallon water in second 6 gallon bucket 1 1 11
Fill first 6 gallon bucket with 1 gallon water which is in second 6 gallon bucket 2 0 11
Empty 11 gallon bucket into second 6 gallon bucket. 2 6 5
Empty 11 gallon bucket - wastage of 5 gallon water 2 6 0
Fill 11 gallon bucket with water in both the 6 gallon buckets 0 0 11
I bought a car with a peculiar 5 digit numbered licence plate which on reversing could still be read. On reversing value is increased by 78633.

Whats the original number if all digits are different?
Answer

Only 0 1 6 8 and 9 can be read upside down. So on rearranging these digits we get the answer as 10968.
Jack and Jill are playing cards for a stake of $1 a game. At the end of the evening, Jack has won 3 games and Jill has won $3. How many games did they play?
Submitted by : Nathalie Drouin
Answer

They played total of 9 games. Jack won 3 games and Jill won 6 games.

If Jack has won three games and Jill has won $3, she lost a dollar for each loss, therefore she has won 6 and lost 3 to make $3 and he won the other 3 that she lost!

Sam and Mala have a conversation.
• Sam says I am certainly not over 40
• Mala says I am 38 and you are atleast 5 years older than me
• Now Sam says you are atleast 39
All the statements by the two are false. How old are they really?
Answer

Sam is 41 and Mala is 37.

Let's invert the teaser and read it like this :
• Sam says I am certainly over 40
• Mala says I am not 38 and you are atmost 4 years older than me
• Now Sam says you are atmost 38
From first statement it is clear that Sam is over 40. Also, from next 2 statements it is clear that Mala is less then 38. Hence the possibilities are :
Sam = 41, 42, 43, 44, 45, ......
Mala = 37, 36, 35, 34, 33, ......

It also says that the difference between their age is maximum 4 years. Hence, there is only one possible pair i.e. 41 and 37, all other combination have differences more then 4.

Hence the answer - Sam is 41 and Mala is 37.
A person travels on a cycle from home to church on a straight road with wind against him. He took 4 hours to reach there.

On the way back to the home, he took 3 hours to reach as wind was in the same direction.

If there is no wind, how much time does he take to travel from home to church?
Answer

Let distance between home and church is D.

A person took 4 hours to reach church. So speed while travelling towards church is D/4.

Similarly, he took 3 hours to reach home. So speed while coming back is D/3.

There is a speed difference of 7*D/12, which is the wind helping person in 1 direction, & slowing him in the other direction. Average the 2 speeds, & you have the speed that person can travel in no wind, which is 7*D/24.

Hence, person will take D / (7*D/24) hours to travel distance D which is 24/7 hours.

Answer is 3 hours 25 minutes 42 seconds
There are N secret agents each know a different piece of secret information. They can telephone each other and exchange all the information they know. After the telephone call, they both know anything that either of them knew before the call.

What are the minimum number of telephone calls needed so that all of the them know everything?
Answer

(2N - 3) telephone calls, for N = 2,3
(2N - 4) telephone calls, for N > 3

Divide the N secret agents into two groups. If N is odd, one group will contain one extra agent.

Consider first group: agent 1 will call up agent 2, agent 2 will call up agent 3 and so on. Similarly in second group, agent 1 will call up agent 2, agent 2 will call up agent 3 and so on. After (N - 2) calls, two agents in each the group will know anything that anyone knew in his group, say they are Y1 & Y2 from group 1 and Z1 & Z2 from group 2.

Now, Y1 will call up Z1 and Y2 will call up Z2. Hence, in next two calls total of 4 agents will know everything.

Now (N - 4) telephone calls are reqiured for remaining (N - 4) secret agents.

Total telephone calls require are
= (N - 2) + 2 + (N - 4)
= 2N - 4

Let\'s take an example. Say there are 4 secret agents W, X, Y & Z. Divide them into two groups of 2 each i.e. (W, X) and (Y, Z). Here, 4 telephone calls are required.
1. W will call up X.
2. Y will call up Z.
3. W, who knows WX will call up Y, who knows YZ.
4. X, who knows WX will call up Z, who knows YZ.
Take an another example. Say there are 5 secret agents J, K, L, M & N. Divide them into two groups i.e. (J, K) and (L, M, N). Here, 6 telephone calls are required.
1. J will call up K.
2. L will call up M.
3. M will call up N. Now M and N know LMN.
4. J, who knows JK will call up M, who knows LMN.
5. K, who knows JK will call up N, who knows LMN.
6. L will call up to anyone of four.
Mrs. F has invited several wives of delegates to the United Nations for an informal luncheon. She plans to seat her 9 guests ina row such that each lady will be able to converse with the person directly to her left and right. She has prepared the following list.

Mrs. F speaks English only.
Mrs. G speaks English and French.
Mrs. H speaks English and Russian.
Mrs. J speaks Russian only.
Mrs. K speaks English only.
Mrs. L speaks French only.
Mrs. M speaks French and German.
Mrs. N speaks English and German.
Mrs. O speaks English only.

How many distinct seating arrangements are possible? Give all possible seating arrangements.

Note that ABCD and DCBA are the same.
Answer

126 distinct seating arrangements are possible.

Mrs. J and Mrs. H must be together and Mrs. J must be at the end as Mrs. J speaks only Russian and Mrs. H is the only other Russian speaker.

Mrs. L speaks only French and there are two others - Mrs. G and Mrs. M - who speak French. Here there are 2 cases.
• CASE A : Mrs. L is at the other end
If Mrs. L is at the other end, either Mrs. G or Mrs. M must seat next to her.
o CASE AA : Mrs. G seats next to Mrs. L
Then, Mrs. M must seat next to Mrs. G and Mrs. N must seat next to Mrs. M. This is because Mrs. M speaks French and German, and Mrs. N is the only other German speaker. Thus, the possible seating arrangement is JHxxxNMGL, where x is the English speakers. Mrs. F, Mrs. K and Mrs. O can be arranged in remaining 3 positions in 3! different ways i.e. 6 ways.
o CASE AB : Mrs. M seats next to Mrs. L
If so, then either Mrs. N or Mrs. G must seat next to Mrs. M
 CASE ABA : Mrs. N seats next to Mrs. M
Thus, the possible seating arrangement is JHxxxxNML, where x is the English speakers. Mrs. F, Mrs. G, Mrs. K and Mrs. O can be arranged in remaining 4 positions in 4! different ways i.e. 24 ways.
 CASE ABB : Mrs. G seats next to Mrs. M
Thus, the possible seating arrangement is JHxxxxGML, where x is the English speakers. Mrs. F, Mrs. K, Mrs. N and Mrs. O can be arranged in remaining 4 positions in 4! different ways i.e. 24 ways.

• CASE B : Mrs. L does not seat at the end
It means that Mrs. G, Mrs. L and Mrs. M must seat together. Also, Mrs. L must seat between Mrs. G and Mrs. M.

o CASE BA : Mrs. G seats left and Mrs. M seats right to Mrs. L i.e. GLM

 CASE BAA : GLM is at the other end
Thus, the possible seating arrangement is JHxxxxGLM, where x is the English speakers. Mrs. F, Mrs. K, Mrs. N and Mrs. O can be arranged in remaining 4 positions in 4! different ways i.e. 24 ways.
 CASE BAB : GLM is not at the other end
Then Mrs. N must seat next to Mrs. M. Now, we have a group of four GLMN where Mrs. G and Mrs. N speak English. Thus, the possible seating arrangement is JHxxxX, where x is the individual English speakers and X is the group of four females with English speakers at the both ends. Thus, there are 4! different ways i.e. 24 ways.

o CASE BB : Mrs. M seats left and Mrs. G seats right to Mrs. L i.e. MLG
Then, Mrs. N must seat next to Mrs. M. Now, we have a group of four NMLG where Mrs. G and Mrs. N speak English. Thus, the possible seating arrangement is JHxxxX, where x is the individual English speakers and X is the group of four females with English speakers at the both ends. Thus, there are 4! different ways i.e. 24 ways.
Thus, total different possible seating arrangements are :
= 6 (case AA) + 24 (case ABA) + 24 (case ABB) + 24 (case BAA) + 24 (case BAB) + 24 (case BB)
= 126 seating arrangements

Thus, 126 distinct seating arrangements are poosible.
What is the smallest number which when divided by 10 leaves a remainder of 9, when divided by 9 leaves a remainder of 8, when divided by 8 leaves a remainder of 7, when divided by 7 leaves a remainder of 6 and so on until when divided by 2 leaves a remainder of 1?
Answer

The smallest such number is 2519.

The easiest way is to find the Least Common Multiple (LCM) of 2, 3, 4, 5, 6, 7, 8 and 9. And subtract 1 from it.

The LCM of 2, 3, 4, 5, 6, 7, 8 and 9 is given by 2520. Hence, the required number is 2519

Three friends divided some bullets equally. After all of them shot 4 bullets the total number of bullets remaining is equal to the bullets each had after division. Find the original number divided.
Answer

18

Assume that initial there were 3*X bullets.

So they got X bullets each after division.

All of them shot 4 bullets. So now they have (X - 4) bullets each.

But it is given that,after they shot 4 bullets each, total number of bullets remaining is equal to the bullets each had after division i.e. X

Therefore, the equation is
3 * (X - 4) = X
3 * X - 12 = X
2 * X = 12
X = 6

Therefore the total bullets before division is = 3 * X = 18
Brain Teaser No : 00114

Everyday in his business a merchant had to weigh amounts from 1 kg to 121 kgs, to the nearest kg. What are the minimum number of different weights required and how heavy should they be?

The minimum number is 5 and they should weigh 1, 3, 9, 27 and 81 kgs
Replace each letter by a digit. Each letter must be represented by the same digit and no beginning letter of a word can be 0.


O N E

O N E

O N E

+ O N E

-------

T E N
Answer

Use trial and error. 0 =1, N = 8 ,E = 2, T = 7

1 8 2

1 8 2

1 8 2

+ 1 8 2

------

7 2 8
A man is on a search for Atlantis and comes upon an island where all the inhabitants know whether Atlantis is still around or not.

However, all of the inhabitants are either Fairies or Trolls and they all use a spell to appear humanoid so you cannot tell which is which. And the Faries always tell the truth and the Trolls always lie, but there is a slight complication, some of the Fairies have gone insane and always lie and some of the Trolls have also gone insane and always tell the truth.

So here is your task: you must ask the first inhabitant that you come to ONE question and from that ONE question you must determine wether Atlantis is still around or not.

What is the question that you must ask?
Answer

There are 2 answers to it:

Answer I"Is the statement that you are reliable equivalent to the statement that Atlantis is still around?"

Answer II"Do you believe that the Statement that you are a Fairy is equivalent to the statement that Atlantis is still around?"
Brain Teaser No : 00276

A frog starts climbing 15 feet wall. Each hour he climbs 3 feet and rests for 30 minutes. During rest, he slips back 2 feet.

How many hours does the frog take to reach the top?
Answer

19 hours

A frog climbs 1 foot per 1 1/2 hours as during 30 minutes rest he slips back 2 feet. This way he will climb 12 feet in 18 hours. In next hour he will climb 3 more feet i.e. he will complete 15 feet in 19 hours and will reach the top of the wall.
If a bear eats 65 pounds in fish every day EXCEPT every 6th day which it only eats 45 pounds of fish.

If the bear continues this, how many pounds of fish will it eat in 200 days?
Submitted by : David
Answer

The bear will eat 12,340 pounds of fish in 200 days.

It is given that on every 6th day beareats 45 pounds of fish i.e. on day number 6, 12, 18, 24, .... 192, 198 the bear eats 45 pounds of fish.

Total number of 6th days = 200/6 = 33 (the bear eats 45 pounds)
Hence, the normal days are = 200 - 33 = 167 (the bear eats 65 pounds)

Thus, in 200 days, the bear will eat
= (167) * (65) + (33) * (45)
= 10855 + 1485
= 12,340 pounds
You have 3 points labelled A, B and C. You then have another 3 points labelled 1, 2 and 3. The aim of the puzzle is to connect point A with point 1, 2 and 3. Point B with point 1, 2 and 3 and point C with point 1, 2 and 3.

Now while connecting the points you have to follow one rule - the lines cannot cross over each other.
A B C

1 2 3
PS : You can arrange the points in order as long as the lines DO NOT cross over each other.
Answer

There is no solution to it, if you consider 2 dimensions. It is impossible to join each of points A, B and C with points 1, 2 and 3 without lines crossing each other.

There is solution, if you consider 3 dimensions. Consider a circular base and a line perpendicular to it passing from the center. Now take any 3 points along the perimeter of the circular base as points 1, 2 and 3. Similarly take any 3 points along the perpendicular line as points A, B and C. Now it is quite simple to join each of points A, B and C with points 1, 2 and 3 without any of the lines crossing each other.

The other possible 3D structure is Pyramid. Take points 1, 2 and 3 as a vertices of the triangular base and points A, B and C along the height of the Pyramid which is perpendicular to the triangular base and passing through the apex.
Brain Teaser No : 00477

Suppose five bales of hay are weighed two at a time in all possible ways. The weights in pounds are 110, 112, 113, 114, 115, 116, 117, 118, 120, and 121.

How much does each bale weigh?
Submitted by : Travis Lara

Answer

They weigh 54, 56, 58, 59, 62 pounds.

Let's assume that the weight of five bales are B1, B2, B3, B4 and B5 pounds respectively. Also, B1 <= B2 <= B3 <= B4 <= B5

It is given that five bales of hay are weighed two at a time in all possible ways. It means that each of the bale is weighted four times.
Thus,
4*(B1 + B2 + B3 + B4 + B5) = (110 + 112 + 113 + 114 + 115 + 116 + 117 + 118 + 120 + 121)
4*(B1 + B2 + B3 + B4 + B5) = 1156
(B1 + B2 + B3 + B4 + B5) = 289 pounds

Now, B1 and B2 must add to 110 as they are the lightest one.
B1 + B2 = 110

Similarly, B4 and B5 must add to 121 as they are the heaviest one.
B4 + B5 = 121

From above three equation, we get B3 = 58 pounds

Also, it is obvious that B1 and B3 will add to 112 - the next possible higher value. Similarly, B3 and B5 will add to 120 - the next possible lower value.
B1 + B3 = 112
B3 + B5 = 120

Substituting B3 = 58, we get B1 = 54 and B5 = 62
From 2 & 3 equations, we get B2 = 56 and B4 = 59

Hence, the weight of five bales are 54, 56, 58, 59 and 62 pounds.

Pinto says, "The horse is not Black."
Sandy says, "The horse is either Brown or Grey."
Andy says, "The horse is Brown."

At least one is telling truth and at least one is lying.

Can you tell the color of the horse?
Answer

The color of the horse can be any color other than Black and Brown.

If the color of the horse is Black - all are lying.

If the color of the horse is Brown - all are telling truth.

Thus, the horse is neither Black nor Brown.

If the color of the horse is Grey - Pinto and Sandy are telling truth whereas Andy is lying.

If the color of the horse is other than Black, Brown and Grey - Pinto is telling truth whereas Sandy and Andy are lying.

You must have noticed that for the given conditions, Pinto is always telling truth whereas Andy is always lying
Brain Teaser No : 00258

Three convicts are brought into the warden's office. He says he can parole one of them and to decide which one he will parole he takes out 5 hats (3 red and 2 white). He stands behind them and places a hat on each one of their heads and puts the other two remaining hats in a drawer.

He tells the prisioners they can look at the others hats and if they can tell which hat they have on they will be the one who is paroled.

The first man looks at the other two and says, "I don't know."

The second man looks at the others hats and says, "I don't know."

The third man who is blind says, "Even though I have not the gift of sight I can tell by what the others have said that the color of my hat is..."

What color is the blind mans hat and how does he know?
Submitted
Answer

The color of blind man's hat is Red.

It is sure that the first man saw either both Red hats or one White hat and one Red hat. There are 6 such possibilities:
1) R R R

2) R R W

3) R W R

4) W R R

5) W R W

6) W W R
In all above possibilities, the first man won't be sure of the color of his hat.

Now, the second man knows that the first man saw either both Red hats or one White hat and one Red hat. And, he also knows that its one of the above 6 possibilities. (like we know ;)) But he says, "I don't know". That means that (2) and (5) are not the possibilities as in either case he would be sure of the color of his hat (Red) by just looking at the third man's color of hat (White).

Now, the blind man knows that there are just 4 possibilities - (1), (3), (4), (6) - and in all, the color of his hat is Red.

Submit
Answer

Users
Answer (48)

BrainVista
Answer

Puzzle A
Friend

Add to
Favourite

Back to
Search Result


Hello sajeesh murali
• my Answers
• my Favourites
• Modify Personal Info
• Subscribe
• Logout













• Jigsaw Puzzle
• Join the Dots
• Marbles Game
• Balls Game
• Towers of Hanoi


• Think Number
• Find A Day





Brain Teaser No








Three Gold (G) coins, three Silver (S) coins and three Copper (C) coins are arranged in a single row as follow:
G S C G S C G S C
• Only 2 adjacent unlike coins can be moved at any one time.
• The moved coins must be in contact with at least one other coin in line. i.e. no pair of coins is to be moved and placed away from the remaining ones.
• No coin pairs can be reversed i.e. a S-C combination must remain in that order in its new positionwhen it is moved.
What is the minimum number of moves required to get all the coins in following order?
C C C S S S G G G
Show all moves.
Answer

Minimum number of moves are 8.
Move Order of Coins
0 G S C G S C G S C
1 G S G S C G S C C
2 G S C G S C C S G
3 G S C G S C C S G
4 G S C C S G S C G
5 G S C C S C S G G
6 G S C C C S S G G
7 C C C S S G G S G
8 C C C S S S G G G
A fly is flying between two trains, each travelling towards each other on the same track at 60 km/h. The fly reaches one engine, reverses itself immediately, and flies back to the other engine, repeating the process each time.

The fly is flying at 90 km/h. If the fly flies 180 km before the trains meet, how far apart were the trains initially?
Answer

Initially, the trains were 240 km apart.

The fly is flying at the speed of 90 km/h and covers 180 km. Hence, the fly flies for 2 hours after trains started.

It's obvious that trains met 2 hours after they started travelling towards each other. Also, trains were travelling at the speed of 60 km/h. So, each train traveled 120 km before they met.

Hence, the trains were 240 km apart initially.

What is the minimum number of numbers needed to form every number from 1 to 7,000?

Example: To form 4884, you would need 2 4s & 2 8s. 4822 requires a 4, a 8, & 2 2s, but you would not count the numbers again that you had already counted from making 4884.
Answer

36

You will need 3 of numbers 0, 7, 8 & 9, & 4 of numbers 1-6.
A drinks machine offers three selections - Tea, Coffee or Random (Either tea or Coffee) but the machine has been wired up wrongly so that each button does not give what it claims.

If each drink costs 50p, how much minimum money do you have to put into the machine to work out which button gives which selection?
Submitted
Answer

You have to put just 50p.

Put 50p and push the button for Random. There are only 2 possibilities. It will give either Tea or Coffee.
• If it gives Tea, then the button named Random is for Tea. The button named Coffee is for Random selection. And the button named Tea is for Coffee.
• If it gives Coffee, then the button named Random is for Coffee. The button named Tea is for Random selection. And the button named Coffee is for Tea.
Thus, you can make out which button is for what by putting just 50p and pressing Random selection first.
You have 13 balls which all look identical. All the balls are the same weight except for one. Using only a balance scale, can find the odd one out with only 3 weighings?

Is it possible to always tell if the odd one out is heavier or lighter than the other balls?
Submitted by : Brett Hurrell
Answer

It is always possible to find odd ball in 3 weighings and in most of the cases it is possible to tell whether the odd ball is heavier or lighter. Only in one case, it is not possible to tell the odd ball is whether heavier or lighter.
1. Take 8 balls and weigh 4 against 4.
o If both are not equal, goto step 2
o If both are equal, goto step 3

2. One of these 8 balls is the odd one. Name the balls on heavier side of the scale as H1, H2, H3 and H4. Similarly, name the balls on the lighter side of the scale as L1, L2, L3 and L4. Either one of H's is heavier or one of L's is lighter. Weigh (H1, H2, L1) against (H3, H4, X) where X is one ball from the remaining 5 balls in intial weighing.
o If both are equal, one of L2, L3, L4 is lighter. Weigh L2 against L3.
 If both are equal, L4 is the odd ball and is lighter.
 If L2 is light, L2 is the odd ball and is lighter.
 If L3 is light, L3 is the odd ball and is lighter.

o If (H1, H2, L1) is heavier side on the scale, either H1 or H2 is heavier. Weight H1 against H2
 If both are equal, there is some error.
 If H1 is heavy, H1 is the odd ball and is heavier.
 If H2 is heavy, H2 is the odd ball and is heavier.

o If (H3, H4, X) is heavier side on the scale, either H3 or H4 is heavier or L1 is lighter. Weight H3 against H4
 If both are equal, L1 is the odd ball and is lighter.
 If H3 is heavy, H3 is the odd ball and is heavier.
 If H4 is heavy, H4 is the odd ball and is heavier.

3. One of the remaining 5 balls is the odd one. Name the balls as C1, C2, C3, C4, C5. Weight (C1, C2, C3) against (X1, X2, X3) where X1, X2, X3 are any three balls from the first weighing of 8 balls.
o If both are equal, one of remaining 2 balls is the odd i.e. either C4 or C5. Weigh C4 with X1
 If both are equal, C5 is the odd ball. But you can not tell whether it is heavier or lighter.
 If C4 is heavy, C4 is the odd ball and is heavier.
 If C4 is light, C4 is the odd ball and is lighter.

o If (C1, C2, C3) is heavier side, one of C1, C2, C3 is the odd ball and is heavier. Weigh C1 and C2.
 If both are equal, C3 is the odd ball and is heavier.
 If C1 is heavy, C1 is the odd ball and is heavier.
 If C2 is heavy, C2 is the odd ball and is heavier.

o If (C1, C2, C3) is lighter side, one of C1, C2, C3 is the odd ball and is lighter. Weigh C1 and C2.
 If both are equal, C3 is the odd ball and is heavier.
 If C1 is light, C1 is the odd ball and is lighter.
 If C2 is light, C2 is the odd ball and is lighter.
How many squares are there in a 5 inch by 5 inch square grid? Note that the grid is made up of one inch by one inch squares.
Submitted by : Kristin Monroe
Answer

There are 55 squares in a 5 by 5 grid.

There are 25 squares of one grid.
There are 16 squares of four grids i.e. 2 by 2
There are 9 squares of nine grids i.e. 3 by 3
There are 4 squares of sixteen grids i.e. 4 by 4
There is 1 square of twenty-five girds i.e. 5 by 5

Hence, there are total 25 + 16 + 9 + 4 + 1 = 55 squares.

You must have noticed one thing that total number squares possible of each size is always a perfact square i.e. 25, 16, 9, 4, 1

For a grid of N by N, the possible number of squares are
= N2 + (N - 1)2 + (N - 2)2 + (N - 3)2 + ......... + 32 + 22 + 12

For 1 by 1 grid, total squares = 12 = 1
For 2 by 2 grid, total squares = 22 + 12 = 5
For 3 by 3 grid, total squares = 32 + 22 + 12 = 14
For 4 by 4 grid, total squares = 42 + 32 + 22 + 12 = 30
For 5 by 5 grid, total squares = 52 + 42 + 32 + 22 + 12 = 55
Five horses ran in the race.
• There were no ties.
• Sikandar did not come first.
• Star was neither first nor last.
• Mughal Glory came in one place after Sikandar.
• Zozo was not second.
• Rangila was two place below Zozo.
In what order did the horses finish?
Answer

It's simple.

Let's find the possible places horses can finish. Possibilities are:
Sikandar - 2,3,4 (not 5th as Mughal Glory came one place after him)
Star - 2,3,4
Mughal Glory - 3,4,5
Zozo - 1,3 (not 4th & 5th as Rangila is two place after him)
Rangila - 3,5

So the result is:
1 Zozo
2 Star
3 Rangila
4 Sikandar
5 Mughal Glory

If you added together the number of 2's in each of the following sets of numbers, which set would contain the most 2's: 1-333, 334-666, or 667-999?
Answer

1-333

The reason why is because 200-299 each begins with a 2!

If one person sends the e-mail to two friends, asking each of them to copy the mail and send it to two of their friends, those in turn send it to two of their friends and so on.

How many e-mails would have been sent by the time it did 30 sets?
Answer

2147483646

First person sent the mail to 2 persons. Those 2 sent the mail to 2 persons each, total 4 persons. Now, those 4 person sent mail to total 8 persons, then 8 to 16 persons, 16 to 32 persons and so on.... Hence, it a series of 2, 4, 8, 16, 32 upto 30 numbers

It is a Geometric series with common ratio 2 and first number is also 2. Summation of such series is given by A * (Rn - 1) / (R - 1) where
A = First term
R = Common Ratio
n = total numbers

So total number of times mail sent by the time it did 30 sets
= 2 * (230 - 1) / (2 - 1)
= 2 * (1073741824 - 1)
= 2 * 1073741823
= 2147483646
Brain Teaser No : 00347

At the entrance to a members club stands a stranger seeking admission. A friend told him that it's easy to get in. You just have to answer a question corrcetly! Answering wrong, however, will result in being shot!

To live a little longer, the man waits in a back alley near the entrance for people to go in. After a while a man comes to the entrance. The door warden asks him: "Twelve?" to which he replies "Six!" and goes in.

"That's easy." our friend thinks, but he waits a little longer.

Another man comes to the door. "Six?" the door warden asks, to which he replies "Three!" and goes in.

"That's too good to be true" our friend thinks, and he was right. Because, when asked "Four?", he answered "Two!" and was found dead in the alley.

What was the correct answer?
Submitted by : Milind Gadagkar
Answer

The correct answer was "Four".

The answer is the number of letters in the word spoken by the door warden.

"Twelve" contains "Six" letters i.e. T, W, E, L, V, E
"Six" contains "Three" letters i.e. S, I, X
Similarly, "Four" contains "Four" letters i.e. F, O, U, R
There is a perfect sphere of diameter 40 cms. resting up against a perfectly straight wall and a perfectly straight floor i.e. the wall and the floor make a perfect right angle.

Can a perfect sphere of diameter 7 cms. pass through the space between the big sphere, the wall and the floor? Support your answer with valid arguments. Don't submit just "Yes" or "No".
Submit
Answer

Users
Answer (23)

BrainVista
Answer

Puzzle A
Friend

Add to
Favourite




For the sake of simplicity, consider two-dimension i.e view sphere as a two dimensional circle with diameter 40 cms.

From Figure I, (40 cms diameter sphere)
OC2 = OD2 + CD2
OC2 = 202 + 202
OC = 28.28427 cms

Also, X is the closest point to origin O on the sphere.
CX = 20 cms (radius)
OX = OC - CX
OX = 28.28427 - 20
OX = 8.28427 cms

From Figure II, (7 cms diameter sphere)
OP2 = OQ2 + PQ2
OP2 = (3.5)2 + (3.5)2
OP = 4.94974 cms

Also, Y is the farthest point to origin O on the sphere.
PY = 3.5 cms (radius)
OY = OP + PY
OY = 4.94974 + 3.5
OY = 8.44974 cms

Now, as OY > OX i.e. smaller sphere requires more space than the space available. Hence, smaller sphere of 7 cms diameter can not pass through the space between the big sphere, the wall and the floor.

The puzzle can be solved by another method.
Draw a line tangent to the big sphere at the point X such that X is the closest point to the origin O on sphere. The tanget will cut X and Y axes at A and B respectively such that OA=OB. [See Fig III] From above, OX=8.28427 cms.

From the right angle triangle OAB, we can deduct that
OA = OB = 11.71572 cms
AB = 16.56854 cms

Now, the diameter of the inscribed circle of right angle triangle is given by d = a + b - c where a <= b < c

The maximum possible diameter of the circle which can pass through the space between the big sphere, the wall and the floor is
= OA + OB - AB
= 11.71572 + 11.71572 - 16.56854
= 6.86291 cms

Hence, the sphere with 7 cms diameter can not pass through the space between the big sphere, the wall and the floor.

Submit
Answer

Users
Answer (23)

BrainVista
Answer

Puzzle A
Friend

Add

Sarika multiplied 414 by certain number and obtained 69958 as the answer. But she found that there is some error in the answer - both the 9s in the answer are wrong and all the other digits are correct.

Can you find the correct answer?
Answer

The correct answer is 60858.

If you divide 69958 by 414, you will get 168.98. Hence, assume some three digit number and multiply it by 414 and use 6**58 as the answer.

Assume three digit number such that
* * *

4 1 4

-------------

* * *

* * * 0

* * * 0 0

-------------

6 * * 5 8
It is obvious that the last digit of the assumed number must be 7.
* * 7

4 1 4

-------------

* * 8

* * 7 0

* * 8 0 0

-------------

6 * * 5 8
Now, the second last digit of the assumed number must be 4 or 9. Also, the first digit of the assumed number must be 1 as the first digit of the answer is 6. Using trial and error for above two conditions, the answer is
1 4 7

4 1 4

-------------

5 8 8

1 4 7 0

5 8 8 0 0

-------------

6 0 8 5 8
Find the least number which when divided by 35, leaves remainder 25; when divided by 45, leaves remainder 35 and when divided by 55, leaves remainder 45.

Answer

3455

The answer is LCM of (35, 45, 55) minus 10.
LCM of (35, 45, 55) is 3465.
Hence, the answer is 3455.
The ratio of Boys to Girls is 6:4. 60% of the boys and 40% of the girls take lunch in the canteen. What % of class takes lunch in canteen?
Answer

Assume there are 6X boys and 4X Girls

Total Students taking lunch in canteen
= (6X)(60/100) + (4X)(40/100)
= 36(X/10) + 16(X/10)
= 52(X/10)

Total students are = 6X + 4X = 10X

% of class taking lunch in canteen
= ((52X/10) * 100 ) / 10X
= 52 %
In the following multiplication, certain digits have been replaced with asterisks (*). Replace all the asterisks such that the problem holds the result.
* * 7

X 3 * *

----------

* 0 * 3

* 1 *

* 5 *

-------------

* 7 * * 3
Answer

A simple one.
1 1 7

X 3 1 9

----------

1 0 5 3

1 1 7

3 5 1

-------------

3 7 3 2 3
How long would it take you to count 1 billion orally if you could count 200 every minute and were given a day off every four years?

Assume that you start counting on 1 January 2001.
SubmitteAnswer

9 Years, 187 Days, 5 Hours, 20 minutes

As you can count 200 per minute, to count 1 billion you require
= 1,000,000,000/200 minutes
= 5,000,000 minutes
= 83,333.3333 hours
= 3,472.2222 days
= 9.512937 years
= 9 Years, 187 Days, 5 Hours, 20 minutes

Note that a day off every four year will be a Leap day. Hence, no need to consider leap year.
dFive students - Akash, Chintan, Jignesh, Mukund and Venky - appeared for an exam. There were total five questions - two multiple choice (a, b or c) and three true/false questions. They answered five questions each and answered as follow.
I II III IV V

--------------------------------------------------

Chintan c b True True False

Akash c c True True True

Jignesh a c False True True

Mukund b a True True False

Venky b b True False True

--------------------------------------------------
Also, no two students got the same number of correct answers.

Can you tell which are the correct answers? What are their individual score?
Answer

The correct answers are b, a, True, False and False. Also, the scores are Jignesh (0), Akash (1), Chintan (2), Venky (3) and Mukund (4).

As no two students got the same number of correct answers, the total number of correct answers must be either 15 (1+2+3+4+5) or 10 (0+1+2+3+4).

Let's find out the maximum number of correct answers possible from the answers given by them.
For Question I = 2 (b or c)
For Question II = 2 (b or c)
For Question III = 4 (True)
For Question IV = 4 (True)
For Question V = 3 (True)

Thus, the maximum number of correct answers possible are 15 (2+2+4+4+3) which means that Akash would have given all correct answers as only he answered True for questions III, IV and V. But then Chintan and Jignesh would have exactly 3 correct answers. And also, Mukund and Venky would have 2 correct answers. So no one got all five correct. One can also arrive at this conclusion by trial-and-error, but that would be bit lengthy.

Now, it is clear that total number of correct answers are 10 (0+1+2+3+4). Questions III and IV both can not be False. If so, total number of correct answers would not be 10. So the student who got all wrong can not be Chintan, Akash and Mukund.

If Venky got all wrong, then Chintan, Jignesh and Mukund each would have atleast 2 correct answers. It means that Akash would have to be the student with only one correct answer and the correct answers for questions I and II would be a and a respectively. But then the total number of correct answers would be 1 (a) + 1 (a) + 1 (False) + 4 (True) + 2 (Flase) = 9.

Thus, Jignesh is the student with all wrong answers. The correct answers are b, a, True, False and False. Also, the scores are Jignesh (0), Akash (1), Chintan (2), Venky (3) and Mukund (4).

Eleven boys and girls wait to take their seats in the same row in a movie theater. There are exactly 11 seats in the row.

They decided that after the first person sits down, the next person has to sit next to the first. The third sits next to one of the first two and so on until all eleven are seated. In other words, no person can take a seat that separates him/her from at least one other person.

How many different ways can this be accomplished? Note that the first person can choose any of the 11 seats.
Answer

There are 1024 different ways.

This is the type of Brain Teaser that can be solved using the method of induction.

If there is just a one person and one seat, that person has only one option.

If there are two persons and two seats, it can be accomplished in 2 different ways.

If there are three persons and three seats, it can be accomplished in 4 different ways. Remember that no person can take a seat that separates him/her from at least one other person.

Similarly, four persons and four seats produce 8 different ways. And five persons with five seats produce 16 different ways.

It can be seen that with each additional person and seat, the different ways increase by the power of two. For six persons with six seats, there are 32 different ways.

For any number N, the different possible ways are 2(N-1)

Thus, for 11 persons and 11 seats, total different ways are 210 i.e. 1024
The secret agent X emailed a code word to his head office. They are "AIM DUE OAT TIE MOD". But four of these five words are fake and only one contains the information.

The agent X also mailed a sentence as a clue - if I tell you any one character of the code word, you would be able to tell the number of vowels in the code word.

Can you tell which is the code word?

Answer

The code word is TIE.

If you were told any one character of MOD, then you would not be able to determine whether the number of vowels are one or two. e.g. if you were told M, there are two words with M - AIM with 2 vowels and MOD with 1 vowel. So you would not be able to say the number of vowels. Same arguments can be given for characters O and D.

Hence, the word with any one of M, O or D is not a code word i.e. AIM, DUE, OAT and MOD are not the code word. Thus, TIE is the code word.
T : two words - TIE and OAT, both with 2 vowels
I : two words - TIE and AIM, both with 2 vowels
E : two words - TIE and DUE, both with 2 vowels.
Brain Teaser No : 00361

Four men - Abraham, Bobby, Clinton and Denial - are standing in a straight line.
1. One man is fair, handsome and unscarred.
2. Two men who are not fair, are each standing next to Abraham.
3. Bobby is the only man standing next to exactly one handsome man.
4. Clinton is the only man not standing next to exactly one scarred man.
Who is fair, handsome and unscarred?

Answer

Clinton is fair, handsome and unscarred.

From (2), both the men standing next to Abraham are not fair. Also, exactly one man is fair, handsom and unscarred. Hence, there are two cases:

Case 1 :: ? (N, ?, ?) : Abraham (Y, Y, N) : ? (N, ?, ?) : ? (?, ?, ?)
Case 2 :: ? (N, ?, ?) : Abraham (?, ?, ?) : ? (N, ?, ?) : ? (Y, Y, N)

Note the representation - Name (Fair, Handsome, Scarred). "Y" stands for Yes and "N" stabds for No. Abraham (Y, Y, N) means Abraham is Fair, Handsome and Unscarred.

It is clear that either Abraham or the man at the extreme right is fair, handsome and unscarred.

From (4), it is deduced that Clinton is standing next to unscarred man and each of the other men standing next to exactly one scarred man.

Case 1 :: Clinton (N, ?, N) : Abraham (Y, Y, N) : ? (N, ?, Y) : ? (?, ?, Y)
Case 2 :: ? (N, ?, Y) : Abraham (?, ?, Y) : ? (N, ?, N) : Clinton (Y, Y, N)

From (3), Bobby is the only man standing next to exactly one handsome man. But in Case 1, Clinton is standing next to exactly one handsome man. Hence, Case 1 is not possible and Case 2 is the correct one.

Case 2 :: ? (N, ?, Y) : Abraham (?, ?, Y) : ? (N, ?, N) : Clinton (Y, Y, N)

Again from (3) and (4), there are 2 possibilities as shown below.

Case 2a :: Denial (N, N, Y) : Abraham (?, N, Y) : Bobby (N, N, N) : Clinton (Y, Y, N)
Case 2b :: Bobby (N, N, Y) : Abraham (?, Y, Y) : Denial (N, N, N) : Clinton (Y, Y, N)

Thus, Clinton is fair, handsome and unscarred. Also, Abraham may be either fair or not fair.
An orange colored glass has Orange juice and white colored glass has Apple juice both of equal volumes. 50ml of the orange juice is taken and poured into the white glass. After that similarly, 50ml from the white glass is poured into the orange glass.

Of the two quantities, the amount of apple juice in the orange glass and the amount of orange juice in the white glass, which one is greater and by how much?
Answer

The two quantities are equal.

Solve it by taking example. Let's assume that both glasses contain 450 ml of juice each.

Now, 50ml of the orange juice is taken and poured into the White glass. Hence, orange colored glass contains 400 ml of Orange juice and white glass contains 450 ml of Apple juice and 50 ml of Orange juice i.e. total of 500 ml from white glass contains 450 ml of Apple juice and 50 ml of Orange juice. It means that every 50 ml from white glass contains 45 ml of Apple juice and 5 ml of Orange juice.

Similary, 50 ml of juice from white glass is poured into orange glass. Now this 50 ml is not a pure apple juice. It contains 45 ml of Apple juice and 5 ml of Orange juice.

Hence, Orange glass contains 405 ml of Orange juice and 45 ml of Apple juice. Similary, white glass contains 405 ml of Apple juice and 45 ml of Orange juice.
Orange Glass White Glass
Orange Juice Apple Juice Orange Juice Apple Juice
Initially 450 ml 0 ml 0 ml 450 ml
50 ml from Orange Glass is poured into White Glass 400 ml 0 ml 50 ml 450 ml
50 ml from White Glass is poured into Orange Glass 405 ml 45 ml 45 ml 405 ml

Now it is clear that the amount of apple juice in the orange glass and the amount of orange juice in the white glass are the same.

P.S. Here we assumed 450 ml as initial quantity in both the glasses just for simplicity. You can try the same by assuming any other number. But the answer is the same.
Brain Teaser No : 00433

Annie, Bunnie, Candy and Dina visited Edy on 14th February.
1. The time of each visit was as follows:
- Annie at 8:00
- Bunnie at 9:00
- Candy at 10:00
- Dina at 11:00
Each time mentioned above may be either AM or PM.
2. Candy did not visit Edy between Bunnie and Dina.
3. At least one female visited Edy between Annie and Bunnie.
4. Annie did not visit Edy before both Candy and Dina.
Can you tell at what time did they individually visit Edy?

Answer

Bunnie (9:00AM) - Dina (11:00AM) - Annie (8:00PM) - Candy (10:00PM)

From the given data, it is clear that at least one female visited Edy in the morning and at least one female visited Edy in the evening. Also, from (4), Annie did not visit Edy first. It means that Annie visited Edy at 8:00 PM

From (3), Bunnie must have visited Edy at 9:00 AM. Also, either Candy or Dina or both visited Edy in the morning.

But from (2), only Dina must have visited Edy in the morning at 11:00 AM and hence, Candy visited Edy at 10:00 PM.

The order of visits must be:
Bunnie (9:00AM) - Dina (11:00AM) - Annie (8:00PM) - Candy (10:00PM)

In training for a competition, you find that swimming downstream (with the current) in a river, you can swim 2 miles in 40 minutes, & upstream (against the current), you can swim 2 miles in 60 minutes.

How long would it take you to swim a mile in still water?
Answer

You are able to swim downstream at 3 miles an hour, & upstream at 2 miles an hour. There is a difference of 1 mile an hour, which is the river helping you in 1 direction, & slowing you in the other direction.

Average the 2 rates, & you have the rate that you can swim in still water, which is 2.5 miles an hour.

You can thus swim a mile in still water in 24 minutes.
Father's age is three years more than three times the son's age. After three years, father's age will be ten years more than twice the son's age.

What is the father's present age?
Answer

Let son's present age is X years.
Hence, father's present age is (3X + 3) years.

After 3 years, son's age will be (X + 3) years.
and father's age will be (3X + 6) years.

But given that after 3 years father's age will be ten years more than twice the son's age.
(3X + 6) = 2 * (X + 3) + 10
3X + 6 = 2X + 16
X = 10

Therefore, father's present age is 33 years.

Submit
Answer

Users
Answer (17)

Brai


Brain Teaser No : 00570
• A is the father of two children - B and D who are of different sexes.
• C is B's spouse.
• E is the same sex as D.
• B and C have the two children - F who is the same sex as B and G who is the same sex as C.
• E's mother, H who is married to L, is the sister of D's mother, M.
• E and E's spouse, I have two children - J and K who are the same sex as I.
Note that no persons have married more than once. Also, there are more number of females than males. Can you tell how many females are there?

Answer

There are 7 females and 6 males.

Assume that there are four sexes - male, female, X and Y. Prepare the following tree based on the data given :

sister

L(m) - H(f) -------------------- M(f) - A(m)

| |

| |

E(x) - I(y) D(x) B(y) - C(x)

| |

| |

J(y) K(y) F(y) G(x)

It is clear that there are altogether 13 persons - 2 males, 2 females, 4 Xs and 5 Ys.

It is given that there are more number of females than male. Hence, all Y must represent female. Thus, there are 7 females and 6 males.
A positive integer that, when added to 1000 gives a sum which is greater than when multiplied by 1000.

Find the positive integer.
Answer

The positive integer is 1.

Sum of 1 and 1000 = 1 + 1000 = 1001
Multiplication of 1 and 1000 = 1 * 1000 = 1000

Thus, sum of 1 and 1000 is greater than the multiplication of 1 and 1000.
Mr. D'souza has bought four cars - Merc, Honda, Ford, Zen - as presents for his sons' birthdays, all of which are next week. Given the following information, what will each son get?

Alan will not get the Honda unless Barry gets the Merc and Denzil gets the Ford. Barry will not get the Ford unless Carl gets the Zen and Alan gets the Merc. Denzil will not get the Zen unless Alan gets the Honda and Barry gets the Merc. Alan will not get the Merc unless Carl gets the Zen and Denzil gets the Ford. Barry will not get the Merc unless Alan gets the Zen and Denzil gets the Ford. Alan will not get the Zen unless Barry gets the Honda and Carl gets the Merc. Carl will not get the Zen unless Barry gets the Honda and Alan gets the Ford. Alan will not get the Ford unless Barry gets the Zen and Denzil gets the Honda. Carl will not get the Merc unless Denzil gets the Honda.
Answer

Let's put given 9 information in a table. The person in Bold Font will not get the corresponding car unless the persons in Normal Font get the corresponding cars. Also, the person will Italics will get the remaining car.
Merc Honda Ford Zen
1 Barry Alan Denzil Carl
2 Alan Denzil Barry Carl
3 Barry Alan Carl Denzil
4 Alan Barry Denzil Carl
5 Barry Carl Denzil Alan
6 Carl Barry Denzil Alan
7 Denzil Barry Alan Carl
8 Carl Denzil Alan Barry
9 Carl Denzil ? ?

Now, let's assume that Alan gets the Merc. Then from (4), Barry gets the Honda, Denzil gets the Ford and Carl gets the Zen. But from (7), Carl will not get the Zen unless Barry gets the Honda and Alan gets the Ford. Thus, it contradicts the original assumption. Hence, Alan will not get the Merc.

Let's assume that Alan gets the Honda. Then from (1), Barry gets the Merc, Denzil gets the Ford and Carl gets the Zen. But from (5) or from (7), it contradicts the original assumption. Hence, Alan will not get the Honda.

Let's assume that Alan gets the Ford. Then from (8), Carl gets the Merc, Denzil gets the Ford and Barry gets the Zen - which does not contradict any of the statement.

Similaly, you can assume that Alan gets the Zen. (which is contradictory to (9))

Hence, Alan gets the Ford, Barry gets the Zen, Carl gets the Merc and Denzil gets the Honda.
Yesterday in a party, I asked Mr. Shah his birthday. With a mischievous glint in his eyes he replied. "The day before yesterday I was 83 years old and next year I will be 86."

Can you figure out what is the Date of Birth of Mr. Shah? Assume that the current year is 2000.
Answer

Mr. Shah's date of birth is 31 December, 1915

Today is 1 January, 2000. The day before yesterday was 30 December, 1999 and Mr. Shah was 83 on that day. Today i.e. 1 January, 2000 - he is 84. On 31 December 2000, he will be 85 and next year i.e. 31 December, 2001 - he will be 86. Hence, the date of birth is 31 December, 1915.

Many people do think of Leap year and date of birth as 29th February as 2000 is the Leap year and there is difference of 3 years in Mr. Shah's age. But that is not the answer.

Brain Teaser No : 00800

There are 4 mathematicians - Brahma, Sachin, Prashant and Nakul - having lunch in a hotel. Suddenly, Brahma thinks of 2 integer numbers greater than 1 and says, "The sum of the numbers is..." and he whispers the sum to Sachin. Then he says, "The product of the numbers is..." and he whispers the product to Prashant. After that following conversation takes place :

Sachin : Prashant, I don't think that we know the numbers.
Prashant : Aha!, now I know the numbers.
Sachin : Oh, now I also know the numbers.
Nakul : Now, I also know the numbers.

What are the numbers? Explain your answer.
Submitted
Answer

The numbers are 4 and 13.

As Sachin is initially confident that they (i.e. he and Prashant) don't know the numbers, we can conclude that -
1) The sum must not be expressible as sum of two primes, otherwise Sachin could not have been sure in advance that Prashant did not know the numbers.
2) The product cannot be less than 12, otherwise there would only be one choice and Prashant would have figured that out also.

Such possible sum are - 11, 17, 23, 27, 29, 35, 37, 41, 47, 51, 53, 57, 59, 65, 67, 71, 77, 79, 83, 87, 89, 93, 95, 97, 101, 107, 113, 117, 119, 121, 123, 125, 127, 131, 135, 137, 143, 145, 147, 149, 155, 157, 161, 163, 167, 171, 173, 177, 179, 185, 187, 189, 191, 197, ....

Let's examine them one by one.

If the sum of two numbers is 11, Sachin will think that the numbers would be (2,9), (3,8), (4,7) or (5,6).

Sachin : "As 11 is not expressible as sum of two primes, Prashant can't know the numbers."

Here, the product would be 18(2*9), 24(3*8), 28(4*7) or 30(5*6). In all the cases except for product 30, Prashant would know the numbers.

- if product of two numbers is 18:
Prashant : "Since the product is 18, the sum could be either 11(2,9) or 9(3,6). But if the sum was 9, Sachin would have deduced that I might know the numbers as (2,7) is the possible prime numbers pair. Hence, the numbers must be 2 and 9." (OR in otherwords, 9 is not in the Possible Sum List)

- if product of two numbers is 24:
Prashant : "Since the product is 24, the sum could be either 14(2,12), 11(3,8) or 10(4,6). But 14 and 10 are not in the Possible Sum List. Hence, the numbers must be 3 and 8."

- if product of two numbers is 28:
Prashant : "Since the product is 28, the sum could be either 16(2,14) or 11(4,7). But 16 is not in the Possible Sum List. Hence, the numbers must be 4 and 7."

- if product of two numbers is 30:
Prashant : "Since the product is 30, the sum could be either 17(2,15), 13(3,10) or 11(5,6). But 13 is not in the Possible Sum List. Hence, the numbers must be either (2,15) or (5,6)." Here, Prashant won't be sure of the numbers.

Hence, Prashant will be sure of the numbers if product is either 18, 24 or 28.

Sachin : "Since Prashant knows the numbers, they must be either (3,8), (4,7) or (5,6)." But he won't be sure. Hence, the sum is not 11.

Summerising data for sum 11:
Possible Sum PRODUCT Possible Sum
2+9 18 2+9=11 (possible)
3+6=9
3+8 24 2+12=14
3+8=11 (possible)
4+6=10
4+7 28 2+12=14
3+8=11 (possible)
4+6=10
5+6 30 2+15=17 (possible)
3+10=13
5+6=11 (possible)


Following the same procedure for 17:
Possible Sum PRODUCT Possible Sum
2+15 30 2+15=17 (possible)
3+10= 13
5+6=11 (possible)
3+14 42 2+21=23 (possible)
3+14=17 (possible)
6+7=13
4+13 52 2+26=28
4+13=17 (possible)
5+12 60 2+30=32
3+20=23 (possible)
4+15=19
5+12=17 (possible)
6+10=16
6+11 66 2+33=35 (possible)
3+22=25
6+11=17 (possible)
7+10 70 2+35=37 (possible)
5+14=19
7+10=17 (possible)
8+9 72 2+36=38
3+24=27 (possible)
4+18=22
6+12=18
8+9=17 (possible)


Here, Prashant will be sure of the numbers if the product is 52.

Sachin : "Since Prashant knows the numbers, they must be (4,13)."

For all other numbers in the Possible Sum List, Prashant might be sure of the numbers but Sachin won't.


Here is the step by step explaination:

Sachin : "As the sum is 17, two numbers can be either (2,15), (3,14), (4,13), (5,12), (6,11), (7,10) or (8,9). Also, as none of them is a prime numbers pair, Prashant won't be knowing numbers either."

Prashant : "Since Sachin is sure that both of us don't know the numbers, the sum must be one of the Possible Sum List. Further, as the product is 52, two numbers can be either (2,26) or (4,13). But if they were (2,26), Sachin would not have been sure in advance that I don't know the numbers as 28 (2+26) is not in the Possible Sum List. Hence, two numbers are 4 and 13."

Sachin : "As Prashant now knows both the numbers, out of all possible products - 30(2,15), 42(3,14), 52(4,13), 60(5,12), 66(6,11), 70(7,10), 72(8,9) - there is one product for which list of all possible sum contains ONLY ONE sum from the Possible Sum List. And also, no such two lists exist. [see table above for 17] Hence, two numbers are 4 and 13."

Nakul figured out both the numbers just as we did by observing the conversation between Sachin and Prashant.

It is interesting to note that there are no other such two numbers. We checked all the possible sums till 500 !!!
Substitute digits for the letters to make the following subtraction problem true.
S A N T A

- C L A U S

-----------------

X M A S
Note that the leftmost letter can't be zero in any word. Also, there must be a one-to-one mapping between digits and letters. e.g. if you substitute 3 for the letter M, no other letter can be 3 and all other M in the puzzle must be 3.
Answer

One of the simplest brain teaser as there are total 26 possible answers.

It is obvious that S=C+1. Since A-S=S, it is clear that A=2*S or 2*s-10. Also, L and X are interchangeable.
SANTA - CLAUS = XMAS
24034 - 16492 = 7542
24034 - 17492 = 6542
24074 - 15432 = 8642
24074 - 18432 = 5642
24534 - 16492 = 8042
24534 - 18492 = 6042
24794 - 16452 = 8342
24794 - 18452 = 6342
24804 - 15462 = 9342
24804 - 19462 = 5342
24974 - 16432 = 8542
24974 - 18432 = 6542
36806 - 27643 = 9163
36806 - 29643 = 7163
36156 - 27693 = 8463
36156 - 28693 = 7463
62132 - 54206 = 7926
62132 - 57206 = 4926
62172 - 53246 = 8926
62172 - 58246 = 3926
62402 - 53276 = 9126
62402 - 59276 = 3126
62712 - 53286 = 9426
62712 - 59286 = 3426
62932 - 58206 = 4726
62932 - 54206 = 8726








APPTITUDE TESTS
EXAMPLE

1.
(a) The gap between the average starting salaries of teachers and those of other professionals has shrunk in recent years.
(b) The average age of first year teachers is same as it was in 1975.
(c) Starting teachers are no longer underpaid.
(d) The extent of a persons formal education is a measure by which to determine his level of salary.
(e) Over the last few years, the average starting salaries of other professionals have increased by 20%
(a) ebd (b) bad (c) abc (d) aec
Answer is d; the statement a,e,c are logically sequenced.
________________________________________
1.
(a) Japan now produces more semiconductors, than US.
(b) Semiconductors are one of the fastest growing industry segments.
(c) A decade ago Japan was producing 24% and the US was producing 22% of the worlds semiconductors, respectively.
(d) 10 years ago Japan ranked third in semiconductor production.
(e) During the last 10 years Japans production of semiconductors has increased by 500% while that of the us has increased by 200%
(a) abd (b) cea (c) edc (d) bcd

2.
(a) Coding program 1 (b) Writing specifications for program 1
(c) Integrating program 1 with other programs (d) Testing program 1
(e) Collecting cheque from the client of the program
(a) edcba (b) abcde (c) badce (d) abdce

3.
(a) Bob is older than Dinku and Ismer (b) Rahul is oldet than Dinku
(c) Rahul is younger than Bob (d) Rahul is older than Ismer (e) Dinku is older than Ismer
(a) edb (b) bcd (c) dab (d) abc

4.
(a) Defining the data type of the variable (b) Using the variable (c) Declaring the variable
(d) Initializing the variable (e) Remove the variable from the memory
(a) cadbe (b) abcde (c) cdb (d) acdbe

5.
(a) In the last six months the number of robberies at gun point in the city has dropped by 18%
(b) Guns are necessary protection against robbers
(c) Strict gun control causes a decrease in violent crime
(d) Most crimes are committed with guns and knives
(e) Six months ago this city's council passed a gun control law
(a) bda (b) acb (c) ebc (d) eac

6.
(a) All missiles follow a fixed trajectory (b) The boomerang requires a high degree of skill
(c) A boomerang is a missile (d) The boomerang is used by Australian aborigines to hunt
(e) A boomerang normally has an elliptical flight path

(a) adc (b) aec (c) cba (d) ebd

7.
(a) Saving the source file (b) Compiler execution (c) Pre-processor execution
(d) Bug fixing (e) Reading the error file
(a) eabcd (b) acbed (c) abced (d) cbeda

8.
(a) But if powers that be, extended any, how will be the first one to take might claim
(b) I don't believe in seeking special privileges because I'm a woman
(c) Let me explain this in context of what happened the other
(a) bac (b) acb (c) bca (d) abc

9.
(a) A long search produce a comprehensive list of 203 manufacturing firms
(b) The number of workers employed by the firms in the area ranged from a dozen to approximately 3500
(c) Those concerned with mining and quarrying, construction ,transport, trade and commerce were excluded
(d) The investigation was confined to manufacturing firms in the area
(a) bcda (b) bcab (c) abcd (d) dabc

10.
(a) The quickly came back with pots laden with water
(b) The water gurgled out and the dying embers hissed and send up little curls of vapour
(c) The poured it on the glowing bed of charcoal
(d) The men jumped up and rushed to the river
(a) acdb (b) bacb (c) dabc (d) dcba
________________________________________
SECTION 2- DATA SUFFICIENCY

Each item has a question followed by two statements :

Mark a: If the question can be answered with the help of statement "1" alone

Mark b: If the question can be answered with the help of statement "2" alone

Mark c: If the question can be answered with the help of both the statements but not with the help of either statement by itself

Mark d: If the question cannot be answered even with the help of both the given statements

Example:

1. Does winking improve eye sight?

1) During the process of winking the focal power of eyes improves

2) Experiments have shown that eye exercise lead to an improvement in eye sight
Answer: d because neither 1 or 2 is adequate.

Questions :
11. Each floor of a 3 storeyed building is occupied and a total of 15 people live in the building. How many live on the first floor?
1) The no. of people living in the first floor is an odd number
2) The no. of people living on the first floor double the number living on the second floor

12. Program 1 can be implemented
1) Program 1 is tested and error free
2) The implementation site is ready

13. The sum of digits of a 5 digit no. is 10. The digit in the ten thousandth place is cube of that of units place. what is the number.
1) The digits in the thousandth, hundredth and tenth place are equal
2) The digit in the units and tenth place are not equal

14. If I deposit Rs.1000 in the bank now and withdraw the amount only at the end of the year how much will I get?
1) The rate of compound interest is 12% per year
2) The interest is deposited in the account at the end of every six months

15. Variable "X" is an address variable.
1) The value of variable "X" is "adbcf"
2) Program has a statement X =&Y

16. Is white color the best reflector of light?
1) The lower a color's reflection index the better its power of reflection
2) White has a reflection index of 0.28

17. Does Mehta work in an advertising agency?
1) Mehta begins work at 9 am in the morning and works till 9 in the night
2) Mehta is a copywriter

18. Is it true that Maggi Noodles success was largely due to its ability to satisfy a latent consumer need?
1) Before the entry of Maggi Noodles, Others did not have access to a food item which was convenient to prepare and could be consumed between meals.
2) Maggi Noodles was an instant hit with ladies who had children in the range of 10 to 12 years

19. Sachin wrote Program 1
1) It is found in the directory c:\user\sachin
2) Sachin tested Program 1

20. Are all Argots also Knicks?
1) All Argots are Drones
2) All Drones are Knicks

21. Does classical music aid plant growth?
1) Music aids in the development of sugar in plants.
2) In an experiment conducted, its was observed that plants exposed to classical music grew by 5cm more than plants not exposed to classical music in the same period.

22. Are cheques the safest method of making a payment.
1) Cheques are more convenient than cash in making and resolving payments.
2) Payment by cheques eliminate the risk involved in handling cash.

23. Networking is working fine.
1) Computer A is able to talk to Computer B
2) Both Computer A & B are Pentium Machines.

24. Is it true that smiling is easier than frowning?
1) Smiling requires the movement of 14 facial muscles while frowning requires the movement of 24 facial muscles.
2) Moving every facial muscles requires the same amount of effort.

25. Is it true that the Carpenter lives on the first floor.?
1) the Barber lives two floors above the black smith who in turn stays one floor above the carpenter.
2) the blacksmith lives two floors above the weaver who lives one floor below the carpenter in a three storeyed building.
________________________________________
SECTION 3 - ANALYTICAL.

Questions 26-29 are based on the following:

At a formal dinner for 8, the host and the hostess are seated at opposite ends of a rectangular table, with 3 persons along each side. Each man must be seated next to at least to 1 woman, and vice versa. Alan is opposite to Diana, who is not the hostess. George has a woman on his right and is opposite to a woman. Helga is at the hostess's right, next to Frank. One person is seated between Belinda and Carol.

26.The 8th person present, Eric must be
(a) the host
(b) seated to Diana's right
(c) seated opposite to Carol
(a) a only (b) c only (c) b and c (d) a, b and c

27. If each person is placed directly opposite to his or her spouse, which of the following pairs must be married.
(a) George and Helga (b) Belinda and Frank
(c) Carol and Frank (d) George and Belinda

28. Which person is not seated next to a person of the same sex.?
(a) Alan (b) Belinda (c) Carol (d) Diana

29. George is bothered by the cigarette smoke of his neighbor and exchanges seats with the person 4 places to his left. Which of the following must be true following the exchange?
(a) No one is seated between two persons of the opposite sex.
(b) one side of the table consists entirely of persons of the same sex.
(c) Either the host or hostess has changed seats
(a) A only (b) C only (c) A and B (d) B and C

Questions 30 - 33 are based on the following:
The hotel Miramar has two wings, the east wing and the west wing. Some east wing rooms but not all, have an ocean view. All west wing rooms have a harbor view. The charge for all rooms is identical except for the following.
There is an extra charge for all harbor view rooms on or above third floor. There is an extra charge for all ocean view rooms except those without balcony. Some harbor view rooms on the first two floors and some east wing rooms without ocean view have kitchen facilities for which there is an extra charge. Only the ocean view and harbor view rooms have balconies.

30. A guest may avoid an extra charge by requesting
(a) A west wing room on one of the first two floors.
(b) A west wing room on the fourth floor without balcony.
(c) An East wing room without balcony. (d) Any room without kitchen.

31. Which of the following must be true if all conditions are as stated?
(a) All rooms above the third floor involves extra charges.
(b) No room without an ocean or harbor view or kitchen facilities involves extra charge.
(c) There is no extra charge for an east wing room without ocean view.
(d) There is no extra charge for any room without Kitchen facilities.

32. which of the following must be false if all conditions are as stated?
(a) some ocean viewing rooms do not involve an extra charge
(b) all rooms with kitchen facilities involve an extra charge
(c) some west viewing rooms above the second floor do not involve an extra charge
(d) some harbor viewing rooms do not involve an extra charge

33. Which of the following can not be determined on the basis of the information given?
(a) whether there are any rooms without a balcony for which extra charge is imposed
(b) whether any room without at kitchen or a view involves an extra charge
(c) whether two extra charges are imposed for any room (d) none of the above

Questions 34 to 37 are based on the following:
Four cards of different suits are dealt one apiece to A, B, C and D.
B says: Mine is not a club. A says: Mine is not a spade.
D says: Mine is not a diamond. C says: Mine is not a spade.
A says: Mine is not a heart.

34. A held
(a) heart (b) clubs (c) diamonds (d) spade

35. B held
(a) heart (b) clubs (c) diamonds (d) spade

36. C held
(a) heart (b) clubs (c) diamonds (d) spade

37. D held
(a) heart (b) clubs (c) diamonds (d) spade

Questions 38 to 40 are based on the following:
In a magical temple there are 3 doorways each leading to the interior of the temple. Every door way has an idol just inside. The magical powers of the temple doubles the flowers a devotee carries every time he/she passes under a doorway. Each devotee has to pass on straight through the doorway and cannot retrace his steps till he comes to the innermost idol.

38. Ram carries X flowers at each idol he places an identical number of flowers Y. He returns from the temple without a single flower. X was most probably
(a) 2 (b) 5 (c) 6 (d) 7

39. In the situation above Y was most probably
(a) 8 (b) 5 (c) 6 (d) 7

40. If Sita took 8 flowers to the temple and offered 4 flowers each to the first two idols then by the time she faces the third idol she has
(a) 40 flowers (b) 36 flowers (c) 52 flowers (d) 56 flowers
________________________________________
SECTION 4 - COMPUTATIONAL.

41. 2 passengers have together 560 kgs of luggage and are charged for the excess above the weight allowed at 10$ and 26$. If all the luggage had belonged to one of them he would have to pay 46$. The amount of luggage each passenger is allowed without any charge is
(a) 100 kg (b) 150 kg (c) 160 kg (d) Insufficient data

42. 6 pigs cost the same as 9 sheep. 27 sheep cost the same as 30 goats. 50 goats cost the same as 3 elephants. If two elephants cost $4800, then the cost of one pig in dollar is
(a) 120 (b) 240 (c) 105 (d) 250

43. A wholesaler allows a discount of 20 % on the list price to the retailer. The retailer sells at 5% below the list price. If the customer pays Rs.19 for an article what profit is made by the retailer on it?
(a) Rs.2 (b) Rs.3 (c) Rs.4 (d) Rs.4.5

44. A circular metal plate of even thickness has 12 holes of radius 1 cm drilled into it. As a result the plate lost 1/6th its original weight. The radius of the circular plate is
(a) 16sqrt2 (b) 8sqrt2 (c) 32sqrt2 (d) sqrt72

45. 3 machines a,b,c can be used to produce a product. Machine a will take 60 hours to produce a million units. Machine b is twice as fast as machine a. Machine c takes the same amount of time as machine a and b taken together. How much time will be required to produce a million units if all the three machines are used simultaneously?
(a) 12 hours (b) 10 hours (c) 8 hours (d) 6 hours

******************
1. If sita is a girl . she likes pot & pen but not cook, she likes hay but not straw, then if she likes star then she not likes
a) earth b) planet c)moon d) data not sufficient

2. My house has a no

1)if my house no is a multiple of 3 (0 * 3,1*3,2*3 etc), then it is a no from 50 through 59.
2) If my house no is not a multiple of 4, then it is a no from 60 through 69.
3) If my house no is not a multiple of 6 then it is a no from 70 through 79.

What is my house no? (From summer’s book)

3. If a manufacturer sold the goods to wholesaler at 15 % profit, Wholesaler sells the same goods at 25 % profit to retailer; again retailer sells the goods to customer at 20% profit. If customer buy the goods at 135 Rs then what was the manufacturing cost ? ( I have changed Questions data )

4. 1)N is assigned a value 0.

2)………………………
3)Increment N by 1.
4)………………………
5)Print Value of N
6)if N is less than 45 then go to step 2.

I forgot the exact steps but question was very easy. Out put was 45,46,47……..100

5. If a boat travels in upstream @ 40 Km/hr and downstream @ 60 Km/hr then find the speed of the stream?

6. Determine how many triangles in the given figure ?












a)16 b)24 c)36 d)56
7. The game of tic-tac-toe is played in a large square divided in to none small squares.
1)Each of two players in turn places his or her mark—usually X or O---in a small square.
2)The player who first gets three marks in a horizontal, vertical or diagonal line wins.
3)A player will always place his or her mark in a line that already contains (a) two of his or her own marks or (b) two of his or her opponents marks-----giving (a)priority over (b). only the last mark to be placed in the game shown above is not given.
Which mark ---X or O---wins the game ?














This question is again from summer’s book.

8.) N O S I E R
+ A S T R A L
7 2 5 6 1 3
In the addition above, the sum represents a word.
[1] Each letter represents a different digit.
[2] No letter represents zero.

What word is represented by 7 2 5 6 1 3. ( Again from summer)

9) A B C
D
E F G
H
I
Each of the digits 1,2,3,4,5,6,7,8,and 9 is :

[1]Represented by a different letter in the figure above.
[2]Positioned in the figure above so that each of
A+B+C, C+D+E, E+F+G, and G+H+I is equal to 13.
Which digit does E represent ? ( Again from summers book)

10) One question on the line intersection ( I forgot what was the condition )












11) 20 men can do a work in 5 days. How many men will be needed to finish the same work in 5 days such that 2 men’s work of the previous is equal to 3 men’s work of latter ?
12) One more summer’s type puzzle that I can’t remember. It was a long one.

13) 10--- 16--- 24--- 27 ----

--- 8 --- 3 ---

--- ----

A ---


What is the value of A ?
Answer -7
14)Consider the three figures given below.













Don’t go for the data given above. I forgot the exact data. Question was same with different data. We had to find the missing no of the last circle.

ANALYTICAL REASONING SECTION
Directions for questions 1-5: The questions are based on the information given below

There are six steps that lead from the first to the second floor. No two people can be on the same step
Mr. A is two steps below Mr. C
Mr. B is a step next to Mr. D
Only one step is vacant ( No one standing on that step )
Denote the first step by step 1 and second step by step 2 etc.
1. If Mr. A is on the first step, Which of the following is true?
(a) Mr. B is on the second step
(b) Mr. C is on the fourth step.
(c) A person Mr. E, could be on the third step
(d) Mr. D is on higher step than Mr. C.
Ans: (d)
2. If Mr. E was on the third step & Mr. B was on a higher step than Mr. E which step must be vacant
(a) step 1
(b) step 2
(c) step 4
(d) step 5
(e) step 6
Ans: (a)
3. If Mr. B was on step 1, which step could A be on?
(a) 2&e only
(b) 3&5 only
(c) 3&4 only
(d) 4&5 only
(e) 2&4 only
Ans: (c)
4. If there were two steps between the step that A was standing and the step that B was standing on, and A was on a higher step than D , A must be on step
(a) 2
(b) 3
(c) 4
(d) 5
(e) 6
Ans: (c)

5. Which of the following is false

i. B&D can be both on odd-numbered steps in one configuration
ii. In a particular configuration A and C must either both an odd numbered steps or both an even-numbered steps
iii. A person E can be on a step next to the vacant step.
(a) i only
(b) ii only
(c) iii only
(d) both i and iii
Ans: (c)

Directions for questions 6-9: The questions are based on the information given below
Six swimmers A, B, C, D, E, F compete in a race. The outcome is as follows.
i. B does not win.
ii. Only two swimmers separate E & D
iii. A is behind D & E
iv. B is ahead of E , with one swimmer intervening
v. F is a head of D
6. Who stood fifth in the race ?
(a) A
(b) B
(c) C
(d) D
(e) E
Ans: (e)
7. How many swimmers seperate A and F ?
(a) 1
(b) 2
(c) 3
(d) 4
(e) cannot be determined
Ans: (d)
8. The swimmer between C & E is
(a) none
(b) F
(c) D
(d) B
(e) A
Ans: (a)


9. If the end of the race, swimmer D is disqualified by the Judges then swimmer B finishes in which place
(a) 1
(b) 2
(c) 3
(d) 4
(e) 5
Ans: (b)
Directions for questions 10-14: The questions are based on the information given below

Five houses lettered A,B,C,D, & E are built in a row next to each other. The houses are lined up in the order A,B,C,D, & E. Each of the five houses has a colored chimney. The roof and chimney of each housemust be painted as follows.
i. The roof must be painted either green,red ,or yellow.
ii. The chimney must be painted either white, black, or red.
iii. No house may have the same color chimney as the color of roof.
iv. No house may use any of the same colors that the every next house uses.
v. House E has a green roof.
vi. House B has a red roof and a black chimney
10. Which of the following is true ?
(a) At least two houses have black chimney.
(b) At least two houses have red roofs.
(c) At least two houses have white chimneys
(d) At least two houses have green roofs
(e) At least two houses have yellow roofs
Ans: (c)
11. Which must be false ?
(a) House A has a yellow roof
(b) House A & C have different color chimney
(c) House D has a black chimney
(d) House E has a white chimney
(e) House B&D have the same color roof.
Ans: (b)
12. If house C has a yellow roof. Which must be true.
(a) House E has a white chimney
(b) House E has a black chimney
(c) House E has a red chimney
(d) House D has a red chimney
(e) House C has a black chimney
Ans: (a)
13. Which possible combinations of roof & chimney can house
I. A red roof 7 a black chimney
II. A yellow roof & a red chimney
III. A yellow roof & a black chimney

(a) I only
(b) II only
(c) III only
(d) I & II only
(e) I&II&III
Ans: (e)
14. What is the maximum total number of green roofs for houses
(a) 1
(b) 2
(c) 3
(d) 4
(e) 5
NOTE: The questions from 15-27 are multiple choice in the paper

15. There are 5 red shoes, 4 green shoes. If one draw randomly a shoe what is the probability of getting a red shoe

Ans 5c1/ 9c1


16. What is the selling price of a car? If the cost of the car is Rs.60 and a profit of 10% over selling price is earned

Ans: Rs 66/-


17. 1/3 of girls , 1/2 of boys go to canteen .What factor and total number of classmates go to canteen.

Ans: Cannot be determined.


18. The price of a product is reduced by 30% . By what percentage should it be increased to make it 100%

Ans: 42.857%


19. There is a square of side 6cm . A circle is inscribed inside the square. Find the ratio of the area of circle to square.
Ans. 11/14
20. There are two candles of equal lengths and of different thickness. The thicker one lasts of six hours. The thinner 2 hours less than the thicker one. Ramesh lights the two candles at the same time. When he went to bed he saw the thicker one is twice the length of the thinner one. How long ago did Ramesh light the two candles .

Ans: 3 hours.
21. If M/N = 6/5,then 3M+2N = ?
22. If p/q = 5/4 , then 2p+q= ?
23. If PQRST is a parallelogram what it the ratio of triangle PQS & parallelogram PQRST .
Ans: 1:2
24. The cost of an item is Rs 12.60. If the profit is 10% over selling price what is the selling price ?
Ans: Rs 13.86/-
25. There are 6 red shoes & 4 green shoes . If two of red shoes are drawn what is the probability of getting red shoes
Ans: 6c2/10c2
26. To 15 lts of water containing 20% alcohol, we add 5 lts of pure water. What is % alcohol.
Ans : 15%
27. A worker is paid Rs.20/- for a full days work. He works 1,1/3,2/3,1/8.3/4 days in a week. What is the total amount paid for that worker ?
Ans : 57.50
28. If the value of x lies between 0 & 1 which of the following is the largest?
(a) x
(b) x2
(c) -x
(d) 1/x

Ans : (d)

DATA SUFFICIENCY SECTION
Directions : For questions in this section mark

(a) If condition (i) alone is sufficient
(b) If condition (ii) alone is sufficient
(c) If both conditions together are sufficient
(d) If condition (i) alone & (ii) alone are sufficient
(e) information not sufficient
1. A man 6 feet tall is standing near a light on the top of a pole What is the length of the shadow cast by the man.
(i) The pole is 18 feet high
(ii) The man is 12 feet from the pole
Ans: (c)
2. Two pipes A and B emptied into a reservoir , pipe A can fill the reservoir in 30 minutes by itself. How long it will take for pipe A and pipe B together to fill up the reservoir.
(i) By itself, pipe B can fill up the reservoir in 20 minutes
(ii) Pipe B has a larger cross-sectional area than pipe A
Ans: (a)
3. K is an integer. Is K is divisible by 12
(i) K is divisible by 4
(ii) K is divisible by 3
Ans: (c)

4. What is the distance from A to B
(i) A is 15 miles from C
(2) C is 25 miles from B
Ans: (e)
5. Was Melissa Brown's novel published?
(i). If Melissa Brown's novel was published she would receive atleast $1000 in royalities during 1978
(ii). Melissa Brown's income for 1978 was over $1000
Ans: (e)
6. Does every bird fly?
(i) Tigers do not fly.
(ii) Ostriches do not fly
Ans: (b)

7. How much does John weigh? Jim weighs 200 pounds.
(i) Toms weight plus Moes weight equal to John's weight.
(ii) John's weight plus Moe's weight equal to Twice Tom's weight.
Ans: (c)


8. Is the figure ABCD is a rectangle if
(i) angle ABC=90(degrees)
(ii) AB=CD

9. Find x+2y
(i). x+y=10
(ii). 2x+4y=20
Ans: (b)


10. Is angle BAC is a right angle
(i) AB=2BC
(2) BC=1.5AC
Ans: (e)
11. Is x greater than y
(i) x=2k
(ii) k=2y
Ans: (e)
12. A piece of string 6 feet long is cut into three smaller pieces. How long is the longest of the three pieces?
(i). Two pieces are the same length.
(ii) One piece is 3 feet 2 inches lone
Ans: (b)
13. How many rolls of wall paper are necessary to cover the walls of a room whose floor and ceiling are rectangles 12 feet wide and 15 feet long
(i) A roll of paper covers 20 sq feet
(ii) There are no windows in the walls
Ans: (e)


14. x and y are integers that are both less than 10. Is x>y?
(i). x is a multiple of 3
(ii). y is a multiple of 2
Ans: (e)
15. Fifty students have signed up for atleast one of the courses GERMAN & ENGLISH, how many of the 50 students are taking GERMANI but not ENGLISH?
(i). 16 students are taking GERMANI & ENGLISH
(ii). The number of students taking ENGLISH but not GERMANI is the same as the number of students taking GERMAN
Ans: (c)
16. Is ABCD is a square ?
A B
X




C D
(i) AD = AB
(ii). x=90(degres)
Ans: (e)
17. How much card board will it take to make a rectangular box with a lid whose base has length 7 inches.
(i). The width of the box 5 inches
(ii). The height of the box will be 4 inches
Ans: (c)
.
18. Did ABC company made profit in 1980?
(i) ABC company made a profit in 1979.
(ii) ABC company made a profit in 1981.
Ans: (e)
19. How much is Janes salary?
(i). Janes salary is 70% of John's salary
(ii). Johns salary is 50% of Mary's salary
Ans: (e)
20. Is x>1
(i) x+y=2
(ii) y<0
Ans: (c)
21. How many of the numbers, x and y are positive? Both x and y are less than 20.
(i) x is less than 5
(ii) x+y =24
Ans: (b)
22. Is the angle ACB is right angle
(1) AC=CB
(2). (AC)2+CB2=AB2
Ans: (b)
23. How far it from town A to town B? Town C is 12 miles east of town A
(i). Town C is south of town B
(ii). It is 9 miles from town B to town C
Ans: (c)
24. A rectangular field is 40 yards long. Find the area of the field.
(i). A fence around the boundary of the field is 140 yards long
(ii). The field is more than 20 yards width
Ans: (a)
25. An industrial plant produces bottles. In 1961 the number of bottles produced by the plant was twice the number of produced in 1960. How many bottles were produced altogether in the year 1960, 61,&62
(i). In 1962 the number of bottles produced was 3 times the number of produced in 1980
(ii). In 1963 the number of bottles produced was one half the total produced in the years 1960,1961,1962.
Ans: (e)
26. Is xy > 1 ? If x & y are both positive
(i) x is less than 1
(ii) y is greater than 1
Ans: (e)
27. Is it a Rhombus
(i) All four sides are equal
(ii) Total internal angle is 360
Ans: (e)
28. How many books are in the book shelf
(i) The book shelf is 12 feet long
(ii). The average weight of each book is 1.2 pound
Ans: (e)
29. What is the area of the circle?
(i) Radius r is given
(ii) Perimeter is 3 times the area
Ans: (a)
ARITHMETIC SECTION

1. If the total distance of a journey is 120 km .If one goes by 60 kmph and comes back at 40kmph what is the average speed during the journey?
Ans: 48kmph
2. A school has 30% students from Maharashtra .Out of these 20% are Bombey students. Find the total percentage of Bombay?
Ans: 6%
3. An equilateral triangle of sides 3 inch each is given. How many equilateral triangles of side 1 inch can be formed from it?
Ans: 9
4. If A/B = 3/5,then 15A = ?
Ans : 9B
5. Each side of a rectangle is increased by 100% .By what percentage does the area increase?
Ans : 300%
6. Perimeter of the back wheel = 9 feet, front wheel = 7 feet on a certain distance, the front wheel gets 10 revolutions more than the back wheel .What is the distance?
Ans : 315 feet.
7. Perimeter of front wheel =30, back wheel = 20. If front wheel revolves 240 times. How many revolutions will the back wheel take?
Ans: 360 times
8. 20% of a 6 litre solution and 60% of 4 litre solution are mixed. What percentage of the mixture of solution
Ans: 36%
9. City A's population is 68000, decreasing at a rate of 80 people per year. City B having population 42000 is increasing at a rate of 120 people per year. In how many years both the cities will have same population?
Ans: 130 years
10. Two cars are 15 kms apart. One is turning at a speed of 50kmph and the other at 40kmph . How much time will it take for the two cars to meet?
Ans: 3/2 hours
11. A person wants to buy 3 paise and 5 paise stamps costing exactly one rupee. If he buys which of the following number of stamps he won't able to buy 3 paise stamps.
Ans: 9
12. There are 12 boys and 15 girls, How many different dancing groups can be formed with 2 boys and 3 girls.

13. Which of the following fractions is less than 1/3
(a) 22/62
(b) 15/46
(c) 2/3
(d) 1
Ans: (b)
14. There are two circles, one circle is inscribed and another circle is circumscribed over a square. What is the ratio of area of inner to outer circle?
Ans: 1 : 2
Directions for questions 15-17: The questions are based on the information given below
Miss Dean wants to rennovate her house. She hires a plumber, a carpenter, a painter, an electrician and an interior decorator. The work to be finished in one working (Monday - Friday ).
Each worker will take the full day to do his job. Miss Dean permits only one person to work each day.
I. The painter can work only after the plumber and the carpenter have finished their jobs
II. The interior decorator must do his job before the electrician.
III. The carpenter cannot work on Monday or Tuesday
15. If the painter work on Thursday, which one of the following alternatives is possible?
(a) The electrician works on Tuesday.
(b). The electrician works on Friday.
(c) The interior decorator works after the painter does.
(d). The painter works on consecutive days.
(e). Miss Dean cannot fit all of the workers int schedule
Ans: (b)
16. If the painter works on Friday which of the following must be false?
(a) . The carpenter may works on Wednesday
(b). The carpenter and the electrician may work on consecutive days
(c). If the carpenter works on Thursday, the electrician has to work on Wednesday
(d). The plumber may work before the electrician does
(e). The electrician may work on Tuesday
Ans: (c)

17. Which argument is possible?

(a). The electrician will works on Tuesday and the interior decorator on Friday
(b). The painter will work on wednesday and plumber on thursday
(c). The carpenter will works on Tuesday and the painter on Friday
(d). THe painter will work on Monday and the carpenter on Thursday
(e). The carpenter will work on Wednesday and the plumber on Thursday
Ans: (e)
CTS SYSTEM
:
The written tests are based on critical reasoning type questions. Word -based problems, verbal ability, pattern recognition and pattern matching, series type, arithematic-based (including functions and permutations) are usually asked.
We are giving some sample tests with questions based on the pattern mentioned above to give you a general idea.
SECTION-1:

Find the next in the sequence:

1. BC CE EG GK ?

a)KN
b)KU
c)KM
d)None

2. AA AB BC CE?

a)EG
b)EH
c)EI
d)None

3. AB EF JK QR ?

a)YZ
b)ZA
c)AB
d)None

4.ACD EGL IKT MOB?

a)QST
b)QSZ
c)QSY
d)None

5.AC CG GO OE?

a)EJ
b)EI
c)EL
d)None

6.AE BH CM DU?

a)EH
b)EZ
c) EB
d)None

7. AD DP PL LV

a)VS
b)VK
c)VI
d)None

8. SE QU EN TI?

a)CN
b)BM
c)AI or AZ
d)None


SECTION-II:

Find the values for the following problem:
f(X)= 2X-1 + f(X-1) if X is not equal to zeroandiff(X=0)=0
9. Value of f(5)

a)15
b)24
c)22
d)None

10. Value of f(f(2))

11. Value of f(16)- f(15)

12. Value of f(16)+f(15)-480

13. If f(f(X))=81 then the value of X=?

14. If f(X)=4f(X-1) then the value of X=?

15.If f(X)= f(X-1)+f(X-2) for X>1 then X= ?

16. If f(X)-f(X-1)=f(X-8) for X>5 then X=



SECTION -III:

In the following questions a 'word' is given which may not have any meaning.Find differnet possible words or palindromes for the word as per the question.

For the following find no of distinct words that can be formed.

17. TYGHHTT

a).420
b)1540
c)840
d)None

18. TYGHHTY

19. TYGHHTT
20. TYGHHTT
21. TYGHASD
22. TYGHHTY

Find the number of possible palindromes for following

23. TYGHHTY
24. TYHHHTYH.
SECTION-IV:

25 to 32 are based on the figures. You have to analyse them and find the odd one out.
Five figures will be given out of which one is not correct.
Refer R.S Agarwal's book on Analytical Reasoning & TMHs Quantitative ability book by Edgar Thorpe.
SECTION -V:

For following first find out the anagram and then note the corresponding meaning.

33.TABLET
Hint: anagram means first u arrange the letters in correct order like TABLET===BATTLE . So ans is FIGHT

34.RUGGED
35.GORE.
36.STASSI.
For all above choices are.

a)resentment
b)fight
c)help
d)monster

37. ENFOLD
38. LAMB
39. RECEDE.
40. PLEASE.

For all the above 4 choices are same

a)cuddle
b)sleeping
c)proclamination
d)ointment.
________________________________________
This is only a sample paper. We are not providing you with all the questions - just some questions to give you a general idea of the test pattern.

SECTION I - 8 questions based on series.

1. These questions involve interchange of letters in a word at particular locations and also interchanging letters adjacent to those particular locations.Certain other conditions may also be given
For eg.
Let the word be ABBAABA
If we apply 25 on this, it means we have to interchange the letters at positions 2 and 5, also we have to change the letters adjacent to positions 2 and 5 i.e.from A to B and B to A.
A B B A A B after Step 1 i.e interchange of 2 and 5 becomes AABABB
Now change adjacent elements of 2 and 5...finally answer becomes
Ans: B A A B B A

Questions 1-5 are based on the pattern with changed numbers as described above
Questions 6-8 are of the following type
To get AAABBD from BBBAAA what number should be applied:-
a) 25
b) 34
c) 25 & 34
d) none

SECTION II

1. Given the following functions
(1) f(n a b c ) = ac if n=1
(2) f(n a b c) = f( n-1 a c b) + f( 1 a b c) + f( n-1 b a c ) if n > 1

Then what is the value f( 2 a b c ) = ?

Ans: f( 2 a c b ) = ab + ac + bc.

2. Similar question on functions.

3. [ Based on the function in the first question] For the function f( 4 a b c ) the number of terms is...?
Hint f( 4 a b c ) = f( 3 a c b ) + f( 1 a b c ) + f( 3 b a c ) etc.


4. What is the value of the function f( 5 a b c ) = ?

SECTION III

Permutations and Combinations.
8 Questions.

1. r = number of flags;n = number of poles;
Any number of flags can be accommodated on any single pole.

1)r=5,n=5 The no. of ways the flags can be arranged ?

Questions 2-5 are based on the above pattern

6. r = 5 n = 3 . If first pole has 2 flags, third pole has 1 flag
How many ways can the remaining be arranged?

Questions 7.& 8. are similar to Question 6.

SECTION IV
Question consisting of figures - Pattern-matching type.
Refer R.S Agarwal's book on Analytical Reasoning & TMHs Quantitative ability book by Edgar Thorpe.



SECTION V
In this section first part of compound word is given. Select meaning of the second part from the choice given:
1. Swan
2. Swans
3. Fool
4. Fools
5. Stare
6. Lady
For all above 4 choices are given.....

Eg. Swan ---> Swansong (compound word)
a) category b) music c) television d) none
Ans: Swansong is compound word. But song is not given as an option. so (b) music is the answer.
Analogies

1. slur : speech : : smudge :?
Ans. writing
2. epaulet : shoulder : : ring :?
Ans.finger
3. vernacular : place : : fingerprint : ?
Ans.identical

Opposites

Q. corpulent
Ans: emaciated
Q. officious
Ans: pragmate
Q. dextrous
Ans: clumsy

The following sentences are broken into 4 sections- A, B, C, D
Choose the part which has a mistake
Mark (E) if you find no mistake.
Q.A)psychologists point out that B)there are human processes C)which does not involve D) the use of words
Ans. (C) which does not involve (do)

Q.A)jack ordered for B)two plates of chicken C)and a glass D)of water
Ans. (A)jack ordered for

The following is a group of questions is based on a passage or a set of conditions for each question.
Select the best answer choice given.
(i). If it is fobidden by law if the object of agreement is the doing of an act,
that is forbidden by law the agreement is void.
(ii). If it is of the nature that,it would defeat the provision of any law
is the agreement is void.if the object of agreement is such that thing got
directly forbidden by law it would defeat the provision of statuary law.
(iii). If the object of agreement is fraddulent it is void.
(iv). An object of agreement is void if it involves or implies to the personnal property of another.
(v). An object of agreement is void where the constant regards as ignored.
(vi). An object of agreement is void where the constant regards is as opposed to public policy.
Q. An algorithm follws a six step process za,zb,zc,zd,ze,zf, it is governed by the following
(i) zd should follow ze
(ii) the first may be za,zd or zf
(iii) zb and zc have to be performed after zd
(iv) zc must be immediately after zb

Q. If za is the first set zd must be
a) 3rd
b) 5th
c) 2nd
d) 4th

Q. If zb must follow za then za can be
a) third or fourth
b) first or second
c) can not be third
d) fouth or fifth
e) none

Q. If ze is third term the number of different operations possible are
The following questions are based on the given statements
Ravi plants six seperate saplings -- x,y,z,w,u,v in rows no 1 to 6 ,according to the follwing conditions
He must plant x before y and u
He must plant y and w
The third has to be z
Q. Which of the following is acceptable
a) xuywzv
b) xvzyuw
c) zuyxwv
d) zvxuwy
e) wyzuvx
Q. Which of the following is true
a) z before v
b) z before x
c) w before u
d) y before u
e) x before w
Q. If he plants v first, then which can be planted second
a) x
b) y
c) z
d) w
e) u

Q. Which of the following describes a correct combination of sapling and row?
a) x,3
b) y,6
c) z,1
d) w,2
e) u,6

Q. If he plants b 6th which would be planted first and second
a) x and w
b) x and y
c)y and x
d)w and z
e) w and u

Q. If he plants w before u and after v he should plant w at
a) first
b) second
c) fourth
d) fifth
e) sixth

Q. At a certain moment a watch shows 2 min lag although it is running fast.
If it showed a 3 min lag at that moment, but also gains by 1/2 min more a day than its current speed
it would show the true time one day sooner than it usually does.
How many mins does the watch gain per day.
a).2
b).5
c).6
d).4
e).75
Q. In 400m race A gives B a start of 7 sec and beats him by 24 sec.
In another race A beats B by 10 sec.the speeds are in the ratio
a)8:7
b)7:6
c)10:8
d)6:8
e)12:10

Q. 3x+4y=10
x3 + y3=6
What is the minimum value of 3x+11y=?

Q. There are 600 tennis players
4% wear wrist band on one wrist
Of the remaining, 25% wear wrist bands on both hands
How many players don't wear a wrist band?
Ans. 432

Q. Three types of tea the a,b,c costs Rs. 95/kg,100/kg and70/kg respectively.
How many kgs of each should be blended to produce 100 kg of mixture worth Rs.90/kg,
given that the quntities of band c are equal
a)70,15,15
b)50,25,25
c)60,20,20
d)40,30,30
Ans. (b)

Q. Two distinct no's are taken from 1,2,3,4......28
Find the probability that their sum is less than 13
DE SHAW AND CO.
:
The paper is technical based - with a major emphasis on C. At the moment we are providing you with a few questions so that you have an idea of the paper and know what to expect.

SECTION-A

Write the programs for the following problems in C.

1. Swap two variables x,y without using a temporary variable.

2. Write algorithm for finding the GCD of a number.

3.Write a program for reversing the given string.

4. The integers from 1 to n are stored in an array in a random
fashion. but one integer is missing. Write a program to find the
missing integer.
Ans). Hint : The sum of n natural numbers is = n(n+1)/2.
if we subtract the above sum from the sum of all the
numbers in the array , the result is nothing but the
missing number.

5. Some bit type of questions has been given on pointers asking to
to find whether it is correct from syntax point of view. and if
it is correct explain what it will do.(around 15 bits).


SECTION-B


6. For the following C program
#define AND &&
#define ARRANGE (a>25 AND a<50)
main()
{int a = 30;
if (ARRANGE)
printf("within range");
else
printf("out of range");
}
What is the output?

7. For the following C program
#define AREA(x)(3.14*x*x)
main()
{floatr1=6.25,r2=2.5,a;
a=AREA(r1);
printf("\n Area of the circle is %f", a);
a=AREA(r2);
printf("\n Area of the circle is %f", a);
}
What is the output?
Ans. Area of the circle is 122.656250
Area of the circle is 19.625000

8. What do the following statements indicate. Explain.
• int(*p)[10]
• int*f()
• int(*pf)()
• int*p[10]
Refer to:
-- Kernighan & Ritchie page no. 122
-- Schaum series page no. 323
9. Write a C program to find whether a stack is progressing in forward
or reverse direction.
10. Write a C program that reverses the linked list.
DEUTSCHE S/W INDIA PVT LTD
:
The written test is purely an aptitude test.It is a one hour test with 50 questions.The test checks your basic mathematics and logical ability.
1. What is the percentage represented by 0.03 * 0.05 ?
(a)0.0015
(b)0.000015
(c)0.15
(d)15
Ans.B

2. (x-a)(x-b)(x-c)....(x-z) = ?
(a) 1
(b) -1
(c) 0
(d) Can't be determined
Ans. C

3. If a = 1, b = 2, c = 3.......z = 26 what is the value of p+q+r ?
(a)33
(b)51
(c)52
(d)48
Ans. B

4. A is 8 miles east of B.
C is 10 miles north of B.
D is 13 miles east of C and E is 2 miles north of D.
Find shortest distance between A and E.
(a) 5 miles
(b) 6miles
(c) 13 miles
(d) 18 miles
Ans. C

5. If z = 1, y = 2.......a = 26. Find the value of z + y + x + .......+a.
(a) 351
(b) 221
(c) 400
(d) 200
Ans. A

6. There are 30 socks in a bag.
Out of these 60 % are green and the rest are blue.
What is the maximum number of times that socks have to be taken out so that atleast 1 blue pair is found.
(a) 21
(b) 2
(c)18
(d) 20
Ans. D

7. How many two digit numbers have their square ending with 8.
(a) 13
(b) 12
(c) 0
(d) 11
Ans. C

8. How many numbers are there between 100 and 300 with 2 in the end and 2 in the beginning.
(a) 10
(b) 9
(c) 11
(d) none of these
Ans. A

9. 0.000006 * 0.0000007 = ?
(a) 0.0000000042
(b) 0.000000000042
(c) 0.0000000000042
(d) 0.00000000000042
Ans. B

10. You have Rs 1000 with 8% p.a compounded every 6 months.
What is the total interest you get after 1 year.
(a) Rs.116.40
(b) Rs.345.60
(c) Rs.224.50
(d) Rs.160
Ans. A

11. If x + y =12,
x - y = 2
Find x + 2y.
(a) 12
(b) 17
(c) 14
(d) none of these
Ans. B

12. With one gallon of petrol a person moves at a speed of 50 mph and covers 16 miles.
3/4th of the distance is covered while moving at 60 mph.
How many gallons does he need to cover 120 miles in 60 mph.

13. A tap drains at x speed while tap B is closed.
When both taps are open they drain at y speed.
What is the speed of draining when only tap B is open
(a) x - y
(b) y-x
(c) x
(d) can't be determined
Ans. B

14. What is twenty percent of 25 % of 20.
(a) 2
(b)1
(c) 5
(d) 4
Ans. B

15. A rectangle has the dimensions 6ft * 4ft.
How many squares of 0.5 inches will it need to completely fill it.
(a) 32000
(b) 12824
(c) 13824
(d) 18324
Ans. C

Directions for questions 16-21: In each question,a series of letters satisfying a certain pattern are given. Identify the pattern and then find the letter/letters that will come in place of the blank/blanks.
16. a, c, e, g, _
(a) h
(b) i
(c) d
(d) j
Ans. B

17. a, e, i, m, q, u, _, _
(a) y, c
(b) b, f
(c) g, i
(d) none
Ans. A

18. ay , bz , cw , dx ,__
(a) gu
(b) ev
(c) fv
(d) eu
Ans. D

19. 1, 2, 3, 5, 7, 11, __
(a) 15
(b) 9
(c) 13
(d) 12
Ans. 13 , series of prime numbers

20. kp , lo , mn , __
(a) nm
(b) np
(c) op
(d) pq
Ans. A

21. abc , zyx , def , wvu , ___
(a) ghi
(b) tsr
(c) ihg
(d) str
Ans. A

22. How is my mother's sister's brother's wife's child related to me?
(a) brother
(b) uncle
(c) cousin
(d) nephew
Ans. A

23.What will my mother's husband's father-in-law's son's daughter to me?
(a) niece
(b) aunt
(c) sister
(d) none of these
Ans. D




SECTION 1- APTITUDE SECTION
Directions for questions 1-3: Complete the sequence given below
1. 5, 5, 13, 13, 21, 21, __

Ans: 29
2. 0, 7, 26, 63, 124, __

Ans: 215 ( hint: n3-1 )
3. 1, 3, 5, 7, __
Ans: 9

4. If a person walks at 4/5th of his usual speed he reaches 40min late. If he walks at his usual speed for how much time does he travel ?
5. Two trains A&B start at opposite points 120km apart at 60kmph. A fly starting along with train A at 120kmph reaches B then returns back to touch A and continues the two and fro movement. By the time two trains meet how much distance would the fly have travelled?
6. In a class 80% have passed in english, 70% passed Hindi, 10% didn't pass either. If 144 students passed both the subjects. What is the total strength of the class?

7. Find the least number which when divided by 7 gives the reminder 6, when divided by 6 gives reminder 5, when divided by 5 gives reminder 4 and so on ?

8. If a man stands in front of sun what is the first letter of the direction which is left to him ?

9. (a)A square is to circle as cube is to
(b)Success is to failure as joy is to

10. (a)Give the synonyms of the following words
(i) Joy
(ii) Inert
(iii) Jolly

(b) Give the opposites of the following words
(i) genuine
(ii) command
(iii) essential
11.Find the odd man out in the following sets
(i) Tiger, Elephant, King Cobra, Dolphin
(ii) Oasis, Lake, Pool, Valcano
(iii) Bengali, Karnataka, Mumbai, Kashmir
(iv) Lapidary, Lancet, Scapel, Surgeon
(v) Requiem, Dirge, Elegy, Paean
12. I bought a cycle 2days before my birthday and I broke it 3 days after my birthday. The day I broke it is Mar2,1956?

Directions: The following questions are to be answered on the basis of the above given statement
i) When is my birthday?
Hint: Keep in mind that 1956 was a leap year.
(ii) What is my age on Mar 4th, 1980?
(iii) My nephew is born exactly 20years after me. If I turned 20 in 1960, what is the nephews age on Feb 28th 1988 ?

13. Monday Aug25, 96 :
Hostess: "Mr A, you forgot your umbrella during the party on last friday. I expected you to collect it on your visit on wednesday as I plan to leave on this Friday."
Directions: The following questions are to be answered on the basis of the above given statement
(i) when A missed umbrella?
(ii)When A is supposed to collect it?
(iii)When K leaves?

14.What is my father's sons son to my son?

Ans. Cousin brother
15. On cutting a solid parabola what would be generated

Ans: Cone

16. What is Eulers formula?

Ans: F+V-E =2; where
F => faces; V => vertices; E => number of edges
17. What is Newton Raphson method used for?

Ans: To find the root of f(x) = 0;

18. How many tangents can be drawn on three circles if they don't lie within each other ?

19. xy - x + 2y = 6 equation is shifted to form equation xy=c what is c ?

20.When x is real what is the least value of (x2-6x+5)/(x2+2x+1)

21.When an object like cube or sphere is seen along x, y, z axis we get the same.Apart from these suggest another object which has similar characteristics as that mentioned above?
Ans: Triangular prism
22. When an object is seen from the front side we can see two concentric squares and top view also without any hidden lines. Draw the side view.
23. In common parlance, A=> B means what
Ans: if A is true B has to be true
23. If A is not invertible and BA = I is not possible, what is implied by this?
Ans: Determinant is Zero.
24. What is a free body diagram used for

25. A die is thrown twice what is the probability that you get same number


26. The sum of two numbers is 55. What is the larger number?


1. a=2, b=3, c=6
Find the value of c/(a+b)-(a+b)/c

2. What does the hexanumber E78 in radix 7.
(a) 12455
(b) 14153
(c) 14256
(d) 13541
(e) 131112
Ans. (d)

3. 10 : 4 seconds :: ? : 6 minutes
Ans. 900

4. Q is not equal to zero and k = (Q x n - s)/2.What is n?
(a) (2 x k + s)/Q
(b) (2 x s x k)/Q
(c) (2 x k - s)/Q
(d) (2 x k + s x Q)/Q
(e) (k + s)/Q

5. From the following statements determing the order of ranking
• M has double the amount as D
• Y has 3 rupess more than half the amount of D
Ans. Data insuffiecient

Questions 6 - 10 are to be answered on the following data
• A causes B or C, but not both
• F occurs only if B occurs
• D occurs if B or C occurs
• E occurs only if C occurs
• J occurs only if E or F occurs
• D causes G,H or both
• H occurs if E occurs
• G occurs if F occurs
6. If A occurs which of the following must occurs
I. F and G
II. E and H
III. D
(a) I only
(b) II only
(c) III only
(d) I,II, & III
(e) I & II (or) II & III but not both
Ans. (e)

7. If B occurs which must occur
(a) D
(b) D and G
(c) G and H
(d) F and G
(e) J
Ans. (a)

8. If J occurs which must have occured
(a) E
(b) either B or C
(c) both E & F
(d) B
(e) both B & C
Ans. (b)

9. Which may occurs as a result of cause not mentioned
I. D
II. A
III. F
(a) I only
(b) II only
(c) I & II
(d) II & III
(e) I,II & III
Ans. (c)
10. E occurs which one cannot occurs
(a) A
(b) F
(c) D
(d) C
(e) J
Ans. (b)




1. At 6'o a clock ticks 6 times.
The time between first and last ticks is 30 seconds.
How long does it tick at 12'o clock.
Ans: 66 sec. (2 marks)

2. Three friends divided some bullets equally.
After all of them shot 4 bullets the total number of bullets remaining is equal to the bullets each had after division.
Find the original number divided.
Ans: 18 (2 marks)
Initially . x x x
Now x-4 x-4 x-4
Equation is 3x-12 = x

3. A ship went on a voyage.
After it had travelled 180 miles a plane statrted with 10 times the speed of the ship.
Find the distance when they meet from starting point.
Ans: 200miles. (2 marks)
Distance travelled by plane = 1/10 distance travelled by ship + 180

5. There are 3 societies A, B, C.
A lent cars to B and C as many as they had already.
After some time B gave as many tractors to A and C
as many as they have. After sometime c did the same thing. At the end of this transaction each one of them had 24.
Find the cars each orginally had.
Ans: A had 39 cars, B had 21 cars & C had 12 cars (4 marks)

6. There N stations on a railroad.
After adding X stations on the rail route 46 additional tickets have to be printed.
Find N and X.
Ans. x=2 and N=11
Let initially, N(N-1) = t
After adding, (N+X)(N+X-1) = t+46
By trail and error method (4 marks)

7. Given that April 1 is tuesday.
A, B, C are 3 persons told that their farewell party was on
• A - May 8, thursday
• B - May 10,tuesday
• C - June 5, friday
Out of A, B, C only one made a completetly true statement concerning date,day and month
The other told two one told the day right and the other the date right..
What is correct date, month, day.
Ans: B - (May 10) SUNDAY
C - June 6 (Friday). (5 marks)

8. The Bulls, Pacers, Lakers and Jazz ran for a contest.
Anup, Sujit, John made the following statements regarding results.
• Anup said either Bulls or Jazz will definitely win
• Sujit said he is confident that Bulls will not win
• John said he is confident that neither Jazz nor Lakers will win
When the result cameit was found that only one of the above three had made a correct statement.
Who has made the correct statement and who has won the contest.
Ans: Sujith; Lakers (5marks )

9. Five people A ,B ,C ,D ,E are related to each other.
Four of them make one true statement each as follows.
(i) B is my father's brother.
(ii) E is my mother-in-law.
(iii)C is my son-in-law's brother
(iv)A is my brother's wife.
Ans: (i) D
(ii) B
(iii) E
(iv) C (10 marks)

10. Some statements are given below:
• L says all of my other four friends have money
• M says that P said that exactly one among them has money
• N says that L said that precisely two among them have money
• O says that M said that three of the others have money
• P, L and N said that they have money
All the above statement are false..
Who has money & who doesn't have any money?

1. Mr.Mathurs jewels have been stolen from his bank locker .
The bank has lockers of 12 people which are arranged in an array of 3 rows and 4 columns like:

• The locker belonging to JONES was to the right of BLACK'S locker and directly above MILLAR'S.
• BOOTH'S locker was directly above MILLAR'S.
• SMITH'S locker was also above GRAY's (though not directly).
• GREEN'S locker was directly below SMITH'S.
• WILSON'S locker was between that of DAVIS and BOOTH.
• MILLAR'S locker was on the bottom row directly to the right of HERD'S.
• WHITE'S locker was on the bottom right hand corner in the same column as BOOTH'S.
Which box belonged to Mr.Mathurs?
Ans: Box number 9 belongs to Mr.Mathurs.

2. Fifty minutes ago if it was four times as many minutes past three o'clock,how many minutes is it to six o'clock?
Ans: Twenty six minutes.

3. If a clock takes 7seconds to strike 7, how long will the same clock take to strike 10?
Ans: The clock strikes for the first time at the start and takes 7 seconds for 6 intervals-thus for one interval time
taken=7/6.
Therefore, for 10 seconds there are 9 intervals and time taken is 9*7/6=10 and 1/2 seconds.

4. Three criminals were arrested for shop lifting.
However, when interrogated only one told the truth in both his statements, while the other two each told one true
statement and one lie.
The statements were:
• ALBERT :(a)Chander passed the merchandise. (b)Bruce created the diversion.
• BRUCE :(a)Albert passed the merchandise. (b)I created the diversion.
• CLIVE :(a)I took the goods out of the shop. (b)Bruce passed them over.
Ans: Albert passed the goods.Bruce created the diversion..Clive took the goods out of the shop.

5. Everyday in his business a merchant had to weigh amounts from 1 kg to 121 kgs, to the nearest kg.
What are the minimum number of weight required and how heavy should they be?
Ans: .The minimum number is 5 and they should weigh 1,3,9,27 and 81 kgs.

6. A hotel has 10 storeys.Which floor is above the floor below the floor, below the floor above the floor, below the
floor above the fifth.
Ans: The sixth floor.

7. Seven members sat around a table for three days for a conference.
The member's names were Abhishek, Amol, Ankur, Anurag,Bhuwan ,Vasu and Vikram.
The meetings were chaired by Vikram.
On the first evening members sat around the table alphabetically.
On the following two nights, Vikram arranged the seatings so that he could have Abhishek as near to him as
possible and abesent minded Vasu as far away as he could.
On no evening did any person have sitting next to him a person who had previously been his neighbour.
How did Vikram manage to seat everybody to the best advantage on the second and third evenings?
Ans:
Second evening:Vikram,Ankur,Abhishek,Amol,Vasu,Anurag and Bhuwan.
Third evening :Vikram,Anurag,Abhishek,Vasu,Bhuwan,Ankur,Amol.

8. Two trains start from stations A and B spaced 50 kms apart at the same time and speed.
As the trains start, a bird flies from one train towards the other and on reaching the second train, it flies back to the
first train.This is repeated till the trains collide.
If the speed of the trains is 25 km/h and that of the bird is 100km/h.
How much did the bird travel till the collision.
Ans: 100 kms.

9. Four prisoners escape from a prison.
The prisoners, Mr East, Mr West, Mr South, Mr North head towards different directions after escaping.
The following information of their escape was supplied:
• The escape routes were The North Road, South Road, East Road and West Road.
• None of the prisoners took the road which was their namesake.
• Mr.East did not take the South Road
• Mr.West did not the South Road.
• The West Road was not taken by Mr.East
What road did each of the prisoners take to make their escape?
Ans: Mr.East took the North Road
Mr.West took the East Road
Mr.North took the South Road
Mr.South took the West Road.

10. Complete the series:
5, 20, 24, 6, 2, 8, ?
Ans: 12 (as 5*4=20, 20+4=24, 24/4=6, 6-4=2, 2*4=8, 8+4=12).
SET C
1) A man collects cigarette stubs and makes one full cigarette with every 8 stubs.
If he gets 64 stubs how many full cigarettes can he smoke.

Ans: 8+1=9


2) A soldier looses his way in a thick jungle. At random he walks from his camp but mathematically in an interesting fashion. First he walks one mile East then half mile to North. Then 1/4 mile to West, then 1/8 mile to South and so on making a loop.
Finally how far he is from his camp and in which direction.

Ans: Distance travelled in north and south directions
1/2 - 1/8 + 1/32 - 1/128 + 1/512 - and so on
= 1/2/((1-(-1/4))
Similarly in east and west directions
1- 1/4 + 1/16 - 1/64 + 1/256 - and so on
= 1/(( 1- ( - 1/4))
Add both the answers

3) How can 1000000000 be written as a product of two factors neither of them containing zeros

Ans: 2 power 9 x 5 power 9

4) Conversation between two mathematcians:
First : I have three childern. The product of their ages is 36.
If you sum their ages, it is exactly same as my neighbour's door number on my left.
The second mathematician verfies the door number and says that it is not sufficient.
Then the first says " Ok one more clue is that my youngest is really the youngest". Immmediately the second mathematician answers .
Can you answer the question asked by the first mathematician?
What are the childeren ages?

Ans 1,6 and 6

5) Light glows for every 13 seconds . How many times did it glow between 1:57:58 and 3:20:47 am.

Ans : 383 + 1 = 384

6) 500 men are arranged in an array of 10 rows and 50 columns according to their heights.
Tallest among each row of all are asked to fall out.
And the shortest among them is A.
Similarly after resuming that to their original podsitions that the shortest among each column are asked to fall out.
And the tallest among them is B .
Now who is taller among A and B ?

Ans A


7) A person with some money spends1/3 for cloths, 1/5 of the remaining for food and 1/4 of the remaining for travel.
He is left with Rs 100/- .
How much did he have with him in the begining ?

Ans: Rs 250/-

8) There are six boxes containing 5 , 7 , 14 , 16 , 18 , 29 balls of either red or blue in colour.
Some boxes contain only red balls and others contain only blue.
One sales man sold one box out of them and then he says
" I have the same number of red balls left out as that of blue ".
Which box is the one he solds out ?

Ans: Total no of balls = 89 and (89-29 /2) = 60/2 = 30
and also 14 + 16 = 5 + 7 + 18 = 30

9) A chain is broken into three pieces of equal lenths containing 3 links each.
It is taken to a backsmith to join into a single continuous one .
How many links are to to be opened to make it ?

Ans : 2.

10) Grass in lawn grows equally thick and in a uniform rate.
It takes 24 days for 70 cows and 60 days for 30 cows to eat the whole of the grass.
How many cows are needed to eat the grass in 96 days.?

Ans : 20
g - grass at the beginning
r - rate at which grass grows, per day
y - rate at which one cow eats grass, per day
n - no of cows to eat the grass in 96 days
g + 24*r = 70 * 24 * y
g + 60*r = 30 * 60 * y
g + 96*r = n * 96 * y
Solving, n = 20.

Section B
1. Replace each letter by a digit.
Each letter must be represented by the same digit and no beginning letter of a word can be 0.

O N E
O N E
O N E
O N E
-------
T E N
Ans: 0 =1, N = 8 ,E = 2, T = 7

2. Ann, Boobie, Cathy and Dave are at their monthly business meeting.
Their occupations are author, biologist, chemist and doctor, but not necessarily in that order.
Dave just told the biologist that Cathy was on her way with doughnuts.
Ann is sitting across from the doctor and next to the chemist.
The doctor was thinking that Boobie was a goofy name for parent's to choose,but didn't say anything.
What is each person's occupation?
Ans: Since Dave spoke to the biologist and Ann sat next to the chemist and across the doctor, Cathy must be the author
and Ann the biologist.
The doctor didn't speak, but David did, so Bobbie is the doctor and Dave the chemist.

3. Sometime after 10:00 PM a murder took place.
A witness claimed that the clock must have stopped at the time of the shooting.
It was later found that the postion of both the hands were the same but their positions had interchanged.
Tell the time of the shooting (both actual and claimed).
Ans: Time of shooting = 11:54 PM
Claimed Time = 10:59 PM

4. Next number in the series is
1 , 2 , 4 , 13 , 31 , 112 , ?
Ans: 224.
No number has digits more than 4. All of them are 1 , 2, 4, 8 , 16 , 32 , 64 converted to numbers in base 5

5. Shahrukh speaks truth only in the morning and lies in the afternoon, whereas Salman speaks truth only in the afternoon. A says that B is Shahrukh. Is it morning or afternoon and who is A - Shahrukh or Salman.
Ans: Afternoon ; A is Salman.

6. Two trains starting at same time, one from Bangalore to Mysore and other in opposite direction arrive at their
destination 1 hr and 4 hours respectively after passing each other. How nuch faster is one train from other?
Ans: Twice

7. There are 6 volumes of books on a rack kept in order ( ie vol.1, vol. 2 and so on ).
Give the position after the following changes were noticed.
• All books have been changed
• Vol.5 was directly to the right of Vol.2
• Vol.4 has Vol.6 to its left and both weren't at Vol.3's place
• Vol.1 has Vol.3 on right and Vol.5 on left
• An even numbered volume is at Vol.5's place
Find the order in which the books are kept now.
Ans: 2 , 5 , 1 , 3 , 6 , 4

8. I bought a car with a peculiar 5 digit numbered licence plate which on reversing could still be read.
On reversing value is increased by 78633.Whats the original number if all digits were different?
Ans: Only 0 1 6 8 and 9 can be readupside down.So on rearranging these digits we get the answer as 10968

9. The shape in the sketch below is that of a square attached to half of a similar square.Divide it into four equal pieces
INCLUDEPICTURE "http://www.freshersworld.com/questions/qimages/ques2_9.gif" \* MERGEFORMATINET
Ans: Hint : the figure can be divided into 12 equal triangles

10. Supposing a clock takes 7 seconds to strike 7. How mlong will it take to strike 10?
Ans: 10 1/2 seconds.

SET D
1. Father's age is three years more than three times the son's age.
After three years, father's age will be ten years more than twice the son's age.
What is the father's present age.
Ans: 33 years. (2 marks)

2. Find the values of each of the alphabets.
N O O N
S O O N
+ M O O N
----------
J U N E
Ans: 9326 (2 marks)

3. There are 20 poles with a constant distance between each pole
A car takes 24 second to reach the 12th pole.
How much will it take to reach the last pole.
Ans: 41.45 seconds (2 marks)
Let the distance between two poles = x
Hence 11x:24::19x:?

4. A car is travelling at a uniform speed.
The driver sees a milestone showing a 2-digit number.
After travelling for an hour the driver sees another milestone with the same digits in reverse order.
After another hour the driver sees another milestone containing the same two digits.
What is the average speed of the driver.

Ans: 45 kmph (4 marks)

5. The minute and the hour hand of a watch meet every 65 minutes.
How much does the watch lose or gain time and by how much?
Ans: Gains; 5/11 minutes (4 marks)

6. Ram, Shyam and Gumnaam are friends.
Ram is a widower and lives alone and his sister takes care of him.
Shyam is a bachelor and his neice cooks his food and looks after his house.
Gumnaam is married to Gita and lives in large house in the same town.
Gita gives the idea that all of them could stay together in the house and share monthly expenses equally.
During their first month of living together, each person contributed Rs.25.
At the end of the month, it was found that Rs 92 was the expense so the remaining amount was distributed equally
among everyone.
The distribution was such that everyone recieved a whole number of Rupees.
How much did each person recieve?
Ans. Rs 2 (4 marks)
(Hint: Ram's sister, Shyam's neice and Gumnaam's wife are the same person)

7. Four persons A, B, C and D are playing cards.
Each person has one card, laid down on the table below him, which has two different colours on either side.
The colours visible on the table are Red, Green, Red and Blue.
They see the color on the reverse side and give the following comment.
A: Yellow or Green
B: Neither Blue nor Green
C: Blue or Yellow
D: Blue or Yellow
Given that out of the 4 people 2 always lie find out the colours on the cards each person.
Section B
1. From a vessel, 1/3rd of the liquid evaporates on the first day.
On the second day 3/4th of the remaining liquid evaporates.
What fraction of the volume is present at the end of the second day.
Ans: 50%
2. An orange glass has orange juice and white glass has apple juice both of equal volumes.
50ml of the orange juice is taken and poured into the apple juice.
50ml from the white glass is poured into the orange glass.
Of the two quantities, the amount of apple juice in the orange glass and the amount of orange juice in the white glass, which one is greater and by how much?

Ans: The two quantities are equal
3. There is a 4 inch cube painted on all sides.
This is cut down into of 1 inch cubes.
What is the no of cubes which have no pointed sides.

Ans: 8
4. Sam and Mala have a conversation.
• Sam says I am certainly not over 40
• Mala says I am 38 and you are atleast 5 years older than me
• Now Sam says you are atleast 39
All the statements by the two are false.
How old are they really?

Ans: Mala = 38 yrs
Sam = 41 yrs.

5. Ram Singh goes to his office in the city, every day from his suburban house.
His driver Gangaram drops him at the railway station in the morning and picks him up in the evening.
Every evening Ram Singh reaches the station at 5 O' Clock.
Gangaram also reaches at the same time.
One day Ram Singh started early from his office and came to the station at 4 O' Clock.
Not wanting to wait for the car he starts walking home. Mangaram starts at normal time, picks him up on the way
and takes him back house, half an hour early.
How much time did Ram Singh walk?
6. In a railway station, there are two trains going.
One in the harbour line and one in the main line, each having a frequency of 10 minutes.
The main line service starts at 5 o'clock and the harbour line starts at 5.02A.M.
A man goes to the station every day to catch the first train that comes.
What is the probability of the man catching the first train?
Ans: 0.8
7. A family X went for a vacation.
Unfortunately it rained for 13 days when they were there.
But whenever it rained in the mornings, they had clear afternoons and vice versa.
In all they enjoyed 11 mornings and 12 afternoons.
How many days did they stay there totally?
Ans: 18
8. A survey was taken among 100 people to find their preference of watching T.V. programmes.
There are 3 channels. Given the no of people who watch
• at least channel 1
• at least channel 2
• at least channel 3
• no channels at all
• atleast channels 1and 3
• atleast channels 1 and 2
• atleast channels 2 and 3
Find the no of people who watched all three.
9. Albert and Fernandes have two leg swimming race.
Both start from opposite ends of the pool.
On the first leg, the boys pass each other at 18 m from the deep end of the pool.
During the second leg they pass at 10 m from the shallow end of the pool.
Both go at constant speed but one of them is faster.
Each boy rests for 4 seconds at the end of the first leg.
What is the length of the pool?
10. Each alphabet stands for one digit in the following multiplication.
T H I S
x I S
---------
X F X X
X X U X
------------
X X N X X
------------
What is the maximum value T can take?

SET E
1.An escalator is descending at constant speed.
A walks down and takes 50 steps to reach the bottom.
B runs down and takes 90 steps in the same time as A takes 10 steps.
How many steps are visible when the escalator is not operating?

Ans: 150 steps
2. Every day a cyclist meets a train at a particular crossing.
The road is straight before the crossing and both are travelling in the same direction.
The cyclist travels with a speed of 10 Kmph.
One day the cyclist comes late by 25 min. and meets the train 5km before the crossing.
What is the speed of the train?

Ans: 60 kmph
3. There are five persons with surnames Mukherjee, Misra, Iyer, Patil and Sharma.
There are 4 persons having first or middle name of Kumar, 3 persons with Mohan, 2 persons with Dev and 1 Anil.
Either Mukherjee and Patil have a first or middle name of Dev or Misra and Iyer have their first or middle name of Dev.
Of Mukherkjee and Misra, either both of them have a first or middle name of Mohan or neither have a first or
middle name of Mohan.
Either Iyer or Sharma has a first or middle name of Kumar but not both.
Who has the first or middle name of Anil?

Ans: Kumar Misra Dev
Mohan Iyer Dev
Kumar Patil Mohan
Mohan Sharma Kumar
4. Boys are allowed to watch football at C.V.Raman auditorium subjected to conditions.
• The boy over age 16 can wear overcoat
• No boy over age 15 can wear cap
• To watch the football either he has to wear overcoat or cap or both
• A boy with an umberella or above 16 or both cannot wear sweater.
• Boys must either not watch football or wear sweater.
What is the appearence of the boy who is watching football.

5. A bird keeper has got P pigeons, M mynas and S sparrows.
The keeper goes for lunch leaving his assistant to watch the birds.
• Suppose p=10, m=5, s=8 when the bird keeper comes back, the assistant informs the x birds have escaped. The bird keeper exclaims: "Oh no! All my sparrows are gone."
How many birds flew away?
• When the bird keeper comes back, the assistant told him that x birds have escaped. The keeper realised that atleast 2 sparrows have escaped.
What is minimum no of birds that can escape?

6. Answer the following questions based on the conditions from the choices A, B, C, D, E as described below:
(A) if a definete conclusion can be drawn from condition 1
(B) if a definete conclusion can be drawn from condition 2
(C) if a definete conclusion can be drawn from condition 1 and 2
(D) if a definete conclusion can be drawn from condition 1 or 2
(E) no conclusion can be drawn using both conditions
• person 1 says N<5
• person says N>5
• person 3 says 3N>20
• person 4 says 3N>10
• person 5 says N<8
What is the value of N?

a) 1. No of persons who speak false being less than no of persons who tells the truth.
2. Person 2 is telling the truth.

b) 1. no of persong telling the truth is greater than no of persons telling lies
2. person 5 is telling the truth.

7. There are N coins on a table and there are two players A & B.
You can take 1 or 2 coins at a time.
The person who takes the last coin is the loser.
A always starts first.
• If N=7
(a) A can always win by taking two coins in his first chanse
(b) B can win only if A takes two coins in his first chance.
(c) B can always win by proper play
(d) none of the above
• 2. A can win by proper play if N is equal to
(a) 13 (b) 37 (c) 22 (d) 34 (e) 48
Ans: (e.)
• 3. B can win by proper play if N is equal to
(a) 25 (b)26 (c) 32 (d) 41 (e) none
• 4. if N<4, can A win by proper play always?
(a) Yes (b) No

8. Two twins have vertain peculiar charcteristics.
One of them always lies on Monday, Wednesday, Friday.
The other always lies on Tuesdays, Thursdays and Saturdays.
On the other days they tell the truth. You are given a conversation.
• Person A-- today is Sunday my name is Anil
• Person B -- today is Tuesday, my name is Bill
What day is today?
Ans: Today is Tuesday.

9. There is a safe with a 5 digit number as the key.
The 4th digit is 4 greater than second digit, while 3rd digit is 3 less than 2nd digit.
The 1st digit is thrice the last digit.
There are 3 pairs whose sum is 11.
Find the number.
Ans: 65292

10. A hotel has two wings,the east wing and the west wing.
Some east wing rooms but not all have an ocean view.
All west wing rooms have a harbour view.
The charge for all rooms is identical, except as follows :
• Extra charge for all harbour view rooms on or above the 3rd floor
• Extra charge for all ocean view rooms except those without balcony
• Extra charge for some harbour rooms on the first two floor & some east wing rooms without ocean view but
having kitchen facilities.
Which of the following cannot be determined on the basis of the nformation given:
I. Whether there are any rooms without a balcony for which an extra charge is imposed.
II. Whether any room without a kitchen or a view involves an extra charge.
III. Whether two extra charges are impsed for any room.
(A) I only
(B) II only
(C) III only
(D) II and III only
(E) I, II and III
(This question is from 1999 Barrons GRE Guide model Test 3 - section 6, question 22)
Ans: (A)

SET F
1. Three friends divided some bullets equally. After all of them shot 4 bullets the total number of bullets remaining is equal to the bullets each had after division. Find the original number divided.
Ans: 18 ( Initially . x x x
Now x-4 x-4 x-4 ) Equation is 3x-12 = x
2. A ship went on a voyage. After it had traveled 180 miles a plane started with 10 times the speed of the ship.
Find the distance when they meet from starting point.
Ans: 200miles. ( Distance traveled by plane = 1/10 distance traveled by ship + 180 )
3. Replace each letter by a digit. Each letter must be represented by the same digit and no beginning letter of a word can be 0.
O N E
O N E
O N E
O N E
T E N
Ans: O =1, N = 8 ,E = 2, T = 7
4. In a railway station, there are two trains going. One in the harbor line and one in the main line, each having a frequency of 10 minutes. The main line service starts at 5 o'clock and the harbor line starts at 5.02A.M. A man goes to the station every day to catch the first train that comes. What is the probability of the man catching the first train?
Ans: 0.8
5. Next number in the series is: 1 , 2 , 4 , 13 , 31 , 112 , ?
Ans: 224. (No number has digits more than 4. All of them are 1 , 2, 4, 8 , 16 , 32 , 64 converted to base 5 )
6. Father's age is three years more than three times the son's age. After three years, father's age will be ten years more than twice the son's age. What is the father's present age?
Ans: 33 years.
7. Light glows for every 13 seconds . How many times did it glow between 1:57:58 and 3:20:47 am.
Ans : 383 + 1 = 384
8. From a vessel, 1/3rd of the liquid evaporates on the first day. On the second day 3/4th of the remaining liquid evaporates. What fraction of the volume is present at the end of the second day.
Ans: 50%
9. Supposing a clock takes 7 seconds to strike 7. How long will it take to strike 10?
Ans: 10 1/2 seconds.
10. There are 20 poles with a constant distance between each pole. A car takes 24 second to reach the 12th pole.
How much will it take to reach the last pole.
Ans: 41.45 seconds (Let the distance between two poles = x, Hence 11x:24::19x:? )
11. How can 1000000000 be written as a product of two factors neither of them containing zeros
Ans: 2 power 9 x 5 power 9
12. Two trains starting at same time, one from Bangalore to Mysore and other in opposite direction arrive at their
destination 1 hr and 4 hours respectively after passing each other. How much faster is one train from other?
Ans: Twice
13. Every day a cyclist meets a train at a particular crossing. The road is straight before the crossing and both are traveling in the same direction. The cyclist travels with a speed of 10kmph. One day the cyclist comes late by 25 min. and meets the train 5km before the crossing. What is the speed of the train?
Ans: 60kmph
14. A man collects cigarette stubs and makes one full cigarette with every 8 stubs. If he gets 64 stubs how many full cigarettes can he smoke.
Ans: 8+1=9
15. The minute and the hour hand of a watch meet every 65 minutes. How much does the watch lose or gain time and by how much?
Ans: Gains; 5/11 minutes
16. A survey was taken among 100 people to find their preference of watching T. V. programs. There are 3 channels. Given the no of people who watch
• at least channel 1
• at least channel 2
• at least channel 3
• no channels at all
• at least channels 1and 3
• at least channels 1 and 2
• at least channels 2 and 3
Find the no of people who watched all three.
Ans.
17. Two trains start from stations A and B spaced 50kms apart at the same time and speed. As the trains start, a bird flies from one train towards the other and on reaching the second train, it flies back to the first train. This is repeated till the trains collide. If the speed of the trains is 25 km/h and that of the bird is 100km/h. How much did the bird travel till the collision.
Ans: 100kms.
18. Four persons A, B, C and D are playing cards. Each person has one card, laid down on the table below him, which has two different colors on either side. The colors visible on the table are Red, Green, Red and Blue. They see the color on the reverse side and give the following comment.
A: Yellow or Green B: Neither Blue nor Green
C: Blue or Yellow D: Blue or Yellow
Given that out of the 4 people 2 always lie find out the colors on the cards each person.
Ans.
19. Sometime after 10:00 PM a murder took place. A witness claimed that the clock must have stopped at the time of the shooting. It was later found that the position of both the hands were the same but their positions had interchanged.
Tell the time of the shooting (both actual and claimed).
Ans: Time of shooting = 11:54 PM
Claimed Time = 10:59 PM
20. Some statements are given below:
• L says all of my other four friends have money
• M says that P said that exactly one among them has money
• N says that L said that precisely two among them have money
• O says that M said that three of the others have money
• P, L and N said that they have money
All the above statement are false. Who has money & who doesn't have any money?
Ans.
21. The Bulls, Pacers, Lakers and Jazz ran for a contest. Anup, Sujit, John made the following statements regarding results.
• Anup said either Bulls or Jazz will definitely win
• Sujit said he is confident that Bulls will not win
• John said he is confident that neither Jazz nor Lakers will win
When the result came, it was found that only one of the above three had made a correct statement. Who has made the correct statement and who has won the contest.
Ans: Sujith; Lakers
22. There are five persons with surnames Mukherjee, Misra, Iyer, Patil and Sharma. There are 4 persons having first or middle name of Kumar, 3 persons with Mohan, 2 persons with Dev and 1 Anil. Either Mukherjee and Patil have a first or middle name of Dev or Misra and Iyer have their first or middle name of Dev. Of Mukherkjee and Misra, either both of them have a first or middle name of Mohan or neither have a first or middle name of Mohan. Either Iyer or Sharma has a first or middle name of Kumar but not both. Who has the first or middle name of Anil?
Ans: Kumar Misra Dev, Mohan Iyer Dev, Kumar Patil Mohan, Mohan Sharma Kumar
23. Ann, Boobie, Cathy and Dave are at their monthly business meeting. Their occupations are author, biologist, chemist and doctor, but not necessarily in that order. Dave just told the biologist that Cathy was on her way with doughnuts. Ann is sitting across from the doctor and next to the chemist. The doctor was thinking that Boobie was a goofy name for parent's to choose, but didn't say anything. What is each person's occupation?
Ans: Since Dave spoke to the biologist and Ann sat next to the chemist and across the doctor, Cathy must be the author and Ann the biologist. The doctor didn't speak, but David did, so Bobbie is the doctor and Dave the chemist.
24. There are 6 volumes of books on a rack kept in order ( i.e. vol.1, vol. 2 and so on ). Give the position after the following changes were noticed.
• All books have been changed
• Vol.5 was directly to the right of Vol.2
• Vol.4 has Vol.6 to its left and both weren't at Vol.3's place
• Vol.1 has Vol.3 on right and Vol.5 on left
• An even numbered volume is at Vol.5's place
Find the order in which the books are kept now.
Ans: 2 , 5 , 1 , 3 , 6 , 4
25. A soldier looses his way in a thick jungle. At random he walks from his camp but mathematically in an interesting fashion. First he walks one mile East then half mile to North. Then 1/4 mile to West, then 1/8 mile to South and so on making a loop. Finally how far he is from his camp and in which direction.
Ans: Distance traveled in north and south directions:
1/2 - 1/8 + 1/32 - 1/128 + 1/512 - ............. = 1/2/((1-(-1/4))
Similarly in east and west directions:
1- 1/4 + 1/16 - 1/64 + 1/256 - ................... = 1/(( 1- ( - 1/4)) Add both the answers
26. Conversation between two mathematicians:
First : I have three children. The product of their ages is 36. If you sum their ages, it is exactly same as my neighbor's door number on my left.
The second mathematician verifies the door number and says that it is not sufficient. Then the first says " Ok one more clue is that my youngest is really the youngest". Immediately the second mathematician answers. Can you answer the question asked by the first mathematician? What are the children ages?
Ans 1,6 and 6
27. 500 men are arranged in an array of 10 rows and 50 columns according to their heights. Tallest among each row of all are asked to fall out. And the shortest among them is A. Similarly after resuming that to their original positions that the shortest among each column are asked to fall out. And the tallest among them is B. Now who is taller among A and B ?
Ans. A

28. There are six boxes containing 5 , 7 , 14 , 16 , 18 , 29 balls of either red or blue in color. Some boxes contain only red balls and others contain only blue. One sales man sold one box out of them and then he says, " I have the same number of red balls left out as that of blue ". Which box is the one he sold out ?
Ans: Total no of balls = 89 and (89-29 /2) = 60/2 = 30 and also 14 + 16 = 5 + 7 + 18 = 30
29. Ram Singh goes to his office in the city, every day from his suburban house. His driver Gangaram drops him at the railway station in the morning and picks him up in the evening. Every evening Ram Singh reaches the station at 5 O' Clock. Gangaram also reaches at the same time. One day Ram Singh started early from his office and came to the station at 4 O' Clock. Not wanting to wait for the car he starts walking home. Mangaram starts at normal time, picks him up on the way and takes him back house, half an hour early. How much time did Ram Singh walk?
Ans.
30. A family X went for a vacation. Unfortunately it rained for 13 days when they were there. But whenever it rained in the mornings, they had clear afternoons and vice versa. In all they enjoyed 11 mornings and 12 afternoons. How many days did they stay there totally?
Ans: 18
31. There are N coins on a table and there are two players A & B. You can take 1 or 2 coins at a time. The person who takes the last coin is the loser. A always starts first.
• If N=7
(a) A can always win by taking two coins in his first chance
(b) B can win only if A takes two coins in his first chance.
(c) B can always win by proper play
(d) none of the above
Ans.
• 2. A can win by proper play if N is equal to
(a) 13 (b) 37 (c) 22 (d) 34 (e) 48
Ans: (e.)
• 3. B can win by proper play if N is equal to
(a) 25 (b)26 (c) 32 (d) 41 (e) none
Ans.
• 4. if N<4, can A win by proper play always?
(a) Yes (b) No
Ans.

SET G
1. There is a 4 inch cube painted on all sides. This is cut down into of 1 inch cubes. What is the no of cubes which have no pointed sides.
Ans: 8
2. At 6'o a clock ticks 6 times. The time between first and last ticks is 30 seconds. How long does it tick at 12'o clock.
Ans: 66 sec.
3. Complete the series: 5, 20, 24, 6, 2, 8, ?
Ans: 12 (as 5*4=20, 20+4=24, 24/4=6, 6-4=2, 2*4=8, 8+4=12).
4. Find the values of each of the alphabets.
N O O N
S O O N
+ M O O N
J U N E
Ans: 9326
5. If a clock takes 7seconds to strike 7, how long will the same clock take to strike 10?
Ans: The clock strikes for the first time at the start and takes 7 seconds for 6 intervals-thus for one interval time taken=7/6. Therefore, for 10 seconds there are 9 intervals and time taken is 9*7/6=10 and 1/2 seconds.
6. An escalator is descending at constant speed. A walks down and takes 50 steps to reach the bottom. B runs down and takes 90 steps in the same time as A takes 10 steps. How many steps are visible when the escalator is not operating?
Ans: 150 steps
7. A chain is broken into three pieces of equal lengths containing 3 links each. It is taken to a blacksmith to join into a single continuous one. How many links are to to be opened to make it ?
Ans : 2.
8. There is a safe with a 5 digit number as the key. The 4th digit is 4 greater than second digit, while 3rd digit is 3 less than 2nd digit. The 1st digit is thrice the last digit. There are 3 pairs whose sum is 11. Find the number.
Ans: 65292
9. An orange glass has orange juice and white glass has apple juice both of equal volumes. 50ml of the orange juice is taken and poured into the apple juice. 50ml from the white glass is poured into the orange glass. Of the two quantities, the amount of apple juice in the orange glass and the amount of orange juice in the white glass, which one is greater and by how much?
Ans: The two quantities are equal
10. The shape in the sketch below is that of a square attached to half of a similar square. Divide it into four equal pieces
INCLUDEPICTURE "http://www.freshersworld.com/questions/ques2_9.gif" \* MERGEFORMATINET
Ans: Hint : the figure can be divided into 12 equal triangles
11. Fifty minutes ago if it was four times as many minutes past three o'clock, how many minutes is it to six o'clock?
Ans: Twenty six minutes.
12. Everyday in his business a merchant had to weigh amounts from 1 kg to 121kgs, to the nearest kg.
What are the minimum number of weight required and how heavy should they be?
Ans: .The minimum number is 5 and they should weigh 1,3,9,27 and 81kgs.
13. A car is traveling at a uniform speed. The driver sees a milestone showing a 2-digit number. After traveling for an hour the driver sees another milestone with the same digits in reverse order. After another hour the driver sees another milestone containing the same two digits. What is the average speed of the driver.
Ans: 45kmph
14. A hotel has 10 storeys. Which floor is above the floor below the floor, below the floor above the floor, below the
floor above the fifth.
Ans: The sixth floor.
15. Albert and Fernandes have two leg swimming race. Both start from opposite ends of the pool. On the first leg, the boys pass each other at 18 m from the deep end of the pool. During the second leg they pass at 10 m from the shallow end of the pool. Both go at constant speed but one of them is faster. Each boy rests for 4 seconds at the end of the first leg. What is the length of the pool?
Ans.
16. Shahrukh speaks truth only in the morning and lies in the afternoon, whereas Salman speaks truth only in the afternoon. A says that B is Shahrukh. Is it morning or afternoon and who is A - Shahrukh or Salman.
Ans: Afternoon ; A is Salman.
17. A person with some money spends1/3 for cloths, 1/5 of the remaining for food and 1/4 of the remaining for travel.
He is left with Rs 100/- . How much did he have with him in the beginning ?
Ans: Rs 250/-
18. Ram, Shyam and Gumnaam are friends.
Ram is a widower and lives alone and his sister takes care of him.
Shyam is a bachelor and his niece cooks his food and looks after his house.
Gumnaam is married to Gita and lives in large house in the same town.
Gita gives the idea that all of them could stay together in the house and share monthly expenses equally.
During their first month of living together, each person contributed Rs.25. At the end of the month, it was found that Rs 92 was the expense so the remaining amount was distributed equally among everyone. The distribution was such that everyone received a whole number of Rupees. How much did each person receive?
Ans. Rs 2 (Hint: Ram's sister, Shyam's niece and Gumnaam's wife are the same person)
19. There are 3 societies A, B, C. A lent cars to B and C as many as they had already. After some time B gave as many tractors to A and C as many as they have. After sometime c did the same thing. At the end of this transaction each one of them had 24. Find the cars each originally had.
Ans: A had 39 cars, B had 21 cars & C had 12 cars
20. Sam and Mala have a conversation.
• Sam says I am certainly not over 40
• Mala says I am 38 and you are at least 5 years older than me
• Now Sam says you are at least 39
All the statements by the two are false. How old are they really?
Ans: Mala = 38 yrs, Sam = 41 yrs.
21. Each alphabet stands for one digit in the following multiplication.
T H I S
x I S
X F X X
X X U X
X X N X X What is the maximum value T can take?
Ans: T max value = 4
22. Grass in lawn grows equally thick and in a uniform rate. It takes 24 days for 70 cows and 60 days for 30 cows to eat the whole of the grass. How many cows are needed to eat the grass in 96 days.?
Ans : 20
[Hint: g - grass at the beginning r - rate at which grass grows, per day
y - rate at which one cow eats grass, per day n - no of cows to eat the grass in 96 days
g + 24*r = 70 * 24 * y g + 60*r = 30 * 60 * y
g + 96*r = n * 96 * y, Solving, n = 20. ]
23. Three criminals were arrested for shop lifting. However, when interrogated only one told the truth in both his statements, while the other two each told one true statement and one lie. The statements were:
• ALBERT :(a) Chander passed the merchandise. (b) Bruce created the diversion.
• BRUCE :(a) Albert passed the merchandise. (b) I created the diversion.
• CLIVE :(a) I took the goods out of the shop. (b) Bruce passed them over.
Ans: Albert passed the goods. Bruce created the diversion. Clive took the goods out of the shop.
24. I bought a car with a peculiar 5 digit numbered license plate which on reversing could still be read. On reversing value is increased by 78633.Whats the original number if all digits were different?
Ans: Only 0 1 6 8 and 9 can be read upside down. So on rearranging these digits we get the answer as 10968
25. There N stations on a railroad. After adding X stations on the rail route 46 additional tickets have to be printed.
Find N and X.
Ans. x=2 and N=11 ( Let initially, N(N-1) = t; After adding, (N+X)(N+X-1) = t+46;Trail and error method )
27. A bird keeper has got P pigeons, M mynas and S sparrows. The keeper goes for lunch leaving his assistant to watch the birds.
• Suppose p=10, m=5, s=8 when the bird keeper comes back, the assistant informs the x birds have escaped. The bird keeper exclaims: "Oh no! All my sparrows are gone."
How many birds flew away?
• When the bird keeper comes back, the assistant told him that x birds have escaped. The keeper realized that at least 2 sparrows have escaped.
What is minimum no of birds that can escape?
Ans.
28. Seven members sat around a table for three days for a conference.
The member's names were Abhishek, Amol, Ankur, Anurag, Bhuwan ,Vasu and Vikram.
The meetings were chaired by Vikram.
On the first evening members sat around the table alphabetically.
On the following two nights, Vikram arranged the seating so that he could have Abhishek as near to him as
possible and absent minded Vasu as far away as he could.
On no evening did any person have sitting next to him a person who had previously been his neighbor.
How did Vikram manage to seat everybody to the best advantage on the second and third evenings?
Ans: Second evening: Vikram, Ankur, Abhishek, Amol, Vasu, Anurag and Bhuwan.
Third evening :Vikram, Anurag, Abhishek, Vasu, Bhuwan, Ankur, Amol.
29. Two twins have certain peculiar characteristics. One of them always lies on Monday, Wednesday, Friday. The other always lies on Tuesdays, Thursdays and Saturdays. On the other days they tell the truth. You are given a conversation.
• Person A-- today is Sunday my name is Anil
• Person B -- today is Tuesday, my name is Bill
What day is today?
Ans: Today is Tuesday.
30. Four prisoners escape from a prison. The prisoners, Mr. East, Mr. West, Mr. South, Mr. North head towards different directions after escaping. The following information of their escape was supplied:
• The escape routes were The North Road, South Road, East Road and West Road.
• None of the prisoners took the road which was their namesake.
• Mr. East did not take the South Road
• Mr. West did not the South Road.
• The West Road was not taken by Mr. East
What road did each of the prisoners take to make their escape?
Ans: Mr. East took the North Road
Mr. West took the East Road
Mr. North took the South Road
Mr. South took the West Road.
31. A hotel has two wings, the east wing and the west wing. Some east wing rooms but not all have an ocean view.
All west wing rooms have a harbor view. The charge for all rooms is identical, except as follows :
• Extra charge for all harbor view rooms on or above the 3rd floor
• Extra charge for all ocean view rooms except those without balcony
• Extra charge for some harbor rooms on the first two floor & some east wing rooms without ocean view but
having kitchen facilities.
Which of the following cannot be determined on the basis of the information given:
I. Whether there are any rooms without a balcony for which an extra charge is imposed.
II. Whether any room without a kitchen or a view involves an extra charge.
III. Whether two extra charges are imposed for any room.
(A) I only (B) II only (C) III only (D) II and III only (E) I, II and III
Ans: (A)
32. Given that April 1 is Tuesday. A, B, C are 3 persons told that their farewell party was on
• A - May 8, Thursday
• B - May 10, Tuesday
• C - June 5, Friday
Out of A, B, C only one made a completely true statement concerning date, day and month. The other told two one told the day right and the other the date right. What is correct date, month, day.
Ans: B - (May 10) SUNDAY , C - June 6 (Friday).
33. Answer the following questions based on the conditions from the choices A, B, C, D, E as described below:
(A) if a definite conclusion can be drawn from condition 1
(B) if a definite conclusion can be drawn from condition 2
(C) if a definite conclusion can be drawn from condition 1 and 2
(D) if a definite conclusion can be drawn from condition 1 or 2
(E) no conclusion can be drawn using both conditions
• person 1 says N<5
• person says N>5
• person 3 says 3N>20
• person 4 says 3N>10
• person 5 says N<8
What is the value of N?
a) 1. No of persons who speak false being less than no of persons who tells the truth.
2. Person 2 is telling the truth.
Ans.
b) 1. no of persons telling the truth is greater than no of persons telling lies
2. person 5 is telling the truth.
Ans.
HYPERLINK "http://www.freshersworld.com/Intsec.htm" 1. 1) a) 10 1 9 2 8 3 7 4 6 5 5 6 4 7 3 8 2 _ _
b) 2 4 16 512 _
write the next elements in the series.
2. A Man is sitting in the last coach of train could not find a seat, so he starts walking to the front coach ,he walks for 5 min and reaches front coach. Not finding a seat he walks back to last coach and when he reaches there, train had completed 5 miles. what is the speed of the train.
3. The Old carry requires tyres to be changed after each 24000 km. if she wants to go for 42000 km journey then how many minimum number of tyres she will need.?
Ans:
4. A coin is so unbalanced that it may come both heads in 2 tosses as it may come tails in a single toss. what is the probabality of getting a head in a single toss.
Ans:
5. A pen, pencil and eraser together cost $1.00. if 2E2P, and 3P>4E then what a single pen will cost??
Ans:.
6. A local forecast service has accuracy of 2/3 says No rain , and Meteriological service having accuracy of 4/3 says Rain. if Preference is as no rain what is the chance of rain??
Ans:
7. Sherlock holmes thrwated the plan to kidnapp Mrs mary when they were questioned Mercy and his two associated shipy and rany.when they were telling the story one of them told one thing wrong and other true, the other told both true, and the last told both false. examining the following tell the roles played by each ??
Mercy:: 1) i wrote the ransome note ??
2) shipy broke into the window
rany 1) shipy wrote the ransome note
2) mercy ran away with the lady
shipy 1)i broke into the window
2)rany wrote the ransome note.?
Ans :
8. Tom asked kim did you like the stamps? she said yes ,me and rob too liked them. kim again said that rob got 3 more than he wd have got if i would have kept 2 more than what he got. Tom asked how many u gave Rob? she replied 2 more than what i got. Tell , how many stamps each rob and kim got?.
Ans:
9. The virgo club members used to meet every week to play cards. Each time they used to seat around a round table and for their memory they used all the possiblecombinations of postions each for a single time only. Can you tell for how many times they met??





A bicycle goes 1 mile in three minutes with the wind and returns back in 4 min. against the wind.How many minutes he will take to cover 1 mile when there is no wind? Ans (drag mouse over it )
2 A man passed one sixth of his life in childhood , one twelfth in youth, And one seventh more as a bachelor , 5yrs after his marriage a son was born Who dies 4yrs before his father at half his final age.What is the total age of the man ? Ans (drag mouse over it )
3 Alpha,Beta Gamma ,Theta and Epsilon.There are 5 different quantities Are given • If and only if alpha=gamma then beta<>epsilon • Alpha-beta = beta-gamma alpha>theta • Alpha>beta and gamma>theta Write the quantities in descending order Ans (drag mouse over it )
4 A thief broke into a jewellery shop and thought of taking all the diamonds with him but while he was leaving came to his bitter senses and thought of taking half with him but he could not resist and took more extra and left . A second thief came and took half of the remaining and two more .A third thief came and took half the remaining and two more .Then fourth thief came and did the same .The fifth thief came and there was no more diamond for him to steal What was the actual no.of diamonds initially ? Ans (drag mouse over it )
5 SEND 9567 What is the value of M,O,N,E MORE 1085 MONEY 10652 Ans (drag mouse over it )
6 Three man A,B,C went in three direction and had stolen a mule , a horse and a camel they ware caught by C.B.I and arrested . During their interrogation they gave the following statements. A: B had stolen a horse B: A and C are both lying and I had stolen nothing. C: A is lying and B had stolen a mule One who had stolen a camel is telling lie and one who had stolen a horse is telling truth. Among A,B,C who had stolen which animal ? Ans (drag mouse over it )
7 A scientist `SAM' arrived late in his lab and found that his watch the minute hand and the hour hand ware coming together in every 65 min. Was his watch gaining or losing time and how much per hour *refer to SHAKUNTALA DEVI'S puzzles to puzzle you NO-63
1. Amarpur is north of Landganj and west of Chutpur
2. Basti is north of Amarpur and west of Fulganj
3. Dania is south and east of Amarpur
4. Landganj is north of Fulganj and east of Dania
5. Fulganj is north of Dania and west of Amarpur
6. Chutpur is south of Fulganj and west of Dania
(A)Which of the towns mentioned is furthest to the north-west ?
*Amarpur
*Basti (Ans)
*Chutpur
*Landganj.
*Fulganj
(B)Which of the following must be both north and east of Fulganj ?
1-Amarpur,
2-Chutpur,
3-Landganj •
1 only •
2 only •
3 only •
1 and 2 •
1 and 3 (ans)
(C) Which of the following towns must be situated both south and west of atlest one other town ? •
Amarpur only •
Amarpur and Fulganj •
Dania and Fulganj Dania ,
Chutpur and Fulganj. (Ans) •
Chutpur,Dania and Landganj.
Another Set of Questions... !!
1) Henny,Axie,Amie are friends
cond:a)Herry or Axies is the oldest
b)If Axie is the oldest,Amie is the youngest
ANS:Amie is the youngest,Axie is oldest

2)A,B,C are 3 girls and there are 770 Apples.For
every 4Apples A takes,B takes 3.For ever 6 Apples ,C
takes 7 Apples?
ANS:261:145:303

3)T,U,V are 3 friends digging groups in fields.If T&U
can complete i groove in 4 days&,U&V can complete 1
groove in 3 days&V&T can complete in 2 days
Find how many days each takes to complete 1 groove
individually.
ANS:24 days =

4)4 mathematician has x apples .if he arranges them
in rows of 3 one will be left.the same is the case
with 5,7,9 apples.But when he arranged them in rows of
11 non will be left.Find the no.of apples
ANS:946
11*6
11*11
11*16
11*21
=2E......11*76
=3D946

5)H starts running after T reaches
1/5th they must when H reach
1/6th if H wants win at what speed H should be run
note:one circle is there,you show this type of
problem

6)There are 4 months,4 daughter and the colour of
their dresses ,and they are aged 1,2,3&4 .stat are in
form as the girl with white dresses is see is daughter

7)There are 5 levels of dolls and each the diff
colors & condition are given
note:this type of problem also refer

8)5 student A,B,C,D,E .one student knows 5
languages.like that up to one langauge.states
*)Spanish is most popular langauge
*)3 persons knows Porchigese
*)B&C are speakes Englishj when D gathered they
switched to Spanish because that is only common
between the three
*)only langauge common between A,B,E is French
*)only langauge common between C,Eis Italian.

Another Set of Questions... !!

1.There are 111 players participating in a singles tennis tournament. The player who is loosing will be out of the tournament.
For each and every match,One new ball is taken. Find the no. of balls required for the entire tournament.

ans:110

2.Two trains are crossing each other.the speed of one train is given. Calculate the time for the engine of one train crosses the other train..
chk. the question correctly..
it is available in sakuntala devi's "puzzle 2 puzzle u"

3.Anan with her son went for shopping she met her husband's mother's only daughterinlaw's sister's husband. find the relation between anan and the mentioned Person.
ans: anan-->sister's husband

4.Question on sets:
In a city,there are 23080 people.
30% read "a" newspaper.
35% read "b" newspaper
40% read "c" neewspaper
1/3 of total read both a and b
25% read both b and c
34% read both a and c
1/3 read all the three.
find.
1.the no. of persons who read a only.(approx,. 4000+)
2.b only(0)
3.c only
4.none of the papers


5)Question on finding the "day"
six persons are there.a.. f
a:the day before yesterday is either friday or saturday.
b:today is saturday.
c:today is not saturday but either friday or saturday.
d:day after tomorrow is not tuesday.
e:......
f:........

answer:friday

6)Question on finding who is familiar with which language.. some five names are given. and five statements are given. analyse all the statements ..
eg. "a" knows somewhat eng but not either urdu or tulu..

answer:

7)I have got some money in my bag.(which is stolen after shopping I and II). I spent 10% of my money for shopping(I) For second time,10% of the remaining Money is spent for shopping(II) The total bill amount=Rs.18. Find the amount which will be remaining in the bag?

answer:Rs.81

1. In 1978, a kg of paper was sold at Rs25/-.
If the paper rate increases at 1.5% more than the inflation rate which is 6.5% a year,
then what wil be the cost of a kg of paper after 2 years?
(a) 29.12
(b) 29.72
(c) 30.12
(d) 32.65
(e) none of these

2. In A,B,C are having some marbles with each of them.
A has given B and C the same number of marbles each of them already have.
Then, B gave C and A the same number of marbles they already have.
Then C gave A and B the same number of marbles they already have.
At the end A,B,and C have equal number of marbles.
(i) If x,y,z are the marbles initially with A,B,C respectively.
Then the number of marbles B have at the end
(a) 2(x-y-z)
(b) 4(x-y-z)
(c) 2(3y-x-z)
(d) x + y-z
Ans. (c)
(ii) If the total number of marbles are 72, then the number of marbles with A at the starting
(a) 20
(b) 30
(c) 32
(d) 39
Ans. (d)

3. If a car starts from A towards B with some velocity.
Due to some problem in the engine after travelling 30km, the car goes with 4/5 th of its actual velocity
The car reaches B 45 min later to the actual time.
If the car engine fails ofter travelling 45km, the car reaches the destination B 36min late to the actual time
What is the initial velocity of car and what is the distance between A and B in km
Ans. 20 & 130.

4. A person has Rs 100/- in his pocket, he can as 25 pencils or 15 books.
He kept 15% of the money for travelling expenses and purchased 5 pencils.
So how many books he can purchase with the remaining money.
5. Ten questions on analogies.
eg: chief : tribe :: governer : state
epaulette : shoulder :: tiara : head
guttural : throat :: gastric : stomach
inept : clever :: languid : active
knife : butcher ::
hammer : carpenter ::

7. In a computer institute 9 languages can be taught.
The module is of 6 months duration and of the six languages only one can be taught each month .
In addition to that BASIC is always taught and should be in first month itself
• WORD PERFECT is to be taught in the preceeding week of WORD STAR.
• FORTRAN can not be taught until COBAL is taught prior to that
• BINO, FIFO can never be taught in single module
languages are BASIC, WORD STAR, WORD PERFECT, FORTRAN, COBAL, BINO, FIFO, LOTUS, C
i) If word star is in 3rd month , what could be in 6th month.
ii) If COBAL is in the 2nd month and BINO in 6th month. FORTRAN will be taught in which month.

8. In a class, except 18 all are above 50 years.
15 are below 50 years of age. How many people are there
(a) 30
(b) 33
(c) 36
(d) none of these.
Ans. (d)

9. A square plate of some size is cut at four corners. Equal squares of the same size are cut and is formed as open box.
If this open box carries 128 ml of oil. What is the size of the side of the plate?
(a) 17
(b) 14
(c) 13
(d) None of these

10. In a square, all the mid points are joined. The inner square is shaded.
If the area of the square is A, what is the area of the shaded area?


1. 5, 5, 13, 13, 21, 21, __

Ans: 29
2. 0, 7, 26, 63, 124, __

Ans: 215 ( hint: n3-1 )
3. 1, 3, 5, 7, __
Ans: 9

4. If a person walks at 4/5th of his usual speed he reaches 40min late. If he walks at his usual speed for how much time does he travel ?
5. Two trains A&B start at opposite points 120km apart at 60kmph. A fly starting along with train A at 120kmph reaches B then returns back to touch A and continues the two and fro movement. By the time two trains meet how much distance would the fly have travelled?
6. In a class 80% have passed in english, 70% passed Hindi, 10% didn't pass either. If 144 students passed both the subjects. What is the total strength of the class?

7. Find the least number which when divided by 7 gives the reminder 6, when divided by 6 gives reminder 5, when divided by 5 gives reminder 4 and so on ?

8. If a man stands in front of sun what is the first letter of the direction which is left to him ?

9. (a)A square is to circle as cube is to
(b)Success is to failure as joy is to

10. (a)Give the synonyms of the following words
(i) Joy
(ii) Inert
(iii) Jolly

(b) Give the opposites of the following words
(i) genuine
(ii) command
(iii) essential
11.Find the odd man out in the following sets
(i) Tiger, Elephant, King Cobra, Dolphin
(ii) Oasis, Lake, Pool, Valcano
(iii) Bengali, Karnataka, Mumbai, Kashmir
(iv) Lapidary, Lancet, Scapel, Surgeon
(v) Requiem, Dirge, Elegy, Paean
12. I bought a cycle 2days before my birthday and I broke it 3 days after my birthday. The day I broke it is Mar2,1956?

Directions: The following questions are to be answered on the basis of the above given statement
i) When is my birthday?
Hint: Keep in mind that 1956 was a leap year.
(ii) What is my age on Mar 4th, 1980?
(iii) My nephew is born exactly 20years after me. If I turned 20 in 1960, what is the nephews age on Feb 28th 1988 ?

13. Monday Aug25, 96 :
Hostess: "Mr A, you forgot your umbrella during the party on last friday. I expected you to collect it on your visit on wednesday as I plan to leave on this Friday."
Directions: The following questions are to be answered on the basis of the above given statement
(i) when A missed umbrella?
(ii)When A is supposed to collect it?
(iii)When K leaves?

14.What is my father's sons son to my son?

Ans. Cousin brother
15. On cutting a solid parabola what would be generated

Ans: Cone

16. What is Eulers formula?

Ans: F+V-E =2; where
F => faces; V => vertices; E => number of edges
17. What is Newton Raphson method used for?

Ans: To find the root of f(x) = 0;

18. How many tangents can be drawn on three circles if they don't lie within each other ?

19. xy - x + 2y = 6 equation is shifted to form equation xy=c what is c ?

20.When x is real what is the least value of (x2-6x+5)/(x2+2x+1)

21.When an object like cube or sphere is seen along x, y, z axis we get the same.Apart from these suggest another object which has similar characteristics as that mentioned above?
Ans: Triangular prism
22. When an object is seen from the front side we can see two concentric squares and top view also without any hidden lines. Draw the side view.
23. In common parlance, A=> B means what
Ans: if A is true B has to be true
23. If A is not invertible and BA = I is not possible, what is implied by this?
Ans: Determinant is Zero.
24. What is a free body diagram used for

25. A die is thrown twice what is the probability that you get same number


26. The sum of two numbers is 55. What is the larger number?

*****************



1. What is the percentage represented by 0.03 * 0.05 ?
(a)0.0015
(b)0.000015
(c)0.15
(d)15
Ans.B

2. (x-a)(x-b)(x-c)....(x-z) = ?
(a) 1
(b) -1
(c) 0
(d) Can't be determined
Ans. C

3. If a = 1, b = 2, c = 3.......z = 26 what is the value of p+q+r ?
(a)33
(b)51
(c)52
(d)48
Ans. B

4. A is 8 miles east of B.
C is 10 miles north of B.
D is 13 miles east of C and E is 2 miles north of D.
Find shortest distance between A and E.
(a) 5 miles
(b) 6miles
(c) 13 miles
(d) 18 miles
Ans. C

5. If z = 1, y = 2.......a = 26. Find the value of z + y + x + .......+a.
(a) 351
(b) 221
(c) 400
(d) 200
Ans. A

6. There are 30 socks in a bag.
Out of these 60 % are green and the rest are blue.
What is the maximum number of times that socks have to be taken out so that atleast 1 blue pair is found.
(a) 21
(b) 2
(c)18
(d) 20
Ans. D

7. How many two digit numbers have their square ending with 8.
(a) 13
(b) 12
(c) 0
(d) 11
Ans. C

8. How many numbers are there between 100 and 300 with 2 in the end and 2 in the beginning.
(a) 10
(b) 9
(c) 11
(d) none of these
Ans. A

9. 0.000006 * 0.0000007 = ?
(a) 0.0000000042
(b) 0.000000000042
(c) 0.0000000000042
(d) 0.00000000000042
Ans. B

10. You have Rs 1000 with 8% p.a compounded every 6 months.
What is the total interest you get after 1 year.
(a) Rs.116.40
(b) Rs.345.60
(c) Rs.224.50
(d) Rs.160
Ans. A

11. If x + y =12,
x - y = 2
Find x + 2y.
(a) 12
(b) 17
(c) 14
(d) none of these
Ans. B

12. With one gallon of petrol a person moves at a speed of 50 mph and covers 16 miles.
3/4th of the distance is covered while moving at 60 mph.
How many gallons does he need to cover 120 miles in 60 mph.

13. A tap drains at x speed while tap B is closed.
When both taps are open they drain at y speed.
What is the speed of draining when only tap B is open
(a) x - y
(b) y-x
(c) x
(d) can't be determined
Ans. B

14. What is twenty percent of 25 % of 20.
(a) 2
(b)1
(c) 5
(d) 4
Ans. B

15. A rectangle has the dimensions 6ft * 4ft.
How many squares of 0.5 inches will it need to completely fill it.
(a) 32000
(b) 12824
(c) 13824
(d) 18324
Ans. C

Directions for questions 16-21: In each question,a series of letters satisfying a certain pattern are given. Identify the pattern and then find the letter/letters that will come in place of the blank/blanks.
16. a, c, e, g, _
(a) h
(b) i
(c) d
(d) j
Ans. B

17. a, e, i, m, q, u, _, _
(a) y, c
(b) b, f
(c) g, i
(d) none
Ans. A

18. ay , bz , cw , dx ,__
(a) gu
(b) ev
(c) fv
(d) eu
Ans. D

19. 1, 2, 3, 5, 7, 11, __
(a) 15
(b) 9
(c) 13
(d) 12
Ans. 13 , series of prime numbers

20. kp , lo , mn , __
(a) nm
(b) np
(c) op
(d) pq
Ans. A

21. abc , zyx , def , wvu , ___
(a) ghi
(b) tsr
(c) ihg
(d) str
Ans. A

22. How is my mother's sister's brother's wife's child related to me?
(a) brother
(b) uncle
(c) cousin
(d) nephew
Ans. A

23.What will my mother's husband's father-in-law's son's daughter to me?
(a) niece
(b) aunt
(c) sister
(d) none of these
Ans. D
*************************
SECTION-1:

Find the next in the sequence:

1. BC CE EG GK ?

a)KN
b)KU
c)KM
d)None

2. AA AB BC CE?

a)EG
b)EH
c)EI
d)None

3. AB EF JK QR ?

a)YZ
b)ZA
c)AB
d)None

4.ACD EGL IKT MOB?

a)QST
b)QSZ
c)QSY
d)None

5.AC CG GO OE?

a)EJ
b)EI
c)EL
d)None

6.AE BH CM DU?

a)EH
b)EZ
c) EB
d)None

7. AD DP PL LV

a)VS
b)VK
c)VI
d)None

8. SE QU EN TI?

a)CN
b)BM
c)AI or AZ
d)None
SECTION-II:

Find the values for the following problem:
f(X)= 2X-1 + f(X-1) if X is not equal to zeroandiff(X=0)=0
9. Value of f(5)

a)15
b)24
c)22
d)None
10. Value of f(f(2))
11. Value of f(16)- f(15)
12. Value of f(16)+f(15)-480
13. If f(f(X))=81 then the value of X=?
14. If f(X)=4f(X-1) then the value of X=?
15.If f(X)= f(X-1)+f(X-2) for X>1 then X= ?
16. If f(X)-f(X-1)=f(X-8) for X>5 then X=
**************************.

Analogies

1. slur : speech : : smudge :?
Ans. writing
2. epaulet : shoulder : : ring :?
Ans.finger
3. vernacular : place : : fingerprint : ?
Ans.identical

Opposites

Q. corpulent
Ans: emaciated
Q. officious
Ans: pragmate
Q. dextrous
Ans: clumsy

The following sentences are broken into 4 sections- A, B, C, D
Choose the part which has a mistake
Mark (E) if you find no mistake.
Q.A)psychologists point out that B)there are human processes C)which does not involve D) the use of words
Ans. (C) which does not involve (do)

Q.A)jack ordered for B)two plates of chicken C)and a glass D)of water
Ans. (A)jack ordered for

Q. An algorithm follws a six step process za,zb,zc,zd,ze,zf, it is governed by the following
(i) zd should follow ze
(ii) the first may be za,zd or zf
(iii) zb and zc have to be performed after zd
(iv) zc must be immediately after zb

Q. If za is the first set zd must be
a) 3rd
b) 5th
c) 2nd
d) 4th
Q. If zb must follow za then za can be
a) third or fourth
b) first or second
c) can not be third
d) fouth or fifth
e) none
Q. If ze is third term the number of different operations possible are
The following questions are based on the given statements
Ravi plants six seperate saplings -- x,y,z,w,u,v in rows no 1 to 6 ,according to the follwing conditions
He must plant x before y and u
He must plant y and w
The third has to be z
Q. Which of the following is acceptable
a) xuywzv
b) xvzyuw
c) zuyxwv
d) zvxuwy
e) wyzuvx
Q. Which of the following is true
a) z before v
b) z before x
c) w before u
d) y before u
e) x before w
Q. If he plants v first, then which can be planted second
a) x
b) y
c) z
d) w
e) u

Q. Which of the following describes a correct combination of sapling and row?
a) x,3
b) y,6
c) z,1
d) w,2
e) u,6

Q. If he plants b 6th which would be planted first and second
a) x and w
b) x and y
c)y and x
d)w and z
e) w and u

Q. If he plants w before u and after v he should plant w at
a) first
b) second
c) fourth
d) fifth
e) sixth

Q. At a certain moment a watch shows 2 min lag although it is running fast.
If it showed a 3 min lag at that moment, but also gains by 1/2 min more a day than its current speed
it would show the true time one day sooner than it usually does.
How many mins does the watch gain per day.
a).2
b).5
c).6
d).4
e).75
Q. In 400m race A gives B a start of 7 sec and beats him by 24 sec.
In another race A beats B by 10 sec.the speeds are in the ratio
a)8:7
b)7:6
c)10:8
d)6:8
e)12:10

Q. 3x+4y=10
x3 + y3=6
What is the minimum value of 3x+11y=?

Q. There are 600 tennis players
4% wear wrist band on one wrist
Of the remaining, 25% wear wrist bands on both hands
How many players don't wear a wrist band?
Ans. 432

Q. Three types of tea the a,b,c costs Rs. 95/kg,100/kg and70/kg respectively.
How many kgs of each should be blended to produce 100 kg of mixture worth Rs.90/kg,
given that the quntities of band c are equal
a)70,15,15
b)50,25,25
c)60,20,20
d)40,30,30
Ans. (b)

Q. Two distinct no's are taken from 1,2,3,4......28
Find the probability that their sum is less than 13

(A) If statement (I) alone is sufficient but statement (II) alone is not sufficient.
(B) If statement(II) alone is sufficient but statement(I) alone is not sufficient.
(C) If both statements together are sufficient but neither of statements alone is sufficient.
(D) If both together are not sufficient.
41. What is John's age?
(I) In 15 years John will be twice as old as Dias would be
(II) Dias was born 5 years ago
Ans. (C)
42. What is the distance from city A to city C in kms?
(I) City A is 90 kms from City B
(II) City B is 30 kms from City C
Ans. (D)
43.Is A=C ? A,B,C are real numbers
(I) A-B=B-C
(II) A-2C = C-2B
Ans. (d)
51. If "PROMPT" is coded as QSPLOS ,then "PLAYER" should be
(a) QMBZFS
(b) QWMFDW
(c) QUREXM
(d) URESTI
Ans. (a)
The questions 52-53 are based on the following data
6 people A,B,C,D,E and F sit around a table for dinner.Since A does not like C, he doesn't sit either opposite or beside C.B and F always like to sit opposite each other.
52. If A is beside F then who is are the two neighbours of B?
(a) D and C
(b) E and C
(c) D and E
(d) Either (a) or (b)
Ans. (c)

53. If D is adjacent to F then who is adjacent to C?
(a) E and B
(b) D and A
(c) D and B
(d) either (a) or (c)
Ans.(d)

54. Complete the sequence A, E ,I ,M ,Q ,U , _ , _
(a) B, F
(b) Y, C
(c) G, I
(d) K, O
Ans.(b)

55. A person travels 6km towards west, then travels 5km towards north ,then finally travels
6km towards west. Where is he with respect to his starting position?
(a) 13km east
(b) 13km northeast
(c) 13km northwest
(d) 13km west
Ans. (c)

56. If A speaks the truth 80% of the times, B speaks the truth 60% of the times.
What is the probability that they tell the truth at the same time
(a) 0.8
(b) 0.48
(c) 0.6
(d) 0.14
Ans.(b)
******************
1.When a bicycle is in motion,the force of friction exerted by the ground on the two wheels is such that it acts
(a) In the backward direction on the front wheel and in the forward direction on the rear wheel.
(b) In the forward direction on the front wheel and in the backward direction on the rear wheel.
(c) In the backward direction on both the front and rear wheels.
(d) In the backward direction on both the front and rear wheels.
Ans. (d)

2. A certain radioactive element A, has a half life = t seconds.
In (t/2) seconds the fraction of the initial quantity of the element so far decayed is nearly
(a) 29%
(b) 15%
(c) 10%
(d) 45%
Ans. (a)

3. Which of the following plots would be a straight line ?
(a) Logarithm of decay rate against logarithm of time
(b) Logarithm of decay rate against logarithm of number of decaying nuclei
(c) Decay rate against time
(d) Number of decaying nuclei against time
Ans. (b)

4. A radioactive element x has an atomic number of 100.
It decays directly into an element y which decays directly into element z.
In both processes a charged particle is emitted.
Which of the following statements would be true?
(a) y has an atomic number of 102
(b) y has an atomic number of 101
(c) z has an atomic number of 100
(d) z has an atomic number of 101
Ans. (b)

5. If the sum of the roots of the equation ax2 + bx + c=0 is equal to the sum of the squares of their reciprocals
then a/c, b/a, c/b are in
(a) AP
(b) GP
(c) HP
(d) None of these
Ans. (c)

6. A man speaks the truth 3 out of 4 times.
He throws a die and reports it to be a 6.
What is the probability of it being a 6?
(a) 3/8
(b) 5/8
(c) 3/4
(d) None of the above
Ans. (a)

7. If cos2A + cos2B + cos2C = 1 then ABC is a
(a) Right angle triangle
(b) Equilateral triangle
(c) All the angles are acute
(d) None of these
Ans. (a)

8. Image of point (3,8) in the line x + 3y = 7 is
(a) (-1,-4)
(b) (-1,4)
(c) (2,-4)
(d) (-2,-4)
Ans. (a)

Q7. In a triode valve in order to increase the saturation current what has to be done
(a) increase plate voltage
(b) reduce distance between grid and plate
(c) increase cathode potential
(d) reduce grid potential

Ans. (d )
Q8. Seven different toys are distributed among 3 children how many different ways are possible?
(a) 7C3
(b) 7P3
(c) 3 7
(d) 7 3

Ans. (c)

Q9. A, B ans C are three speakers. They have to speak randomly along with another 5 speakers in a function.
A has to speak before B and B has to speak before C. What is the probability.

Ans. 1/6
Q10. If dy = (secx + ytanx)dx, Then the curve is

(a) x = ycosx
(b) x = ysinx
(c) x = ytanx
(d) x = ysecx

Ans. (a)
Q11. Two series are 16,21,26.... and 17,21,25.....
What is the sum of first hundred common numbers
(a) 101100
(b) 110100
(c) 101110
(d) 110101
Ans. (a)

Q12. There are two sections in a question paper each contain five questions. A students has to answer 6 questions.
Maximum no. of questions that can be answered from any section is 4. How many ways he can attempt the paper?

(a) 50
(b) 100
(c) 120
(d) 200
Ans. (d)
Q13. a and b are two numbers selected randomly from 1,2,3.... 25 what is the probability of a and b are not equal.
(a) 1/25
(b) 24/25
(c) 13/25
(d) 2/25
Ans. (b)

Q14. The sum of the series 1 + 1(1+1/n) + 3(1+1/n)2 + ..... is equal to?
Ans. n2
Q15. Two circles of different radii intersects each other what is the maximum no of intersections
(a) 0
(b) 1
(c) 2
(d) 3
Ans. (c)

Q16. If x= sin-1(t), y = log(1-t2), find d2y/dx2 when t=1/2
(a) 1
(b) 0
(c) -8/3
(d) -2/3
Ans. (c)

Q17. If x approaches infinity , then ex dx )/( e2xdx) is ?
(a) 1
(b) 0
(c) -1
(d) 2
Ans. (a)

Q18. If f(x)=1-cos(1-cosx)/x4 is continuos at f(0) then what is x
(a) 1
(b) 0
(c) 1/4
(d) -1/4
Ans. (c)

Q19. For the word SURITI, if you arrange the letters in dictionary order then what is its rank?
(a) 234
(b) 235
(c) 236
(d) 237
Ans. (c)

Q20. Period of sin ((2t + 3) / 6 pi)
(a) 6pi
(b) 6pi2
(c) 3pi
Ans. (b)

Q21 - Q23. Four questions given on the below data
X,Yand Z are senior engineers. A,B,C,D are junior engineers. Company wants to select 4 enginers. Two will be senior and two will be juniors. The company wants these engineers to work in the most productive way so they respect each person's likes/dislikes.
• Y is not friends with A
• Z is not friends with C
• B is not friends with A
1. If B is selected then who will be the remaining 4 members ?
2. If C is selected, Z and ___ cannot be selected?
3. D is always selected if ___ is selected?

Q24. A speaks truth 70% of the times, B speaks truth 80% of the times.
What is the probability that both are contradicting each other is ?

Q25. ((2x-3)/((x2 +x+1)2 )dx is ?

Q26. Ram starts from A walking 2 km North and turns right and walks 4 km and turns right again and walks 4 km and turns right again and walks 4 km and meets Radha at Bwalking in the opposite direction to Ram .
a) Which direction does Ram walk after the first turn?
b) Distance between A and B

Q27. If the equation x2 - 3x + a = 0 has the roots (0,1) then value of a is ?

Verbal
ACUMEN
a. exactness
b. potential
c. shrewdness
d. bluntness
e. None of these
BEHEST
a. behavior
b. hold down
c. hold up
d. relieve
e. condemn
DISCRETION
a. prudence
b. consistency
c. precipice
d. disturbance
e. distemper

ORDAIN
a. arrange
b. command
c. contribute
d. establish
e. control

FLORID
a. ornate
b. thriving
c. artistic
d. elegant
e. None of these
PENITENCE
a. liking
b. insightful
c. attractive
d. penetrable
e. compunction

WHET
a. stimulate
b. humorous
c. inculate
d. dampen
e. None of these

INCENTIVE
a. reflex
b. amplitude
c. inflection
d. provocation
e. escutcheon

LATITUDE
a. scope
b. segment
c. globule
d. legislature
e. lamentation
MORTIFY
a. make a cavity
b. displease
c. humiliate
d. relapse
e. murder

ADAGE
a. advice
b. proverb
c. enlargement
d. advantage
e. usage

TO DISPEL
a. to dissipate
b. to dissent
c. to distort
d. to disfigure
e. to dissect
ERRATIC
a. unromantic
b. free
c. popular
d. steady
e. unknown

TO MERIT
a. to embrace
b. to devote
c. to deserve
d. to combine
e. to display

RAPT
a. lively
b. concealed
c. engrossed
d. prototype
e. None of these

TO HEAP
a. to pile
b. to forbid
c. to proceed
d. to share
e. to stoop

CAJOLE
a. coax
b. motivate
c. profound
d. mollify
e. evade
OVULATE
a. penury
b. immunize
c. fertilize
d. reproduce
e. incisions
ABODE
a. clay
b. obstacle
c. dwelling
d. bind
e. to beguile

POTENTIAL
a. latent
b. hysterical
c. conventional
d. symmetrical
e. conscientious

EXTRICATE
a. terminate
b. isolate
c. liberate
d. simplify
e. frustrate

DISPARITY
a. inequality
b. impartiality
c. unfairness
d. twist
e. None of these

TO CONFISCATE
a. to harass
b. to repulse
c. to console
d. to appropriate
e. to congregate
PIOUS
a. historic
b. devout
c. multiple
d. fortunate
e. authoritative

LETHARGY
a. reminiscence
b. category
c. fallacy
d. unanimity
e. stupor

CARGO
a. cabbage
b. camel
c. lance
d. freight
e. flax
OVATION
a. oration
b. gesture
c. emulation
d. applause
e. nourish
**********************
Aptitude
1. A family, planning a weekend trip, decides to spend not more than a total of 8 hours driving. By leaving early in the morning, they can average 40 miles per hour on the way to their destination. Due to the heavy Sunday traffic, they can average only 30 miles per hour on the return trip. What is the farthest distance from home they can plan to go?
(a) 120 miles or less (b) Between 120and 140 miles (c) 140 miles
(d) Between 140 and 160 miles (e) 160 miles or more
2. A car is filled with four and half gallons of fuel for a round trip. If the amount of fuel taken while going is 1/4 more than the amount taken for coming, what is the amount of fuel consumed while coming back?
(a) Less than 2 gallons (b) 2 gallons (c) 2 1/2 gallons
(d) 3 gallons (e) More than 3 gallons

3. A 3-gallon mixture contains one part S and two parts R. In order to change it to a mixture containing 25% S, how much R should be added?
(a) 1/2 gallon (b) 2/3 gallon (c) 3/4 gallon (d) 1 gallon (e) 1 1/2 gallon

4. A tree grows only 3/5 as fast as the one beside it. In four years the combined growth of the two trees is eight feet.
How much does the shorter tree grow in two years?
(a) Less than 2 feet (b) 2 feet (c) 2 1/2 feet
(d) 3 feet (e) more than 3 feet.
5. Wind flows at 160 miles in 330 minutes, for traveling 80 miles how much time does it require?
(a) 1 hour 30 minutes (b) 1 hour 45 minutes (c) 2 hours
(d) 2 hours 45 minutes (e) 3 hours

6. A stationary engine has enough fuel to run 12 hours when its tank is 4/5 full. How long will it run when the tank is 1/3 full?
(a) Less than 2 hours (b) 2 hours (c) 3 hours
(d) 4 hours (e) 5 hours
7. If A is traveling at 72 km per hour on a highway. B is traveling at a speed of 25 meters per second on a highway. What is the difference in their speeds in meters per second?
(a) 1/2 m/sec (b) 1 m/sec (c) 1 1/2 m/sec
(d) 2 m/sec (e) 3 m/sec
8. A salesperson by mistake multiplied a number and got the answer as 3, instead of dividing the number by 3. What is the answer he should have actually got?
(a) 0 (b) 1/3 (c) 1 (d) 2 (e) 3
9. If the length of a rectangle is increased by 30% and the width is decreased by 20%, then the area is increased by...
(a) 10% (b) 5% (c) 4% (d) 20% (e) 25%

10. In the class of 40 students, 30 speak Hindi and 20 speak English. What is the lowest possible number of students who speak both the languages?
(a) 5 (b) 20 (c) 15 (d) 10 (e) 30

11. The most economical prices among the following prices is:
(a) 10 kilo for Rs.160 (b) 2 kilo for Rs.30 (c) 4 kilo for Rs.70
(d) 20 kilo for Rs.340 (e) 8 kilo for Rs.130

12. A truck contains 150 small packages, some weighing 1 kg each and some weighing 2 kg each. how many packages weighing 2 kg each are in the truck if the total weight of all the packages is 264 kg?
(a) 36 (b) 52 (c) 88 (d) 124 (e) 114
13. A man was arrested for exceeding the speed limit by 10 miles an hour. A second man was charged with exceeding the same limit by twice as much. The second man was driving 35 miles per hour. What was the speed limit?
(a) 10 miles per hour (b) 15 miles per hour (c) 20 miles per hour
(d) 25 miles per hour (e) 30 miles per hour

14. One year ago Pandit was three times his sister's age. Next year he will be only twice her age. How old will Pandit be after five years?
(a) 8 (b) 12 (c) 11 (d) 13 (e) 15
15. If two pencils cost 8 cents, then how much do 5 pencils cost?
(a) 18 cents (b) 20 cents (c) 22 cents (d) 23 cents (e) 24 cents

Passage
All the questions are of multiple choice type. You have to answer the questions based on the preceding paragraph. All the questions have the same answer choice. The choices are as given below:
(a) True.
(b) False.
(c) Cannot Say.

Consider the following paragraph:
Researchers in Mumbai have found that certain types of gallstones can be dissolved by injecting them with a gasoline additive in the form of ether. The ether is injected through a tube directly into the gall bladder. The one-day treatment works only on cholesterol-based stones, not those composed largely of calcium. However, as the cholesterol stones are by far the most common type, for millions of gallstone sufferers the treatment should offer a welcome alternative to surgery, the commonest option in most hospitals.

# "It takes more than one day for ether to dissolve a calcium-based gallstone".

# "Gallstones can only be dissolved by injections".

# "Gallstones can quickly be cured with surgery".

# "Ether is largely used for dissolving gallstones".
# "Calcium stones can be cured in one day".
# "Hundreds of people contains calcium stones".
Consider the following paragraph:
My father had no brothers, but his three sisters are all married and each has two children. My grandfather has two sons.

# "My father was the only child".

# "I have only one uncle".

# "One of my aunts is a spinster".

# "I have six cousins on my father's side".

# "My grandfather was the only son".
Consider the following paragraph:
In the Totalitarian days, the words have very much devalued. In the present day, they are becoming domestic, that is, the words will be much more devalued. In that days, the words will be very much effected in political area. But at present, the words came very cheap. We can say they come free at cost.

# "In Totalitarian society, words are devalued".

# "Totalitarians will have to come much about words".

# "In the Totalitarian society the words are used for the political speeches".

Consider the following paragraph:
In past helicopters were forced to ground or crash because of the formation of the ice on the rotors and engines. A new electronic device has been developed which can detect the water content in the atmosphere and warns the pilot, if the temperature is below freezing temperature; about the formation of ice on the rotors and wings.

# "The electronic device can avoid the formation of ice on the wings".

# "There will be malfunction of rotor and engine because of formation of ice".

# "The helicopters were to be crashed or grounded".

# "There is only one device that warns about the formation of ice".

Consider the following paragraph:
Human existence is not susceptible of arbitrary division between consciousness and unconsciousness. The conscious world invaders and shapes the activities of the unconscious, while many of the great achievements of humanity's waking hours where wholly or partly inspired by dreams. Even if it could argued that dreams precede experience such a dichotomy could not be drawn, as the influence of the dreaming on the waking state would remain unclear, but as yet no common vocabulary exists to record the substance of prenatal dreaming.

# "Sleep can be a creative state".

Consider the following paragraph:
FLORA 3-piece sofa-set is at the top of our upholstery range. This high-backed quality sofa-set boasts an impressive specification which starts with a hardwood frame in teak and a padded front edge ensuring really deep, long-lasting comfort. Seat cushions are of high resilience foam and back cushions of softest hollow fill. The whole set is carefully upholstered throughout in a choice of superb fabrics ranging from cotton print to velvet.

# "The padding is there to ensure that the furniture will last for a long time".

# "The firm sells other upholstery furniture".
Consider the following paragraph:
Hacking is a crime made possible by a relatively new technology, which is one of the reasons it is often poorly understood and reported. Many computers, but by no means all, are now linked together in networks which allow users on one computer to communicate with others on the same network. If a computer is not networked, no manipulation of its data from another machine possible. So long as users are authorized, networking is just a way of making work easier more productive. Hacking, on the other hand is the unauthorized use of networks or unauthorized entry into the computers themselves. Most people do not break into the networks they use, since they are already accredited users.

# "Hackers do not work for the firms whose networks they break into".
# "Hacking is the only vulnerability of the computers for the usage of the data".
# "Hacking is done mostly due to the lack of computer knowledge".
Consider the following paragraph:
Polycythemia often occurs in people who have chronic lung disease, but can appear spontaneously in healthy individuals. Excessive numbers of red blood cells manufactured by the body and the individual then develops a very healthy-looking, ruddy complexion. The blood becomes thicker and is liable to clot and block major blood vessels. High blood pressure is another frequent complication. Treatment involves venesection, in which a liter or so of blood is removed from the body. Medication may also be given to reduce the numbers of red blood cells manufactured in the body.

# "Lung disease frequently precedes polycythemia".

Consider the following paragraph:
Bindweed is only effectively controlled by applying a solution of brushwood-killer to the growing tips. It is necessary to unwind a suitable length from the host plant before treatment, but this is not so very difficult, and it does not seem essential to find and treat every leader on the same weed. The solution should be made up in a can which is carried in one hand, while the other, in a rubber glove, inserts the leaders in the can. If the the leaders can be laid out on the the ground , they can easily be wetted with a small brush. As long as the weather is calm, there is no real risk of damage to adjacent plants, and in two or three weeks the weeds should have disappeared.

# "Brushwood-killer can pose a threat to other plants in the garden".

Consider the following paragraph:
Senior managers in a leading company said that new Japanese investment in India was transforming the car industry, and warned that jobs were under threat from Japanese competition. They stated that increasing competition would be coupled with an inevitable downturn in the car market and the recent rise interest rates which has already hit demand.

# "The managers issued their warning after a rise in the interest rates".

# "According to the senior managers, the Japanese investment in India will lead to a glut in the car market".

# "Some senior managers said that more people will want to by new cars in the future".

# "The perception of the senior managers is the new Japanese investment in India is leading to more automation of the car industry".

# "The increased rate of interest will mean that Japanese firms will cease to operate in this country".

# "The increase in loan interest will adversely affect car sales".
# "Japanese workers are taking over the jobs of Indian industry".
# "Managers said that interests in car will go down after seeing the raise in interest rates".
# "People are very interested to buy the cars".
Consider the following paragraph:
The new Starfire has an advanced four-cylinder engine with catalytic converter and uses only unleaded petrol. Versatility is a major feature of the range and the 1500 and 1800 models have the same high level of specification inside and out. The only obvious visual difference, internally and externally, is the use of alloy wheels on the 1800 version, together with a discreet change in badging. The StarFire 2000 is distinguished by its tailgate spoiler and the rectangular fog and driving lamps integrated into the front bumper which are also included in the specification.

# "Internally, the Starfire 2000 looks like the 1500 model".

Consider the following paragraph:
Pierre Claude Jean Allouez explored lake superior from 1665 to 1667. At his little mission station near the western end of the lake, he heard from the Indians of a great river to the west. Pierre Jacques Marquette determined to investigate. In 1673, accompanied by Louis Jolliet and five others, he left St. Ignace mission and ascended the fox river, which flows into green bay crossed over to Wisconsin river and followed it to the upper Mississippi. The party then descended the Mississippi to the mouth of Arkansas. These Frenchmen were not first Europeans to sight or travel the Mississippi. De Soto and Moscoso had done so a century and a half before. The report of the exploration was rushed back to Quebec, where, in 1672, Count Frontenac had arrived as Governor of the province. He and his friend, the remarkable La Salle-who earlier may have penetrated the Ohio river valley-listened with deep interest.

# "Allouez explored the western end of lake superior".

# "Marquette and his party were not the first French men to travel the Mississippi river".

# "La Salle listened with deep interest- the report of exploration of De Soto and Moscoso".

# "La Salle explored the Mississippi river valley".
Consider the following paragraph:
Dr. Goddard was the first to fire a rocket that reached a speed faster than the speed of sound. He was the first to develop a gyroscopic steering apparatus for rockets. He was the first to use vanes in the jet stream for rocket stabilization during the initial phase of a rocket flight. And he was the first to patent the idea of step rockets. After proving on paper and in actual tests that a rocket can travel in vacuum, he developed the mathematical theory of rocket propulsion and rocket flight, including basic designs for long-range rockets. All of this information was available to our military men before World War II, but evidently its immediate use didn't seem applicable. Near the end of World War II we started intense work on rocket-powered guided missiles, using the experiments and developments of Dr. Goddard and the American Rocket Society.

# "The stabilization problem of rockets in the initial phase was solved by Dr. Goddard."

# "Rockets can travel faster than sound, thanks to gyroscopic steering."

# "Goddard lived before World War II".

# "After careful mathematical calculations, Dr. Goddard proved that rockets can travel in vacuum".

Consider the following paragraph:
In March 1513, de Leon sailed off confidently from Peuto Rico for the Bahamas. Landing briefly at San Salvador, Bahamas, he wound through unchartered islands until he sighted an extensive coastline. He had no reason to suspect that it is anything more than an island, but he followed the coast for a day without rounding its end or finding a suitable landing place. He named the "island" La Florida. This name came to be applied by the Spanish to the entire Southeastern United States and beyond. Then, near the 30th parallel, de Leon landed at the mouth of the St. Johns river. Determined to be the first to circumnavigate the "island", he turned south, traced the coast around the tip of the peninsula, moved to the west, perhaps reaching Tampa bay. After 7 weeks, he gave up hope of circling the northern tip of this "island"; it was incredibly large and he may have suspected that he had discovered the long sought mainland. If so, it all belonged to his King, for he had earlier planted the Spanish flag and claimed Florida for Ferdinand.

# "de Leon is from Spain, ruled by Ferdinand".

# "de Leon is very patriotic".

# "de Leon discovered part of US during his journey".

Consider the following paragraph:
James Madison understood that interests groups will inevitably develop within a free political system. The problem, as Madison saw it, was to prevent any single interest group from becoming so strong that it was able to dominate the political system. This could be accomplished by legislating restrictions on political behavior, but that solution meant a sacrifice of some of the freedom that Madison prized so highly. A better solution, he thought, was to extend the territorial scope of the government. This would allow for greater diversity of interests in the nation, and a greater number of groups competing for power. Each interest group would thereby find it more difficult to appeal to a majority of the people, and to dominate the political process.

# "The more interest groups there are in a political system, the less freedom there is for everyone."

# "Legislating restrictions on political behavior is sometimes the only method of preserving political freedom."

# "Increasing the territorial scope of a government can help to preserve freedom."

# "According to Madison, in a free political system, interest groups are undesirable."

Consider the following paragraph:
The regulations and expenses to invent, patent and market new ideas and products imposes a heavy burden on inventors. The cost is often absorbed by large corporations with research and development facilities they provide. Corporations also help creative people contribute to society without suffering the loss of income or security of the private inventor. The realities of this arrangement are that many good ideas are never brought into the marketplace and the cost of products on the market is high because of the development cost. However, protection provided by the patents and the safety to the public to avoid placing harmful products on the market is important to maintain. Thus, as is often the case, rules and regulations have their favorable and unfavorable consequences.

# "The regulations and expenses to invent, patent and market new ideas is an expensive proposition to the inventors".

# "Good ideas are never brought into the marketplace because of the costs involved in inventing, patenting marketing them".

# "Corporations steal the individual inventor of their inventions".
Consider the following paragraph:
Being born female and black were two handicaps Gwendolyn Brooks states that she faced from her birth, in 1917, in Kansas. Brooks was determined to succeed. Despite the lack of encouragement she received from her teachers and others, she was determined to write and found the first publisher for one of her poems when she was 11. In 1945, she marketed and sold her first book; national recognition ensued. She applied for and received grants and fellowships from such organizations as the AAAL and the Guggenheim Foundation. Later she received the Pulitzer prize for poetry; she was the first black woman to receive such an honor. Brooks' reaction to fame is atypical. She continues to work and work hard. She writes, travels, and helps many who are interested in writing. Especially important for her is increasing her knowledge of her black heritage and encouraging other people to do the same. She encourages dedication to the art to would-be writers.

# "Brooks' story illustrates the power of strong determination".

# "She became the author of a book in her teens".

# "Gwendolyn received the Pulitzer prize for her first poetry".

Consider the following paragraph:
A cave is a natural opening in the ground extending beyond the zone of light and large enough to permit the entry of man. Occurring in a wide variety of rock types and caused by widely differing geological processes, caves range in size from single small rooms to interconnecting passages many miles long. The scientific study of caves is called speleology. It is a composite science based on geology, hydrology, biology and archeology, and thus holds special interest for earth scientists. Caves have been natural attractions since prehistoric times. Prolific evidence of early man's interest has been discovered in caves scattered throughout the world. Skeletons of some of the earliest manlike creatures (Australopithecines) have been discovered in cave deposit in South Africa, and the first evidence of primitive Neanderthal man was found in Germany. Cro-Magnon man created his remarkable murals on the walls of caves in France.

# "Primitive human form originated in Germany".

# "Study of caves is the study of earth, water, life and early man".

# "Cro-Magnon man was more intelligent than Neanderthal man".

# "Caves are a natural attraction because they reveal information about the early man".

Consider the following paragraph:
Although invaders represent the threat to the conservation of flora and fauna, there are two special cases in which invasion have been deliberately brought about. One is the desire to control pests by natural predators, which may have to be brought from other countries. The second is releasing organisms into the wild (or on to farms, from which they might escape) that are completely novel, because they have been genetically engineered. There is nothing intrinsically sinister about engineered organisms, but any novelty must be regarded as potential invader.

# "Pests are more dangerous than their natural predators".

Consider the following paragraph:
Life in colonial times was harsh, and the refinements of the mother country were ordinarily lacking. The colonists, however, soon began to mold their English culture into the fresh environment of new land. The influence of religion permeated the entire way of life. In most Southern colonies, the Anglican church was the legally established church. In New England, the Puritans were dominant; and in Pennsylvania, the Quakers. Especially in the New England colonies, the local or village church was the hub of community life; the authorities strictly enforced the Sabbath and sometimes banished non-believers. Unfortunately, the same sort of religious intolerance, bigotry and superstition associated with the age of Reformation in Europe also prevailed in some of the colonies, though on a lesser scale. In the last half of the 17th century, during sporadic outbreak of religious fanaticism and hysteria, Connecticut authorities tried and hanged several women as "witches". Early in the 17th century, some other witchcraft persecution occurred in Virginia. As the decades passed, however, religious tolerance developed in colonies.

# "New England was part of the Southern colonies".

# "During the mid 17th century there was significant improvement in religious tolerance and superstition".

# "Life in colonial times was harsh due to the strong influence of religion".

# "The Anglican church used to govern the people in most Southern colonies".
Consider the following paragraph:
Confucius said that to know the future we have to understand the past. in his time, transport, communications and scientific knowledge were less developed than they are today. news took weeks to travel whereas today satellite links connect the continents virtually instantaneously, but our technological advances in the field of communications seem not to improved our capacity to understand one another.

# "We understand each other better now than in Confucius' time because we can travel more quickly".

# "In Confucius' day people were more intelligent".

# "We have made great improvements in transport since Confucius' day".

# "Technological advances in communication and human capacity to understand one another are directly proportional".

# "In Confucius' day time news took months to travel".

# "According to Confucius the past has a linkage to the future".
# "Even with the fast developments of the technology we can't live happily".
Consider the following paragraph:
Every form of art is protected by copyright, upon the expiration of which the property passes to the public domain and becomes freely available to anyone wishing to exploit it commercially. the time has come when all treasures should pass to the control of a trust, and by this be made readily available to anyone on payment of a fee or royalty. The income from the works Van Gogh would alone be enormous. Those who now gain financial benefit from his genius should make some contribution to the welfare of the arts in general.

# "Instead of buying a ticket, museum goers should pay a fee to a trust for the benefit of arts".

# "It is not desirable to pass the control of treasures to a trust".

# "Van Gogh's paintings are not protected by copyright".

# "All artworks must be managed by a trust, so that the income generated can be used for the welfare of the arts".

# "Copyright in art is valid only for a limited period of time".

# "Van Gogh's descendants should be asked to make some contribution to the arts".
# "Van Gogh's works are under this copy right rule".
# "People are free to go to the public because of the copy right rule".
# "People gives to theater and collect the money for development".
# "We have asked the Van Gogh descendants to help for the developments of art".
Consider the following paragraph:
Organizing the home can be perceived as conferring power, so large numbers of women are unwilling to let go of chores, even when they have careers. A survey found that, out of 65 new marriages, not one single wife expected her husband to share work equally. According to the Family Policy Studies Center, 81% of working wives return home to do all the cooking. The average male has nearly half as much as more free time at weekends than his wife, and the typical new father spends just 37 seconds a day talking to his baby.

# "Only career women perceive organizing the home as conferring power".

# "The average wife has half as much free time at weekends as her husband".

# "The family planning studies center shows that 81% working wives do all the cooking at home".

# "19% working wives do not want to do the cooking at home".
# "Housewives want the husbands to take part equally in the household".
# "Wives have half as much leisure time as the husbands have".
# "39% of the men will work equally in the house in cleaning and washing".
Consider the following paragraph:
Statistics show that millions of vehicles have been carried by shuttle over the past 30 years through Alpine tunnels without one ever catching fire. In the Alpine tunnels, drivers and passengers sit in their vehicles on the shuttle trains. Only one vehicle has ever caught fire on the busy French motorail equivalent system. This sort of accident is not possible in a closed shuttle. Assertion that a vehicle fire will lead to catastrophe have no basis. Since the resources exist to detect, control and extinguish a fire, and to remove any persons present safely to an adjoining wagon, leaving any surviving fire facing rapid extinction within a wagon built to contain fire for 30 minutes, catastrophe seems very unlikely.

# " It is theoretically possible for a vehicle to catch fire even in a closed wagon".

# " The French motorail system is inferior to the shuttle train system."
# "No accident can occur in the closed tunnels".
# "Fire is allowed to live for 30 min".
# "All the cars that travel in the tunnels will be carried by rail shutters".
***********************
APTITUDE SECTION
Q1. Mr. Shah decided to walk down the escalator of a tube station. He found
that if he walks down 26 steps, he requires 30 seconds to reach the bottom.
However, if he steps down 34 stairs he would only require 18 seconds to
get to the bottom. If the time is measured from the moment the top step begins
to descend to the time he steps off the last step at the bottom, find out
the height of the stair way in steps?
Ans.46 steps.
Q2. The average age of 10 members of a committee is the same as it was 4
years ago, because an old member has been replaced by a young member. Find
how much younger is the new member ?
Ans.40 years.
Q3. Three containers A, B and C have volumes a, b, and c respectively; and
container A is full of water while the other two are empty. If from container
A water is poured into container B which becomes 1/3 full, and into
container C which becomes 1/2 full, how much water is left in container A?
Q4. ABCE is an isosceles trapezoid and ACDE is a rectangle. AB = 10 and EC = 20.
What is the length of AE?
Ans. AE = 10.
Q5. In the given figure, PA and PB are tangents to the circle at A and B respectively and
the chord BC is parallel to tangent PA. If AC = 6 cm, and length of the tangent AP
is 9 cm, then what is the length of the chord BC?
Ans. BC = 4 cm.
Q6. Three cards are drawn at random from an ordinary pack of cards. Find
the probability that they will consist of a king, a queen and an ace.
Ans. 64/2210.
Q7. A number of cats got together and decided to kill between them 999919
mice. Every cat killed an equal number of mice. Each cat killed more mice
than there were cats. How many cats do you think there were ?
Ans. 991.
Q8. If Log2 x - 5 Log x + 6 = 0, then what would the value / values of x
be?
Ans. x = e2 or e3.
Q9. The square of a two digit number is divided by half the number. After
36 is added to the quotient, this sum is then divided by 2. The digits of the
resulting number are the same as those in the original number, but they
are in reverse order. The ten's place of the original number is equal to twice
the difference between its digits. What is the number?
Ans. 46
Q10. Can you tender a one rupee note in such a manner that there shall be
total 50 coins but none of them would be 2 paise coins.?
Ans. 45 one paisa coins, 2 five paise coins, 2 ten paise coins, and 1 twenty-five paise coins.
Q11. A monkey starts climbing up a tree 20ft. tall. Each hour, it hops 3ft.
and slips back 2ft. How much time would it take the monkey to reach the
top?
Ans.18 hours.
Q12. What is the missing number in this series?
8 2 14 6 11 ? 14 6 18 12
Ans. 9
Q13. A certain type of mixture is prepared by mixing brand A at Rs.9 a kg.
with brand B at Rs.4 a kg. If the mixture is worth Rs.7 a kg., how many
kgs. of brand A are needed to make 40kgs. of the mixture?
Ans. Brand A needed is 24kgs.
Q14. A wizard named Nepo says "I am only three times my son's age. My father
is 40 years more than twice my age. Together the three of us are a mere 1240
years old." How old is Nepo?
Ans. 360 years old.
Q15. One dog tells the other that there are two dogs in front of me. The
other one also shouts that he too had two behind him. How many are they?
Ans. Three.
Q16. A man ate 100 bananas in five days, each day eating 6 more than the
previous day. How many bananas did he eat on the first day?
Ans. Eight.
Q17. If it takes five minutes to boil one egg, how long will it take to boil
four eggs?
Ans. Five minutes.
Q18. The minute hand of a clock overtakes the hour hand at intervals of 64
minutes of correct time. How much a day does the clock gain or lose?
Ans.32 8/11 minutes.
Q19. Solve for x and y:
1/x - 1/y = 1/3, 1/x2 + 1/y2 = 5/9.
Ans. x = 3/2 or -3 and y = 3 or -3/2.
Q20. Daal is now being sold at Rs. 20 a kg. During last month its rate was
Rs. 16 per kg. By how much percent should a family reduce its consumption so
as to keep the expenditure fixed?
Ans. 20 %.
Q21. Find the least value of 3x + 4y if x2y3 = 6.
Ans. 10.
Q23. Can you find out what day of the week was January 12, 1979?
Ans. Friday.
Q24. A garrison of 3300 men has provisions for 32 days, when given at a rate
of 850 grams per head. At the end of 7 days a reinforcement arrives and it was
found that now the provisions will last 8 days less, when given at the rate
of 825 grams per head. How, many more men can it feed?
Ans. 1700 men.

Q25. From 5 different green balls, four different blue balls and three
different red balls, how many combinations of balls can be chosen taking at least
one green and one blue ball?
Ans. 3720.
Q26. Three pipes, A, B, & C are attached to a tank. A & B can fill it in 20
& 30 minutes respectively while C can empty it in 15 minutes. If A, B & C
are kept open successively for 1 minute each, how soon will the tank be filled?
Ans. 167 minutes.
Q27. A person walking 5/6 of his usual rate is 40 minutes late. What is his
usual time?

Ans. 3 hours 20 minutes.
******************
VERBAL SECTION
Directions: Give the synonyms for the following words
1. Depreciation: deflation, depression, devaluation, fall, slump
2. Depricate : feel and express disapproval,
3. Incentive : thing one encourages one to do (stimulus)
4. Echelon : level of authority or responsibility
5. Innovation : make changes or introduce new things
6. Intermittent : externally stopping and then starting
7. Detrimental: harmful
8. Conciliation : make less angry or more friendly

9. Orthodox: conventional or traditional, superstitious

10. Fallible : liable to error

11. Volatile : ever changing

12. Manifest: clear and obvious

13. Connotation : suggest or implied meaning of expression

14. Reciprocal: reverse or opposite

15. Agrarian : related to agriculture

16. Vacillate : undecided or dilemma

17. Expedient : fitting proper, desirable

18. Simulate : produce artificially resembling an existing one.

19. Access : to approah

20. Compensation: salary

21. Truncate : shorten by cutting

22. Adherence : stick

23. Heterogenous: non similar things

24. Surplus : excessive

25. Assess : determine the amount or value

26. Congnizance : knowledge

27. Retrospective : review

28. Naive : innocent,rustic

29. Equivocate : tallying on both sides, lie, mislead

30. Postulate : frame a theory

31. Latent : dormant, secret

32. Fluctuation : wavering,

33. Eliminate : to reduce

34. Affinity : strong liking

35. Expedite : hasten

36. Console : to show sympathy

37. Adversary : opposition

38. Affable : lovable or approachable

39. Decomposition : rotten

40. Agregious : apart from the crowd, especially bad

41. Conglomaration: group, collection

42. Aberration: deviation

43. Aurgury : prediction

44. Crediability : ability to common belief, quality of being credible

45. Coincident: incidentally

46. Constituent : accompanying

47. Differential : having or showing or making use of

48. Litigation : engaging in a law suit

49. Maratorium: legally or offficiallly determined period of dealy before
fulfillment of the agreement of paying of debts.

50. Negotiate : discuss or bargain

51. Preparation : act of preparing

52. Preponderant : superiority of power or quality

53. Relevance : quality of being relevant

54. Apparatus : appliances

55. Ignorance : blindness, in experience

56. Obsession: complex enthusiasm

57. precipitate : speed,active
SERIES SECTION
Directions: In the following questions complete the series
NOTE: This section is quite tough and consists of 26 questions to be done in 10 minutes. Please keep track of time.
1. A C B D E F G I - I H K J L
Ans. H


2. A I Z B E Y C I X D I - G E N J W
Ans. W


3. A D G J M P - R W T S
Ans. S


4. A B C E F G I J K - M L O N P
Ans. M

5. A B F G K L P Q - T S V U W
Ans. U


6. J W X U V S T - Q P S E T
Ans. Q


7. A R H X Y T D T W S T - N P T K R
Ans. P


8. F M B I P Z V I E V - I R Y O U


9. N Z I Y C X KW F - J F V M Y
Ans. V


10. A A S A S P A S P K A - R Q T S U
Ans. S


11. A E C P S - T R U E
Ans. U


12. B B P R D D L N F F I K - H Q J I K
Ans. H


13 A Z E X I V M T - R Q N S O
Ans. Q


14. A B D G K P - L I W U X
Ans. U


15. B C D A E G H I F J L M N L K N M O
Ans. K


16. X W E F G V U H I J K - P N S R T
Ans. T


17. O D J T O P Q N O E R T - Q O U V W
Ans. O


18. P R N U U P E J R B B - H V U N E
Ans. E


19.L U L M G M N F N P S - O N Q P S
Ans. P
NUMERICAL ABILITY
1. 420% OF 7.79 = ?
Ans. 32.718

2. 3427 / 16.53 = ?
Ans. 202
3. 10995 /95 = ?
Ans.115.7365
4. 43+557-247 =?
Ans. 353
5. 3107*3.082= ?
Ans. 9591
6. 48.7 + 24.9 - 8.7 = ?
Ans. 64.90
7.525.0/47.8 = ?
Ans. 11
8. (135-30-14)*7 - 6 +2 = ?
Ans. 3
9. 3/8 * 5.04=?
Ans. 1.89
10. 697 /219 = ?
Ans. 3.18
11.8/64 +64/16 =?
Ans. 4.14
12. 298 * 312 / 208 = ?
Ans. 453.54
13. 0.33 *1496 /13 = ?
Ans. 37.98
14.0.26 + 1/8 = ?
Ans. 0.385
15. 66.17+1/3= ?
Ans. 67.03
16. 2.84+1/4= ?
Ans. 3.09
17. 33% OF 450 = ?
Ans. 148.5
18. 907.54 / 0,3073= ?
Ans.3002
19.There are two categories of persons in ratio A:B = 2:3. A type earns 2.5 dollars/hr and
B type 1 dollar/hr total money earned by both is 24dollars. Then what is the total number of persons
Ans. 15
20. Total balls are z, the number of red balls is n and the remaining are blak balls,then the % of black
balls equal to ?
Ans. (z - n) / z*100
21. If A = C, B = 2D what should be done to make the ratio same. i.e.a/b = c/d
Ans. Multiply A by 2
22. If P=Total number of components, Q = number of defective components .What is the % of non defective components?
Ans. (p-q) / p*100
23. If the cost of an article is x , first discount given is y% of cost, second discount given is z% of cost .
The selling price of x is

Ans. x (1-y / 100) (1- z / 100)
24.Which of the following are prime numbers
(a) 119
(b) 115
(c) 127
(d) none
Ans. (c)
25. A / B = C; C > D then
(a) A is always greater than D
(b) C is always greater than D
(c) B is always less than D
(d) None of these
Ans. (a)
26. If B>C and AAns. ( A + B )C
27. If for H hours of work the salary is S and the employee gets x hours of medical leave, then what is the salary/hr ?
Ans. s/H-x
28. ( 1/6 of 596) / (0.695) = ?
Ans. 142
29. 35-30 + 4/7 - 5 + 1 = ?

Ans. 3
30. 10995 + 95 = ?
Ans. 11090
31. If on a salary of Rs."S" per month,one has to pay one tax of x Rs. and a second type of tax of y Rs
then % of salary taken home is?

Ans. s-(x+y)/s * 100
32. B>A then which expression will be highest value
(a) A-B
(b) AB
(c) A+B
(d) Can't Say
Ans. (b)
33. K, L are men who take home a salary of x, y respectively.The total amount taken home is
Ans. Kx + Ly

34. If out of X bulbs y bulbs are broken;The % of non broken bulbs
Ans. (x-y) / x*100

35. If on a salary s per month, a tax of x% of the salary and another of r% of the salary is deducted what
is the income.

Ans. s*(1-(x+r)/100
36. 0.512 * 18902358 =?
Ans. 9678007.296
37. If the % of defective balls is 10% balls,and the number of defective balls is 5.The number of balls is

Ans.50

38. 6.29% of 2.8 =?
Ans. 0.18
39. 0.398 * 456= ?
Ans. 181.49

40. 0 < x < 1 which is greater
(a) 1/x2
(b) 1/x
(c) x
(d) x2

Ans. (a)
41. If c = a/b; a-1 = c, what is the relation between a and b?

Ans. b = a/a-1
42. What is the sum of 7 consecutive odd numbers with 27 as the fourth number
Ans.189

FLOWCHART SECTION
Directions: There are 7 flow charts and each has 5-6 blank rectangles/diamonds with
subquestion number in the rectangle/diamond. You have to fill the blank from the
5 options given against respective question number
NOTE:These types of questions are not at all tough.You have to understand the logic and then it is very easy to fill the blanks. Some information is provided for getting to the answers.There will be blanks which have to be filled.
Examples of flow charts asked to be filled :
(1) There are 3 boxes of 3 balls each. you have to select the heaviest among all.
(2) There are red and black balls in a box. You select some balls from the blocks.If the ball chosen is red then you get one point. If the chosen ball is ball black and previous ball is red then you get two points. For winning u have to get seven points. No point for selecting consecutive balls of the same color.
(3)Classify objects in class A, class B and scrap. for classfing you have to do different tests such as weight test, material test etc.
(4)There is production process in which action depends on temperature and pressure and we have some temperature and pressure controls.Draw a flowchart to complete the process.
(5)Find max. and min. of the 12 nos.in an array.Arrang the array in ascending order and find the maximum and
minimum value in the array
(6)Diffrent age group are given and also diffrent salary slabs are given. Depending on the salary group as well as his group you have to classify the group of people in particular class.
***********************
1. Last month of an year
(a) January (b) February (c) December (d) November

2. Select the odd one
(a) January (b) February (c) Wednesday (d) November

3. Select the antonym of capture from the following
(a) attack (b) Release (c) condemn (d) None of the above

4. Find the antonym of autumn
(a) Spring (b) Winter (c) Summer (d) None of the above

5. One skirt requires 3.75 yards of cloth. How many skirts you can make from 45 yards?
Ans: 12 skirts

6. How can you make a square from two triangles?

7. Is the meaning of Client and Customer,
(a) same (b) contradictory (c) no relation

8. Is the meaning of It's and Its,
(a) same (b) contradictory (c) no relation

9. Is the meaning of Canvas and Canvass,
(a) same (b) contradictory (c) no relation

10. Is the meaning of Ingenious and Ingenuous,
(a) same (b) contradictory (c) no relation

11. Is the meaning of Credible and Credulous,
(a) same (b) contradictory (c) no relation

12. Select the odd one out.
(a) 1/4 (b) 1/3 (c) 1/6 (d) 1/18

13. Select the least from the following.
(a) 0.99 (b) 1 (c) 81 (d) 0.333

14. Find the next number in the series. 1, 0.5, 0.25, 0.125
Ans: 0.0625

15. One dollar is saved in one month. Then how much dollar is saved in one day?
Ans: 1/30 =0.0333$

16. Y catches 5 times more fishes than X. If total number of fishes caught by X and Y is 48, then number of fishes caught by X?
Ans: 8

17. Y catches 5 times more fishes than X. If total number of fishes caught by X and Y is 42, then number of fishes caught by X?
Ans: 7

18. If a train covers 600m in 0.5 seconds, how long it will cover in 10 seconds?
Ans: 3000m = 3km

19. The girl's age is twice that of boy, if the boy is four years old. After four years the age
of the girl is
Ans: 12 years

20. Sister's age is twice than that of the brother. If the brother's age is six, what is the sister's age after two years?
Ans: 14 Yrs.

21. Two lemons cost 10 cents. Then one and a half dozen cost
Ans: 90 cents

22. A clock is late by 1 minute 27 seconds in a month. Then how much will it be late in 1 day?
Ans: 2.9 seconds

23. Which of the following figures together will make a triangle?
Ans: a,b,c,d

24. Make a square by drawing only one line
Ans: line 2-5, square 2-3-4-5-2

25. Which of the following is the odd one? crew, constellation, companion, league, participants.
Ans: companion

26. Opposite of Remote?
(a) Far (b) Near (c) Huge (d) Village

27. Statement A: All great men are ridiculous;
Statement B: I am ridiculous ;
Inference : I am a great man;
(a) True (b) False (c) Not clear

28. Statement: Normal children are active;
Inference: All children are active;
(a) True (b) False (c) Uncertain

29. Next number in the series 1, 1/2, 1/4, 1/8 ?
Ans: 1/16

30. In 6 seconds a light flashes once. In one hour how many times it will flash?
Ans: 601 times

31. At 20% discount, a cycle is sold at a selling price of 2500 Rs. What is the actual price?
Ans: Rs. 3125

32. Statement A: A & B have same age;
Statement B: B is younger than C;
Inference : A is younger than C;
(a) True (b) False (c) Uncertain

33. All chickens lay eggs (True/False)
Ans: False

34. A invests $12000, B invests $8000, C invests $6000 and they got a profit of $1200. How much share A got more than B and C?
Ans: 2/13 and 3/13


In a class composed of x girls and y boys what part of the class is composed of girls
A.y/(x + y) B.x/xy C.x/(x + y) D.y/xy (Ans.C)

What is the maximum number of half-pint bottles of cream that can be filled with a 4-gallon can of cream(2 pt.=1 qt. and 4 qt.=1 gal)
A.16 B.24 C.30 D.64 (Ans.D)

.If the operation,^ is defined by the equation x ^ y = 2x + y,what is the value of a in 2 ^ a = a ^ 3

A.0 B.1 C.-1 D.4 (Ans.B)

A coffee shop blends 2 kinds of coffee,putting in 2 parts of a 33p. a gm. grade to 1 part of a 24p. a gm.If the mixture is changed to 1 part of the 33p. a gm. to 2 parts of the less expensive grade,how much will the shop save in blending 100 gms.

A.Rs.90 B.Rs.1.00 C.Rs.3.00 D.Rs.8.00 (Ans.C)

There are 200 questions on a 3 hr examination.Among these questions are 50 mathematics problems.It is suggested that twice as much time be spent on each maths problem as for each other question.How many minutes should be spent on mathematics problems

A.36 B.72 C.60 D.100 (Ans.B)

In a group of 15,7 have studied Latin, 8 have studied Greek, and 3 have not studied either.How many of these studied both Latin and Greek

A.0 B.3 C.4 D.5 (Ans.B)

If 13 = 13w/(1-w) ,then (2w)2 =

A.1/4 B.1/2 C.1 D.2 (Ans.C)

If a and b are positive integers and (a-b)/3.5 = 4/7, then


(A) b < a (B) b > a (C) b = a (D) b >= a (Ans. A)

In june a baseball team that played 60 games had won 30% of its game played. After a phenomenal winning streak this team raised its average to 50% .How many games must the team have won in a row to attain this average?


A. 12 B. 20 C. 24 D. 30 (Ans. C)

M men agree to purchase a gift for Rs. D. If three men drop out how much more will each have to contribute towards the purchase of the gift/

A. D/(M-3) B. MD/3 C. M/(D-3) D. 3D/(M2-3M) (Ans. D)

. A company contracts to paint 3 houses. Mr.Brown can paint a house in 6 days while Mr.Black would take 8 days and Mr.Blue 12 days. After 8 days Mr.Brown goes on vacation and Mr. Black begins to work for a period of 6 days. How many days will it take Mr.Blue to complete the contract?

A. 7 B. 8 C. 11 D. 12 (Ans.C)

2 hours after a freight train leaves Delhi a passenger train leaves the same station travelling in the same direction at an average speed of 16 km/hr. After travelling 4 hrs the passenger train overtakes the freight train. The average speed of the freight train was?

A. 30 B. 40 C.58 D. 60 (Ans. B)

If 9x-3y=12 and 3x-5y=7 then 6x-2y = ?

A.-5 B. 4 C. 2 D. 8 (Ans. D)

There are 5 red shoes, 4 green shoes. If one draw randomly a shoe what is the probability of getting a red shoe (Ans 5c1/ 9c1)

What is the selling price of a car? If the cost of the car is Rs.60 and a profit of 10% over selling price is earned (Ans: Rs 66/-)

1/3 of girls , 1/2 of boys go to canteen .What factor and total number of classmates go to canteen.
Ans: Cannot be determined.


The price of a product is reduced by 30% . By what percentage should it be increased to make it 100% (Ans: 42.857%)

There is a square of side 6cm . A circle is inscribed inside the square. Find the ratio of the area of circle to square. (Ans. 11/14 )

There are two candles of equal lengths and of different thickness. The thicker one lasts of six hours. The thinner 2 hours less than the thicker one. Ramesh lights the two candles at the same time. When he went to bed he saw the thicker one is twice the length of the thinner one. How long ago did Ramesh light the two candles .
Ans: 3 hours.

If M/N = 6/5,then 3M+2N = ?

If p/q = 5/4 , then 2p+q= ?

If PQRST is a parallelogram what it the ratio of triangle PQS & parallelogram PQRST . (Ans: 1:2 )

The cost of an item is Rs 12.60. If the profit is 10% over selling price what is the selling price ?

(Ans: Rs 13.86/-

There are 6 red shoes & 4 green shoes . If two of red shoes are drawn what is the probability of getting red shoes (Ans: 6c2/10c2)

To 15 lts of water containing 20% alcohol, we add 5 lts of pure water. What is % alcohol. (Ans : 15% )

A worker is paid Rs.20/- for a full days work. He works 1,1/3,2/3,1/8.3/4 days in a week. What is the total amount paid for that worker ? (Ans : 57.50 )

If the value of x lies between 0 & 1 which of the following is the largest?

(a) x b) x2 (c) –x (d) 1/x (Ans : (d) )

If the total distance of a journey is 120 km .If one goes by 60 kmph and comes back at 40kmph what is the average speed during the journey? Ans: 48kmph

A school has 30% students from Maharashtra .Out of these 20% are Bombey students. Find the total percentage of Bombay? (Ans: 6%)

An equilateral triangle of sides 3 inch each is given. How many equilateral triangles of side 1 inch can be formed from it? (Ans: 9)

If A/B = 3/5,then 15A = ? (Ans : 9B)

Each side of a rectangle is increased by 100% .By what percentage does the area increase? (Ans : 300%)

Perimeter of the back wheel = 9 feet, front wheel = 7 feet on a certain distance, the front wheel gets 10 revolutions more than the back wheel .What is the distance? Ans : 315 feet

Perimeter of front wheel =30, back wheel = 20. If front wheel revolves 240 times. How many revolutions will the back wheel take? Ans: 360 times

20% of a 6 litre solution and 60% of 4 litre solution are mixed. What percentage of the mixture of solution

(Ans: 36%)

City A's population is 68000, decreasing at a rate of 80 people per year. City B having population 42000 is increasing at a rate of 120 people per year. In how many years both the cities will have same population? (Ans: 130 years)

Two cars are 15 kms apart. One is turning at a speed of 50kmph and the other at 40kmph . How much time will it take for the two cars to meet? (Ans: 3/2 hours)

A person wants to buy 3 paise and 5 paise stamps costing exactly one rupee. If he buys which of the following number of stamps he won't able to buy 3 paise stamps. Ans: 9

There are 12 boys and 15 girls, How many different dancing groups can be formed with 2 boys and 3 girls.

Which of the following fractions is less than 1/3

(a) 22/62 (b) 15/46 (c) 2/3 (d) 1 (Ans: (b))

There are two circles, one circle is inscribed and another circle is circumscribed over a square. What is the ratio of area of inner to outer circle? Ans: 1 : 2

Three types of tea the a,b,c costs Rs. 95/kg,100/kg and70/kg respectively.How many kgs of each should be blended to produce 100 kg of mixture worth Rs.90/kg, given that the quntities of band c are equal

a)70,15,15 b)50,25,25 c)60,20,20 d)40,30,30 (Ans. (b))

in a class, except 18 all are above 50 years.15 are below 50 years of age. How many people are there

(a) 30 (b) 33 (c) 36 (d) none of these. (Ans. (d))

If a boat is moving in upstream with velocity of 14 km/hr and goes downstream with a velocity of 40 km/hr, then what is the speed of the stream ?
(a) 13 km/hr (b) 26 km/hr (c) 34 km/hr (d) none of these (Ans. A)

Find the value of ( 0.75 * 0.75 * 0.75 - 0.001 ) / ( 0.75 * 0.75 - 0.075 + 0.01)
(a) 0.845 (b) 1.908 (c) 2.312 (d) 0.001 (Ans. A)

A can have a piece of work done in 8 days, B can work three times faster than the A, C can work five times faster than A. How many days will they take to do the work together ?

(a) 3 days (b) 8/9 days (c) 4 days (d) can't say (Ans. B)

A car travels a certain distance taking 7 hrs in forward journey, during the return journey increased speed 12km/hr takes the times 5 hrs.What is the distancetravelled

(a) 210 kms (b) 30 kms (c) 20 kms (c) none of these (Ans. B)

Instead of multiplying a number by 7, the number is divided by 7. What is the percentage of error obtained ?

Find (7x + 4y ) / (x-2y) if x/2y = 3/2 ?

(a) 6 (b) 8 (c) 7 (d) data insufficient (Ans. C)

A man buys 12 lts of liquid which contains 20% of the liquid and the rest is water. He then mixes it with 10 lts of another mixture with 30% of liquid.What is the % of water in the new mixture?

If a man buys 1 lt of milk for Rs.12 and mixes it with 20% water and sells it for Rs.15, then what is the percentage of gain?

Pipe A can fill a tank in 30 mins and Pipe B can fill it in 28 mins.If 3/4th of the tank is filled by Pipe B alone and both are opened, how much time is required by both the pipes to fill the tank completely ?

If on an item a company gives 25% discount, they earn 25% profit. If they now give 10% discount then what is the profit percentage.

(a) 40% (b) 55% (c) 35% (d) 30% (Ans. D)

A certain number of men can finish a piece of work in 10 days. If however there were 10 men less it will take 10 days more for the work to be finished. How many men were there originally?

(a) 110 men (b) 130 men (c) 100 men (d) none of these (Ans. A)

In simple interest what sum amounts of Rs.1120/- in 4 years and Rs.1200/- in 5 years ?

(a) Rs. 500 (b) Rs. 600 (c) Rs. 800 (d) Rs. 900 (Ans. C)

If a sum of money compound annually amounts of thrice itself in 3 years. In how many years will it become 9 times itself.
(a) 6 (b) 8 (c) 10 (d) 12 (Ans A)

Two trains move in the same direction at 50 kmph and 32 kmph respectively. A man in the slower train observes the 15 seconds elapse before the faster train completely passes by him. What is the length of faster train ?
(a) 100m (b) 75m (c) 120m (d) 50m (Ans B)

How many mashes are there in 1 squrare meter of wire gauge if each mesh
is 8mm long and 5mm wide ?

(a) 2500 (b) 25000 (c) 250 (d) 250000 (Ans B)

x% of y is y% of ?
(a) x/y (b) 2y (c) x (d) can't be determined Ans. C

The price of sugar increases by 20%, by what % should a housewife reduce the consumption of sugar so that expenditure on sugar can be same as before ?

(a) 15% (b) 16.66% (c) 12% (d) 9% (Ans B)

A man spends half of his salary on household expenses, 1/4th for rent, 1/5th for travel expenses, the man deposits the rest in a bank. If his monthly deposits in the bank amount 50, what is his monthly salary ?
(a) Rs.500 (b) Rs.1500 (c) Rs.1000 (d) Rs. 900 (Ans C)

The population of a city increases @ 4% p.a. There is an additional annual increase of 4% of the population due to the influx of job seekers, find the % increase in population after 2 years ?

The ratio of the number of boys and girls in a school is 3:2 Out of these 10% the boys and 25% of girls are scholarship holders. % of students who are not scholarship holders.?

15 men take 21 days of 8 hrs. each to do a piece of work. How many days of 6 hrs. each would it take for 21 women if 3 women do as much work as 2 men?
(a) 30 (b) 20 (c) 19 (d) 29 (Ans. A)

A cylinder is 6 cms in diameter and 6 cms in height. If spheres of the same size are made from the material obtained, what is the diameter of each sphere?

(a) 5 cms (b) 2 cms (c) 3 cms (d) 4 cms (Ans C)

A rectangular plank (2)1/2 meters wide can be placed so that it is on either side of the diagonal of a square shown below.(Figure is not available)What is the area of the plank? ( Ans :7*(2)1/2 )

What is the smallest number by which 2880 must be divided in order to make it into a perfect square ?
(a) 3 (b) 4 (c) 5 (d) 6 (Ans. C)

A father is 30 years older than his son however he will be only thrice as old as the son after 5 years what is father's present age ?
(a) 40 yrs (b) 30 yrs (c) 50 yrs (d) none of these (Ans. A)

An article sold at a profit of 20% if both the cost price and selling price would be Rs.20/- the profit would be 10% more. What is the cost price of that article?


SECTION 1- APTITUDE SECTION
Directions for questions 1-3: Complete the sequence given below
1. 5, 5, 13, 13, 21, 21, __

Ans: 29
2. 0, 7, 26, 63, 124, __

Ans: 215 ( hint: n3-1 )
3. 1, 3, 5, 7, __
Ans: 9

4. If a person walks at 4/5th of his usual speed he reaches 40min late. If he walks at his usual speed for how much time does he travel ?
5. Two trains A&B start at opposite points 120km apart at 60kmph. A fly starting along with train A at 120kmph reaches B then returns back to touch A and continues the two and fro movement. By the time two trains meet how much distance would the fly have travelled?
6. In a class 80% have passed in english, 70% passed Hindi, 10% didn't pass either. If 144 students passed both the subjects. What is the total strength of the class?

7. Find the least number which when divided by 7 gives the reminder 6, when divided by 6 gives reminder 5, when divided by 5 gives reminder 4 and so on ?

8. If a man stands in front of sun what is the first letter of the direction which is left to him ?

9. (a)A square is to circle as cube is to
(b)Success is to failure as joy is to

10. (a)Give the synonyms of the following words
(i) Joy
(ii) Inert
(iii) Jolly

(b) Give the opposites of the following words
(i) genuine
(ii) command
(iii) essential
11.Find the odd man out in the following sets
(i) Tiger, Elephant, King Cobra, Dolphin
(ii) Oasis, Lake, Pool, Valcano
(iii) Bengali, Karnataka, Mumbai, Kashmir
(iv) Lapidary, Lancet, Scapel, Surgeon
(v) Requiem, Dirge, Elegy, Paean
12. I bought a cycle 2days before my birthday and I broke it 3 days after my birthday. The day I broke it is Mar2,1956?

Directions: The following questions are to be answered on the basis of the above given statement
i) When is my birthday?
Hint: Keep in mind that 1956 was a leap year.
(ii) What is my age on Mar 4th, 1980?
(iii) My nephew is born exactly 20years after me. If I turned 20 in 1960, what is the nephews age on Feb 28th 1988 ?

13. Monday Aug25, 96 :
Hostess: "Mr A, you forgot your umbrella during the party on last friday. I expected you to collect it on your visit on wednesday as I plan to leave on this Friday."
Directions: The following questions are to be answered on the basis of the above given statement
(i) when A missed umbrella?
(ii)When A is supposed to collect it?
(iii)When K leaves?

14.What is my father's sons son to my son?

Ans. Cousin brother
15. On cutting a solid parabola what would be generated

25. A die is thrown twice what is the probability that you get same number


26. The sum of two numbers is 55. What is the larger number?

*********************
1. What is the percentage represented by 0.03 * 0.05 ?
(a)0.0015
(b)0.000015
(c)0.15
(d)15
Ans.B

2. (x-a)(x-b)(x-c)....(x-z) = ?
(a) 1
(b) -1
(c) 0
(d) Can't be determined
Ans. C

3. If a = 1, b = 2, c = 3.......z = 26 what is the value of p+q+r ?
(a)33
(b)51
(c)52
(d)48
Ans. B

4. A is 8 miles east of B.
C is 10 miles north of B.
D is 13 miles east of C and E is 2 miles north of D.
Find shortest distance between A and E.
(a) 5 miles
(b) 6miles
(c) 13 miles
(d) 18 miles
Ans. C

5. If z = 1, y = 2.......a = 26. Find the value of z + y + x + .......+a.
(a) 351
(b) 221
(c) 400
(d) 200
Ans. A

6. There are 30 socks in a bag.
Out of these 60 % are green and the rest are blue.
What is the maximum number of times that socks have to be taken out so that atleast 1 blue pair is found.
(a) 21
(b) 2
(c)18
(d) 20
Ans. D

7. How many two digit numbers have their square ending with 8.
(a) 13
(b) 12
(c) 0
(d) 11
Ans. C

8. How many numbers are there between 100 and 300 with 2 in the end and 2 in the beginning.
(a) 10
(b) 9
(c) 11
(d) none of these
Ans. A

9. 0.000006 * 0.0000007 = ?
(a) 0.0000000042
(b) 0.000000000042
(c) 0.0000000000042
(d) 0.00000000000042
Ans. B

10. You have Rs 1000 with 8% p.a compounded every 6 months.
What is the total interest you get after 1 year.
(a) Rs.116.40
(b) Rs.345.60
(c) Rs.224.50
(d) Rs.160
Ans. A

11. If x + y =12,
x - y = 2
Find x + 2y.
(a) 12
(b) 17
(c) 14
(d) none of these
Ans. B

12. With one gallon of petrol a person moves at a speed of 50 mph and covers 16 miles.
3/4th of the distance is covered while moving at 60 mph.
How many gallons does he need to cover 120 miles in 60 mph.

13. A tap drains at x speed while tap B is closed.
When both taps are open they drain at y speed.
What is the speed of draining when only tap B is open
(a) x - y
(b) y-x
(c) x
(d) can't be determined
Ans. B

14. What is twenty percent of 25 % of 20.
(a) 2
(b)1
(c) 5
(d) 4
Ans. B

15. A rectangle has the dimensions 6ft * 4ft.
How many squares of 0.5 inches will it need to completely fill it.
(a) 32000
(b) 12824
(c) 13824
(d) 18324
Ans. C

Directions for questions 16-21: In each question,a series of letters satisfying a certain pattern are given. Identify the pattern and then find the letter/letters that will come in place of the blank/blanks.
16. a, c, e, g, _
(a) h
(b) i
(c) d
(d) j
Ans. B

17. a, e, i, m, q, u, _, _
(a) y, c
(b) b, f
(c) g, i
(d) none
Ans. A

18. ay , bz , cw , dx ,__
(a) gu
(b) ev
(c) fv
(d) eu
Ans. D

19. 1, 2, 3, 5, 7, 11, __
(a) 15
(b) 9
(c) 13
(d) 12
Ans. 13 , series of prime numbers

20. kp , lo , mn , __
(a) nm
(b) np
(c) op
(d) pq
Ans. A

21. abc , zyx , def , wvu , ___
(a) ghi
(b) tsr
(c) ihg
(d) str
Ans. A

22. How is my mother's sister's brother's wife's child related to me?
(a) brother
(b) uncle
(c) cousin
(d) nephew
Ans. A

23.What will my mother's husband's father-in-law's son's daughter to me?
(a) niece
(b) aunt
(c) sister
(d) none of these
Ans. D
*******************
Analogies

1. slur : speech : : smudge :?
Ans. writing
2. epaulet : shoulder : : ring :?
Ans.finger
3. vernacular : place : : fingerprint : ?
Ans.identical
Opposites

Q. corpulent
Ans: emaciated
Q. officious
Ans: pragmate
Q. dextrous
Ans: clumsy

The following sentences are broken into 4 sections- A, B, C, D
Choose the part which has a mistake
Mark (E) if you find no mistake.
Q.A)psychologists point out that B)there are human processes C)which does not involve D) the use of words
Ans. (C) which does not involve (do)

Q.A)jack ordered for B)two plates of chicken C)and a glass D)of water
Ans. (A)jack ordered for

The following is a group of questions is based on a passage or a set of conditions for each question.
Select the best answer choice given.
(i). If it is fobidden by law if the object of agreement is the doing of an act,
that is forbidden by law the agreement is void.
(ii). If it is of the nature that,it would defeat the provision of any law
is the agreement is void.if the object of agreement is such that thing got
directly forbidden by law it would defeat the provision of statuary law.
(iii). If the object of agreement is fraddulent it is void.
(iv). An object of agreement is void if it involves or implies to the personnal property of another.
(v). An object of agreement is void where the constant regards as ignored.
(vi). An object of agreement is void where the constant regards is as opposed to public policy.
Q. An algorithm follws a six step process za,zb,zc,zd,ze,zf, it is governed by the following
(i) zd should follow ze
(ii) the first may be za,zd or zf
(iii) zb and zc have to be performed after zd
(iv) zc must be immediately after zb

Q. If za is the first set zd must be
a) 3rd
b) 5th
c) 2nd
d) 4th

Q. If zb must follow za then za can be
a) third or fourth
b) first or second
c) can not be third
d) fouth or fifth
e) none

Q. If ze is third term the number of different operations possible are
The following questions are based on the given statements
Ravi plants six seperate saplings -- x,y,z,w,u,v in rows no 1 to 6 ,according to the follwing conditions
He must plant x before y and u
He must plant y and w
The third has to be z
Q. Which of the following is acceptable
a) xuywzv
b) xvzyuw
c) zuyxwv
d) zvxuwy
e) wyzuvx
Q. Which of the following is true
a) z before v
b) z before x
c) w before u
d) y before u
e) x before w
Q. If he plants v first, then which can be planted second
a) x
b) y
c) z
d) w
e) u

Q. Which of the following describes a correct combination of sapling and row?
a) x,3
b) y,6
c) z,1
d) w,2
e) u,6

Q. If he plants b 6th which would be planted first and second
a) x and w
b) x and y
c)y and x
d)w and z
e) w and u

Q. If he plants w before u and after v he should plant w at
a) first
b) second
c) fourth
d) fifth
e) sixth

Q. At a certain moment a watch shows 2 min lag although it is running fast.
If it showed a 3 min lag at that moment, but also gains by 1/2 min more a day than its current speed
it would show the true time one day sooner than it usually does.
How many mins does the watch gain per day.
a).2
b).5
c).6
d).4
e).75
Q. In 400m race A gives B a start of 7 sec and beats him by 24 sec.
In another race A beats B by 10 sec.the speeds are in the ratio
a)8:7
b)7:6
c)10:8
d)6:8
e)12:10

Q. 3x+4y=10
x3 + y3=6
What is the minimum value of 3x+11y=?

Q. There are 600 tennis players
4% wear wrist band on one wrist
Of the remaining, 25% wear wrist bands on both hands
How many players don't wear a wrist band?
Ans. 432

Q. Three types of tea the a,b,c costs Rs. 95/kg,100/kg and70/kg respectively.
How many kgs of each should be blended to produce 100 kg of mixture worth Rs.90/kg,
given that the quntities of band c are equal
a)70,15,15
b)50,25,25
c)60,20,20
d)40,30,30
Ans. (b)

Q. Two distinct no's are taken from 1,2,3,4......28
Find the (b)
****************************
The questions 41-46 are based on the following pattern. The problems below contain a question
and two statements giving certain data. You have to decide whether the data given in the
statements are sufficient for answering the questions. The correct answer is
(A) If statement (I) alone is sufficient but statement (II) alone is not sufficient.
(B) If statement (II) alone is sufficient but statement(I) alone is not sufficient.
(C) If both statements together are sufficient but neither of statements alone is sufficient.
(D) If both together are not sufficient.
41. What is John's age?
(I) In 15 years John will be twice as old as Dias would be
(II) Dias was born 5 years ago
Ans. (C)
42. What is the distance from city A to city C in kms?
(I) City A is 90 kms from City B
(II) City B is 30 kms from City C
Ans. (D)
43.Is A=C ? A,B,C are real numbers
(I) A-B=B-C
(II) A-2C = C-2B
Ans. (C)
44. What is the 30th term of a given sequence?
(I) The first two terms of the sequence are 1, 1/2
(II) The common difference is -1/2
Ans. (A)
45. Was Avinash early, on time or late for work?
(I) He thought his watch was 10 minutes fast
(II) Actually his watch was 5 minutes slow
Ans. (D)
46. What is the value of A if A is an integer?
(I) A4 = 1
(II) A3 + 1 = 0
Ans. (B)
47. A person travels 12 km in the southward direction and then travels 5km to the right and then travels 15km toward the right and finally travels 5km towards the east, how far is he from his starting place?
(a) 5.5 kms
(b) 3 km
(c) 13 km
(d) 6.4 km
Ans. (b)
48. X's father's wife's father's granddaughter uncle will be related to X as
(a) Son
(b) Nephew
(c) Uncle
(d) Grandfather
Ans. (c)
49. Find the next number in the series 1, 3, 7, 13, 21, 31
(a) 43
(b) 33
(c) 41
(d) 45
Ans. (a)
50. If in a certain code "RANGE" is coded as 12345 and "RANDOM" is coded as 123678.
Then the code for the word "MANGO" would be
(a) 82357
(b) 89343
(c) 84629
(d) 82347
Ans. (d)
51. If "PROMPT" is coded as QSPLOS, then "PLAYER" should be
(a) QMBZFS
(b) QWMFDW
(c) QUREXM
(d) URESTI
Ans. (a)
*************************
1.When a bicycle is in motion, the force of friction exerted by the ground on the two wheels is such that it acts
(a) In the backward direction on the front wheel and in the forward direction on the rear wheel.
(b) In the forward direction on the front wheel and in the backward direction on the rear wheel.
(c) In the backward direction on both the front and rear wheels.
(d) In the backward direction on both the front and rear wheels.
Ans. (d)

2. A certain radioactive element A, has a half life = t seconds.
In (t/2) seconds the fraction of the initial quantity of the element so far decayed is nearly
(a) 29%
(b) 15%
(c) 10%
(d) 45%
Ans. (a)

3. Which of the following plots would be a straight line ?
(a) Logarithm of decay rate against logarithm of time
(b) Logarithm of decay rate against logarithm of number of decaying nuclei
(c) Decay rate against time
(d) Number of decaying nuclei against time
Ans. (b)

4. A radioactive element x has an atomic number of 100.
It decays directly into an element y which decays directly into element z.
In both processes a charged particle is emitted.
Which of the following statements would be true?
(a) y has an atomic number of 102
(b) y has an atomic number of 101
(c) z has an atomic number of 100
(d) z has an atomic number of 101
Ans. (b)

5. If the sum of the roots of the equation ax2 + bx + c=0 is equal to the sum of the squares of their reciprocals
then a/c, b/a, c/b are in
(a) AP
(b) GP
(c) HP
(d) None of these
Ans. (c)

6. A man speaks the truth 3 out of 4 times.
He throws a die and reports it to be a 6.
What is the probability of it being a 6?
(a) 3/8
(b) 5/8
(c) 3/4
(d) None of the above
Ans. (a)

7. If cos2A + cos2B + cos2C = 1 then ABC is a
(a) Right angle triangle
(b) Equilateral triangle
(c) All the angles are acute
(d) None of these
Ans. (a)

8. Image of point (3,8) in the line x + 3y = 7 is
(a) (-1,-4)
(b) (-1,4)
(c) (2,-4)
(d) (-2,-4)
Ans. (a)

9. The mass number of a nucleus is
(a) Always less than its atomic number
(b) Always more than its atomic number
(c) Sometimes more than and sometimes equal to its atomic number
(d) None of the above
Ans. (c)

10. The maximum KE of the photoelectron emitted from a surface is dependent on
(a) The intensity of incident radiation
(b) The potential of the collector electrode
(c) The frequency of incident radiation
(d) The angle of incidence of radiation of the surface
Ans. (c)

11. Which of the following is not an essential condition for interference
(a) The two interfering waves must be propagated in almost the same direction or
the two interfering waves must intersect at a very small angle
(b) The waves must have the same time period and wavelength
(c) Amplitude of the two waves should be the same
(d) The interfering beams of light must originate from the same source
Ans. (c)
***************************
Q1. Two bodies changed from p1v1 to p2v2 state in two ways. The heat supplied is delta Q and work done is delta W
Then what is constant in these two processes
(a) delta q
(b) delta w
(c) delta q + delta w
(d) delta q - delta w
Ans. (d)

Q2. _______ have same atomic number and same mass number are
(a) Isotopes
(b) Isotones
(c) Isomers
(d) Isobars
Ans. (c)

Q3. When a free electron is placed in a plane of electro magnetic then it moves in
(a) in the direction of the electric field
(b) in the direction of magnetic field
(c) of propagation of wave
(d) of the plane containing magnetic field and propagation direction.

Q4. Name the phenomena in which one proton is jumped from one isomer to another isomer to create two different elements

(a) functional isomerisim
(b) sterio merisim
(c) tauto merisim
(d) penta merisim

Ans. (c)
Q5. In the below compounds which one has 40% C ,6.7% H and 53.3 % O what is its empherical formula
(a) CHO
(b) CH2
(c) C2H2O2
(d) C2H3O2
Ans: (b)

Q6. X rays are coming from X ray tube, the wavelength is _______ a certain wavelength/s

(a) below
(b) above
(c) inbetween
(d) out of

Ans. (c)

Q7. In a triode valve in order to increase the saturation current what has to be done
(a) increase plate voltage
(b) reduce distance between grid and plate
(c) increase cathode potential
(d) reduce grid potential

Ans. (d )

Q8. Seven different toys are distributed among 3 children how many different ways are possible?
(a) 7C3
(b) 7P3
(c) 3 7
(d) 7 3

Ans. (c)

Q9. A, B ans C are three speakers. They have to speak randomly along with another 5 speakers in a function.
A has to speak before B and B has to speak before C. What is the probability.

Ans. 1/6

Q10. If dy = (secx + ytanx)dx, Then the curve is

(a) x = ycosx
(b) x = ysinx
(c) x = ytanx
(d) x = ysecx

Ans. (a)
Q11. Two series are 16,21,26.... and 17,21,25.....
What is the sum of first hundred common numbers
(a) 101100
(b) 110100
(c) 101110
(d) 110101
Ans. (a)

Q12. There are two sections in a question paper each contain five questions. A students has to answer 6 questions.
Maximum no. of questions that can be answered from any section is 4. How many ways he can attempt the paper?

(a) 50
(b) 100
(c) 120
(d) 200
Ans. (d)
Q13. a and b are two numbers selected randomly from 1,2,3.... 25 what is the probability of a and b are not equal.
(a) 1/25
(b) 24/25
(c) 13/25
(d) 2/25
Ans. (b)

Q14. The sum of the series 1 + 1(1+1/n) + 3(1+1/n)2 + ..... is equal to?
Ans. n2
Q15. Two circles of different radii intersects each other what is the maximum no of intersections
(a) 0
(b) 1
(c) 2
(d) 3
Ans. (c)

Q16. If x= sin-1(t), y = log(1-t2), find d2y/dx2 when t=1/2
(a) 1
(b) 0
(c) -8/3
(d) -2/3
Ans. (c)

Q17. If x approaches infinity , then ex dx )/( e2xdx) is ?
(a) 1
(b) 0
(c) -1
(d) 2
Ans. (a)

Q18. If f(x)=1-cos(1-cosx)/x4 is continuos at f(0) then what is x
(a) 1
(b) 0
(c) 1/4
(d) -1/4
Ans. (c)

Q19. For the word SURITI, if you arrange the letters in dictionary order then what is its rank?
(a) 234
(b) 235
(c) 236
(d) 237
Ans. (c)

Q20. Period of sin ((2t + 3) / 6 pi)
(a) 6pi
(b) 6pi2
(c) 3pi
Ans. (b)

Q21 - Q23. Four questions given on the below data
X,Yand Z are senior engineers. A,B,C,D are junior engineers. Company wants to select 4 enginers. Two will be senior and two will be juniors. The company wants these engineers to work in the most productive way so they respect each person's likes/dislikes.
• Y is not friends with A
• Z is not friends with C
• B is not friends with A
4. If B is selected then who will be the remaining 4 members ?
5. If C is selected, Z and ___ cannot be selected?
6. D is always selected if ___ is selected?

Q24. A speaks truth 70% of the times, B speaks truth 80% of the times.
What is the probability that both are contradicting each other is ?

Q25. ((2x-3)/((x2 +x+1)2 )dx is ?

Q26. Ram starts from A walking 2 km North and turns right and walks 4 km and turns right again and walks 4 km and turns right again and walks 4 km and meets Radha at Bwalking in the opposite direction to Ram .
a) Which direction does Ram walk after the first turn?
b) Distance between A and B

Q27. If the equation x2 - 3x + a = 0 has the roots (0,1) then value of a is ?

Q28. A and B's temperature are 10c and 20 having same surface , then their ratio of rate of emmisions is ?

Q29. An atomic particle exists and has a particlular decay rate . It is in a train . When the train moves, a person observes for whether the decay rate
(a) increases
(b) decreases
(c) depend on the directions of movement of train


Q30. Which of the following exchanges positive ions
(a).cl-
(b) nh2-
(c) ch2
Ans. (b)

Q31. After execution of CMP, a instruction in Intel 8085 microprocessor
(a) ZF is set and CY is reset.
(b) ZF is set CY is unchanged
(c) ZF is reset, CY is set
(d) ZF is reset , CY is unchanged .

Ans. ZF is set and CY is reset


Q32. The best tool for editing a graphic image is ?

Q33. Network scheme defines
a.)one to one
b.) many to many
c.) one to ,many ?


Q34. A person wants to measures the length of a rod.First he measures with standing ideally then he maeasures by
moving parrel to the rod
(a)the length will decrease in second case
(b)length will be same
(c) length will increse in the second case.

Q35. One U-230 nucleus is placed in a train moving by velocity emiting alpha rays .When the train is at rest the
distance between nucleus and alpha particle is x . One passenger is observing the particle . When the train is moving
what is the distance between particle and nucleus ?
(a) x
(b) x + vt
(c) x - vt

Q36. What is the resulting solution when benzene and toluene are mixed ?

Q37. If the word FADENCOMT equals 345687921 then
1. What is FEAT
2. Find representation of 2998

Q38. Given 10 alphabets out of which 5 are to be chosen. How many words can be made with atleast one repetition.

Q39. Arrange by acidic values : phenol, nitrotolouene and o-cresol?

Q40. Find sum of 3 + 5/(1+22) + 7/(1 + 22 + 32) + ......
Ans. 3n/(1 + n)

The following are few sample questions that maybe asked in the software paper.We haven't been able to give the values in certain problems ; only the type of questions have been mentioned.
Q What sorting algos have their best and worst case times equal ?
Ans. O(nlogn) for mergesort and heap sort
Q. What page replacement algo . has minimumn number of page faults ?
Ans. Optimality algorithm
Q. What is the use of virtual base class in c++
Ans. Multiple lines between derived classes.
Q. Find the eccentricity of a given node in a directed graph
Q. Convert the infix to postfix for A-(B+C)*(D/E)
Ans. ABC+DE/*-
Q. What is swapping
Q. Assignment operator targets to
Ans. l-value
Q. A byte addressable computer has memory capacity of 2 power m Kbytes and can perform 2 power n operations
an instruction involving three operands and one operator needs maximum of ---bits
Ans. 3m + n
Q. In round robin scheduling, if time quatum is too large then it degenerates to
Ans. FCFS
Q. What is network schema?
Q. Packet Burst is ______
Q. Picard's method uses _______?
Ans. Successive Differentiation.




Verbal
ACUMEN
a. exactness
b. potential
c. shrewdness
d. bluntness
e. None of these
BEHEST
a. behavior
b. hold down
c. hold up
d. relieve
e. condemn
DISCRETION
a. prudence
b. consistency
c. precipice
d. disturbance
e. distemper

ORDAIN
a. arrange
b. command
c. contribute
d. establish
e. control

FLORID
a. ornate
b. thriving
c. artistic
d. elegant
e. None of these
PENITENCE
a. liking
b. insightful
c. attractive
d. penetrable
e. compunction

WHET
a. stimulate
b. humorous
c. inculate
d. dampen
e. None of these

INCENTIVE
a. reflex
b. amplitude
c. inflection
d. provocation
e. escutcheon

LATITUDE
a. scope
b. segment
c. globule
d. legislature
e. lamentation
MORTIFY
a. make a cavity
b. displease
c. humiliate
d. relapse
e. murder

ADAGE
a. advice
b. proverb
c. enlargement
d. advantage
e. usage

TO DISPEL
a. to dissipate
b. to dissent
c. to distort
d. to disfigure
e. to dissect
ERRATIC
a. unromantic
b. free
c. popular
d. steady
e. unknown

TO MERIT
a. to embrace
b. to devote
c. to deserve
d. to combine
e. to display

RAPT
a. lively
b. concealed
c. engrossed
d. prototype
e. None of these

TO HEAP
a. to pile
b. to forbid
c. to proceed
d. to share
e. to stoop

CAJOLE
a. coax
b. motivate
c. profound
d. mollify
e. evade
OVULATE
a. penury
b. immunize
c. fertilize
d. reproduce
e. incisions
ABODE
a. clay
b. obstacle
c. dwelling
d. bind
e. to beguile

POTENTIAL
a. latent
b. hysterical
c. conventional
d. symmetrical
e. conscientious

EXTRICATE
a. terminate
b. isolate
c. liberate
d. simplify
e. frustrate

DISPARITY
a. inequality
b. impartiality
c. unfairness
d. twist
e. None of these

TO CONFISCATE
a. to harass
b. to repulse
c. to console
d. to appropriate
e. to congregate
PIOUS
a. historic
b. devout
c. multiple
d. fortunate
e. authoritative

LETHARGY
a. reminiscence
b. category
c. fallacy
d. unanimity
e. stupor

CARGO
a. cabbage
b. camel
c. lance
d. freight
e. flax
OVATION
a. oration
b. gesture
c. emulation
d. applause
e. nourish


Aptitude
1. A family, planning a weekend trip, decides to spend not more than a total of 8 hours driving. By leaving early in the morning, they can average 40 miles per hour on the way to their destination. Due to the heavy Sunday traffic, they can average only 30 miles per hour on the return trip. What is the farthest distance from home they can plan to go?
(a) 120 miles or less (b) Between 120and 140 miles (c) 140 miles
(d) Between 140 and 160 miles (e) 160 miles or more
2. A car is filled with four and half gallons of fuel for a round trip. If the amount of fuel taken while going is 1/4 more than the amount taken for coming, what is the amount of fuel consumed while coming back?
(a) Less than 2 gallons (b) 2 gallons (c) 2 1/2 gallons
(d) 3 gallons (e) More than 3 gallons

3. A 3-gallon mixture contains one part S and two parts R. In order to change it to a mixture containing 25% S, how much R should be added?
(a) 1/2 gallon (b) 2/3 gallon (c) 3/4 gallon (d) 1 gallon (e) 1 1/2 gallon

4. A tree grows only 3/5 as fast as the one beside it. In four years the combined growth of the two trees is eight feet.
How much does the shorter tree grow in two years?
(a) Less than 2 feet (b) 2 feet (c) 2 1/2 feet
(d) 3 feet (e) more than 3 feet.
5. Wind flows at 160 miles in 330 minutes, for traveling 80 miles how much time does it require?
(a) 1 hour 30 minutes (b) 1 hour 45 minutes (c) 2 hours
(d) 2 hours 45 minutes (e) 3 hours

6. A stationary engine has enough fuel to run 12 hours when its tank is 4/5 full. How long will it run when the tank is 1/3 full?
(a) Less than 2 hours (b) 2 hours (c) 3 hours
(d) 4 hours (e) 5 hours
7. If A is traveling at 72 km per hour on a highway. B is traveling at a speed of 25 meters per second on a highway. What is the difference in their speeds in meters per second?
(a) 1/2 m/sec (b) 1 m/sec (c) 1 1/2 m/sec
(d) 2 m/sec (e) 3 m/sec
8. A salesperson by mistake multiplied a number and got the answer as 3, instead of dividing the number by 3. What is the answer he should have actually got?
(a) 0 (b) 1/3 (c) 1 (d) 2 (e) 3
9. If the length of a rectangle is increased by 30% and the width is decreased by 20%, then the area is increased by...
(a) 10% (b) 5% (c) 4% (d) 20% (e) 25%

10. In the class of 40 students, 30 speak Hindi and 20 speak English. What is the lowest possible number of students who speak both the languages?
(a) 5 (b) 20 (c) 15 (d) 10 (e) 30

11. The most economical prices among the following prices is:
(a) 10 kilo for Rs.160 (b) 2 kilo for Rs.30 (c) 4 kilo for Rs.70
(d) 20 kilo for Rs.340 (e) 8 kilo for Rs.130

12. A truck contains 150 small packages, some weighing 1 kg each and some weighing 2 kg each. how many packages weighing 2 kg each are in the truck if the total weight of all the packages is 264 kg?
(a) 36 (b) 52 (c) 88 (d) 124 (e) 114
13. A man was arrested for exceeding the speed limit by 10 miles an hour. A second man was charged with exceeding the same limit by twice as much. The second man was driving 35 miles per hour. What was the speed limit?
(a) 10 miles per hour (b) 15 miles per hour (c) 20 miles per hour
(d) 25 miles per hour (e) 30 miles per hour

14. One year ago Pandit was three times his sister's age. Next year he will be only twice her age. How old will Pandit be after five years?
(a) 8 (b) 12 (c) 11 (d) 13 (e) 15
15. If two pencils cost 8 cents, then how much do 5 pencils cost?
(a) 18 cents (b) 20 cents (c) 22 cents (d) 23 cents (e) 24 cents

Passage
All the questions are of multiple choice type. You have to answer the questions based on the preceding paragraph. All the questions have the same answer choice. The choices are as given below:
(a) True.
(b) False.
(c) Cannot Say.

Consider the following paragraph:
Researchers in Mumbai have found that certain types of gallstones can be dissolved by injecting them with a gasoline additive in the form of ether. The ether is injected through a tube directly into the gall bladder. The one-day treatment works only on cholesterol-based stones, not those composed largely of calcium. However, as the cholesterol stones are by far the most common type, for millions of gallstone sufferers the treatment should offer a welcome alternative to surgery, the commonest option in most hospitals.

# "It takes more than one day for ether to dissolve a calcium-based gallstone".

# "Gallstones can only be dissolved by injections".

# "Gallstones can quickly be cured with surgery".

# "Ether is largely used for dissolving gallstones".
# "Calcium stones can be cured in one day".
# "Hundreds of people contain calcium stones".
Consider the following paragraph:
My father had no brothers, but his three sisters are all married and each has two children. My grandfather has two sons.

# "My father was the only child".

# "I have only one uncle".

# "One of my aunts is a spinster".

# "I have six cousins on my father's side".

# "My grandfather was the only son".
Consider the following paragraph:
In the Totalitarian days, the words have very much devalued. In the present day, they are becoming domestic, that is, the words will be much more devalued. In that days, the words will be very much effected in political area. But at present, the words came very cheap. We can say they come free at cost.

# "In Totalitarian society, words are devalued".

# "Totalitarians will have to come much about words".

# "In the Totalitarian society the words are used for the political speeches".

Consider the following paragraph:
In past helicopters were forced to ground or crash because of the formation of the ice on the rotors and engines. A new electronic device has been developed which can detect the water content in the atmosphere and warns the pilot, if the temperature is below freezing temperature; about the formation of ice on the rotors and wings.

# "The electronic device can avoid the formation of ice on the wings".

# "There will be malfunction of rotor and engine because of formation of ice".

# "The helicopters were to be crashed or grounded".

# "There is only one device that warns about the formation of ice".

Consider the following paragraph:
Human existence is not susceptible of arbitrary division between consciousness and unconsciousness. The conscious world invaders and shapes the activities of the unconscious, while many of the great achievements of humanity's waking hours where wholly or partly inspired by dreams. Even if it could argued that dreams precede experience such a dichotomy could not be drawn, as the influence of the dreaming on the waking state would remain unclear, but as yet no common vocabulary exists to record the substance of prenatal dreaming.

# "Sleep can be a creative state".

Consider the following paragraph:
FLORA 3-piece sofa-set is at the top of our upholstery range. This high-backed quality sofa-set boasts an impressive specification which starts with a hardwood frame in teak and a padded front edge ensuring really deep, long-lasting comfort. Seat cushions are of high resilience foam and back cushions of softest hollow fill. The whole set is carefully upholstered throughout in a choice of superb fabrics ranging from cotton print to velvet.

# "The padding is there to ensure that the furniture will last for a long time".

# "The firm sells other upholstery furniture".
Consider the following paragraph:
Hacking is a crime made possible by a relatively new technology, which is one of the reasons it is often poorly understood and reported. Many computers, but by no means all, are now linked together in networks which allow users on one computer to communicate with others on the same network. If a computer is not networked, no manipulation of its data from another machine possible. So long as users are authorized, networking is just a way of making work easier more productive. Hacking, on the other hand is the unauthorized use of networks or unauthorized entry into the computers themselves. Most people do not break into the networks they use, since they are already accredited users.

# "Hackers do not work for the firms whose networks they break into".
# "Hacking is the only vulnerability of the computers for the usage of the data".
# "Hacking is done mostly due to the lack of computer knowledge".
Consider the following paragraph:
Polycythemia often occurs in people who have chronic lung disease, but can appear spontaneously in healthy individuals. Excessive numbers of red blood cells manufactured by the body and the individual then develops a very healthy-looking, ruddy complexion. The blood becomes thicker and is liable to clot and block major blood vessels. High blood pressure is another frequent complication. Treatment involves venesection, in which a liter or so of blood is removed from the body. Medication may also be given to reduce the numbers of red blood cells manufactured in the body.

# "Lung disease frequently precedes polycythemia".

Consider the following paragraph:
Bindweed is only effectively controlled by applying a solution of brushwood-killer to the growing tips. It is necessary to unwind a suitable length from the host plant before treatment, but this is not so very difficult, and it does not seem essential to find and treat every leader on the same weed. The solution should be made up in a can which is carried in one hand, while the other, in a rubber glove, inserts the leaders in the can. If the the leaders can be laid out on the the ground , they can easily be wetted with a small brush. As long as the weather is calm, there is no real risk of damage to adjacent plants, and in two or three weeks the weeds should have disappeared.

# "Brushwood-killer can pose a threat to other plants in the garden".

Consider the following paragraph:
Senior managers in a leading company said that new Japanese investment in India was transforming the car industry, and warned that jobs were under threat from Japanese competition. They stated that increasing competition would be coupled with an inevitable downturn in the car market and the recent rise interest rates which has already hit demand.

# "The managers issued their warning after a rise in the interest rates".

# "According to the senior managers, the Japanese investment in India will lead to a glut in the car market".

# "Some senior managers said that more people will want to by new cars in the future".

# "The perception of the senior managers is the new Japanese investment in India is leading to more automation of the car industry".

# "The increased rate of interest will mean that Japanese firms will cease to operate in this country".

# "The increase in loan interest will adversely affect car sales".
# "Japanese workers are taking over the jobs of Indian industry".
# "Managers said that interests in car will go down after seeing the raise in interest rates".
# "People are very interested to buy the cars".
Consider the following paragraph:
The new Starfire has an advanced four-cylinder engine with catalytic converter and uses only unleaded petrol. Versatility is a major feature of the range and the 1500 and 1800 models have the same high level of specification inside and out. The only obvious visual difference, internally and externally, is the use of alloy wheels on the 1800 version, together with a discreet change in badging. The StarFire 2000 is distinguished by its tailgate spoiler and the rectangular fog and driving lamps integrated into the front bumper which are also included in the specification.

# "Internally, the Starfire 2000 looks like the 1500 model".

Consider the following paragraph:
Pierre Claude Jean Allouez explored lake superior from 1665 to 1667. At his little mission station near the western end of the lake, he heard from the Indians of a great river to the west. Pierre Jacques Marquette determined to investigate. In 1673, accompanied by Louis Jolliet and five others, he left St. Ignace mission and ascended the fox river, which flows into green bay crossed over to Wisconsin river and followed it to the upper Mississippi. The party then descended the Mississippi to the mouth of Arkansas. These Frenchmen were not first Europeans to sight or travel the Mississippi. De Soto and Moscoso had done so a century and a half before. The report of the exploration was rushed back to Quebec, where, in 1672, Count Frontenac had arrived as Governor of the province. He and his friend, the remarkable La Salle-who earlier may have penetrated the Ohio river valley-listened with deep interest.

# "Allouez explored the western end of lake superior".

# "Marquette and his party were not the first French men to travel the Mississippi river".

# "La Salle listened with deep interest- the report of exploration of De Soto and Moscoso".

# "La Salle explored the Mississippi river valley".
Consider the following paragraph:
Dr. Goddard was the first to fire a rocket that reached a speed faster than the speed of sound. He was the first to develop a gyroscopic steering apparatus for rockets. He was the first to use vanes in the jet stream for rocket stabilization during the initial phase of a rocket flight. And he was the first to patent the idea of step rockets. After proving on paper and in actual tests that a rocket can travel in vacuum, he developed the mathematical theory of rocket propulsion and rocket flight, including basic designs for long-range rockets. All of this information was available to our military men before World War II, but evidently its immediate use didn't seem applicable. Near the end of World War II we started intense work on rocket-powered guided missiles, using the experiments and developments of Dr. Goddard and the American Rocket Society.

# "The stabilization problem of rockets in the initial phase was solved by Dr. Goddard."

# "Rockets can travel faster than sound, thanks to gyroscopic steering."

# "Goddard lived before World War II".

# "After careful mathematical calculations, Dr. Goddard proved that rockets can travel in vacuum".

Consider the following paragraph:
In March 1513, de Leon sailed off confidently from Peuto Rico for the Bahamas. Landing briefly at San Salvador, Bahamas, he wound through unchartered islands until he sighted an extensive coastline. He had no reason to suspect that it is anything more than an island, but he followed the coast for a day without rounding its end or finding a suitable landing place. He named the "island" La Florida. This name came to be applied by the Spanish to the entire Southeastern United States and beyond. Then, near the 30th parallel, de Leon landed at the mouth of the St. Johns river. Determined to be the first to circumnavigate the "island", he turned south, traced the coast around the tip of the peninsula, moved to the west, perhaps reaching Tampa bay. After 7 weeks, he gave up hope of circling the northern tip of this "island"; it was incredibly large and he may have suspected that he had discovered the long sought mainland. If so, it all belonged to his King, for he had earlier planted the Spanish flag and claimed Florida for Ferdinand.

# "de Leon is from Spain, ruled by Ferdinand".

# "de Leon is very patriotic".

# "de Leon discovered part of US during his journey".

Consider the following paragraph:
James Madison understood that interests groups will inevitably develop within a free political system. The problem, as Madison saw it, was to prevent any single interest group from becoming so strong that it was able to dominate the political system. This could be accomplished by legislating restrictions on political behavior, but that solution meant a sacrifice of some of the freedom that Madison prized so highly. A better solution, he thought, was to extend the territorial scope of the government. This would allow for greater diversity of interests in the nation, and a greater number of groups competing for power. Each interest group would thereby find it more difficult to appeal to a majority of the people, and to dominate the political process.

# "The more interest groups there are in a political system, the less freedom there is for everyone."

# "Legislating restrictions on political behavior is sometimes the only method of preserving political freedom."

# "Increasing the territorial scope of a government can help to preserve freedom."

# "According to Madison, in a free political system, interest groups are undesirable."

Consider the following paragraph:
The regulations and expenses to invent, patent and market new ideas and products imposes a heavy burden on inventors. The cost is often absorbed by large corporations with research and development facilities they provide. Corporations also help creative people contribute to society without suffering the loss of income or security of the private inventor. The realities of this arrangement are that many good ideas are never brought into the marketplace and the cost of products on the market is high because of the development cost. However, protection provided by the patents and the safety to the public to avoid placing harmful products on the market is important to maintain. Thus, as is often the case, rules and regulations have their favorable and unfavorable consequences.

# "The regulations and expenses to invent, patent and market new ideas is an expensive proposition to the inventors".

# "Good ideas are never brought into the marketplace because of the costs involved in inventing, patenting marketing them".

# "Corporations steal the individual inventor of their inventions".
Consider the following paragraph:
Being born female and black were two handicaps Gwendolyn Brooks states that she faced from her birth, in 1917, in Kansas. Brooks was determined to succeed. Despite the lack of encouragement she received from her teachers and others, she was determined to write and found the first publisher for one of her poems when she was 11. In 1945, she marketed and sold her first book; national recognition ensued. She applied for and received grants and fellowships from such organizations as the AAAL and the Guggenheim Foundation. Later she received the Pulitzer prize for poetry; she was the first black woman to receive such an honor. Brooks' reaction to fame is atypical. She continues to work and work hard. She writes, travels, and helps many who are interested in writing. Especially important for her is increasing her knowledge of her black heritage and encouraging other people to do the same. She encourages dedication to the art to would-be writers.

# "Brooks' story illustrates the power of strong determination".

# "She became the author of a book in her teens".

# "Gwendolyn received the Pulitzer prize for her first poetry".

Consider the following paragraph:
A cave is a natural opening in the ground extending beyond the zone of light and large enough to permit the entry of man. Occurring in a wide variety of rock types and caused by widely differing geological processes, caves range in size from single small rooms to interconnecting passages many miles long. The scientific study of caves is called speleology. It is a composite science based on geology, hydrology, biology and archeology, and thus holds special interest for earth scientists. Caves have been natural attractions since prehistoric times. Prolific evidence of early man's interest has been discovered in caves scattered throughout the world. Skeletons of some of the earliest manlike creatures (Australopithecines) have been discovered in cave deposit in South Africa, and the first evidence of primitive Neanderthal man was found in Germany. Cro-Magnon man created his remarkable murals on the walls of caves in France.

# "Primitive human form originated in Germany".

# "Study of caves is the study of earth, water, life and early man".

# "Cro-Magnon man was more intelligent than Neanderthal man".

# "Caves are a natural attraction because they reveal information about the early man".

Consider the following paragraph:
Although invaders represent the threat to the conservation of flora and fauna, there are two special cases in which invasion have been deliberately brought about. One is the desire to control pests by natural predators, which may have to be brought from other countries. The second is releasing organisms into the wild (or on to farms, from which they might escape) that are completely novel, because they have been genetically engineered. There is nothing intrinsically sinister about engineered organisms, but any novelty must be regarded as potential invader.

# "Pests are more dangerous than their natural predators".

Consider the following paragraph:
Life in colonial times was harsh, and the refinements of the mother country were ordinarily lacking. The colonists, however, soon began to mold their English culture into the fresh environment of new land. The influence of religion permeated the entire way of life. In most Southern colonies, the Anglican church was the legally established church. In New England, the Puritans were dominant; and in Pennsylvania, the Quakers. Especially in the New England colonies, the local or village church was the hub of community life; the authorities strictly enforced the Sabbath and sometimes banished non-believers. Unfortunately, the same sort of religious intolerance, bigotry and superstition associated with the age of Reformation in Europe also prevailed in some of the colonies, though on a lesser scale. In the last half of the 17th century, during sporadic outbreak of religious fanaticism and hysteria, Connecticut authorities tried and hanged several women as "witches". Early in the 17th century, some other witchcraft persecution occurred in Virginia. As the decades passed, however, religious tolerance developed in colonies.

# "New England was part of the Southern colonies".

# "During the mid 17th century there was significant improvement in religious tolerance and superstition".

# "Life in colonial times was harsh due to the strong influence of religion".

# "The Anglican church used to govern the people in most Southern colonies".
Consider the following paragraph:
Confucius said that to know the future we have to understand the past. in his time, transport, communications and scientific knowledge were less developed than they are today. news took weeks to travel whereas today satellite links connect the continents virtually instantaneously, but our technological advances in the field of communications seem not to improved our capacity to understand one another.

# "We understand each other better now than in Confucius' time because we can travel more quickly".

# "In Confucius' day people were more intelligent".

# "We have made great improvements in transport since Confucius' day".

# "Technological advances in communication and human capacity to understand one another are directly proportional".

# "In Confucius' day time news took months to travel".

# "According to Confucius the past has a linkage to the future".
# "Even with the fast developments of the technology we can't live happily".
Consider the following paragraph:
Every form of art is protected by copyright, upon the expiration of which the property passes to the public domain and becomes freely available to anyone wishing to exploit it commercially. the time has come when all treasures should pass to the control of a trust, and by this be made readily available to anyone on payment of a fee or royalty. The income from the works Van Gogh would alone be enormous. Those who now gain financial benefit from his genius should make some contribution to the welfare of the arts in general.

# "Instead of buying a ticket, museum goers should pay a fee to a trust for the benefit of arts".

# "It is not desirable to pass the control of treasures to a trust".

# "Van Gogh's paintings are not protected by copyright".

# "All artworks must be managed by a trust, so that the income generated can be used for the welfare of the arts".

# "Copyright in art is valid only for a limited period of time".

# "Van Gogh's descendants should be asked to make some contribution to the arts".
# "Van Gogh's works are under this copy right rule".
# "People are free to go to the public because of the copy right rule".
# "People gives to theater and collect the money for development".
# "We have asked the Van Gogh descendants to help for the developments of art".
Consider the following paragraph:
Organizing the home can be perceived as conferring power, so large numbers of women are unwilling to let go of chores, even when they have careers. A survey found that, out of 65 new marriages, not one single wife expected her husband to share work equally. According to the Family Policy Studies Center, 81% of working wives return home to do all the cooking. The average male has nearly half as much as more free time at weekends than his wife, and the typical new father spends just 37 seconds a day talking to his baby.

# "Only career women perceive organizing the home as conferring power".

# "The average wife has half as much free time at weekends as her husband".

# "The family planning studies center shows that 81% working wives do all the cooking at home".

# "19% working wives do not want to do the cooking at home".
# "Housewives want the husbands to take part equally in the household".
# "Wives have half as much leisure time as the husbands have".
# "39% of the men will work equally in the house in cleaning and washing".
Consider the following paragraph:
Statistics show that millions of vehicles have been carried by shuttle over the past 30 years through Alpine tunnels without one ever catching fire. In the Alpine tunnels, drivers and passengers sit in their vehicles on the shuttle trains. Only one vehicle has ever caught fire on the busy French motorail equivalent system. This sort of accident is not possible in a closed shuttle. Assertion that a vehicle fire will lead to catastrophe have no basis. Since the resources exist to detect, control and extinguish a fire, and to remove any persons present safely to an adjoining wagon, leaving any surviving fire facing rapid extinction within a wagon built to contain fire for 30 minutes, catastrophe seems very unlikely.

# " It is theoretically possible for a vehicle to catch fire even in a closed wagon".

# " The French motorail system is inferior to the shuttle train system."
# "No accident can occur in the closed tunnels".
# "Fire is allowed to live for 30 min".
# "All the cars that travel in the tunnels will be carried by rail shutters".
***********************
APTITUDE SECTION
Q1. Mr. Shah decided to walk down the escalator of a tube station. He found
that if he walks down 26 steps, he requires 30 seconds to reach the bottom.
However, if he steps down 34 stairs he would only require 18 seconds to
get to the bottom. If the time is measured from the moment the top step begins
to descend to the time he steps off the last step at the bottom, find out
the height of the stair way in steps?
Ans.46 steps.
Q2. The average age of 10 members of a committee is the same as it was 4
years ago, because an old member has been replaced by a young member. Find
how much younger is the new member ?
Ans.40 years.
Q3. Three containers A, B and C have volumes a, b, and c respectively; and
container A is full of water while the other two are empty. If from container
A water is poured into container B which becomes 1/3 full, and into
container C which becomes 1/2 full, how much water is left in container A?
Q4. ABCE is an isosceles trapezoid and ACDE is a rectangle. AB = 10 and EC = 20.
What is the length of AE?
Ans. AE = 10.
Q5. In the given figure, PA and PB are tangents to the circle at A and B respectively and
the chord BC is parallel to tangent PA. If AC = 6 cm, and length of the tangent AP
is 9 cm, then what is the length of the chord BC?
Ans. BC = 4 cm.
Q6. Three cards are drawn at random from an ordinary pack of cards. Find
the probability that they will consist of a king, a queen and an ace.
Ans. 64/2210.
Q7. A number of cats got together and decided to kill between them 999919
mice. Every cat killed an equal number of mice. Each cat killed more mice
than there were cats. How many cats do you think there were ?
Ans. 991.
Q8. If Log2 x - 5 Log x + 6 = 0, then what would the value / values of x
be?
Ans. x = e2 or e3.
Q9. The square of a two digit number is divided by half the number. After
36 is added to the quotient, this sum is then divided by 2. The digits of the
resulting number are the same as those in the original number, but they
are in reverse order. The ten's place of the original number is equal to twice
the difference between its digits. What is the number?
Ans. 46
Q10. Can you tender a one rupee note in such a manner that there shall be
total 50 coins but none of them would be 2 paise coins.?
Ans. 45 one paisa coins, 2 five paise coins, 2 ten paise coins, and 1 twenty-five paise coins.
Q11. A monkey starts climbing up a tree 20ft. tall. Each hour, it hops 3ft.
and slips back 2ft. How much time would it take the monkey to reach the
top?
Ans.18 hours.
Q12. What is the missing number in this series?
8 2 14 6 11 ? 14 6 18 12
Ans. 9
Q13. A certain type of mixture is prepared by mixing brand A at Rs.9 a kg.
with brand B at Rs.4 a kg. If the mixture is worth Rs.7 a kg., how many
kgs. of brand A are needed to make 40kgs. of the mixture?
Ans. Brand A needed is 24kgs.
Q14. A wizard named Nepo says "I am only three times my son's age. My father
is 40 years more than twice my age. Together the three of us are a mere 1240
years old." How old is Nepo?
Ans. 360 years old.
Q15. One dog tells the other that there are two dogs in front of me. The
other one also shouts that he too had two behind him. How many are they?
Ans. Three.
Q16. A man ate 100 bananas in five days, each day eating 6 more than the
previous day. How many bananas did he eat on the first day?
Ans. Eight.

Q17. If it takes five minutes to boil one egg, how long will it take to boil
four eggs?
Ans. Five minutes.
Q18. The minute hand of a clock overtakes the hour hand at intervals of 64
minutes of correct time. How much a day does the clock gain or lose?
Ans.32 8/11 minutes.
Q19. Solve for x and y:
1/x - 1/y = 1/3, 1/x2 + 1/y2 = 5/9.
Ans. x = 3/2 or -3 and y = 3 or -3/2.
Q20. Daal is now being sold at Rs. 20 a kg. During last month its rate was
Rs. 16 per kg. By how much percent should a family reduce its consumption so
as to keep the expenditure fixed?
Ans. 20 %.
Q21. Find the least value of 3x + 4y if x2y3 = 6.
Ans. 10.
Q23. Can you find out what day of the week was January 12, 1979?
Ans. Friday.
Q24. A garrison of 3300 men has provisions for 32 days, when given at a rate
of 850 grams per head. At the end of 7 days a reinforcement arrives and it was
found that now the provisions will last 8 days less, when given at the rate
of 825 grams per head. How, many more men can it feed?
Ans. 1700 men.
Q25. From 5 different green balls, four different blue balls and three
different red balls, how many combinations of balls can be chosen taking at least
one green and one blue ball?
Ans. 3720.
Q26. Three pipes, A, B, & C are attached to a tank. A & B can fill it in 20
& 30 minutes respectively while C can empty it in 15 minutes. If A, B & C
are kept open successively for 1 minute each, how soon will the tank be filled?
Ans. 167 minutes.
Q27. A person walking 5/6 of his usual rate is 40 minutes late. What is his
usual time?

Ans. 3 hours 20 minutes.

VERBAL SECTION
Directions: Give the synonyms for the following words
1. Depreciation: deflation, depression, devaluation, fall, slump
2. Depricate : feel and express disapproval,
3. Incentive : thing one encourages one to do (stimulus)
4. Echelon : level of authority or responsibility
5. Innovation : make changes or introduce new things
6. Intermittent : externally stopping and then starting
7. Detrimental: harmful
8. Conciliation : make less angry or more friendly

9. Orthodox: conventional or traditional, superstitious

10. Fallible : liable to error

11. Volatile : ever changing

12. Manifest: clear and obvious

13. Connotation : suggest or implied meaning of expression

14. Reciprocal: reverse or opposite

15. Agrarian : related to agriculture

16. Vacillate : undecided or dilemma

17. Expedient : fitting proper, desirable

18. Simulate : produce artificially resembling an existing one.

19. Access : to approah

20. Compensation: salary

21. Truncate : shorten by cutting

22. Adherence : stick

23. Heterogenous: non similar things

24. Surplus : excessive

25. Assess : determine the amount or value

26. Congnizance : knowledge

27. Retrospective : review

28. Naive : innocent,rustic

29. Equivocate : tallying on both sides, lie, mislead

30. Postulate : frame a theory

31. Latent : dormant, secret

32. Fluctuation : wavering,

33. Eliminate : to reduce

34. Affinity : strong liking

35. Expedite : hasten

36. Console : to show sympathy

37. Adversary : opposition

38. Affable : lovable or approachable

39. Decomposition : rotten

40. Agregious : apart from the crowd, especially bad

41. Conglomaration: group, collection

42. Aberration: deviation

43. Aurgury : prediction

44. Crediability : ability to common belief, quality of being credible

45. Coincident: incidentally

46. Constituent : accompanying

47. Differential : having or showing or making use of

48. Litigation : engaging in a law suit

49. Maratorium: legally or offficiallly determined period of dealy before
fulfillment of the agreement of paying of debts.

50. Negotiate : discuss or bargain

51. Preparation : act of preparing

52. Preponderant : superiority of power or quality

53. Relevance : quality of being relevant

54. Apparatus : appliances

55. Ignorance : blindness, in experience

56. Obsession: complex enthusiasm

57. precipitate : speed,active
SERIES SECTION
Directions: In the following questions complete the series
NOTE: This section is quite tough and consists of 26 questions to be done in 10 minutes. Please keep track of time.
1. A C B D E F G I - I H K J L
Ans. H


2. A I Z B E Y C I X D I - G E N J W
Ans. W


3. A D G J M P - R W T S
Ans. S


4. A B C E F G I J K - M L O N P
Ans. M

5. A B F G K L P Q - T S V U W
Ans. U


6. J W X U V S T - Q P S E T
Ans. Q


7. A R H X Y T D T W S T - N P T K R
Ans. P


8. F M B I P Z V I E V - I R Y O U


9. N Z I Y C X KW F - J F V M Y
Ans. V


10. A A S A S P A S P K A - R Q T S U
Ans. S


11. A E C P S - T R U E
Ans. U


12. B B P R D D L N F F I K - H Q J I K
Ans. H


13 A Z E X I V M T - R Q N S O
Ans. Q


14. A B D G K P - L I W U X
Ans. U


15. B C D A E G H I F J L M N L K N M O
Ans. K

16. X W E F G V U H I J K - P N S R T
Ans. T

17. O D J T O P Q N O E R T - Q O U V W
Ans. O

18. P R N U U P E J R B B - H V U N E
Ans. E


19.L U L M G M N F N P S - O N Q P S
Ans. P
NUMERICAL ABILITY
1. 420% OF 7.79 =?
Ans. 32.718

2. 3427 / 16.53 = ?
Ans. 202
3. 10995 /95 = ?
Ans.115.7365
4. 43+557-247 =?
Ans. 353
5. 3107*3.082=?
Ans. 9591
6. 48.7 + 24.9 - 8.7 =?
Ans. 64.90
7.525.0/47.8 =?
Ans. 11
8. (135-30-14)*7 - 6 +2 =?
Ans. 3
9. 3/8 * 5.04=?
Ans. 1.89
10. 697 /219 =?
Ans. 3.18
11.8/64 +64/16 =?
Ans. 4.14
12. 298 * 312 / 208 = ?
Ans. 453.54
13. 0.33 *1496 /13 = ?
Ans. 37.98
14.0.26 + 1/8 = ?
Ans. 0.385
15. 66.17+1/3= ?
Ans. 67.03
16. 2.84+1/4= ?
Ans. 3.09
17. 33% OF 450 = ?
Ans. 148.5
18. 907.54 / 0,3073= ?
Ans.3002
19.There are two categories of persons in ratio A:B = 2:3. A type earns 2.5 dollars/hr and
B type 1 dollar/hr total money earned by both is 24dollars. Then what is the total number of persons
Ans. 15
20. Total balls are z, the number of red balls is n and the remaining are blak balls,then the % of black
balls equal to ?
Ans. (z - n) / z*100
21. If A = C, B = 2D what should be done to make the ratio same. i.e.a/b = c/d
Ans. Multiply A by 2
22. If P=Total number of components, Q = number of defective components .What is the % of non defective components?
Ans. (p-q) / p*100
23. If the cost of an article is x , first discount given is y% of cost, second discount given is z% of cost .
The selling price of x is

Ans. x (1-y / 100) (1- z / 100)
24. Which of the following are prime numbers?
(a) 119
(b) 115
(c) 127
(d) none
Ans. (c)
25. A / B = C; C > D then
(a) A is always greater than D
(b) C is always greater than D
(c) B is always less than D
(d) None of these
Ans. (a)
26. If B>C and AAns. ( A + B )C
27. If for H hours of work the salary is S and the employee gets x hours of medical leave, then what is the salary/hr ?
Ans. s/H-x
28. ( 1/6 of 596) / (0.695) = ?
Ans. 142
29. 35-30 + 4/7 - 5 + 1 = ?

Ans. 3
30. 10995 + 95 = ?
Ans. 11090
31. If on a salary of Rs."S" per month,one has to pay one tax of x Rs. and a second type of tax of y Rs
then % of salary taken home is?

Ans. s-(x+y)/s * 100
32. B>A then which expression will be highest value
(a) A-B
(b) AB
(c) A+B
(d) Can't Say
Ans. (b)
33. K, L are men who take home a salary of x, y respectively.The total amount taken home is
Ans. Kx + Ly

34. If out of X bulbs y bulbs are broken;The % of non broken bulbs
Ans. (x-y) / x*100

35. If on a salary s per month, a tax of x% of the salary and another of r% of the salary is deducted what
is the income.

Ans. s*(1-(x+r)/100
36. 0.512 * 18902358 =?
Ans. 9678007.296
37. If the % of defective balls is 10% balls,and the number of defective balls is 5.The number of balls is

Ans.50

38. 6.29% of 2.8 =?
Ans. 0.18
39. 0.398 * 456= ?
Ans. 181.49

40. 0 < x < 1 which is greater
(a) 1/x2
(b) 1/x
(c) x
(d) x2

Ans. (a)
41. If c = a/b; a-1 = c, what is the relation between a and b?

Ans. b = a/a-1
42. What is the sum of 7 consecutive odd numbers with 27 as the fourth number
Ans.189

1. Last month of an year
(a) January (b) February (c) December (d) November

2. Select the odd one
(a) January (b) February (c) Wednesday (d) November

3. Select the antonym of capture from the following
(a) attack (b) Release (c) condemn (d) None of the above

4. Find the antonym of autumn
(a) Spring (b) Winter (c) Summer (d) None of the above

5. One skirt requires 3.75 yards of cloth. How many skirts you can make from 45 yards?
Ans: 12 skirts

6. How can you make a square from two triangles?

7. Is the meaning of Client and Customer,
(a) same (b) contradictory (c) no relation

8. Is the meaning of It's and Its,
(a) same (b) contradictory (c) no relation

9. Is the meaning of Canvas and Canvass,
(a) same (b) contradictory (c) no relation

10. Is the meaning of Ingenious and Ingenuous,
(a) same (b) contradictory (c) no relation

11. Is the meaning of Credible and Credulous,
(a) same (b) contradictory (c) no relation

12. Select the odd one out.
(a) 1/4 (b) 1/3 (c) 1/6 (d) 1/18

13. Select the least from the following.
(a) 0.99 (b) 1 (c) 81 (d) 0.333

14. Find the next number in the series. 1, 0.5, 0.25, 0.125
Ans: 0.0625

15. One dollar is saved in one month. Then how much dollar is saved in one day?
Ans: 1/30 =0.0333$

16. Y catches 5 times more fishes than X. If total number of fishes caught by X and Y is 48, then number of fishes caught by X?
Ans: 8

17. Y catches 5 times more fishes than X. If total number of fishes caught by X and Y is 42, then number of fishes caught by X?
Ans: 7

18. If a train covers 600m in 0.5 seconds, how long it will cover in 10 seconds?
Ans: 3000m = 3km

19. The girl's age is twice that of boy, if the boy is four years old. After four years the age
of the girl is
Ans: 12 years

20. Sister's age is twice than that of the brother. If the brother's age is six, what is the sister's age after two years?
Ans: 14 Yrs.

21. Two lemons cost 10 cents. Then one and a half dozen cost
Ans: 90 cents

22. A clock is late by 1 minute 27 seconds in a month. Then how much will it be late in 1 day?
Ans: 2.9 seconds

23. Which of the following figures together will make a triangle?
Ans: a,b,c,d

24. Make a square by drawing only one line
Ans: line 2-5, square 2-3-4-5-2

25. Which of the following is the odd one? crew, constellation, companion, league, participants.
Ans: companion

26. Opposite of Remote?
(a) Far (b) Near (c) Huge (d) Village

27. Statement A: All great men are ridiculous;
Statement B: I am ridiculous ;
Inference : I am a great man;
(a) True (b) False (c) Not clear

28. Statement: Normal children are active;
Inference: All children are active;
(a) True (b) False (c) Uncertain

29. Next number in the series 1, 1/2, 1/4, 1/8 ?
Ans: 1/16

30. In 6 seconds a light flashes once. In one hour how many times it will flash?
Ans: 601 times

31. At 20% discount, a cycle is sold at a selling price of 2500 Rs. What is the actual price?
Ans: Rs. 3125

32. Statement A: A & B have same age;
Statement B: B is younger than C;
Inference : A is younger than C;
(a) True (b) False (c) Uncertain

33. All chickens lay eggs (True/False)
Ans: False

34. A invests $12000, B invests $8000, C invests $6000 and they got a profit of $1200. How much share A got more than B and C?
Ans: 2/13 and 3/13

***************************
1.
(a) The gap between the average starting salaries of teachers and those of other professionals has shrunk in recent years.
(b) The average age of first year teachers is same as it was in 1975.
(c) Starting teachers are no longer underpaid.
(d) The extent of a persons formal education is a measure by which to determine his level of salary.
(e) Over the last few years, the average starting salaries of other professionals have increased by 20%
(a) ebd (b) bad (c) abc (d) aec
Answer is d; the statement a,e,c are logically sequenced.

1.
(a) Japan now produces more semiconductors, than US.
(b) Semiconductors are one of the fastest growing industry segments.
(c) A decade ago Japan was producing 24% and the US was producing 22% of the worlds semiconductors, respectively.
(d) 10 years ago Japan ranked third in semiconductor production.
(e) During the last 10 years Japans production of semiconductors has increased by 500% while that of the us has increased by 200%
(a) abd (b) cea (c) edc (d) bcd

2.
(a) Coding program 1 (b) Writing specifications for program 1
(c) Integrating program 1 with other programs (d) Testing program 1
(e) Collecting cheque from the client of the program
(a) edcba (b) abcde (c) badce (d) abdce

3.
(a) Bob is older than Dinku and Ismer (b) Rahul is oldet than Dinku
(c) Rahul is younger than Bob (d) Rahul is older than Ismer (e) Dinku is older than Ismer
(a) edb (b) bcd (c) dab (d) abc

4.
(a) Defining the data type of the variable (b) Using the variable (c) Declaring the variable
(d) Initializing the variable (e) Remove the variable from the memory
(a) cadbe (b) abcde (c) cdb (d) acdbe

5.
(a) In the last six months the number of robberies at gun point in the city has dropped by 18%
(b) Guns are necessary protection against robbers
(c) Strict gun control causes a decrease in violent crime
(d) Most crimes are committed with guns and knives
(e) Six months ago this city's council passed a gun control law
(a) bda (b) acb (c) ebc (d) eac

6.
(a) All missiles follow a fixed trajectory (b) The boomerang requires a high degree of skill
(c) A boomerang is a missile (d) The boomerang is used by Australian aborigines to hunt
(e) A boomerang normally has an elliptical flight path

(a) adc (b) aec (c) cba (d) ebd

7.(a) Saving the source file (b) Compiler execution (c) Pre-processor execution
(d) Bug fixing (e) Reading the error file
(a) eabcd (b) acbed (c) abced (d) cbeda

8.
(a) But if powers that be, extended any, how will be the first one to take might claim
(b) I don't believe in seeking special privileges because I'm a woman
(c) Let me explain this in context of what happened the other
(a) bac (b) acb (c) bca (d) abc

9.
(a) A long search produce a comprehensive list of 203 manufacturing firms
(b) The number of workers employed by the firms in the area ranged from a dozen to approximately 3500
(c) Those concerned with mining and quarrying, construction ,transport, trade and commerce were excluded
(d) The investigation was confined to manufacturing firms in the area
(a) bcda (b) bcab (c) abcd (d) dabc

10.
(a) The quickly came back with pots laden with water
(b) The water gurgled out and the dying embers hissed and send up little curls of vapour
(c) The poured it on the glowing bed of charcoal
(d) The men jumped up and rushed to the river
(a) acdb (b) bacb (c) dabc (d) dcba

SECTION 2- DATA SUFFICIENCY

Each item has a question followed by two statements :

Mark a: If the question can be answered with the help of statement "1" alone

Mark b: If the question can be answered with the help of statement "2" alone

Mark c: If the question can be answered with the help of both the statements but not with the help of either statement by itself

Mark d: If the question cannot be answered even with the help of both the given statements

Example:

1. Does winking improve eye sight?

1) During the process of winking the focal power of eyes improves

2) Experiments have shown that eye exercise lead to an improvement in eye sight
Answer: d because neither 1 or 2 is adequate.

Questions :
11. Each floor of a 3 storeyed building is occupied and a total of 15 people live in the building. How many live on the first floor?
1) The no. of people living in the first floor is an odd number
2) The no. of people living on the first floor double the number living on the second floor

12. Program 1 can be implemented
1) Program 1 is tested and error free
2) The implementation site is ready

13. The sum of digits of a 5 digit no. is 10. The digit in the ten thousandth place is cube of that of units place. what is the number.
1) The digits in the thousandth, hundredth and tenth place are equal
2) The digit in the units and tenth place are not equal

14. If I deposit Rs.1000 in the bank now and withdraw the amount only at the end of the year how much will I get?
1) The rate of compound interest is 12% per year
2) The interest is deposited in the account at the end of every six months

15. Variable "X" is an address variable.
1) The value of variable "X" is "adbcf"
2) Program has a statement X =&Y

16. Is white color the best reflector of light?
1) The lower a color's reflection index the better its power of reflection
2) White has a reflection index of 0.28

17. Does Mehta work in an advertising agency?
1) Mehta begins work at 9 am in the morning and works till 9 in the night
2) Mehta is a copywriter

18. Is it true that Maggi Noodles success was largely due to its ability to satisfy a latent consumer need?
1) Before the entry of Maggi Noodles, Others did not have access to a food item which was convenient to prepare and could be consumed between meals.
2) Maggi Noodles was an instant hit with ladies who had children in the range of 10 to 12 years

19. Sachin wrote Program 1
1) It is found in the directory c:\user\sachin
2) Sachin tested Program 1

20. Are all Argots also Knicks?
1) All Argots are Drones
2) All Drones are Knicks

21. Does classical music aid plant growth?
1) Music aids in the development of sugar in plants.
2) In an experiment conducted, its was observed that plants exposed to classical music grew by 5cm more than plants not exposed to classical music in the same period.

22. Are cheques the safest method of making a payment.
1) Cheques are more convenient than cash in making and resolving payments.
2) Payment by cheques eliminate the risk involved in handling cash.

23. Networking is working fine.
1) Computer A is able to talk to Computer B
2) Both Computer A & B are Pentium Machines.

24. Is it true that smiling is easier than frowning?
1) Smiling requires the movement of 14 facial muscles while frowning requires the movement of 24 facial muscles.
2) Moving every facial muscles requires the same amount of effort.

25. Is it true that the Carpenter lives on the first floor.?
1) the Barber lives two floors above the black smith who in turn stays one floor above the carpenter.
2) the blacksmith lives two floors above the weaver who lives one floor below the carpenter in a three storeyed building.

SECTION 3 - ANALYTICAL.

Questions 26-29 are based on the following:

At a formal dinner for 8, the host and the hostess are seated at opposite ends of a rectangular table, with 3 persons along each side. Each man must be seated next to at least to 1 woman, and vice versa. Alan is opposite to Diana, who is not the hostess. George has a woman on his right and is opposite to a woman. Helga is at the hostess's right, next to Frank. One person is seated between Belinda and Carol.

26.The 8th person present, Eric must be
(a) the host
(b) seated to Diana's right
(c) seated opposite to Carol
(a) a only (b) c only (c) b and c (d) a, b and c

27. If each person is placed directly opposite to his or her spouse, which of the following pairs must be married.
(a) George and Helga (b) Belinda and Frank
(c) Carol and Frank (d) George and Belinda

28. Which person is not seated next to a person of the same sex.?
(a) Alan (b) Belinda (c) Carol (d) Diana

29. George is bothered by the cigarette smoke of his neighbor and exchanges seats with the person 4 places to his left. Which of the following must be true following the exchange?
(a) No one is seated between two persons of the opposite sex.
(b) one side of the table consists entirely of persons of the same sex.
(c) Either the host or hostess has changed seats
(a) A only (b) C only (c) A and B (d) B and C

Questions 30 - 33 are based on the following:
The hotel Miramar has two wings, the east wing and the west wing. Some east wing rooms but not all, have an ocean view. All west wing rooms have a harbor view. The charge for all rooms is identical except for the following.
There is an extra charge for all harbor view rooms on or above third floor. There is an extra charge for all ocean view rooms except those without balcony. Some harbor view rooms on the first two floors and some east wing rooms without ocean view have kitchen facilities for which there is an extra charge. Only the ocean view and harbor view rooms have balconies.

30. A guest may avoid an extra charge by requesting
(a) A west wing room on one of the first two floors.
(b) A west wing room on the fourth floor without balcony.
(c) An East wing room without balcony. (d) Any room without kitchen.

31. Which of the following must be true if all conditions are as stated?
(a) All rooms above the third floor involves extra charges.
(b) No room without an ocean or harbor view or kitchen facilities involves extra charge.
(c) There is no extra charge for an east wing room without ocean view.
(d) There is no extra charge for any room without Kitchen facilities.

32. which of the following must be false if all conditions are as stated?
(a) some ocean viewing rooms do not involve an extra charge
(b) all rooms with kitchen facilities involve an extra charge
(c) some west viewing rooms above the second floor do not involve an extra charge
(d) some harbor viewing rooms do not involve an extra charge

33. Which of the following can not be determined on the basis of the information given?
(a) whether there are any rooms without a balcony for which extra charge is imposed
(b) whether any room without at kitchen or a view involves an extra charge
(c) whether two extra charges are imposed for any room (d) none of the above

Questions 34 to 37 are based on the following:
Four cards of different suits are dealt one apiece to A, B, C and D.
B says: Mine is not a club. A says: Mine is not a spade.
D says: Mine is not a diamond. C says: Mine is not a spade.
A says: Mine is not a heart.

34. A held
(a) heart (b) clubs (c) diamonds (d) spade

35. B held
(a) heart (b) clubs (c) diamonds (d) spade

36. C held
(a) heart (b) clubs (c) diamonds (d) spade

37. D held
(a) heart (b) clubs (c) diamonds (d) spade

Questions 38 to 40 are based on the following:
In a magical temple there are 3 doorways each leading to the interior of the temple. Every door way has an idol just inside. The magical powers of the temple doubles the flowers a devotee carries every time he/she passes under a doorway. Each devotee has to pass on straight through the doorway and cannot retrace his steps till he comes to the innermost idol.

38. Ram carries X flowers at each idol he places an identical number of flowers Y. He returns from the temple without a single flower. X was most probably
(a) 2 (b) 5 (c) 6 (d) 7

39. In the situation above Y was most probably
(a) 8 (b) 5 (c) 6 (d) 7

40. If Sita took 8 flowers to the temple and offered 4 flowers each to the first two idols then by the time she faces the third idol she has
(a) 40 flowers (b) 36 flowers (c) 52 flowers (d) 56 flowers

SECTION 4 - COMPUTATIONAL.

41. 2 passengers have together 560 kgs of luggage and are charged for the excess above the weight allowed at 10$ and 26$. If all the luggage had belonged to one of them he would have to pay 46$. The amount of luggage each passenger is allowed without any charge is
(a) 100 kg (b) 150 kg (c) 160 kg (d) Insufficient data

42. 6 pigs cost the same as 9 sheep. 27 sheep cost the same as 30 goats. 50 goats cost the same as 3 elephants. If two elephants cost $4800, then the cost of one pig in dollar is
(a) 120 (b) 240 (c) 105 (d) 250

43. A wholesaler allows a discount of 20 % on the list price to the retailer. The retailer sells at 5% below the list price. If the customer pays Rs.19 for an article what profit is made by the retailer on it?
(a) Rs.2 (b) Rs.3 (c) Rs.4 (d) Rs.4.5

44. A circular metal plate of even thickness has 12 holes of radius 1 cm drilled into it. As a result the plate lost 1/6th its original weight. The radius of the circular plate is
(a) 16sqrt2 (b) 8sqrt2 (c) 32sqrt2 (d) sqrt72

45. 3 machines a,b,c can be used to produce a product. Machine a will take 60 hours to produce a million units. Machine b is twice as fast as machine a. Machine c takes the same amount of time as machine a and b taken together. How much time will be required to produce a million units if all the three machines are used simultaneously?
(a) 12 hours (b) 10 hours (c) 8 hours (d) 6 hours

AXES TECHNOLOGY
:
The written test is purely technical with stress on Networking, C, Operating Systems, Data Structures etc. You should be clear with the fundamentals of these and other core subjects. Questions are mainly multiple-choice though this may vary.
1. One of the following is my secret word: AIM DUE MOD OAT TIE. With the list in front of you, if I were to tell you any one of my secret word, then you would be able to tell me the number of vowels in my secret word. Which is my secret word?
Ans. TIE

2. In the following figure: A B C
D
E F G
H
I
Each of the digits 1, 2, 3, 4, 5, 6, 7, 8, and 9 is:
a) Represented by a different letter in the figure above.
b) Positioned in the figure above so that each of A + B + C, C + D +E, E + F + G, and G + H + I is equal to 13.
Which digit does E represent?
Ans. E is 4

3. One of Mr. Horton, his wife, their son, and Mr. Horton's mother is a doctor and another is a lawyer.
a) If the doctor is younger than the lawyer, then the doctor and the lawyer are not blood relatives.
b) If the doctor is a woman, then the doctor and the lawyer are blood relatives.
c) If the lawyer is a man, then the doctor is a man.
Whose occupation you know?
Ans. Mr. Horton: he is the doctor.

4. Here is a picture of two cubes:
INCLUDEPICTURE "http://freshersworld.com/questions/aztec.gif" \* MERGEFORMATINET

a) The two cubes are exactly alike.
b) The hidden faces indicated by the dots have the same alphabet on them.
Which alphabet-q, r, w, or k is on the faces indicated by the dots?
Ans. q

5. In the following figure:
A D
B G E
C F
Each of the seven digits from 0, 1, 2, 3, 4, 5, 6, 7, 8, and 9 is:
a) Represented by a different letter in the figure above.
b) Positioned in the figure above so that A*B*C, B*G*E, and D*E*F are equal.
Which digit does G represent?
Ans. G represents the digit 2.

6. Mr. and Mrs. Aye and Mr. and Mrs. Bee competed in a chess tournament. Of the three games played:
a) In only the first game were the two players married to each other.
b) The men won two games and the women won one game.
c) The Ayes won more games than the Bees.
d) Anyone who lost game did not play the subsequent game.
Who did not lose a game?
Ans. Mrs. Bee did not lose a game.

7. Three piles of chips--pile I consists one chip, pile II consists of chips, and pile III consists of three chips--are to be used in game played by Anita and Brinda. The game requires:
a) That each player in turn take only one chip or all chips from just one pile.
b) That the player who has to take the last chip loses.
c) That Anita now have her turn.
From which pile should Anita draw in order to win?
Ans. Pile II

8. Of Abdul, Binoy, and Chandini:
a) Each member belongs to the Tee family whose members always tell the truth or to the El family whose members always lie.
b) Abdul says ''Either I belong or Binoy belongs to a different family from the other two."
Whose family name do you know?
Ans. Binoy's family--El.

***********************
For a motorist there are three ways going from City A to City C. By way of bridge the distance is 20 miles and toll is $0.75. A tunnel between the two cities is a distance of 10 miles and toll is $1.00 for the vehicle and driver and $0.10 for each passenger. A two-lane highway without toll goes east for 30 miles to city B and then 20 miles in a northwest direction to City C.


1. Which is the shortest route from B to C

(a) Directly on toll free highway to City C
(b) The bridge
(c) The Tunnel
(d) The bridge or the tunnel
(e) The bridge only if traffic is heavy on the toll free highway

Ans. (a)


2. The most economical way of going from City A to City B, in terms of toll and distance is to use the

(a) tunnel
(b) bridge
(c) bridge or tunnel
(d) toll free highway
(e) bridge and highway

Ans. (a)


3. Jim usually drives alone from City C to City A every working day. His firm deducts a percentage of employee pay for lateness. Which factor would most influence his choice of the bridge or the tunnel ?

(a) Whether his wife goes with him
(b) scenic beauty on the route
(c) Traffic conditions on the road, bridge and tunnel
(d) saving $0.25 in tolls
(e) price of gasoline consumed in covering additional 10 miles on the bridge

Ans. (a)


4. In choosing between the use of the bridge and the tunnel the chief factor(s) would be:
I. Traffic and road conditions
II. Number of passengers in the car
III. Location of one's homes in the center or outskirts of one of the cities
IV. Desire to save $0.25

(a) I only
(b) II only
(c) II and III only
(d) III and IV only
(e) I and II only

Ans. (a)


Q2.
The letters A, B, C, D, E, F and G, not necessarily in that order, stand for seven consecutive integers from 1 to 10
D is 3 less than A
B is the middle term
F is as much less than B as C is greater than D
G is greater than F

1. The fifth integer is
(a) A
(b) C
(c) D
(d) E
(e) F

Ans. (a)

2. A is as much greater than F as which integer is less than G
(a) A
(b) B
(c) C
(d) D
(e) E

Ans. (a)

3. If A = 7, the sum of E and G is
(a) 8
(b) 10
(c) 12
(d) 14
(e) 16

Ans. (a)

4. A - F = ?
(a) 1
(b) 2
(c) 3
(d) 4
(e) Cannot be determined

Ans. (a)


5. An integer T is as much greater than C as C is greater than E. T can be written as A + E. What is D?
(a) 2
(b) 3
(c) 4
(d) 5
(e) Cannot be determined

Ans. (a)


6. The greatest possible value of C is how much greater than the smallest possible value of D?
(a) 2
(b) 3
(c) 4
(d) 5
(e) 6

Ans. (a)



Q3.
1. All G's are H's
2. All G's are J's or K's
3. All J's and K's are G's
4. All L's are K's
5. All N's are M's
6. No M's are G's


1. If no P's are K's, which of the following must be true?

(a) All P's are J's
(b) No P is a G
(c) No P is an H
(d) If any P is an H it is a G
(e) If any P is a G it is a J

Ans. (a)


2. Which of the following can be logically deduced from the conditions stated?

(a) No M's are H's
(b) No M's that are not N's are H's
(c) No H's are M's
(d) Some M's are H's
(e) All M's are H's

Ans. (a)


3. Which of the following is inconsistent with one or more of the conditions?

(a) All H's are G's
(b) All H's that are not G's are M's
(c) Some H's are both M's and G's
(d) No M's are H's
(e) All M's are H's

Ans. (a)


4. The statement "No L's are J's" is
I. Logically deducible from the conditions stated
II. Consistent with but not deducible from the conditions stated
III. Deducible from the stated conditions together with the additional statement "No J's are K's"

(a) I only
(b) II only
(c) III only
(d) II and III only
(e) Neither I, II nor III

Ans. (a)



Q5.
In country X, democratic, conservative and justice parties have fought three civil wars in twenty years. TO restore stability an agreement is reached to rotate the top offices President, Prime Minister and Army Chief among the parties so that each party controls one and only one office at all times. The three top office holders must each have two deputies, one from each of the other parties. Each deputy must choose a staff composed of equally members of his or her chiefs party and member of the third party.

1. When Justice party holds one of the top offices, which of the following cannot be true

(a) Some of the staff members within that office are justice party members
(b) Some of the staff members within that office are democratic party members
(c) Two of the deputies within the other offices are justice party members
(d) Two of the deputies within the other offices are conservative party members
(e) Some of the staff members within the other offices are justice party members.

Ans. (a)


2. When the democratic party holds presidency, the staff of the prime minister's deputies are composed
I. One-fourth of democratic party members
II. One-half of justice party members and one-fourth of conservative party members
III. One-half of conservative party members and one-fourth of justice party members.

(a) I only
(b) I and II only
(c) II or III but not both
(d) I and II or I and III
(e) None of these

Ans. (a)


3. Which of the following is allowable under the rules as stated:

(a) More than half of the staff within a given office belonging to a single party
(b) Half of the staff within a given office belonging to a single party
(c) Any person having a member of the same party as his or her immediate superior
(d) Half the total number of staff members in all three offices belonging to a single party
(e) Half the staff members in a given office belonging to parties different from the party of the top office holder in that office.

Ans. (a)


4. The office of the Army Chief passes from Conservative to Justice party. Which of the following must be fired.

(a) The democratic deputy and all staff members belonging to Justice party
(b) Justice party deputy and all his or hers staff members
(c) Justice party deputy and half of his Conservative staff members in the chief of staff office
(d) The Conservative deputy and all of his or her staff members belonging to Conservative party
(e) No deputies and all staff members belonging to conservative parties.

Ans. (a)



Q6.
In recommendations to the board of trustees a tuition increase of $500 per year, the president of the university said "There were no student demonstrations over the previous increases of $300 last year and $200 the year before". If the president's statement is accurate then which of the following can be validly inferred from the information given:
I. Most students in previous years felt that the increases were justified because of increased operating costs.
II. Student apathy was responsible for the failure of students to protest the previous tuition increases.
III. Students are not likely to demonstrate over new tuition increases.

(a) I only
(b) II only
(c) I or II but not both
(d) I, II and III
(e) None

Ans. (a)


Part II -- Problems on ages ( Simple Linear Equations etc.)
Part III -- Blood Relations (Check R S Aggarwal)

Part IV -- Series problems

6,9,14,21,(30)
2,10,(30),68,130,(222)
9,15,23,33,(45)
5,11,19,29,(41),55
2,12,30,56,90,(132)
1,3,7,(15)31
3,9,21,(45),93
35,24,15,8,(3)
2,12,30,56,90,(132)
3,11,19,29,(39),51
Part V -- Analogy

1. fans : bleachers::

(a) cheerleaders : pompoms
(b) audience:seats
(c) team:goalposts
(d) conductor:podium
(e) referee:decision

Ans. (a)


2. archipelago:islands::

(a) arbor:bower
(b) garden:flower
(c) mountain:valley
(d) sand:dune
(e) constellation:star

Ans. (a)


3. crow:boastful ::

(a) smirk:witty
(b) conceal:s;y
(c) pout:sulky
(d) blush:coarse
(e) bluster:unhappy

Ans. (a)


4. bracket:shelf ::

(a) hammer:anvil
(b) girder:rivet
(c) strut:rafter
(d) valve:pipe
(e) bucket:well

Ans. (a)


5. taxonomy:classification ::

(a) etymology:derivation
(b) autonomy:authorization
(c) economy:rationalization
(d) tautology:justification
(e) ecology:urbanisation

Ans. (a)


6. moderator:debate ::

(a) legislator:election
(b) chef:banquet
(c) auditor:lecture
(d) conspirator:plot
(e) umpire:game

Ans. (a)


7. glossary:words ::

(a) catalogue:dates
(b) atlas:maps
(c) almanac:synonyms
(d) thesaurus:rhymes
(e) lexicon:numbers

Ans. (a)

8. lumber: bear ::

(a) roost:hen
(b) bray:donkey
(c) waddle:goose
(d) swoop:hawk
(e) chirp:sparrow

Ans. (a)

9. celerity:snail ::

(a) indolence:sloth
(b) cunning:weasel
(c) curiosity:cat
(d) humility:peacock
(e) obstinacy:mule

Ans. (a)



10. wood:sand ::

(a) coal:burn
(b) brick:lay
(c) oil:polish
(d) metal:burnish
(e) stone:quarry

Ans. (a)


11. carpenter:saw ::

(a) stenographer:typist
(b) painter:brush
(c) lawyer:brief
(d) runner:sneakers
(e) seamstress:scissors

Ans. (a)


12. horns:bull ::

(a) mane:lion
(b) wattles:turkey
(c) antlers:stag
(d) hooves:horse
(e) wings:eagle

Ans. (a)


13. gullible:duped ::

(a) credible:cheated
(b) careful:cautioned
(c) malleable:moulded
(d) myopic:mislead
(e) articulate:silenced

Ans. (a)


14. marathon:stamina ::

(a) relay:independence
(b) hurdle:perseverance
(c) sprint:celerity
(d) job:weariness
(e) ramble:directness

Ans. (a)


15. Skin:man ::

(a) hide:animal
(b) jump:start
(c) peel:potato
(d) eat:food
(e) wool:cloth

Ans. (a)


16. Bamboo:Shoot ::

(a) Bean:Sprout
(b) Peas:Pod
(c) Potato:Eye
(d) Carrot:Root
(e) Leaf:Stem

Ans. (a)


18. Deflect:Missile ::

(a) Siege:Castle
(b) Distract:Attraction
(c) Protect:Honour
(d) Drop:Catch
(e) Score:Goal

Ans. (a)


19. Editor:magazine ::

(a) captain:ship
(b) actor:movie
(c) director:film
(d) player:team
(e) jockey:horse

Ans. (a)


21. Volcano : Lava ::

(a) Fault:earthquate
(b) crack:wall
(c) tunnel:dig
(d) water:swim
(e) floor:polish

Ans. (a)

Other Sample Analogies:

22. Agitator : Firebrand :: Renegade : Turncoat

23. Burst : Sound :: Tinder : Fire

24. Star : cluster :: Tree : clump

25. Piston : Cylinder :: elevator : shaft

26. Mitigate : punishment :: commute : sentence

27. Erudite : scholar :: illiterate : ignorant

28. Fire : Ashes :: explosion : debris

29. mason : wall :: Author : Book

30. Fire : Ashes :: Event : memories




SECTION II CODING

1. If LOAD = MPBE and DRIVE = ESJWF
Then LADDLER = ?

Ans. MBEEMFS


2. START = WALKA and BUDPI = XZFMR
Then STUPID = ?

Ans. WAZMRF


Q3 - Q7

If A=Z, B=Y, C=X, .......... Z=A
Then

3. LIMIT = ?

Ans. ORNRG


4. SOUR = ?

Ans. HLFI


5. POCKET = ?

Ans. KLXPUG


6. GROUP = ?

Ans. TILFK


7. ZERD = ?

Ans. AVIL



Q8 - Q9
Here each letter is coded as
A = D, B=E, C=F


8. SHOOT = ?

Ans. VKRRW


9. VWDUW = ?

Ans. YZGXZ


10. If DBMDVUUB = CALCUTTA
Then BOMBAY = ?

Ans. ANLAZX


************
ANALYTICAL REASONING SECTION
Directions for questions 1-5: The questions are based on the information given below

There are six steps that lead from the first to the second floor. No two people can be on the same step
Mr. A is two steps below Mr. C
Mr. B is a step next to Mr. D
Only one step is vacant ( No one standing on that step )
Denote the first step by step 1 and second step by step 2 etc.
1. If Mr. A is on the first step, Which of the following is true?
(a) Mr. B is on the second step
(b) Mr. C is on the fourth step.
(c) A person Mr. E, could be on the third step
(d) Mr. D is on higher step than Mr. C.
Ans: (d)
2. If Mr. E was on the third step & Mr. B was on a higher step than Mr. E which step must be vacant
(a) step 1
(b) step 2
(c) step 4
(d) step 5
(e) step 6
Ans: (a)
3. If Mr. B was on step 1, which step could A be on?
(a) 2&e only
(b) 3&5 only
(c) 3&4 only
(d) 4&5 only
(e) 2&4 only
Ans: (c)
4. If there were two steps between the step that A was standing and the step that B was standing on, and A was on a higher step than D , A must be on step
(a) 2
(b) 3
(c) 4
(d) 5
(e) 6
Ans: (c)

5. Which of the following is false

i. B&D can be both on odd-numbered steps in one configuration
ii. In a particular configuration A and C must either both an odd numbered steps or both an even-numbered steps
iii. A person E can be on a step next to the vacant step.
(a) i only
(b) ii only
(c) iii only
(d) both i and iii
Ans: (c)

Directions for questions 6-9: The questions are based on the information given below
Six swimmers A, B, C, D, E, F compete in a race. The outcome is as follows.
i. B does not win.
ii. Only two swimmers separate E & D
iii. A is behind D & E
iv. B is ahead of E , with one swimmer intervening
v. F is a head of D
6. Who stood fifth in the race ?
(a) A
(b) B
(c) C
(d) D
(e) E
Ans: (e)
7. How many swimmers seperate A and F ?
(a) 1
(b) 2
(c) 3
(d) 4
(e) cannot be determined
Ans: (d)
8. The swimmer between C & E is
(a) none
(b) F
(c) D
(d) B
(e) A
Ans: (a)


9. If the end of the race, swimmer D is disqualified by the Judges then swimmer B finishes in which place
(a) 1
(b) 2
(c) 3
(d) 4
(e) 5
Ans: (b)
Directions for questions 10-14: The questions are based on the information given below

Five houses lettered A,B,C,D, & E are built in a row next to each other. The houses are lined up in the order A,B,C,D, & E. Each of the five houses has a colored chimney. The roof and chimney of each housemust be painted as follows.
i. The roof must be painted either green,red ,or yellow.
ii. The chimney must be painted either white, black, or red.
iii. No house may have the same color chimney as the color of roof.
iv. No house may use any of the same colors that the every next house uses.
v. House E has a green roof.
vi. House B has a red roof and a black chimney
10. Which of the following is true ?
(a) At least two houses have black chimney.
(b) At least two houses have red roofs.
(c) At least two houses have white chimneys
(d) At least two houses have green roofs
(e) At least two houses have yellow roofs
Ans: (c)
11. Which must be false ?
(a) House A has a yellow roof
(b) House A & C have different color chimney
(c) House D has a black chimney
(d) House E has a white chimney
(e) House B&D have the same color roof.
Ans: (b)
12. If house C has a yellow roof. Which must be true.
(a) House E has a white chimney
(b) House E has a black chimney
(c) House E has a red chimney
(d) House D has a red chimney
(e) House C has a black chimney
Ans: (a)
13. Which possible combinations of roof & chimney can house
I. A red roof 7 a black chimney
II. A yellow roof & a red chimney
III. A yellow roof & a black chimney

(a) I only
(b) II only
(c) III only
(d) I & II only
(e) I&II&III
Ans: (e)
14. What is the maximum total number of green roofs for houses
(a) 1
(b) 2
(c) 3
(d) 4
(e) 5
NOTE: The questions from 15-27 are multiple choice in the paper

15. There are 5 red shoes, 4 green shoes. If one draw randomly a shoe what is the probability of getting a red shoe

Ans 5c1/ 9c1


16. What is the selling price of a car? If the cost of the car is Rs.60 and a profit of 10% over selling price is earned

Ans: Rs 66/-


17. 1/3 of girls , 1/2 of boys go to canteen .What factor and total number of classmates go to canteen.

Ans: Cannot be determined.


18. The price of a product is reduced by 30% . By what percentage should it be increased to make it 100%

Ans: 42.857%


19. There is a square of side 6cm . A circle is inscribed inside the square. Find the ratio of the area of circle to square.
Ans. 11/14
20. There are two candles of equal lengths and of different thickness. The thicker one lasts of six hours. The thinner 2 hours less than the thicker one. Ramesh lights the two candles at the same time. When he went to bed he saw the thicker one is twice the length of the thinner one. How long ago did Ramesh light the two candles .

Ans: 3 hours.
21. If M/N = 6/5,then 3M+2N = ?
22. If p/q = 5/4 , then 2p+q= ?
23. If PQRST is a parallelogram what it the ratio of triangle PQS & parallelogram PQRST .
Ans: 1:2
24. The cost of an item is Rs 12.60. If the profit is 10% over selling price what is the selling price ?
Ans: Rs 13.86/-
25. There are 6 red shoes & 4 green shoes . If two of red shoes are drawn what is the probability of getting red shoes
Ans: 6c2/10c2
26. To 15 lts of water containing 20% alcohol, we add 5 lts of pure water. What is % alcohol.
Ans : 15%
27. A worker is paid Rs.20/- for a full days work. He works 1,1/3,2/3,1/8.3/4 days in a week. What is the total amount paid for that worker ?
Ans : 57.50
28. If the value of x lies between 0 & 1 which of the following is the largest?
(a) x
(b) x2
(c) -x
(d) 1/x

Ans : (d)

DATA SUFFICIENCY SECTION
Directions : For questions in this section mark

(a) If condition (i) alone is sufficient
(b) If condition (ii) alone is sufficient
(c) If both conditions together are sufficient
(d) If condition (i) alone & (ii) alone are sufficient
(e) information not sufficient
1. A man 6 feet tall is standing near a light on the top of a pole What is the length of the shadow cast by the man.
(i) The pole is 18 feet high
(ii) The man is 12 feet from the pole
Ans: (c)
2. Two pipes A and B emptied into a reservoir , pipe A can fill the reservoir in 30 minutes by itself. How long it will take for pipe A and pipe B together to fill up the reservoir.
(i) By itself, pipe B can fill up the reservoir in 20 minutes
(ii) Pipe B has a larger cross-sectional area than pipe A
Ans: (a)
3. K is an integer. Is K is divisible by 12
(i) K is divisible by 4
(ii) K is divisible by 3
Ans: (c)

4. What is the distance from A to B
(i) A is 15 miles from C
(2) C is 25 miles from B
Ans: (e)
5. Was Melissa Brown's novel published?
(i). If Melissa Brown's novel was published she would receive atleast $1000 in royalities during 1978
(ii). Melissa Brown's income for 1978 was over $1000
Ans: (e)
6. Does every bird fly?
(i) Tigers do not fly.
(ii) Ostriches do not fly
Ans: (b)

7. How much does John weigh? Jim weighs 200 pounds.
(i) Toms weight plus Moes weight equal to John's weight.
(ii) John's weight plus Moe's weight equal to Twice Tom's weight.
Ans: (c)


8. Is the figure ABCD is a rectangle if
(i) angle ABC=90(degrees)
(ii) AB=CD

9. Find x+2y
(i). x+y=10
(ii). 2x+4y=20
Ans: (b)


10. Is angle BAC is a right angle
(i) AB=2BC
(2) BC=1.5AC
Ans: (e)
11. Is x greater than y
(i) x=2k
(ii) k=2y
Ans: (e)
12. A piece of string 6 feet long is cut into three smaller pieces. How long is the longest of the three pieces?
(i). Two pieces are the same length.
(ii) One piece is 3 feet 2 inches lone
Ans: (b)
13. How many rolls of wall paper are necessary to cover the walls of a room whose floor and ceiling are rectangles 12 feet wide and 15 feet long
(i) A roll of paper covers 20 sq feet
(ii) There are no windows in the walls
Ans: (e)


14. x and y are integers that are both less than 10. Is x>y?
(i). x is a multiple of 3
(ii). y is a multiple of 2
Ans: (e)
15. Fifty students have signed up for atleast one of the courses GERMAN & ENGLISH, how many of the 50 students are taking GERMANI but not ENGLISH?
(i). 16 students are taking GERMANI & ENGLISH
(ii). The number of students taking ENGLISH but not GERMANI is the same as the number of students taking GERMAN
Ans: (c)
17. How much card board will it take to make a rectangular box with a lid whose base has length 7 inches.
(i). The width of the box 5 inches
(ii). The height of the box will be 4 inches
Ans: (c)
.
18. Did ABC company made profit in 1980?
(i) ABC company made a profit in 1979.
(ii) ABC company made a profit in 1981.
Ans: (e)
19. How much is Janes salary?
(i). Janes salary is 70% of John's salary
(ii). Johns salary is 50% of Mary's salary
Ans: (e)
20. Is x>1
(i) x+y=2
(ii) y<0
Ans: (c)
21. How many of the numbers, x and y are positive? Both x and y are less than 20.
(i) x is less than 5
(ii) x+y =24
Ans: (b)
22. Is the angle ACB is right angle
(1) AC=CB
(2). (AC)2+CB2=AB2
Ans: (b)
23. How far it from town A to town B? Town C is 12 miles east of town A
(i). Town C is south of town B
(ii). It is 9 miles from town B to town C
Ans: (c)
24. A rectangular field is 40 yards long. Find the area of the field.
(i). A fence around the boundary of the field is 140 yards long
(ii). The field is more than 20 yards width
Ans: (a)
25. An industrial plant produces bottles. In 1961 the number of bottles produced by the plant was twice the number of produced in 1960. How many bottles were produced altogether in the year 1960, 61,&62
(i). In 1962 the number of bottles produced was 3 times the number of produced in 1980
(ii). In 1963 the number of bottles produced was one half the total produced in the years 1960,1961,1962.
Ans: (e)
26. Is xy > 1 ? If x & y are both positive
(i) x is less than 1
(ii) y is greater than 1
Ans: (e)
27. Is it a Rhombus
(i) All four sides are equal
(ii) Total internal angle is 360
Ans: (e)
28. How many books are in the book shelf
(i) The book shelf is 12 feet long
(ii). The average weight of each book is 1.2 pound
Ans: (e)
29. What is the area of the circle?
(i) Radius r is given
(ii) Perimeter is 3 times the area
Ans: (a)
ARITHMETIC SECTION

1. If the total distance of a journey is 120 km .If one goes by 60 kmph and comes back at 40kmph what is the average speed during the journey?
Ans: 48kmph
2. A school has 30% students from Maharashtra .Out of these 20% are Bombey students. Find the total percentage of Bombay?
Ans: 6%
3. An equilateral triangle of sides 3 inch each is given. How many equilateral triangles of side 1 inch can be formed from it?
Ans: 9
4. If A/B = 3/5,then 15A = ?
Ans : 9B
5. Each side of a rectangle is increased by 100% .By what percentage does the area increase?
Ans : 300%
6. Perimeter of the back wheel = 9 feet, front wheel = 7 feet on a certain distance, the front wheel gets 10 revolutions more than the back wheel .What is the distance?
Ans : 315 feet.
7. Perimeter of front wheel =30, back wheel = 20. If front wheel revolves 240 times. How many revolutions will the back wheel take?
Ans: 360 times
8. 20% of a 6 litre solution and 60% of 4 litre solution are mixed. What percentage of the mixture of solution
Ans: 36%
9. City A's population is 68000, decreasing at a rate of 80 people per year. City B having population 42000 is increasing at a rate of 120 people per year. In how many years both the cities will have same population?
Ans: 130 years
10. Two cars are 15 kms apart. One is turning at a speed of 50kmph and the other at 40kmph . How much time will it take for the two cars to meet?
Ans: 3/2 hours
11. A person wants to buy 3 paise and 5 paise stamps costing exactly one rupee. If he buys which of the following number of stamps he won't able to buy 3 paise stamps.
Ans: 9
12. There are 12 boys and 15 girls, How many different dancing groups can be formed with 2 boys and 3 girls.

13. Which of the following fractions is less than 1/3
(a) 22/62
(b) 15/46
(c) 2/3
(d) 1
Ans: (b)
14. There are two circles, one circle is inscribed and another circle is circumscribed over a square. What is the ratio of area of inner to outer circle?
Ans: 1 : 2
Directions for questions 15-17: The questions are based on the information given below
Miss Dean wants to rennovate her house. She hires a plumber, a carpenter, a painter, an electrician and an interior decorator. The work to be finished in one working (Monday - Friday ).
Each worker will take the full day to do his job. Miss Dean permits only one person to work each day.
I. The painter can work only after the plumber and the carpenter have finished their jobs
II. The interior decorator must do his job before the electrician.
III. The carpenter cannot work on Monday or Tuesday
15. If the painter work on Thursday, which one of the following alternatives is possible?
(a) The electrician works on Tuesday.
(b). The electrician works on Friday.
(c) The interior decorator works after the painter does.
(d). The painter works on consecutive days.
(e). Miss Dean cannot fit all of the workers int schedule
Ans: (b)
16. If the painter works on Friday which of the following must be false?
(a) . The carpenter may works on Wednesday
(b). The carpenter and the electrician may work on consecutive days
(c). If the carpenter works on Thursday, the electrician has to work on Wednesday
(d). The plumber may work before the electrician does
(e). The electrician may work on Tuesday
Ans: (c)

17. Which argument is possible?

(a). The electrician will works on Tuesday and the interior decorator on Friday
(b). The painter will work on wednesday and plumber on thursday
(c). The carpenter will works on Tuesday and the painter on Friday
(d). THe painter will work on Monday and the carpenter on Thursday
(e). The carpenter will work on Wednesday and the plumber on Thursday
Ans: (e)
SECTION-1:

Find the next in the sequence:

1. BC CE EG GK ?

a)KN
b)KU
c)KM
d)None

2. AA AB BC CE?

a)EG
b)EH
c)EI
d)None

3. AB EF JK QR ?

a)YZ
b)ZA
c)AB
d)None

4.ACD EGL IKT MOB?

a)QST
b)QSZ
c)QSY
d)None

5.AC CG GO OE?

a)EJ
b)EI
c)EL
d)None

6.AE BH CM DU?

a)EH
b)EZ
c) EB
d)None

7. AD DP PL LV

a)VS
b)VK
c)VI
d)None

8. SE QU EN TI?

a)CN
b)BM
c)AI or AZ
d)None


SECTION-II:

Find the values for the following problem:
f(X)= 2X-1 + f(X-1) if X is not equal to zeroandiff(X=0)=0
9. Value of f(5)

a)15
b)24
c)22
d)None

10. Value of f(f(2))

11. Value of f(16)- f(15)

12. Value of f(16)+f(15)-480

13. If f(f(X))=81 then the value of X=?

14. If f(X)=4f(X-1) then the value of X=?

15.If f(X)= f(X-1)+f(X-2) for X>1 then X= ?

16. If f(X)-f(X-1)=f(X-8) for X>5 then X=

SECTION -III:

In the following questions a 'word' is given which may not have any meaning.Find differnet possible words or palindromes for the word as per the question.

For the following find no of distinct words that can be formed.

17. TYGHHTT

a).420
b)1540
c)840
d)None

18. TYGHHTY

19. TYGHHTT
20. TYGHHTT
21. TYGHASD
22. TYGHHTY

Find the number of possible palindromes for following

23. TYGHHTY
24. TYHHHTYH.
SECTION-IV:

25 to 32 are based on the figures. You have to analyse them and find the odd one out.
Five figures will be given out of which one is not correct.
Refer R.S Agarwal's book on Analytical Reasoning & TMHs Quantitative ability book by Edgar Thorpe.
SECTION -V:

For following first find out the anagram and then note the corresponding meaning.

33.TABLET
Hint: anagram means first u arrange the letters in correct order like TABLET===BATTLE . So ans is FIGHT

34.RUGGED
35.GORE.
36.STASSI.
For all above choices are.

a)resentment
b)fight
c)help
d)monster

37. ENFOLD
38. LAMB
39. RECEDE.
40. PLEASE.

For all the above 4 choices are same

a)cuddle
b)sleeping
c)proclamination
d)ointment.

This is only a sample paper. We are not providing you with all the questions - just some questions to give you a general idea of the test pattern.

SECTION I - 8 questions based on series.

1. These questions involve interchange of letters in a word at particular locations and also interchanging letters adjacent to those particular locations.Certain other conditions may also be given
For eg.
Let the word be ABBAABA
If we apply 25 on this, it means we have to interchange the letters at positions 2 and 5, also we have to change the letters adjacent to positions 2 and 5 i.e.from A to B and B to A.
A B B A A B after Step 1 i.e interchange of 2 and 5 becomes AABABB
Now change adjacent elements of 2 and 5...finally answer becomes
Ans: B A A B B A

Questions 1-5 are based on the pattern with changed numbers as described above
Questions 6-8 are of the following type
To get AAABBD from BBBAAA what number should be applied:-
a) 25
b) 34
c) 25 & 34
d) none

SECTION II

1. Given the following functions
(1) f(n a b c ) = ac if n=1
(2) f(n a b c) = f( n-1 a c b) + f( 1 a b c) + f( n-1 b a c ) if n > 1

Then what is the value f( 2 a b c ) = ?

Ans: f( 2 a c b ) = ab + ac + bc.

2. Similar question on functions.

3. [ Based on the function in the first question] For the function f( 4 a b c ) the number of terms is...?
Hint f( 4 a b c ) = f( 3 a c b ) + f( 1 a b c ) + f( 3 b a c ) etc.


4. What is the value of the function f( 5 a b c ) = ?

SECTION III

Permutations and Combinations.
8 Questions.

1. r = number of flags;n = number of poles;
Any number of flags can be accommodated on any single pole.

1)r=5,n=5 The no. of ways the flags can be arranged ?

Questions 2-5 are based on the above pattern

6. r = 5 n = 3 . If first pole has 2 flags, third pole has 1 flag
How many ways can the remaining be arranged?

Questions 7.& 8. are similar to Question 6.

SECTION IV
Question consisting of figures - Pattern-matching type.
Refer R.S Agarwal's book on Analytical Reasoning & TMHs Quantitative ability book by Edgar Thorpe.



SECTION V
In this section first part of compound word is given. Select meaning of the second part from the choice given:
1. Swan
2. Swans
3. Fool
4. Fools
5. Stare
6. Lady
For all above 4 choices are given.....

Eg. Swan ---> Swansong (compound word)
a) category b) music c) television d) none
Ans: Swansong is compound word. But song is not given as an option. so (b) music is the answer.
Analogies

1. slur : speech : : smudge :?
Ans. writing
2. epaulet : shoulder : : ring :?
Ans.finger
3. vernacular : place : : fingerprint : ?
Ans.identical

Opposites

Q. corpulent
Ans: emaciated
Q. officious
Ans: pragmate
Q. dextrous
Ans: clumsy

The following sentences are broken into 4 sections- A, B, C, D
Choose the part which has a mistake
Mark (E) if you find no mistake.
Q.A)psychologists point out that B)there are human processes C)which does not involve D) the use of words
Ans. (C) which does not involve (do)

Q.A)jack ordered for B)two plates of chicken C)and a glass D)of water
Ans. (A)jack ordered for

The following is a group of questions is based on a passage or a set of conditions for each question.
Select the best answer choice given.
(i). If it is fobidden by law if the object of agreement is the doing of an act,
that is forbidden by law the agreement is void.
(ii). If it is of the nature that,it would defeat the provision of any law
is the agreement is void.if the object of agreement is such that thing got
directly forbidden by law it would defeat the provision of statuary law.
(iii). If the object of agreement is fraddulent it is void.
(iv). An object of agreement is void if it involves or implies to the personnal property of another.
(v). An object of agreement is void where the constant regards as ignored.
(vi). An object of agreement is void where the constant regards is as opposed to public policy.
Q. An algorithm follws a six step process za,zb,zc,zd,ze,zf, it is governed by the following
(i) zd should follow ze
(ii) the first may be za,zd or zf
(iii) zb and zc have to be performed after zd
(iv) zc must be immediately after zb

Q. If za is the first set zd must be
a) 3rd
b) 5th
c) 2nd
d) 4th
Q. If zb must follow za then za can be
a) third or fourth
b) first or second
c) can not be third
d) fouth or fifth
e) none

Q. If ze is third term the number of different operations possible are
The following questions are based on the given statements
Ravi plants six seperate saplings -- x,y,z,w,u,v in rows no 1 to 6 ,according to the follwing conditions
He must plant x before y and u
He must plant y and w
The third has to be z
Q. Which of the following is acceptable
a) xuywzv
b) xvzyuw
c) zuyxwv
d) zvxuwy
e) wyzuvx
Q. Which of the following is true
a) z before v
b) z before x
c) w before u
d) y before u
e) x before w
Q. If he plants v first, then which can be planted second
a) x
b) y
c) z
d) w
e) u

Q. Which of the following describes a correct combination of sapling and row?
a) x,3
b) y,6
c) z,1
d) w,2
e) u,6

Q. If he plants b 6th which would be planted first and second
a) x and w
b) x and y
c)y and x
d)w and z
e) w and u

Q. If he plants w before u and after v he should plant w at
a) first
b) second
c) fourth
d) fifth
e) sixth

Q. At a certain moment a watch shows 2 min lag although it is running fast.
If it showed a 3 min lag at that moment, but also gains by 1/2 min more a day than its current speed
it would show the true time one day sooner than it usually does.
How many mins does the watch gain per day.
a).2
b).5
c).6
d).4
e).75
Q. In 400m race A gives B a start of 7 sec and beats him by 24 sec.
In another race A beats B by 10 sec.the speeds are in the ratio
a)8:7
b)7:6
c)10:8
d)6:8
e)12:10

Q. 3x+4y=10
x3 + y3=6
What is the minimum value of 3x+11y=?

Q. There are 600 tennis players
4% wear wrist band on one wrist
Of the remaining, 25% wear wrist bands on both hands
How many players don't wear a wrist band?
Ans. 432

Q. Three types of tea the a,b,c costs Rs. 95/kg,100/kg and70/kg respectively.
How many kgs of each should be blended to produce 100 kg of mixture worth Rs.90/kg,
given that the quntities of band c are equal
a)70,15,15
b)50,25,25
c)60,20,20
d)40,30,30
Ans. (b)

Q. Two distinct no's are taken from 1,2,3,4......28
Find the probability that their sum is less than 13

(a)0.0015
(b)0.000015
(c)0.15
(d)15
Ans.B

2. (x-a)(x-b)(x-c)....(x-z) = ?
(a) 1
(b) -1
(c) 0
(d) Can't be determined
Ans. C

3. If a = 1, b = 2, c = 3.......z = 26 what is the value of p+q+r ?
(a)33
(b)51
(c)52
(d)48
Ans. B

4. A is 8 miles east of B.
C is 10 miles north of B.
D is 13 miles east of C and E is 2 miles north of D.
Find shortest distance between A and E.
(a) 5 miles
(b) 6miles
(c) 13 miles
(d) 18 miles
Ans. C

5. If z = 1, y = 2.......a = 26. Find the value of z + y + x + .......+a.
(a) 351
(b) 221
(c) 400
(d) 200
Ans. A

6. There are 30 socks in a bag.
Out of these 60 % are green and the rest are blue.
What is the maximum number of times that socks have to be taken out so that atleast 1 blue pair is found.
(a) 21
(b) 2
(c)18
(d) 20
Ans. D

7. How many two digit numbers have their square ending with 8.
(a) 13
(b) 12
(c) 0
(d) 11
Ans. C

8. How many numbers are there between 100 and 300 with 2 in the end and 2 in the beginning.
(a) 10
(b) 9
(c) 11
(d) none of these
Ans. A

9. 0.000006 * 0.0000007 = ?
(a) 0.0000000042
(b) 0.000000000042
(c) 0.0000000000042
(d) 0.00000000000042
Ans. B

10. You have Rs 1000 with 8% p.a compounded every 6 months.
What is the total interest you get after 1 year.
(a) Rs.116.40
(b) Rs.345.60
(c) Rs.224.50
(d) Rs.160
Ans. A

11. If x + y =12,
x - y = 2
Find x + 2y.
(a) 12
(b) 17
(c) 14
(d) none of these
Ans. B

12. With one gallon of petrol a person moves at a speed of 50 mph and covers 16 miles.
3/4th of the distance is covered while moving at 60 mph.
How many gallons does he need to cover 120 miles in 60 mph.

13. A tap drains at x speed while tap B is closed.
When both taps are open they drain at y speed.
What is the speed of draining when only tap B is open
(a) x - y
(b) y-x
(c) x
(d) can't be determined
Ans. B

14. What is twenty percent of 25 % of 20.
(a) 2
(b)1
(c) 5
(d) 4
Ans. B

15. A rectangle has the dimensions 6ft * 4ft.
How many squares of 0.5 inches will it need to completely fill it.
(a) 32000
(b) 12824
(c) 13824
(d) 18324
Ans. C

Directions for questions 16-21: In each question,a series of letters satisfying a certain pattern are given. Identify the pattern and then find the letter/letters that will come in place of the blank/blanks.
16. a, c, e, g, _
(a) h
(b) i
(c) d
(d) j
Ans. B

17. a, e, i, m, q, u, _, _
(a) y, c
(b) b, f
(c) g, i
(d) none
Ans. A

18. ay , bz , cw , dx ,__
(a) gu
(b) ev
(c) fv
(d) eu
Ans. D

19. 1, 2, 3, 5, 7, 11, __
(a) 15
(b) 9
(c) 13
(d) 12
Ans. 13 , series of prime numbers

20. kp , lo , mn , __
(a) nm
(b) np
(c) op
(d) pq
Ans. A

21. abc , zyx , def , wvu , ___
(a) ghi
(b) tsr
(c) ihg
(d) str
Ans. A

22. How is my mother's sister's brother's wife's child related to me?
(a) brother
(b) uncle
(c) cousin
(d) nephew
Ans. A

23.What will my mother's husband's father-in-law's son's daughter to me?
(a) niece
(b) aunt
(c) sister
(d) none of these
***********************
SECTION 1- APTITUDE SECTION
Directions for questions 1-3: Complete the sequence given below
1. 5, 5, 13, 13, 21, 21, __

Ans: 29
2. 0, 7, 26, 63, 124, __

Ans: 215 ( hint: n3-1 )
3. 1, 3, 5, 7, __
Ans: 9

4. If a person walks at 4/5th of his usual speed he reaches 40min late. If he walks at his usual speed for how much time does he travel ?
5. Two trains A&B start at opposite points 120km apart at 60kmph. A fly starting along with train A at 120kmph reaches B then returns back to touch A and continues the two and fro movement. By the time two trains meet how much distance would the fly have travelled?
6. In a class 80% have passed in english, 70% passed Hindi, 10% didn't pass either. If 144 students passed both the subjects. What is the total strength of the class?

7. Find the least number which when divided by 7 gives the reminder 6, when divided by 6 gives reminder 5, when divided by 5 gives reminder 4 and so on ?

8. If a man stands in front of sun what is the first letter of the direction which is left to him ?

9. (a)A square is to circle as cube is to
(b)Success is to failure as joy is to

10. (a)Give the synonyms of the following words
(i) Joy
(ii) Inert
(iii) Jolly

(b) Give the opposites of the following words
(i) genuine
(ii) command
(iii) essential
11.Find the odd man out in the following sets
(i) Tiger, Elephant, King Cobra, Dolphin
(ii) Oasis, Lake, Pool, Valcano
(iii) Bengali, Karnataka, Mumbai, Kashmir
(iv) Lapidary, Lancet, Scapel, Surgeon
(v) Requiem, Dirge, Elegy, Paean
12. I bought a cycle 2days before my birthday and I broke it 3 days after my birthday. The day I broke it is Mar2,1956?

Directions: The following questions are to be answered on the basis of the above given statement
i) When is my birthday?
Hint: Keep in mind that 1956 was a leap year.
(ii) What is my age on Mar 4th, 1980?
(iii) My nephew is born exactly 20years after me. If I turned 20 in 1960, what is the nephews age on Feb 28th 1988 ?

13. Monday Aug25, 96 :
Hostess: "Mr A, you forgot your umbrella during the party on last friday. I expected you to collect it on your visit on wednesday as I plan to leave on this Friday."
Directions: The following questions are to be answered on the basis of the above given statement
(i) when A missed umbrella?
(ii)When A is supposed to collect it?
(iii)When K leaves?

14.What is my father's sons son to my son?

Ans. Cousin brother
15. On cutting a solid parabola what would be generated

Ans: Cone

16. What is Eulers formula?

Ans: F+V-E =2; where
F => faces; V => vertices; E => number of edges
17. What is Newton Raphson method used for?

Ans: To find the root of f(x) = 0;

18. How many tangents can be drawn on three circles if they don't lie within each other ?

19. xy - x + 2y = 6 equation is shifted to form equation xy=c what is c ?

20.When x is real what is the least value of (x2-6x+5)/(x2+2x+1)

21.When an object like cube or sphere is seen along x, y, z axis we get the same.Apart from these suggest another object which has similar characteristics as that mentioned above?
Ans: Triangular prism
22. When an object is seen from the front side we can see two concentric squares and top view also without any hidden lines. Draw the side view.
23. In common parlance, A=> B means what
Ans: if A is true B has to be true
23. If A is not invertible and BA = I is not possible, what is implied by this?
Ans: Determinant is Zero.
24. What is a free body diagram used for

25. A die is thrown twice what is the probability that you get same number


***************
1. In 1978, a kg of paper was sold at Rs25/-.
If the paper rate increases at 1.5% more than the inflation rate which is 6.5% a year,
then what wil be the cost of a kg of paper after 2 years?
(a) 29.12
(b) 29.72
(c) 30.12
(d) 32.65
(e) none of these

2. In A,B,C are having some marbles with each of them.
A has given B and C the same number of marbles each of them already have.
Then, B gave C and A the same number of marbles they already have.
Then C gave A and B the same number of marbles they already have.
At the end A,B,and C have equal number of marbles.
(i) If x,y,z are the marbles initially with A,B,C respectively.
Then the number of marbles B have at the end
(a) 2(x-y-z)
(b) 4(x-y-z)
(c) 2(3y-x-z)
(d) x + y-z
Ans. (c)
(ii) If the total number of marbles are 72, then the number of marbles with A at the starting
(a) 20
(b) 30
(c) 32
(d) 39
Ans. (d)

3. If a car starts from A towards B with some velocity.
Due to some problem in the engine after travelling 30km, the car goes with 4/5 th of its actual velocity
The car reaches B 45 min later to the actual time.
If the car engine fails ofter travelling 45km, the car reaches the destination B 36min late to the actual time
What is the initial velocity of car and what is the distance between A and B in km
Ans. 20 & 130.

4. A person has Rs 100/- in his pocket, he can as 25 pencils or 15 books.
He kept 15% of the money for travelling expenses and purchased 5 pencils.
So how many books he can purchase with the remaining money.
5. Ten questions on analogies.
eg: chief : tribe :: governer : state
epaulette : shoulder :: tiara : head
guttural : throat :: gastric : stomach
inept : clever :: languid : active
knife : butcher ::
hammer : carpenter ::

. In a computer institute 9 languages can be taught.
The module is of 6 months duration and of the six languages only one can be taught each month .
In addition to that BASIC is always taught and should be in first month itself
• WORD PERFECT is to be taught in the preceeding week of WORD STAR.
• FORTRAN can not be taught until COBAL is taught prior to that
• BINO, FIFO can never be taught in single module
languages are BASIC, WORD STAR, WORD PERFECT, FORTRAN, COBAL, BINO, FIFO, LOTUS, C
i) If word star is in 3rd month , what could be in 6th month.
ii) If COBAL is in the 2nd month and BINO in 6th month. FORTRAN will be taught in which month.

8. In a class, except 18 all are above 50 years.
15 ae below 50 years of age. How many people are there
(a) 30
(b) 33
(c) 36
(d) none of these.
Ans. (d)

9. A square plate of some size is cut at four corners. Equal squares of the same size are cut and is formed as open box.
If this open box carries 128 ml of oil. What is the size of the side of the plate?
(a) 17
(b) 14
(c) 13
(d) None of these

10. In a square, all the mid points are joined. The inner square is shaded.
If the area of the square is A, what is the area of the shaded area?
11. Two questions on basic angles i.e given a circle, a few chords or diameter is drawn etc.
**************************
2. Three friends divided some bullets equally.
After all of them shot 4 bullets the total number of bullets remaining is equal to the bullets each had after division.
Find the original number divided.
Ans: 18 (2 marks)
Initially . x x x
Now x-4 x-4 x-4
Equation is 3x-12 = x

3. A ship went on a voyage.
After it had travelled 180 miles a plane statrted with 10 times the speed of the ship.
Find the distance when they meet from starting point.
Ans: 200miles. (2 marks)
Distance travelled by plane = 1/10 distance travelled by ship + 180

5. There are 3 societies A, B, C.
A lent cars to B and C as many as they had already.
After some time B gave as many tractors to A and C
as many as they have. After sometime c did the same thing. At the end of this transaction each one of them had 24.
Find the cars each orginally had.
Ans: A had 39 cars, B had 21 cars & C had 12 cars (4 marks)

6. There N stations on a railroad.
After adding X stations on the rail route 46 additional tickets have to be printed.
Find N and X.
Ans. x=2 and N=11
Let initially, N(N-1) = t
After adding, (N+X)(N+X-1) = t+46
By trail and error method (4 marks)

7. Given that April 1 is tuesday.
A, B, C are 3 persons told that their farewell party was on
• A - May 8, thursday
• B - May 10,tuesday
• C - June 5, friday
Out of A, B, C only one made a completetly true statement concerning date,day and month
The other told two one told the day right and the other the date right..
What is correct date, month, day.
Ans: B - (May 10) SUNDAY
C - June 6 (Friday). (5 marks)

8. The Bulls, Pacers, Lakers and Jazz ran for a contest.
Anup, Sujit, John made the following statements regarding results.
• Anup said either Bulls or Jazz will definitely win
• Sujit said he is confident that Bulls will not win
• John said he is confident that neither Jazz nor Lakers will win
When the result cameit was found that only one of the above three had made a correct statement.
Who has made the correct statement and who has won the contest.
Ans: Sujith; Lakers (5marks )

9. Five people A ,B ,C ,D ,E are related to each other.
Four of them make one true statement each as follows.
(i) B is my father's brother.
(ii) E is my mother-in-law.
(iii)C is my son-in-law's brother
(iv)A is my brother's wife.
Ans: (i) D
(ii) B
(iii) E
(iv) C (10 marks)

10. Some statements are given below:
• L says all of my other four friends have money
• M says that P said that exactly one among them has money
• N says that L said that precisely two among them have money
• O says that M said that three of the others have money
• P, L and N said that they have money
All the above statement are false..
Who has money & who doesn't have any money?
(5 marks)

• The locker belonging to JONES was to the right of BLACK'S locker and directly above MILLAR'S.
• BOOTH'S locker was directly above MILLAR'S.
• SMITH'S locker was also above GRAY's (though not directly).
• GREEN'S locker was directly below SMITH'S.
• WILSON'S locker was between that of DAVIS and BOOTH.
• MILLAR'S locker was on the bottom row directly to the right of HERD'S.
• WHITE'S locker was on the bottom right hand corner in the same column as BOOTH'S.
Which box belonged to Mr.Mathurs?
Ans: Box number 9 belongs to Mr.Mathurs.

2. Fifty minutes ago if it was four times as many minutes past three o'clock,how many minutes is it to six o'clock?
Ans: Twenty six minutes.

3. If a clock takes 7seconds to strike 7, how long will the same clock take to strike 10?
Ans: The clock strikes for the first time at the start and takes 7 seconds for 6 intervals-thus for one interval time
taken=7/6.
Therefore, for 10 seconds there are 9 intervals and time taken is 9*7/6=10 and 1/2 seconds.

4. Three criminals were arrested for shop lifting.
However, when interrogated only one told the truth in both his statements, while the other two each told one true
statement and one lie.
The statements were:
• ALBERT :(a)Chander passed the merchandise. (b)Bruce created the diversion.
• BRUCE :(a)Albert passed the merchandise. (b)I created the diversion.
• CLIVE :(a)I took the goods out of the shop. (b)Bruce passed them over.
Ans: Albert passed the goods.Bruce created the diversion..Clive took the goods out of the shop.

5. Everyday in his business a merchant had to weigh amounts from 1 kg to 121 kgs, to the nearest kg.
What are the minimum number of weight required and how heavy should they be?
Ans: .The minimum number is 5 and they should weigh 1,3,9,27 and 81 kgs.

6. A hotel has 10 storeys.Which floor is above the floor below the floor, below the floor above the floor, below the
floor above the fifth.
Ans: The sixth floor.

7. Seven members sat around a table for three days for a conference.
The member's names were Abhishek, Amol, Ankur, Anurag,Bhuwan ,Vasu and Vikram.
The meetings were chaired by Vikram.
On the first evening members sat around the table alphabetically.
On the following two nights, Vikram arranged the seatings so that he could have Abhishek as near to him as
possible and abesent minded Vasu as far away as he could.
On no evening did any person have sitting next to him a person who had previously been his neighbour.
How did Vikram manage to seat everybody to the best advantage on the second and third evenings?
Ans:
Second evening:Vikram,Ankur,Abhishek,Amol,Vasu,Anurag and Bhuwan.
Third evening :Vikram,Anurag,Abhishek,Vasu,Bhuwan,Ankur,Amol.

8. Two trains start from stations A and B spaced 50 kms apart at the same time and speed.
As the trains start, a bird flies from one train towards the other and on reaching the second train, it flies back to the
first train.This is repeated till the trains collide.
If the speed of the trains is 25 km/h and that of the bird is 100km/h.
How much did the bird travel till the collision.
Ans: 100 kms.

9. Four prisoners escape from a prison.
The prisoners, Mr East, Mr West, Mr South, Mr North head towards different directions after escaping.
The following information of their escape was supplied:
• The escape routes were The North Road, South Road, East Road and West Road.
• None of the prisoners took the road which was their namesake.
• Mr.East did not take the South Road
• Mr.West did not the South Road.
• The West Road was not taken by Mr.East
What road did each of the prisoners take to make their escape?
Ans: Mr.East took the North Road
Mr.West took the East Road
Mr.North took the South Road
Mr.South took the West Road.

10. Complete the series:
5, 20, 24, 6, 2, 8, ?
Ans: 12 (as 5*4=20, 20+4=24, 24/4=6, 6-4=2, 2*4=8, 8+4=12).
3
1) A man collects cigarette stubs and makes one full cigarette with every 8 stubs.
If he gets 64 stubs how many full cigarettes can he smoke.

Ans: 8+1=9


2) A soldier looses his way in a thick jungle. At random he walks from his camp but mathematically in an interesting fashion. First he walks one mile East then half mile to North. Then 1/4 mile to West, then 1/8 mile to South and so on making a loop.
Finally how far he is from his camp and in which direction.

Ans: Distance travelled in north and south directions
1/2 - 1/8 + 1/32 - 1/128 + 1/512 - and so on
= 1/2/((1-(-1/4))
Similarly in east and west directions
1- 1/4 + 1/16 - 1/64 + 1/256 - and so on
= 1/(( 1- ( - 1/4))
Add both the answers

3) How can 1000000000 be written as a product of two factors neither of them containing zeros

Ans: 2 power 9 x 5 power 9

4) Conversation between two mathematcians:
First : I have three childern. The product of their ages is 36.
If you sum their ages, it is exactly same as my neighbour's door number on my left.
The second mathematician verfies the door number and says that it is not sufficient.
Then the first says " Ok one more clue is that my youngest is really the youngest". Immmediately the second mathematician answers .
Can you answer the question asked by the first mathematician?
What are the childeren ages?

Ans 1,6 and 6

5) Light glows for every 13 seconds . How many times did it glow between 1:57:58 and 3:20:47 am.

Ans : 383 + 1 = 384

6) 500 men are arranged in an array of 10 rows and 50 columns according to their heights.
Tallest among each row of all are asked to fall out.
And the shortest among them is A.
Similarly after resuming that to their original podsitions that the shortest among each column are asked to fall out.
And the tallest among them is B .
Now who is taller among A and B ?

Ans A


7) A person with some money spends1/3 for cloths, 1/5 of the remaining for food and 1/4 of the remaining for travel.
He is left with Rs 100/- .
How much did he have with him in the begining ?

Ans: Rs 250/-

8) There are six boxes containing 5 , 7 , 14 , 16 , 18 , 29 balls of either red or blue in colour.
Some boxes contain only red balls and others contain only blue.
One sales man sold one box out of them and then he says
" I have the same number of red balls left out as that of blue ".
Which box is the one he solds out ?

Ans: Total no of balls = 89 and (89-29 /2) = 60/2 = 30
and also 14 + 16 = 5 + 7 + 18 = 30

9) A chain is broken into three pieces of equal lenths containing 3 links each.
It is taken to a backsmith to join into a single continuous one .
How many links are to to be opened to make it ?

Ans : 2.

10) Grass in lawn grows equally thick and in a uniform rate.
It takes 24 days for 70 cows and 60 days for 30 cows to eat the whole of the grass.
How many cows are needed to eat the grass in 96 days.?

Ans : 20
g - grass at the beginning
r - rate at which grass grows, per day
y - rate at which one cow eats grass, per day
n - no of cows to eat the grass in 96 days
g + 24*r = 70 * 24 * y
g + 60*r = 30 * 60 * y
g + 96*r = n * 96 * y
Solving, n = 20.

Section B
1. Replace each letter by a digit.
Each letter must be represented by the same digit and no beginning letter of a word can be 0.

O N E
O N E
O N E
O N E
-------
T E N
Ans: 0 =1, N = 8 ,E = 2, T = 7

2. Ann, Boobie, Cathy and Dave are at their monthly business meeting.
Their occupations are author, biologist, chemist and doctor, but not necessarily in that order.
Dave just told the biologist that Cathy was on her way with doughnuts.
Ann is sitting across from the doctor and next to the chemist.
The doctor was thinking that Boobie was a goofy name for parent's to choose,but didn't say anything.
What is each person's occupation?
Ans: Since Dave spoke to the biologist and Ann sat next to the chemist and across the doctor, Cathy must be the author
and Ann the biologist.
The doctor didn't speak, but David did, so Bobbie is the doctor and Dave the chemist.

3. Sometime after 10:00 PM a murder took place.
A witness claimed that the clock must have stopped at the time of the shooting.
It was later found that the postion of both the hands were the same but their positions had interchanged.
Tell the time of the shooting (both actual and claimed).
Ans: Time of shooting = 11:54 PM
Claimed Time = 10:59 PM

4. Next number in the series is
1 , 2 , 4 , 13 , 31 , 112 , ?
Ans: 224.
No number has digits more than 4. All of them are 1 , 2, 4, 8 , 16 , 32 , 64 converted to numbers in base 5

5. Shahrukh speaks truth only in the morning and lies in the afternoon, whereas Salman speaks truth only in the afternoon. A says that B is Shahrukh. Is it morning or afternoon and who is A - Shahrukh or Salman.
Ans: Afternoon ; A is Salman.

6. Two trains starting at same time, one from Bangalore to Mysore and other in opposite direction arrive at their
destination 1 hr and 4 hours respectively after passing each other. How nuch faster is one train from other?
Ans: Twice

7. There are 6 volumes of books on a rack kept in order ( ie vol.1, vol. 2 and so on ).
Give the position after the following changes were noticed.
• All books have been changed
• Vol.5 was directly to the right of Vol.2
• Vol.4 has Vol.6 to its left and both weren't at Vol.3's place
• Vol.1 has Vol.3 on right and Vol.5 on left
• An even numbered volume is at Vol.5's place
Find the order in which the books are kept now.
Ans: 2 , 5 , 1 , 3 , 6 , 4

8. I bought a car with a peculiar 5 digit numbered licence plate which on reversing could still be read.
On reversing value is increased by 78633.Whats the original number if all digits were different?
Ans: Only 0 1 6 8 and 9 can be readupside down.So on rearranging these digits we get the answer as 10968

9. The shape in the sketch below is that of a square attached to half of a similar square.Divide it into four equal pieces
INCLUDEPICTURE "http://freshersworld.com/questions/qimages/ques2_9.gif" \* MERGEFORMATINET
Ans: Hint : the figure can be divided into 12 equal triangles

10. Supposing a clock takes 7 seconds to strike 7. How mlong will it take to strike 10?
Ans: 10 1/2 seconds.
4
1. Father's age is three years more than three times the son's age.
After three years, father's age will be ten years more than twice the son's age.
What is the father's present age.
Ans: 33 years. (2 marks)

2. Find the values of each of the alphabets.
N O O N
S O O N
+ M O O N
----------
J U N E
Ans: 9326 (2 marks)

3. There are 20 poles with a constant distance between each pole
A car takes 24 second to reach the 12th pole.
How much will it take to reach the last pole.
Ans: 41.45 seconds (2 marks)
Let the distance between two poles = x
Hence 11x:24::19x:?

4. A car is travelling at a uniform speed.
The driver sees a milestone showing a 2-digit number.
After travelling for an hour the driver sees another milestone with the same digits in reverse order.
After another hour the driver sees another milestone containing the same two digits.
What is the average speed of the driver.

Ans: 45 kmph (4 marks)

5. The minute and the hour hand of a watch meet every 65 minutes.
How much does the watch lose or gain time and by how much?
Ans: Gains; 5/11 minutes (4 marks)

6. Ram, Shyam and Gumnaam are friends.
Ram is a widower and lives alone and his sister takes care of him.
Shyam is a bachelor and his neice cooks his food and looks after his house.
Gumnaam is married to Gita and lives in large house in the same town.
Gita gives the idea that all of them could stay together in the house and share monthly expenses equally.
During their first month of living together, each person contributed Rs.25.
At the end of the month, it was found that Rs 92 was the expense so the remaining amount was distributed equally
among everyone.
The distribution was such that everyone recieved a whole number of Rupees.
How much did each person recieve?
Ans. Rs 2 (4 marks)
(Hint: Ram's sister, Shyam's neice and Gumnaam's wife are the same person)

7. Four persons A, B, C and D are playing cards.
Each person has one card, laid down on the table below him, which has two different colours on either side.
The colours visible on the table are Red, Green, Red and Blue.
They see the color on the reverse side and give the following comment.
A: Yellow or Green
B: Neither Blue nor Green
C: Blue or Yellow
D: Blue or Yellow
Given that out of the 4 people 2 always lie find out the colours on the cards each person.
Section B
1. From a vessel, 1/3rd of the liquid evaporates on the first day.
On the second day 3/4th of the remaining liquid evaporates.
What fraction of the volume is present at the end of the second day.
Ans: 50%
2. An orange glass has orange juice and white glass has apple juice both of equal volumes.
50ml of the orange juice is taken and poured into the apple juice.
50ml from the white glass is poured into the orange glass.
Of the two quantities, the amount of apple juice in the orange glass and the amount of orange juice in the white glass, which one is greater and by how much?

Ans: The two quantities are equal
3. There is a 4 inch cube painted on all sides.
This is cut down into of 1 inch cubes.
What is the no of cubes which have no pointed sides.

Ans: 8
4. Sam and Mala have a conversation.
• Sam says I am certainly not over 40
• Mala says I am 38 and you are atleast 5 years older than me
• Now Sam says you are atleast 39
All the statements by the two are false.
How old are they really?

Ans: Mala = 38 yrs
Sam = 41 yrs.

5. Ram Singh goes to his office in the city, every day from his suburban house.
His driver Gangaram drops him at the railway station in the morning and picks him up in the evening.
Every evening Ram Singh reaches the station at 5 O' Clock.
Gangaram also reaches at the same time.
One day Ram Singh started early from his office and came to the station at 4 O' Clock.
Not wanting to wait for the car he starts walking home. Mangaram starts at normal time, picks him up on the way
and takes him back house, half an hour early.
How much time did Ram Singh walk?
6. In a railway station, there are two trains going.
One in the harbour line and one in the main line, each having a frequency of 10 minutes.
The main line service starts at 5 o'clock and the harbour line starts at 5.02A.M.
A man goes to the station every day to catch the first train that comes.
What is the probability of the man catching the first train?
Ans: 0.8
7. A family X went for a vacation.
Unfortunately it rained for 13 days when they were there.
But whenever it rained in the mornings, they had clear afternoons and vice versa.
In all they enjoyed 11 mornings and 12 afternoons.
How many days did they stay there totally?
Ans: 18
8. A survey was taken among 100 people to find their preference of watching T.V. programmes.
There are 3 channels. Given the no of people who watch
• at least channel 1
• at least channel 2
• at least channel 3
• no channels at all
• atleast channels 1and 3
• atleast channels 1 and 2
• atleast channels 2 and 3
Find the no of people who watched all three.
9. Albert and Fernandes have two leg swimming race.
Both start from opposite ends of the pool.
On the first leg, the boys pass each other at 18 m from the deep end of the pool.
During the second leg they pass at 10 m from the shallow end of the pool.
Both go at constant speed but one of them is faster.
Each boy rests for 4 seconds at the end of the first leg.
What is the length of the pool?
10. Each alphabet stands for one digit in the following multiplication.
T H I S
x I S
---------
X F X X
X X U X
------------
X X N X X
------------
What is the maximum value T can take?
5
.An escalator is descending at constant speed.
A walks down and takes 50 steps to reach the bottom.
B runs down and takes 90 steps in the same time as A takes 10 steps.
How many steps are visible when the escalator is not operating?

Ans: 150 steps
2. Every day a cyclist meets a train at a particular crossing.
The road is straight before the crossing and both are travelling in the same direction.
The cyclist travels with a speed of 10 Kmph.
One day the cyclist comes late by 25 min. and meets the train 5km before the crossing.
What is the speed of the train?

Ans: 60 kmph
3. There are five persons with surnames Mukherjee, Misra, Iyer, Patil and Sharma.
There are 4 persons having first or middle name of Kumar, 3 persons with Mohan, 2 persons with Dev and 1 Anil.
Either Mukherjee and Patil have a first or middle name of Dev or Misra and Iyer have their first or middle name of Dev.
Of Mukherkjee and Misra, either both of them have a first or middle name of Mohan or neither have a first or
middle name of Mohan.
Either Iyer or Sharma has a first or middle name of Kumar but not both.
Who has the first or middle name of Anil?
Kumar Misra Dev
Ans: Kumar Misra Dev
Mohan Iyer Dev
Kumar Patil Mohan
Mohan Sharma Kumar
4. Boys are allowed to watch football at C.V.Raman auditorium subjected to conditions.
• The boy over age 16 can wear overcoat
• No boy over age 15 can wear cap
• To watch the football either he has to wear overcoat or cap or both
• A boy with an umberella or above 16 or both cannot wear sweater.
• Boys must either not watch football or wear sweater.
What is the appearence of the boy who is watching football.

5. A bird keeper has got P pigeons, M mynas and S sparrows.
The keeper goes for lunch leaving his assistant to watch the birds.
• Suppose p=10, m=5, s=8 when the bird keeper comes back, the assistant informs the x birds have escaped. The bird keeper exclaims: "Oh no! All my sparrows are gone."
How many birds flew away?
• When the bird keeper comes back, the assistant told him that x birds have escaped. The keeper realised that atleast 2 sparrows have escaped.
What is minimum no of birds that can escape?

6. Answer the following questions based on the conditions from the choices A, B, C, D, E as described below:
(A) if a definete conclusion can be drawn from condition 1
(B) if a definete conclusion can be drawn from condition 2
(C) if a definete conclusion can be drawn from condition 1 and 2
(D) if a definete conclusion can be drawn from condition 1 or 2
(E) no conclusion can be drawn using both conditions
• person 1 says N<5
• person says N>5
• person 3 says 3N>20
• person 4 says 3N>10
• person 5 says N<8
What is the value of N?

a) 1. No of persons who speak false being less than no of persons who tells the truth.
2. Person 2 is telling the truth.

b) 1. no of persong telling the truth is greater than no of persons telling lies
2. person 5 is telling the truth.

7. There are N coins on a table and there are two players A & B.
You can take 1 or 2 coins at a time.
The person who takes the last coin is the loser.
A always starts first.
• If N=7
(a) A can always win by taking two coins in his first chanse
(b) B can win only if A takes two coins in his first chance.
(c) B can always win by proper play
(d) none of the above
• 2. A can win by proper play if N is equal to
(a) 13 (b) 37 (c) 22 (d) 34 (e) 48
Ans: (e.)
• 3. B can win by proper play if N is equal to
(a) 25 (b)26 (c) 32 (d) 41 (e) none
• 4. if N<4, can A win by proper play always?
(a) Yes (b) No

8. Two twins have vertain peculiar charcteristics.
One of them always lies on Monday, Wednesday, Friday.
The other always lies on Tuesdays, Thursdays and Saturdays.
On the other days they tell the truth. You are given a conversation.
• Person A-- today is Sunday my name is Anil
• Person B -- today is Tuesday, my name is Bill
What day is today?
Ans: Today is Tuesday.

9. There is a safe with a 5 digit number as the key.
The 4th digit is 4 greater than second digit, while 3rd digit is 3 less than 2nd digit.
The 1st digit is thrice the last digit.
There are 3 pairs whose sum is 11.
Find the number.
Ans: 65292

10. A hotel has two wings,the east wing and the west wing.
Some east wing rooms but not all have an ocean view.
All west wing rooms have a harbour view.
The charge for all rooms is identical, except as follows :
• Extra charge for all harbour view rooms on or above the 3rd floor
• Extra charge for all ocean view rooms except those without balcony
• Extra charge for some harbour rooms on the first two floor & some east wing rooms without ocean view but
having kitchen facilities.
Which of the following cannot be determined on the basis of the nformation given:
I. Whether there are any rooms without a balcony for which an extra charge is imposed.
II. Whether any room without a kitchen or a view involves an extra charge.
III. Whether two extra charges are impsed for any room.
(A) I only
(B) II only
(C) III only
(D) II and III only
(E) I, II and III
(This question is from 1999 Barrons GRE Guide model Test 3 - section 6, question 22)
Ans: (A)
6
. Three friends divided some bullets equally. After all of them shot 4 bullets the total number of bullets remaining is equal to the bullets each had after division. Find the original number divided.
Ans: 18 ( Initially . x x x
Now x-4 x-4 x-4 ) Equation is 3x-12 = x
2. A ship went on a voyage. After it had traveled 180 miles a plane started with 10 times the speed of the ship.
Find the distance when they meet from starting point.
Ans: 200miles. ( Distance traveled by plane = 1/10 distance traveled by ship + 180 )
3. Replace each letter by a digit. Each letter must be represented by the same digit and no beginning letter of a word can be 0.
O N E
O N E
O N E
O N E
T E N
Ans: O =1, N = 8 ,E = 2, T = 7
4. In a railway station, there are two trains going. One in the harbor line and one in the main line, each having a frequency of 10 minutes. The main line service starts at 5 o'clock and the harbor line starts at 5.02A.M. A man goes to the station every day to catch the first train that comes. What is the probability of the man catching the first train?
Ans: 0.8
5. Next number in the series is: 1 , 2 , 4 , 13 , 31 , 112 , ?
Ans: 224. (No number has digits more than 4. All of them are 1 , 2, 4, 8 , 16 , 32 , 64 converted to base 5 )
6. Father's age is three years more than three times the son's age. After three years, father's age will be ten years more than twice the son's age. What is the father's present age?
Ans: 33 years.
7. Light glows for every 13 seconds . How many times did it glow between 1:57:58 and 3:20:47 am.
Ans : 383 + 1 = 384
8. From a vessel, 1/3rd of the liquid evaporates on the first day. On the second day 3/4th of the remaining liquid evaporates. What fraction of the volume is present at the end of the second day.
Ans: 50%
9. Supposing a clock takes 7 seconds to strike 7. How long will it take to strike 10?
Ans: 10 1/2 seconds.
10. There are 20 poles with a constant distance between each pole. A car takes 24 second to reach the 12th pole.
How much will it take to reach the last pole.
Ans: 41.45 seconds (Let the distance between two poles = x, Hence 11x:24::19x:? )
11. How can 1000000000 be written as a product of two factors neither of them containing zeros
Ans: 2 power 9 x 5 power 9
12. Two trains starting at same time, one from Bangalore to Mysore and other in opposite direction arrive at their
destination 1 hr and 4 hours respectively after passing each other. How much faster is one train from other?
Ans: Twice
13. Every day a cyclist meets a train at a particular crossing. The road is straight before the crossing and both are traveling in the same direction. The cyclist travels with a speed of 10kmph. One day the cyclist comes late by 25 min. and meets the train 5km before the crossing. What is the speed of the train?
Ans: 60kmph
14. A man collects cigarette stubs and makes one full cigarette with every 8 stubs. If he gets 64 stubs how many full cigarettes can he smoke.
Ans: 8+1=9
15. The minute and the hour hand of a watch meet every 65 minutes. How much does the watch lose or gain time and by how much?
Ans: Gains; 5/11 minutes
16. A survey was taken among 100 people to find their preference of watching T. V. programs. There are 3 channels. Given the no of people who watch
• at least channel 1
• at least channel 2
• at least channel 3
• no channels at all
• at least channels 1and 3
• at least channels 1 and 2
• at least channels 2 and 3
Find the no of people who watched all three.
Ans.
17. Two trains start from stations A and B spaced 50kms apart at the same time and speed. As the trains start, a bird flies from one train towards the other and on reaching the second train, it flies back to the first train. This is repeated till the trains collide. If the speed of the trains is 25 km/h and that of the bird is 100km/h. How much did the bird travel till the collision.
Ans: 100kms.
18. Four persons A, B, C and D are playing cards. Each person has one card, laid down on the table below him, which has two different colors on either side. The colors visible on the table are Red, Green, Red and Blue. They see the color on the reverse side and give the following comment.
A: Yellow or Green B: Neither Blue nor Green
C: Blue or Yellow D: Blue or Yellow
Given that out of the 4 people 2 always lie find out the colors on the cards each person.
Ans.
19. Sometime after 10:00 PM a murder took place. A witness claimed that the clock must have stopped at the time of the shooting. It was later found that the position of both the hands were the same but their positions had interchanged.
Tell the time of the shooting (both actual and claimed).
Ans: Time of shooting = 11:54 PM
Claimed Time = 10:59 PM
20. Some statements are given below:
• L says all of my other four friends have money
• M says that P said that exactly one among them has money
• N says that L said that precisely two among them have money
• O says that M said that three of the others have money
• P, L and N said that they have money
All the above statement are false. Who has money & who doesn't have any money?
Ans.
21. The Bulls, Pacers, Lakers and Jazz ran for a contest. Anup, Sujit, John made the following statements regarding results.
• Anup said either Bulls or Jazz will definitely win
• Sujit said he is confident that Bulls will not win
• John said he is confident that neither Jazz nor Lakers will win
When the result came, it was found that only one of the above three had made a correct statement. Who has made the correct statement and who has won the contest.
Ans: Sujith; Lakers
22. There are five persons with surnames Mukherjee, Misra, Iyer, Patil and Sharma. There are 4 persons having first or middle name of Kumar, 3 persons with Mohan, 2 persons with Dev and 1 Anil. Either Mukherjee and Patil have a first or middle name of Dev or Misra and Iyer have their first or middle name of Dev. Of Mukherkjee and Misra, either both of them have a first or middle name of Mohan or neither have a first or middle name of Mohan. Either Iyer or Sharma has a first or middle name of Kumar but not both. Who has the first or middle name of Anil?
Ans: Kumar Misra Dev, Mohan Iyer Dev, Kumar Patil Mohan, Mohan Sharma Kumar
23. Ann, Boobie, Cathy and Dave are at their monthly business meeting. Their occupations are author, biologist, chemist and doctor, but not necessarily in that order. Dave just told the biologist that Cathy was on her way with doughnuts. Ann is sitting across from the doctor and next to the chemist. The doctor was thinking that Boobie was a goofy name for parent's to choose, but didn't say anything. What is each person's occupation?
Ans: Since Dave spoke to the biologist and Ann sat next to the chemist and across the doctor, Cathy must be the author and Ann the biologist. The doctor didn't speak, but David did, so Bobbie is the doctor and Dave the chemist.
24. There are 6 volumes of books on a rack kept in order ( i.e. vol.1, vol. 2 and so on ). Give the position after the following changes were noticed.
• All books have been changed
• Vol.5 was directly to the right of Vol.2
• Vol.4 has Vol.6 to its left and both weren't at Vol.3's place
• Vol.1 has Vol.3 on right and Vol.5 on left
• An even numbered volume is at Vol.5's place
Find the order in which the books are kept now.
Ans: 2 , 5 , 1 , 3 , 6 , 4
25. A soldier looses his way in a thick jungle. At random he walks from his camp but mathematically in an interesting fashion. First he walks one mile East then half mile to North. Then 1/4 mile to West, then 1/8 mile to South and so on making a loop. Finally how far he is from his camp and in which direction.
Ans: Distance traveled in north and south directions:
1/2 - 1/8 + 1/32 - 1/128 + 1/512 - ............. = 1/2/((1-(-1/4))
Similarly in east and west directions:
1- 1/4 + 1/16 - 1/64 + 1/256 - ................... = 1/(( 1- ( - 1/4)) Add both the answers
26. Conversation between two mathematicians:
First : I have three children. The product of their ages is 36. If you sum their ages, it is exactly same as my neighbor's door number on my left.
The second mathematician verifies the door number and says that it is not sufficient. Then the first says " Ok one more clue is that my youngest is really the youngest". Immediately the second mathematician answers. Can you answer the question asked by the first mathematician? What are the children ages?
Ans 1,6 and 6
27. 500 men are arranged in an array of 10 rows and 50 columns according to their heights. Tallest among each row of all are asked to fall out. And the shortest among them is A. Similarly after resuming that to their original positions that the shortest among each column are asked to fall out. And the tallest among them is B. Now who is taller among A and B ?
Ans. A

28. There are six boxes containing 5 , 7 , 14 , 16 , 18 , 29 balls of either red or blue in color. Some boxes contain only red balls and others contain only blue. One sales man sold one box out of them and then he says, " I have the same number of red balls left out as that of blue ". Which box is the one he sold out ?
Ans: Total no of balls = 89 and (89-29 /2) = 60/2 = 30 and also 14 + 16 = 5 + 7 + 18 = 30
29. Ram Singh goes to his office in the city, every day from his suburban house. His driver Gangaram drops him at the railway station in the morning and picks him up in the evening. Every evening Ram Singh reaches the station at 5 O' Clock. Gangaram also reaches at the same time. One day Ram Singh started early from his office and came to the station at 4 O' Clock. Not wanting to wait for the car he starts walking home. Mangaram starts at normal time, picks him up on the way and takes him back house, half an hour early. How much time did Ram Singh walk?
Ans.
30. A family X went for a vacation. Unfortunately it rained for 13 days when they were there. But whenever it rained in the mornings, they had clear afternoons and vice versa. In all they enjoyed 11 mornings and 12 afternoons. How many days did they stay there totally?
Ans: 18
31. There are N coins on a table and there are two players A & B. You can take 1 or 2 coins at a time. The person who takes the last coin is the loser. A always starts first.
• If N=7
(a) A can always win by taking two coins in his first chance
(b) B can win only if A takes two coins in his first chance.
(c) B can always win by proper play
(d) none of the above
Ans.
• 2. A can win by proper play if N is equal to
(a) 13 (b) 37 (c) 22 (d) 34 (e) 48
Ans: (e.)
• 3. B can win by proper play if N is equal to
(a) 25 (b)26 (c) 32 (d) 41 (e) none
Ans.
• 4. if N<4, can A win by proper play always?
(a) Yes (b) No
Ans.
32. Mr. Mathurs jewels have been stolen from his bank locker. The bank has lockers of 12 people which are arranged in an array of 3 rows and 4 columns like:
• The locker belonging to JONES was to the right of BLACK'S locker and directly above MILLAR'S.
• BOOTH'S locker was directly above MILLAR'S.
• SMITH'S locker was also above GRAY's (though not directly).
• GREEN'S locker was directly below SMITH'S.
• WILSON'S locker was between that of DAVIS and BOOTH.
• MILLAR'S locker was on the bottom row directly to the right of HERD'S.
• WHITE'S locker was on the bottom right hand corner in the same column as BOOTH'S.
Which box belonged to Mr. Mathurs?
Ans: Box number 9 belongs to Mr. Mathurs.
33. Five people A ,B ,C ,D ,E are related to each other. Four of them make one true statement each as follows.
(i) B is my father's brother.
(ii) E is my mother-in-law.
(iii) C is my son-in-law's brother
(iv) A is my brother's wife.
Ans: (i) D (ii) B (iii) E (iv) C
7
1. There is a 4 inch cube painted on all sides. This is cut down into of 1 inch cubes. What is the no of cubes which have no pointed sides.
Ans: 8
2. At 6'o a clock ticks 6 times. The time between first and last ticks is 30 seconds. How long does it tick at 12'o clock.
Ans: 66 sec.
3. Complete the series: 5, 20, 24, 6, 2, 8, ?
Ans: 12 (as 5*4=20, 20+4=24, 24/4=6, 6-4=2, 2*4=8, 8+4=12).
4. Find the values of each of the alphabets.
N O O N
S O O N
+ M O O N
J U N E
Ans: 9326
5. If a clock takes 7seconds to strike 7, how long will the same clock take to strike 10?
Ans: The clock strikes for the first time at the start and takes 7 seconds for 6 intervals-thus for one interval time taken=7/6. Therefore, for 10 seconds there are 9 intervals and time taken is 9*7/6=10 and 1/2 seconds.
6. An escalator is descending at constant speed. A walks down and takes 50 steps to reach the bottom. B runs down and takes 90 steps in the same time as A takes 10 steps. How many steps are visible when the escalator is not operating?
Ans: 150 steps
7. A chain is broken into three pieces of equal lengths containing 3 links each. It is taken to a blacksmith to join into a single continuous one. How many links are to to be opened to make it ?
Ans : 2.
8. There is a safe with a 5 digit number as the key. The 4th digit is 4 greater than second digit, while 3rd digit is 3 less than 2nd digit. The 1st digit is thrice the last digit. There are 3 pairs whose sum is 11. Find the number.
Ans: 65292
9. An orange glass has orange juice and white glass has apple juice both of equal volumes. 50ml of the orange juice is taken and poured into the apple juice. 50ml from the white glass is poured into the orange glass. Of the two quantities, the amount of apple juice in the orange glass and the amount of orange juice in the white glass, which one is greater and by how much?
Ans: The two quantities are equal
10. The shape in the sketch below is that of a square attached to half of a similar square. Divide it into four equal pieces
INCLUDEPICTURE "http://freshersworld.com/questions/ques2_9.gif" \* MERGEFORMATINET
Ans: Hint : the figure can be divided into 12 equal triangles
11. Fifty minutes ago if it was four times as many minutes past three o'clock, how many minutes is it to six o'clock?
Ans: Twenty six minutes.
12. Everyday in his business a merchant had to weigh amounts from 1 kg to 121kgs, to the nearest kg.
What are the minimum number of weight required and how heavy should they be?
Ans: .The minimum number is 5 and they should weigh 1,3,9,27 and 81kgs.
13. A car is traveling at a uniform speed. The driver sees a milestone showing a 2-digit number. After traveling for an hour the driver sees another milestone with the same digits in reverse order. After another hour the driver sees another milestone containing the same two digits. What is the average speed of the driver.
Ans: 45kmph
14. A hotel has 10 storeys. Which floor is above the floor below the floor, below the floor above the floor, below the
floor above the fifth.
Ans: The sixth floor.
15. Albert and Fernandes have two leg swimming race. Both start from opposite ends of the pool. On the first leg, the boys pass each other at 18 m from the deep end of the pool. During the second leg they pass at 10 m from the shallow end of the pool. Both go at constant speed but one of them is faster. Each boy rests for 4 seconds at the end of the first leg. What is the length of the pool?
Ans.
16. Shahrukh speaks truth only in the morning and lies in the afternoon, whereas Salman speaks truth only in the afternoon. A says that B is Shahrukh. Is it morning or afternoon and who is A - Shahrukh or Salman.
Ans: Afternoon ; A is Salman.
17. A person with some money spends1/3 for cloths, 1/5 of the remaining for food and 1/4 of the remaining for travel.
He is left with Rs 100/- . How much did he have with him in the beginning ?
Ans: Rs 250/-
18. Ram, Shyam and Gumnaam are friends.
Ram is a widower and lives alone and his sister takes care of him.
Shyam is a bachelor and his niece cooks his food and looks after his house.
Gumnaam is married to Gita and lives in large house in the same town.
Gita gives the idea that all of them could stay together in the house and share monthly expenses equally.
During their first month of living together, each person contributed Rs.25. At the end of the month, it was found that Rs 92 was the expense so the remaining amount was distributed equally among everyone. The distribution was such that everyone received a whole number of Rupees. How much did each person receive?
Ans. Rs 2 (Hint: Ram's sister, Shyam's niece and Gumnaam's wife are the same person)
19. There are 3 societies A, B, C. A lent cars to B and C as many as they had already. After some time B gave as many tractors to A and C as many as they have. After sometime c did the same thing. At the end of this transaction each one of them had 24. Find the cars each originally had.
Ans: A had 39 cars, B had 21 cars & C had 12 cars
20. Sam and Mala have a conversation.
• Sam says I am certainly not over 40
• Mala says I am 38 and you are at least 5 years older than me
• Now Sam says you are at least 39
All the statements by the two are false. How old are they really?
Ans: Mala = 38 yrs, Sam = 41 yrs.
21. Each alphabet stands for one digit in the following multiplication.
T H I S
x I S
X F X X
X X U X
X X N X X What is the maximum value T can take?
Ans: T max value = 4
22. Grass in lawn grows equally thick and in a uniform rate. It takes 24 days for 70 cows and 60 days for 30 cows to eat the whole of the grass. How many cows are needed to eat the grass in 96 days.?
Ans : 20
[Hint: g - grass at the beginning r - rate at which grass grows, per day
y - rate at which one cow eats grass, per day n - no of cows to eat the grass in 96 days
g + 24*r = 70 * 24 * y g + 60*r = 30 * 60 * y
g + 96*r = n * 96 * y, Solving, n = 20. ]
23. Three criminals were arrested for shop lifting. However, when interrogated only one told the truth in both his statements, while the other two each told one true statement and one lie. The statements were:
• ALBERT :(a) Chander passed the merchandise. (b) Bruce created the diversion.
• BRUCE :(a) Albert passed the merchandise. (b) I created the diversion.
• CLIVE :(a) I took the goods out of the shop. (b) Bruce passed them over.
Ans: Albert passed the goods. Bruce created the diversion. Clive took the goods out of the shop.
24. I bought a car with a peculiar 5 digit numbered license plate which on reversing could still be read. On reversing value is increased by 78633.Whats the original number if all digits were different?
Ans: Only 0 1 6 8 and 9 can be read upside down. So on rearranging these digits we get the answer as 10968
25. There N stations on a railroad. After adding X stations on the rail route 46 additional tickets have to be printed.
Find N and X.
Ans. x=2 and N=11 ( Let initially, N(N-1) = t; After adding, (N+X)(N+X-1) = t+46;Trail and error method )
27. A bird keeper has got P pigeons, M mynas and S sparrows. The keeper goes for lunch leaving his assistant to watch the birds.
• Suppose p=10, m=5, s=8 when the bird keeper comes back, the assistant informs the x birds have escaped. The bird keeper exclaims: "Oh no! All my sparrows are gone."
How many birds flew away?
• When the bird keeper comes back, the assistant told him that x birds have escaped. The keeper realized that at least 2 sparrows have escaped.
What is minimum no of birds that can escape?
Ans.
28. Seven members sat around a table for three days for a conference.
The member's names were Abhishek, Amol, Ankur, Anurag, Bhuwan ,Vasu and Vikram.
The meetings were chaired by Vikram.
On the first evening members sat around the table alphabetically.
On the following two nights, Vikram arranged the seating so that he could have Abhishek as near to him as
possible and absent minded Vasu as far away as he could.
On no evening did any person have sitting next to him a person who had previously been his neighbor.
How did Vikram manage to seat everybody to the best advantage on the second and third evenings?
Ans: Second evening: Vikram, Ankur, Abhishek, Amol, Vasu, Anurag and Bhuwan.
Third evening :Vikram, Anurag, Abhishek, Vasu, Bhuwan, Ankur, Amol.
29. Two twins have certain peculiar characteristics. One of them always lies on Monday, Wednesday, Friday. The other always lies on Tuesdays, Thursdays and Saturdays. On the other days they tell the truth. You are given a conversation.
• Person A-- today is Sunday my name is Anil
• Person B -- today is Tuesday, my name is Bill
What day is today?
Ans: Today is Tuesday.
30. Four prisoners escape from a prison. The prisoners, Mr. East, Mr. West, Mr. South, Mr. North head towards different directions after escaping. The following information of their escape was supplied:
• The escape routes were The North Road, South Road, East Road and West Road.
• None of the prisoners took the road which was their namesake.
• Mr. East did not take the South Road
• Mr. West did not the South Road.
• The West Road was not taken by Mr. East
What road did each of the prisoners take to make their escape?
Ans: Mr. East took the North Road
Mr. West took the East Road
Mr. North took the South Road
Mr. South took the West Road.
31. A hotel has two wings, the east wing and the west wing. Some east wing rooms but not all have an ocean view.
All west wing rooms have a harbor view. The charge for all rooms is identical, except as follows :
• Extra charge for all harbor view rooms on or above the 3rd floor
• Extra charge for all ocean view rooms except those without balcony
• Extra charge for some harbor rooms on the first two floor & some east wing rooms without ocean view but
having kitchen facilities.
Which of the following cannot be determined on the basis of the information given:
I. Whether there are any rooms without a balcony for which an extra charge is imposed.
II. Whether any room without a kitchen or a view involves an extra charge.
III. Whether two extra charges are imposed for any room.
(A) I only (B) II only (C) III only (D) II and III only (E) I, II and III
Ans: (A)
32. Given that April 1 is Tuesday. A, B, C are 3 persons told that their farewell party was on
• A - May 8, Thursday
• B - May 10, Tuesday
• C - June 5, Friday
Out of A, B, C only one made a completely true statement concerning date, day and month. The other told two one told the day right and the other the date right. What is correct date, month, day.
Ans: B - (May 10) SUNDAY , C - June 6 (Friday).
33. Answer the following questions based on the conditions from the choices A, B, C, D, E as described below:
(A) if a definite conclusion can be drawn from condition 1
(B) if a definite conclusion can be drawn from condition 2
(C) if a definite conclusion can be drawn from condition 1 and 2
(D) if a definite conclusion can be drawn from condition 1 or 2
(E) no conclusion can be drawn using both conditions
• person 1 says N<5
• person says N>5
• person 3 says 3N>20
• person 4 says 3N>10
• person 5 says N<8
What is the value of N?
a) 1. No of persons who speak false being less than no of persons who tells the truth.
2. Person 2 is telling the truth.
Ans.
b) 1. no of persons telling the truth is greater than no of persons telling lies
2. person 5 is telling the truth.
Ans.
HYPERLINK "http://freshersworld.com/Intsec.htm"
LUCENT TECHNOLOGY
The written test is of two parts:
Section A : Aptitude Test
Section B : Technical Test
The aptitude is a mixture of verbal, analytical and quantitative questions. It is based on a pattern similar to the CAT and GRE tests. Also questions on the patterns of these in R.S Agarwal can be practiced.
The technical section tests your knowledge in C with Data Structures and Pointers, Computer Networks, DBMS and Operating Systems
People with electronics background can expect to be tested on core subjects.
1. 6*12*15 is the volume of some material. How many cubes of edge 3 can be inserted into it ?
a. 20 b. 30 c. 40 d. 46
2. Two pipes can fill a tank in 10 and 12 hours respectively while third pipe will make the tank empty in 20 hours. If all three pipes operate simultaneously, in how many hours the tank will be filled ?
a. 11 Hrs b. 9 Hrs 15 minutes c. 8 Hrs d. 7 Hrs 30 minutes
3. Cost of an item is x. It's value increases by p% and decreases by p% Now the new value is 1 rupee, what is the actual value ?
Ans.(1000)/(1000-p*p).
4. A right circular cylinder and a cone are there. Base radius of cone is equal to radius of cylinder. What is the ratio of height to slant side if their volume are the same?
5. Distance between two poles is 50 meters. A train goes by 48 at a speed of kmph. In one minute how many poles will be crossed by the train ?
6. A pole seen from a certain distance at an angle of 15 degrees and 100 meters ahead by 30 degrees. What is the height of pole ?
7. For 15 people--each has to pay Rs.20.For 20 people--each has to pay Rs.18. For 40 people--how much has each to pay ?
8. If p=2q then q=r*r , if p-odd then q is even, whether r is even or odd ?
a) first condition is sufficient b) second condition is sufficient
c) both are sufficient d) both are not sufficient
9. If he sells 40 mangoes, he will get the selling price of 4 mangoes extra, What is his percentage increase in profit ?
a. 25% b. 30% c. 15% d.18%
10. 100 glasses are there. A servant has to supply glasses to a person If he supplies the glasses without any damage he will get 3 paise otherwise he will loose 3 paise. At the end of supplying 100 glasses if he gets 270 paise, how many glasses were supplied safely.
a. 100 b. 98 c. 95 d. 93
11. Q is not equal to zero and k = (Q x n - s)/2 find n?
(a) (2 x k + s)/Q (b) (2 x s x k)/Q (c) (2 x k - s)/Q (d) (2 x k + s x Q)/Q (e) (k + s)/Q
Questions 12 - 16:
A causes B or C, but not both
F occurs only if B occurs
D occurs if B or C occurs
E occurs only if C occurs
J occurs only if E or F occurs
D causes G, H or both
H occurs if E occurs
G occurs if F occurs
12. If A occurs which of the following must occurs
I. F & G II. E and H III. D
(a) I only (b) II only (c) III only
(d) I, II, III (e) I, II (or) II, III but not both
13. If B occurs which must occur
(a) D (b) D and G (c) G and H (d) F and G (e) J
14. If J occurs which must have occurred
(a) E (b) either B or C (c) both E F (d) B (e) both B and C
15. Which may occurs as a result of cause not mentioned:
I. D II. A III. F
(a) 1 only (b) 2 only (c) 1 and 2 (d) 2 and 3 (e) 1,2,3
16. E occurs which one cannot occurs
(a) A (b) F (c) D (d) C (e) J
GENERAL APTITUDE TEST (Time:30 min)
SECTION 1:READING COMPREHENSION
.......................................
..(passage of one and half page.)
The question & answers based on this passage were given. There were 5 questions.
You shouldn't expect the same passage to be given to you but we havw written down the questions to give you a general
view of the type of RC questions that you can expect
1. A reductive cycle is one in which
a. an employer attempts to reduce costs
b. the work-force is gradually reduced in number
c. costs decreases as....
d. there is less productive effort on the part of employees

Ans. (d)
2. If a substantial number of employees remain the reductive cycle one may assume that
a. the org. is enjoying increased business
b. the personnel dept....
c. the boss is not...
d. there is an unwholessome..

Ans. (a)
3. The passage indicates that unionisation of a worker...
a. opportunities for the workers realise..
b. opportunities for the workers to ...
c. more please working...
d. greater fringe benifits include

Ans. (d)
4. According to the author management failures in supervision mainly attributable to
a. carrying...
b. a soft-hearted...
c. ignorance.
d. lack of consideration.

Ans. (d)
5. Employees will get together to seek an improvement of conditions because of dissatisfaction stemming from
i. social just
ii. economic
iii. moral decadence

a. i only
b. ii only
c. i and ii only
d. i & iii only

Ans. (d)
SECTION II:QUANTITATIVE AND NUMERICAL ABILITY

6. The monthly personal maintenance allowance for a family of an employee is determined by the average age of family and size of the family. Jacob was drawing rs.570 as PMA after he got married 6 years back.At present he draws Rs 720 as PMA with his family of 3 after the birth of son?
a. 2 years
b. 3 years
c. 4 years
d. 5 years

Ans. (d)
7. White collar sells a shirt for an amount. Due to off season sale white collar started offering a discount of 20% on tag price. come diwali, white collar offered a further discount of 10% on the reduced price. If i get the shirt for Rs 108 what was the original price?
a. Rs.180
b. 160
c. 145
d. none

Ans. (d)
8. A vertical stick 10-cm long casts a shadow 6 cm long on the ground under similar conditions a tower casts a shadow 10m long determine the height of the tower to the 2nd place of the decimal.
a. 16.67m
b. 17.70m
c. 16.8m
d. none

Ans. (a)
9. In the fig given below ...... which curve rep.the monthly savage(the fig.cannot be recollected by the candidate)
a. LR
b. ST
c. PQ
d. none

Ans. (b)
10. Two poles of height 7m and 12m stand on a play ground. If the distance between their feet is 12m,find the distance between their tops.
a. 12m
b. 13m
c. 11m
d. none

Ans. (b)
SECTION III : ANALOGY
11. saint : holiness : : mongol :
a. chinese
b. barbarity
c. tribal
d. courageousness

Ans. (d)
12. entomologist : insects : : ornithologist :
a. marine life
b. birds
c. wild life
d. astronomy

Ans. (b)


13. ostracize : exclude : :
a. war : death
b. shipping : transport
c. war : surrender
d. population : people

Ans. (a)


14. rain : downpour : : joy :
a. happiness
b. triumph
c. ecstasy
d. laughter

Ans. (b)
15. niggardly : generous : : dolorous :
a. understandable
b. practical
c. happy
d. ostentatious
Ans. (d)
Direction : (18-20) : In column one are listed certain groups of individual while in column 2 are listedsome of the characteristics against each group of individuals which are common to each member of that group
Coloumn1 Column 2
BCDFH TU
DEGH QRS
DEF PN
ACDE NS
GDE PVQ
DEG VQRS
CDE VNO
BCD VTS
ABD PT
18. Which characteristcs are found either in 'E' or 'F' or in both but not in 'H'?
a. QRS
b. NOP
c. PVT
d. UVW

Ans. (b)
19. Which character is common to 'B' & 'C' but is not present in 'H'?
a. U
b. V
c. W
d. T

Ans. (a)
20. Which characteristics are common to 'C' & 'E' but are not present in 'F'?
a. T,V
b. V,W
c. T,U
d. O,P

Ans. (d)
SECTION V : VERBAL ABILITY
In the following questions mark the option that is nearest to the given word

21. Synonym for quixotic

a. rapid
b. exotic
c. longing
d. timid
e. idealistic

Ans. (e)
22. Antonym of recondite
a. unfriendly
b. easily comprehensive
c. closely juxtaposed
d. broad minded
e. sardonic

Ans. (b)
23. antonym of servile
a. moral
b. pue
c. futile
d. foul
e. haughty

Ans. (e)
24. Antonym of tacit
a. spoken
b. allowed
c. neutral
d. impertinent
e. unwanted

Ans. (a)
25. Synonym for macabre
a. musical
b. frightening
c. chewed
d. wicked
e. exceptional

Ans. (b)
SECTION VI : ANALYTICAL ABILITY
26. Fran: i want to stay out of ...
Sid:that's not true....
a. makes unfair demands on her students
b. only gives grades to a few favoured students
c. never gives out grades of A

Ans. (c)

For Q27 - 28 A small passage was given.
The answers though are the following
27. The major logical.weakness of the argument above is the fact that
a. it draws meaningful distinction between hands out and subsidies
b. it supports only the subsidy and does not provide the other side view.
c. it draws no meaningful distinction between hands out and subsidies
d. subsidies should not be given
e. none of the above

Ans. (c)
28. Which of the following persons would be most likely to disagree with the conclusions reached above argument
a. the president..
b. the patenee of a new device designed to make the widget absolute
c. the worker who worked with the patentee

Ans. (b)
29.- 30 (a small passage )
In a laboratory study,160 rabbits in an experimental group were injected with serum d, while 160 rabbits in a control group were injected with in two weeks, 39% of the experimental group rabbits had contracted jungle fever, a highly contagious and usually fatal disease therefore jungle fever must be caused by some substance similar to the substance
found in serum d
29. The above argument would be most greately strengthened if it were shown that
a. the normal...
b. 40% of ..
c. serum...
d. the blood of jungle fever victims invariably contains a high level of a certain toxic substance also

Ans. (d)
30. The above argument would be most seriously weakened if it were shown that
a. none of the rabbits in the experimental group had had jungle fever prior to the start of the experiment
b. the rabbits in the experimental group had had jungle fever prior to the start of the experiment.
c. some of the rabbits in the experimental group had had jungle fever prior to the start of the experiment .
d. four of the rabbits had had jungle fever prior to the start of the experiment .
e. one of the rabbits in the experimental group had had jungle fever prior to the start of the experiment

Ans. (e)
***************************
1. If a boat is moving in upstream with velocity of 14 km/hr and goes downstream with a velocity of 40 km/hr, then what is the speed of the stream ?

(a) 13 km/hr
(b) 26 km/hr
(c) 34 km/hr
(d) none of these
Ans. A
2. Find the value of ( 0.75 * 0.75 * 0.75 - 0.001 ) / ( 0.75 * 0.75 - 0.075 + 0.01)

(a) 0.845
(b) 1.908
(c) 2.312
(d) 0.001
Ans. A
3. A can have a piece of work done in 8 days, B can work three times faster than the A, C can work five times faster than A. How many days will they take to do the work together ?

(a) 3 days
(b) 8/9 days
(c) 4 days
(d) can't say
Ans. B
4. A car travels a certain distance taking 7 hrs in forward journey, during the return journey increased speed 12km/hr takes the times 5 hrs.What is the distance travelled

(a) 210 kms
(b) 30 kms
(c) 20 kms
(c) none of these
Ans. B
5. Instead of multiplying a number by 7, the number is divided by 7. What is the percentage of error obtained ?
6. Find (7x + 4y ) / (x-2y) if x/2y = 3/2 ?

(a) 6
(b) 8
(c) 7
(d) data insufficient
Ans. C
7. A man buys 12 lts of liquid which contains 20% of the liquid and the rest is water. He then mixes it with 10 lts of another mixture with 30% of liquid.What is the % of water in the new mixture?

8. If a man buys 1 lt of milk for Rs.12 and mixes it with 20% water and sells it for Rs.15, then what is the percentage of gain?

9. Pipe A can fill a tank in 30 mins and Pipe B can fill it in 28 mins.If 3/4th of the tank is filled by Pipe B alone and both are opened, how much time is required by both the pipes to fill the tank completely ?
10. If on an item a company gives 25% discount, they earn 25% profit. If they now give 10% discount then what is the profit percentage.
(a) 40%
(b) 55%
(c) 35%
(d) 30%
Ans. D
11. A certain number of men can finish a piece of work in 10 days. If however there were 10 men less it will take 10 days more for the work to be finished. How many men were there originally?

(a) 110 men
(b) 130 men
(c) 100 men
(d) none of these
Ans. A
12. In simple interest what sum amounts of Rs.1120/- in 4 years and Rs.1200/- in 5 years ?
(a) Rs. 500
(b) Rs. 600
(c) Rs. 800
(d) Rs. 900
Ans. C
13. If a sum of money compound annually amounts of thrice itself in 3 years. In how many years
will it become 9 times itself.

(a) 6
(b) 8
(c) 10
(d) 12
Ans A
14. Two trains move in the same direction at 50 kmph and 32 kmph respectively. A man in the slower train
observes the 15 seconds elapse before the faster train completely passes by him.
What is the length of faster train ?

(a) 100m
(b) 75m
(c) 120m
(d) 50m
Ans B
15. How many mashes are there in 1 squrare meter of wire gauge if each mesh
is 8mm long and 5mm wide ?

(a) 2500
(b) 25000
(c) 250
(d) 250000
Ans B
16. x% of y is y% of ?

(a) x/y
(b) 2y
(c) x
(d) can't be determined
Ans. C
17. The price of sugar increases by 20%, by what % should a housewife reduce the consumption of sugar so that expenditure on sugar can be same as before ?

(a) 15%
(b) 16.66%
(c) 12%
(d) 9%
Ans B
18. A man spends half of his salary on household expenses, 1/4th for rent, 1/5th for travel expenses, the man deposits the rest in a bank. If his monthly deposits in the bank amount 50, what is his monthly salary ?

(a) Rs.500
(b) Rs.1500
(c) Rs.1000
(d) Rs. 900
Ans C
20. The population of a city increases @ 4% p.a. There is an additional annual increase of 4% of the population due to the influx of job seekers, find the % increase in population after 2 years ?
21. The ratio of the number of boys and girls in a school is 3:2 Out of these 10% the boys and 25% of girls are scholarship holders. % of students who are not scholarship holders.?
22. 15 men take 21 days of 8 hrs. each to do a piece of work. How many days of 6 hrs. each would it take for 21 women if 3 women do as much work as 2 men?

(a) 30
(b) 20
(c) 19
(d) 29
Ans. A
23. A cylinder is 6 cms in diameter and 6 cms in height. If spheres of the same size are made from the material obtained, what is the diameter of each sphere?

(a) 5 cms
(b) 2 cms
(c) 3 cms
(d) 4 cms
Ans C
24. A rectangular plank (2)1/2 meters wide can be placed so that it is on either side of the diagonal of a square shown below.(Figure is not available)What is the area of the plank?
Ans :7*(2)1/2
25. The difference b/w the compound interest payble half yearly and the simple interest on a
certain sum lent out at 10% p.a for 1 year is Rs 25. What is the sum?

(a) Rs. 15000
(b) Rs. 12000
(c) Rs. 10000
(d) none of these
Ans C
26. What is the smallest number by which 2880 must be divided in order to make it into a
perfect square ?

(a) 3
(b) 4
(c) 5
(d) 6
Ans. C
27. A father is 30 years older than his son however he will be only thrice as old as the son after 5 years
what is father's present age ?

(a) 40 yrs
(b) 30 yrs
(c) 50 yrs
(d) none of these
Ans. A

28. An article sold at a profit of 20% if both the cost price and selling price would be Rs.20/- the profit would be 10% more. What is the cost price of that article?
29. If an item costs Rs.3 in '99 and Rs.203 in '00.What is the % increase in price?

(a) 200/3 %
(b) 200/6 %
(c) 100%
(d) none of these
Ans. A

LOGICAL SECTION
Directions: For questions 30-39 fill the missing number or letter in the given series
30. a, c, e, g, _

(a) h
(b) i
(c) d
(d) j
Ans. B

31. a, e, i, m, q, u, _, _

(a) y,c
(b) b,f
(c) g,i
(d) none
Ans. A
32. ay , bz , cw , dx ,__

(a) gu
(b) ev
(c) fv
(d) eu
Ans. D
33. 1, 2, 3, 5, 7, 11, __

(a) 15
(b) 9
(c) 13
(d) 12
Ans. C (series of prime numbers)
34. kp , lo , mn , __

(a) nm
(b) np
(c) op
(d) pq
Ans. A
35. R,M,__,F,D,__

(a) I, C
(b) A, Q
(c) L, N
(d) B, Q
Ans. A
36. ___, ayw, gec, mki, sqo

(a) awx
(b) usq
(c) prs
(d) lmn
Ans. B
37. 1, 3, 4, 8, 15, 27, __

(a) 60
(b) 59
(c) 43
(d) 50
Ans D
38. 0, 2, 3, 5, 8, 10, 15, 17, 24, 26,__

(a) 45
(b) 55
(c) 35
(d) 48
Ans. C
39. 2, 5, 9, 19, 37,__

(a) 64
(b) 55
(c) 75
(d) 40
Ans C
Directions for questions 40 to 45: Select the alternative that logically follows form the two given statements.
40. All scientists are fools. All fools are literates.

(a)All literates are scientists
(b) All scientists are literates
(c) No scientists are literates
(d) Both (a) and (b) are correct
Ans. B
41. No apple is an orange. All bananas are oranges.

(a) All apples are oranges
(b) Some apples are oranges
(c) No apple is a banana
(d) None of the above
Ans. A
42. All pens are elephants. Some elephants are cats.

(a) Some pens are cats
(b) No pens are cats
(c) All pens are cats
(d) None of the above
Ans. D
43. All shares are debentures.No debentures are deposits.

(a) All shares are deposits
(b) Some shares are deposits
(c) No shares are deposits
(d) None of the above
Ans. C
44. Many fathers are brothers. All brothers are priests.

(a) No father is a priest
(b) Many fathers are not priests
(c) Many fathers are priests
(d) Both (b) and (c)
Ans. B
45. Some green are blue. No blue are white.

(a) No green are white
(b) Some green are white
(c) No green are white
(d) None of the above
Ans. B
46. If the word "CODING" is represented as DPEJOH , then the word "CURFEW" can be represented?

(a) dvsgfx
(b) dvshfx
(c) dgshfx
(d) dtsgfy
Ans. A
47. If in a certain code "RANGE" is coded as 12345 and "RANDOM" is coded as 123678, then the code for the word "MANGO" would be

(a) 82357
(b) 84563
(c) 82346
(d) 82543
Ans. D
Directions for questions 48-50:The questions are based on the following data
In a class of 150 students 55 speak English;85 speak Telugu and 30 speak neither English nor Telugu
48. How many speak both English and Telugu?

(a) 10
(b) 15
(c) 20
(d) 12
Ans. C
49.How many speak only Telugu?

(a) 55
(b) 45
(c) 65
(d) none of the above
Ans.C
50.How many speak at least one of the two languages?

(a) 120
(b) 100
(c) 250
(d) 50
Ans. A

Refer R.S Agarwal books for more questions of the same kind
Verbal -- Page 254 problems 53 to56
246 eg.2
Page 104 Exercise.3a (Series Questions)
Page 354-355 8,13,
Page 115

Nonverbal -- Pages 5,41,54,108,145,158
MISTRAL SOLUTIONS
:
:
The written test consists of two sections: Aptitude Section and C Section. The aptitude section consists of 25 questions to be solved in minutes. The C Section also consists of 25 questions, but to be solved in minutes.
Both the sections are of objective type.
APPTITUDE TEST
1. At A&R auto repair shop, 2 mechanics can repair six cars in three hours. By this same reasoning how many mechanics will it take to repair 22 cars in five hours?
a. 8 b. 5 c. 7 d. 6 e. not sure

2. A worker of the CIA(cybernetic information association) is translating information in to a code sequence, so that competitors who might steal the information will not be able to make use of it. The worker is currently translating a word that u can find somewhere in this problem into this code. What number completes this word.: 8 22 10 6 22 13 24 ?
a. 5 b. 16 c. 22 d. 26 e. not sure

3.I really need to get a new watch. It correctly reads the time as 4:12pm,but three hours later it reads 8pm.two hours after that it reads 10.32pm.what time will it read when it is actually 3.42am?
a. 6:46am b. 1:38am c. 8:24am d. 6:26am e. not sure

4.how many cards are there in a full tarot deck?
a. 52 b. 78 c. 96 d. 113 e. not sure

5.what is Excalibur?
a. King b. Mage c. Mythical city d. sword e. not sure

6.How many days are there in a fortnight?
a. 15 b. 14 c. 10 d. 2 e. not sure

7.What was sundial used for?
a. regulating temperature b. refracting light c. measuring time
d. scrambling radio waves e. not sure

8.who wrote hunchback of Notre dame?
a. Hugo b. Pierre c. Walters d. Anderson e. not sure

9.What is Clark Kent also known as??
a. Dr. Death b. Human Torch c. Superman d. Lone Avenger e. not sure

10.A dresser drawer contains 15 garments. If 40% of those are blouses. How many are not blouses?
a. 6 b. 8 c. 9 d. 10 e. not sure

11. If the length of each of the sides of 3square garden plots is increased by 50%,by what % is the sum of the areas of the 3plots increased?
a. 375% b. 200% c. 150% d. 125% e. not sure

13. If the length of a rectangle is increased by 20% and the width of the same rectangle is decreased by 20% then the area of the rectangle?
a. decr.by 20% b. decr.by 4% c. unchanged d. incr.by 20% e. not sure

14. If n and p r both odd numbers, which of the following numbers must be an even number??
a. n+p b. np c. np+2 d. n+p+1 e. not sure

15. If 2 places r one inch apart on a map, then they r actually 160miles apart. (the scale on the map is 1inch=160miles)if Seton is 2 7/8 inches from Monroe on the map, how many miles is it from Seton to Monroe?
a. 460 b. 300 c. 27 d. 360 e. not sure

16. A screw driver and a hammer currently have the same price. If the price of the screw driver rises by 5% and the price of hammer goes up by 3%, by what percent will the cost of 3 screwdrivers and 3 hammers price?
a. 3% b. 4% c. 8% d. 25% e. not sure

17. If the average (or arithmetic mean) of six numbers is 4.5, what is the sum of the numbers?
a. 4.5 b. 24 c. 27 d. 30 e. not sure

Read the following and answer the questions below:

The office staff of XYZ corporation presently consists of 3 bookkeepers (A, B, C) and 5 Secretaries (D, E, F, G, H). Management is planning to open a new office in another city using 3 secretaries and 2 book keepers of the present staffs. To do so they plan to separate certain individuals who do not function well together. The following guidelines were established to set up the new office.
A) Book keepers A & C are constantly finding fault with one another and should not be send as a team to the new office
B) C & E function well alone but not as a team . They should be separated.
C) D & G have not been on speaking term for many months. They should not go together.
D) Since D & F have been competing for promotion, they should not be a team.


18. If A is to be moved as one of the book keepers, which of the following cannot be the possible working unit
a. ABDEH b. ABDGH c. ABEFH d. ABEGH e. not sure

19. If C &F are moved to the new office, how many combinations are possible.
a. 1 b. 2 c. 3 d. 4 e. not sure

20. If C is send to the new office which member of the staff cannot go with C
a. B b. D c. F d. G e. not sure

21. Under the guidelines developed, which of the following must go to the new office.
a. B b. D c. E d. G e. not sure

22. If D goes to the new office which of the following is (are ) true?
i. C cannot go
ii. A cannot go
iii. H must also go
a. i only b. ii only c. i&iii only d. i, ii, iii e. not sure

23. At luncheon table were 12 men are seated, one-half of the men belongs to club A, one-third belongs to club B and one-fourth belongs to both club. How many belongs to neither.
a. 3 b. 4 c. 5 d. 6 e. not sure

For each analogy, find the answer that best completes the problem:

24. Thieves: Den :: Cards: ?
a. Game b. Deck c. Set d. Group e. not sure

25. Body: Helmet :: Finger: ?
a. Thimble b. Nail c. Glove d. Bandage e. not sure
Question 1 to 5 have to be answered on the basis of the information given below:
On Sunday, December 23, four ships were berthed at the Port.
• Ship W left at 4 PM on Sunday, December 23, for a series of 8-day cruises to Bermuda and Nassau.
• Ship X left at 4:30 PM on Sunday, December 23, for a series of alternating11-day and 13-day cruises.
• Ship Y sailed at 5 PM on Sunday, December 23, for a series of 5-day cruises to Bermuda.
• Ship Z sailed on Monday, Decmeber 24, for a series of 7-day cruises to Nassau.
Each cruise begins on the day after departure.
Each ship is scheduled to return to the Port early in the morning after the last day of the cruise and leave again in the afternoon of the same day.
( From 1999 Barrons GRE book Model Test 3 - Section 5 - Q8 to Q12)
1. On December 31, which ships will be sailing from the Port on a New Year's Eve.
(a) W and X
(b) X and Y
(c) W and Z
(d) X and Z
(e) X, Y and Z
Ans: (c)
2. On how many sailing dates between December 24 and February 28 will ship W be moored alongside another ship
(a) 0
(b) 2
(c) 4
(d) 5
(e) 6
Ans: (d)
3. On how many occasions between December 24 and February 28 will three ships be moored at the Port.
(a) 0
(b) 1
(c) 2
(d) 3
(e) 4
Ans: (a)
4. On which day of the week will these four ships make most of their departures?
(a) Sunday
(b) Monday
(c) Tuesday
(d) Thursday
(e) Saturday
Ans: (b)
5. On which days of the week in the period between December 24 and February 28 will the pier be least crowded?
(a) Tuesday and Friday
(b) Tuesday and Thursday
(c) Friday and Saturday
(d) Wednesday and Thursday
(e) Thursday and Saturday
Ans: (a)

6. A family with a husband, his wife and their child are at one side of river.
They want to cross the river on a boat. The child can't be left alone.
The only available boat can hold only one person and the boatboy.
Only the boatboy can row the boat.
What is the minimum number of trips from on bank to the other, that the boatboy has to make
for the whole family to reach the other side.
Question 7 to 10 have to be answered on the basis of the information given below:
The workweek in a small business is a five-day workweek running from Monday through Friday.
In each workweek, activities L,M,N,O and P must all be done.The work is subject to the following restrictions:
• L must be done earlier in the week than O and earlier than P
• M must be done earlier in the week than N and earler than O
• No more than one of the activities can ever be done on any one day

7.Which of the following is an acceptable schedule starting from Monday to Friday
a) L, M, N, O, P
b) M, N, O, N, M
c) O, N, L, P, M
d) P, O, L, M, L
e) P, O, L, M, N
Ans. (a)

8. Which of the following pair of activies could be done on Monday and Tuesday
a) L and O
b) M and L
c) M and P
d) N and O
e) O and M
Ans. (b)

9.If P and N are done on Thursday and Friday, then which of the following is true
a) L is done on Tuesday
b) L is done on Wednesday
c) M is done on Monday
d) O is done on Tuesday
e) O is done on Wednesday
Ans. (e)

10. Which of the following could be true
a) L on Friday
b) M on Thursday
c) N on Monday
d) O on Monday
e) P on Tuesday
Ans. (e)

Aptitude Section

Q. 5 men or 8 women do equal amount of work in a day. a job requires 3 men and 5 women to finish the job in 10 days how many woman are required to finish the job in 14 days.

a) 10
b) 7
c) 6
d) 12
Ans 7


Q. A simple interest amount of rs 5000 for six month is rs 200. what is the anual rate of interest?

a) 10%
b) 6%
c) 8%
d) 9%
Ans 8%
Q. In objective test a correct ans score 4 marks and on a wrong ans 2 marks are ---. a student score 480 marks from 150 question. how many ans were correct?
a) 120
b) 130
c) 110
d) 150
Ans130.
Q. An artical sold at amount of 50% the net sale price is rs 425 .what is the list price of the artical?

a) 500
b) 488
c) 480
d) 510
Ans 500
1. Three beauty pageant finalists-Cindy, Amy and Linda-The winner was musician. The one who was not last or first was a math major. The one who came in third had black hair. Linda had red hair. Amy had no musical abilities. Who was first?
(A) Cindy (B) Amy (C) Linda (D) None of these

2. Two twins have certain peculiar characteristics. One of them always lies on Monday, Wednesday, Friday. The other always lies on Tuesdays, Thursday and Saturdays. On the other days they tell the truth. You are given a conversation.
Person A- today is Sunday, my name is Anil
Person B-today is Tuesday, my name is Bill What day is today?
(A) Sunday (B) Tuesday (C) Monday (D) Thursday

3. The difference of a number and its reciprocal is 1/2.The sum of their squares is
(A) 9/4 (B) 4/5 (C) 5/3 (D) 7/4

4. The difference of a number and its square is 870.What is the number?
(A) 42 (B) 29 (C) 30 (D) 32

5. A trader has 100 Kg of wheat, part of which he sells at 5% profit and the rest at 20% profit. He gains 15% on the whole. Find how much is sold at 5% profit?
(A) 60 (B) 50 (C) 66.66 (D) 33.3

6. Which of the following points are collinear?
(A) (3,5) (4,6) (2,7) (B) (3,5) (4,7) (2,3)
(C) (4,5) (4,6) (2,7) (D) (6,7) (7,8) (2,7)

7. A man leaves office daily at 7pm.a driver with car comes from his home to pick him from office and bring back home. One day he gets free at 5.30 and instead of waiting for driver he starts walking towards home. In the way he meets the car and returns home on car. He reaches home 20 minutes earlier than usual. In how much time does the man reach home usually?
(A) 1 hr 20 min (B) 1 hr (C) 1 hr 10 min (D) 55 min

8. If m:n = 2:3,the value of 3m+5n/6m-n is
(A) 7/3 (B) 3/7 (C) 5/3 (D) 3/5

9. A dog taken four leaps for every five leaps of hare but three leaps of the dog is equal to four leaps of the hare. Compare speed?
(A) 12:16 (B) 19:20 (C) 16:15 (D) 10:12

10. A watch ticks 90 times in 95 seconds. And another watch ticks 315 times in 323 secs.
Directions: Each of the following question has a question and two statements labelled as (i) and (ii). Use the data/information given in (i) and (ii) to decide whether the data are sufficient to answer the question record your answer as
A) If you can get the answer from (1)alone but not from (2)
B) If you can get the answer from (2)alone but not from (1)
C) If can get the answer from (1)and (2)together ,although neither statement by itself suffice
D) If statement (1)alone suffices and statement (2) alone also suffice.
E) If can't get the answer from statements (1) and (2) together and you need more data.
Q1)What will be the population of city X in 1991?
1) Population of the city has 55% annual growth rate
2) in 1991,the population of city X was 8 million
Ans:C

Q2) Was it Rani's birthday yesterday?
1)Lata spends Rs.100 on Rani's birthday
2)Lata spent Rs.100 yesterday
Ans: E

Q3)Is 3*5 or is 4*6 greater ?
1)a*b =b*a
2)a*b is the remainder of ab%(a+b)
Ans:B

Q4)Will the graph X-Y pass through the origin?
1) x proportional to the Y
2)increment in y per units rise of x is fixed.
Ans:E
Q5) What was the value of the machine 2 years ago?
1) the deprecition of the value of the machine per year is 10%
2)present value of the machine is rs 8000/
Ans:C
Q6) What will be the area of a square that can be inscribed in a circle?
1) Radius of the circle is T
2) Length of a diagonal of the square is 2r
Ans:D
Q7) Can it be concluded that the port made more profit in 1988 than in 1987
1) 1987
Total tonnage handled by the port 10 million tonnes
Expenditure made by the port to handle one tonne of cargo
Rs.20/-
2) 1988
Total tonnage handled by the port 12.5 million tonnes
Expenditure made by the port to handle one tonne of cargo Rs 25/-
Ans: E
Q8) There are two figures viz., a circle and a square. Which having greater area?
1) Perimeter of the circle is the same as the perimeter of the square.
2) Eleven times the radius is equal to seven times the length of one side of the square.
Ans: D
Q9) A candidate who was found to be under weightin medical test had been selected provisionally subject to his attainment of 60Kg weight within one year. What should be the percentage increase of his weightso that selection is confirmed after one year.
1) Weight (Kg)=16+8 Height (ft) is standard equation for the Indian population. The candidates height is 5.5
2) His present weight is 55Kg.
Ans: D

Q10) Is angle µ=90
1) sin**2(µ)+cos**2(µ)=1
2) sin**2(µ)-+cos**2(µ)=1
Ans: E

Q11) What will be the average age of workers of an Institution after two years?
1) Present average age is 35 years
2) There are total 20 workers in the Institution
Ans: A

Q12) Can it be concluded that firestry is getting increasing importance in India? ( Disregarding the change in money value )
1)
Name of the plan
Expenditure on Forest (Crores of rupees)

First five year plan
Second five year plan
10
19

2)
Name of the plan
Expenditure on Forest
(Crores of rupees)

First five year plan
Second five year plan
46
92.5

Ans: E

Q13) Is AB>AM ( A Triangle is given )
1) AB2) M is any point other than B and C on BC
Ans: E

Q14) Is X^2+Y^21) 02) 0Ans: C

Q15) Can it be concluded that angle ABO = angle ODC
1) ABCD is a Parallelogram and O is the point of intersection of the diagonals.
2) Angle DOC =75deg. and angle DAO =35deg.
Ans: A

Q16) What is the value of x+y?
1) 2y=x+6
2) 5x=10y-30
Ans: E

Q17) How many students are there in the class?
1) 30 students play foot ball and 40 play cricket .
2)Each student plays either foot ball or cricket or both.
Ans: E

Q18) What is the value of a:b?
1) a=x+10%ofx
2) b=a+10%ofa
Ans: B

Q19) What is the maximum value of the expression 5+8x-8x^2?
1) x is real
2) x is not positive
Ans: C

Q20) What will be the value of the greatest angle of the triangle ABC?
1) Angles of the triangle are in the ration 2:5:3
2) The side opposite to the greatest angle is the longest side.
Ans: A

Q21) What is the range of values of x?
1)( x-2 ) / ( 2x + 5 ) < 1/3
2)2x /3 + 17/3 > 3x - 20
Ans: D

Q22) Of the two which one is the greater -- -3/x , -3/y?
1) x,y>0
2) xAns: C

Q23) What percentage of the candidates passed both in science and mathematics?
1) 52 percent of the candidates failed in science
2) 42% of the candidates failed in mathematics
Ans: C

Q24) How much pure H2SO4 (Hydro Sulphuric Acid) should be added to bring down the percentage of impuritity to 5%?
1). 50 liters of pure H2SO4 was diluted
2). dilution was to the extent of 20%
Ans:C

Q25) What is the cost of building when archtects feeses was 70,000
1. Architect gets 10% for the first Rs. 50000 of the cost of building
2. Architect gets 3% on the cost of the building over 50000
Ans:C

Q26) What is the value of BC?( here one triangle figure is there )
1). AP=4
2). PQ=5
Ans: E

Q27) What is the area of the shaded portion (assume AB, CD are arcs of two circles with centre at 0.)Here one arc figure is there
1). CA=20m
2). CB=5m
Ans:C

Q28) What is the area of the greatest circle that can be out from rectangular paper
1). length of the paper is 30cm
2). Width of the paper is 21cm
Ans:B

Q29) Y is what percentage of X?
1). 0.3x=Y
2). 3x-10y=0
Ans:D

Q30) What is the area of the trapezium abcd where ab is 5cm
1. BC=7CM
2. AB+CD= 16CM


Directions :- Each sentence below has one or two blanks.Choose the word from the set of words for each blank that best fits the meaning of the sentence as a whole.
Q31) The air was bitter cold, the temperature well below the freezing point , yet they found themselves ------ freely as they clambered up the steep northern slope
Ans: disporting

Q32) We were taken when we heard of his defection , never having suspected that he was anything but loyal. So capable had been his ---- or and devotion to cease
Ans: presentiment

Q33) War and peace are mutually ------- states of being and war to preserve peace is not a paradox . It is a -----
Ans: incompatible -- contradiction

Q34) Although the injury appeared ------, the examination by the ophthomologist revealed that he would need immediate surgery to save his sight.
Ans: superficial
Q35 to Q40 - On similar pattern as above.

Antonyms
Q41. corroborative ---- refutable
Q42. obnoxious ---- harmless
Q43. sanction ---- hinder
Q44. empirical ---- experimental
Q45. aborigine ---- emigrant

Directions for questions 56- 60 . Questions 56 -60 are based on the following information:
A port has four berths W,V,X,Y. Of these two are general cargo berth, one is a fertiliser berth and one is for liquid cargo, When vessel A arrived it was berthed at berth V but vessel B which along with A had to wait prior to berthing as vessel C was working in berth Y and vessel D was working in berth W .Vessel E came to unload fertiliser and did not have to wait. All are specilised berths i.e. general cargo vessel has to work only in a general cargo berth. So is true for fertiliser vessel and liquid cargo vessel.
Q56. The vessel E should be alotted to the berth.
Ans: X

Q57. Which of the following berth can accept a vessel carrying liquid cargo--W, V, X, Y
Ans: V

Q58. Which of the following is not a general cargo vessel--A ,B, C, D, E
Ans:A

Q59. Total number of general cargo vessels mentioned in the above description is
Ans:3

Q60. Whcih of the following allotments is possible
Ans: B to W
*********************
ANTONYMS
1. Disregarded
(a) heed
(b) hopeful
(c) evade
(d) dense
Ans. (a)
2. Obviate
(a) becloud
(b) necessitate
(c) rationalize
(d) execute
Ans. (b)
3. Superficial
(a) profound
(b) exaggerated
(c) subjective
(d) spirited
Ans. (a)
4. Abide
(a) retract an offer
(b) refuse to endure
(c) shield from harm
(d) exonerate
Ans. (b)

5. Acerbity
(a) noteworthiness
(b) hypocrisy
(c) mildness of temperament
(d) lack of anxiety
Ans. (c)

Directions: Each question or group of questions is based on a passage or set of conditions. For each question, select the best answer choice given.
Quesitions 6-9
In a certain society, there are two marriage groups, Red and Brown. No marriage is permitted within a group. On marriage, males become part of their wife's group: women remain in their own group. Children belong to the same group as their parents. Widowers and divorced males revert to the group of their birth. Marriage to more than one person at the same time and marriage to a direct descendant are forbidden.
6. A Brown female could have had
I. a grandfather born Red
II. a grandmother born Red
III. two grandfathers born Brown
(a) I only
(b) II only
(c) I and II only
(d) II and III only
(e) I,II and III
Ans. (c)

7. A male born into the Brown group may have
(a) an uncle in either group
(b) a Brown daughter
(c) a Brown son
(d) a son-in-law born into the Red group
(e) a daughter-in-law in the Red group
Ans. (a)

8. Which of the following is not permitted under the rules stated?
(a) A Brown male marrying his father's sister
(b) A Red female marrying her mother's brother
(c) A man born Red, who is now a widower, marrying his brother's widow
(d) A widower marrying his wife's sister
(e) A widow marrying her divorced daughter's ex-husband
Ans. (b)

9. If widowers and divorced males retained the group they had upon marrying, which of the following would have been permissible?(Assume no previous marriages occurred)
(a) A woman marrying her dead sister's husband
(b) A woman marrying her divorced daughter's ex-husband.
(c) A widower marrying his brother's daughter
(d) A woman marrying her mother's brother, who is a widower
(e) A divorced male marrying his ex-wife's divorced sister
Ans. (d)
Questions 10-13
Tom wishes to enroll in Latin AA, Sanskrit A, Armenian Literature 221, and Celtic Literature 701.
Latin AA meets five days a week, either from 9 to 11 A.M or from 2 to 4 P.M.
Sanskrit A meets either Tuesday and Thursday from 12 noon to 3 P.M., or Monday, Wednesday, and Friday
from 10 A.M to 12 noon.
Armenian Literature 221 meets either Monday, Wednesday, and Friday from 12:30 to 2 P.M., or Tuesday and Thursday
from 10:30 A.M to 12:30 P.M
Celtic Literature 701 meets by arrangement with the instructor, the only requirement being that it meet for one four-hour session or two two-hour sessions per week, between 9A.M and 4 P.M from Monday to Friday, beginning on the hour.

10. Which combination is impossible for Tom?
(a) Latin in the morning, Sanskrit on Tuesday and Thursday, and Armenian Literature on Monday, Wednesday, Friday
(b) Latin in the afternoon and Sanskrit and Armenian Literature on Monday, Wednesday, and Friday.
(c) Latin in the afternoon, Sanskrit on Monday, Wednesday, and Friday,and Armenian Literature on Tuesday and Thursday
(d) Latin in the morning and Sanskrit and Armenian Literature on Monday, Wednesday, and Friday
(e) Latin in the afternoon, Armenian Literature on Monday, Wednesda and Friday, and Celtic Literature on Tuesday
Ans. (d)

11. Which of the following gives the greatest number of alternatives for scheduling Celtic Literature, assuming that all other courses
(a) Latin in the afternoon and Armenian Literature Monday, Wednesday and Friday
(b) Sanskrit on Tuesday and Thursday and Armenian Literature on Monday, Wednesday and Friday
(c) Latin in the afternoon and Armenian Literature Tuesday and Thursday
(d) Latin in the morning and Sanskrit on Tuesday and Thursday
(e) Sanskrit on Monday, Wednesday, and Friday. and Armenian Literature on Tuesday and Thursday
Ans. (a)

12. If the Celtic instructor insists on holding at least one session on Friday, in which of the following can Tom enroll?
(I) Armenian Literature on Monday, Wednesday, and Friday
(II) Sanskrit on Monday, Wednesday, and Friday
(a) I only
(b) II only
(c) both I and II
(d) I or II but not both
(e) neither I nor II
Ans. (d)

13. Which of the following additional courses, meeting as indicated, can Tom take?
(a) Maths--Monday, Wednesday, and Friday from 10A.M to 12 noon
(b) French--Monday, Wednesday, and Friday from 11A.M to 12:30 P.M
(c) English--Tuesday and Thursday from 2 to 4 P.M
(d) Japenese--Tuesday and Thursday from 1 to 3 P.M
(e) Old Norse-Icelandic--Monday only from 12 to 3 P.M
Ans. (b)
Questions 14-18
(1) Ashland is north of East Liverpool and west of Coshocton
(2) Bowling Green is north of Ashland and west of Fredericktown
(3) Dover is south and east of Ashland
(4) East Liverpool is north of Fredricktown and east of Dover
(5) Fredricktown is north of Dover and west of Ashland
(6) Coshocton is south of Fredricktown and west of Dover
14. Which of the towns mentioned is furthest to the northwest ?
(a) Ashland
(b) Bowling Green
(c) Coshocton
(d) East Liverpool
(e) Fredericktown
Ans. (b)

15. Which of the following must be both north and east of Fredricktown?
(I) Ashland
(II) Coshocton
(III) East Liverpool
(a) I only
(b) II only
(c) III only
(d) I and II
(e) I and III
Ans. (e)

16. Which of the following towns must be situated both south and west of at least one other town?
(a) Ashland only
(b) Ashland and Fredricktown
(c) Dover and Fredricktown
(d) Dover,Coshocton and Fredricktown
(e) Dover,Coshocton and East Liverpool
Ans. (d)

17. Which of the following statements, if true, would make the information in the numbered statements more specific?
(a) Coshocton is north of Dover
(b) East Liverpool is north of Dover
(c) Ashland is east of Bowling Green
(d) Coshocton is east of Fredericktown
(e) Bowling Green is north of Fredericktown
Ans. (a)

18. Which of the numbered statements gives information that can be deduced from one or more of the other statement?
(a) (1)
(b) (2)
(c) (3)
(d) (4)
(e) (6)
Ans. (c)

Questions 19-22
Spelunkers International offers exploring tours in eight caves: Abbott, Benny, Caeser, Dangerfield, Ewell, Fields, Guinness, and Hope
(1) Class 1 spelunkers may not attempt cave Ewell, Fields or Hope
(2) Class 2 spelunkers may not attempt Hope
(3) Class 3 spelunkers may attempt any cave
(4) Cave Caesar may be attempted only by spelunkers who have previously explored cave Benny
(5) Cave Fields may be attempted only by spelunkers who have previously explored cave Ewell
(6) Only two of caves Benny, Caeser, Ewell, Fields, and Hope may be attempted by any explorer in a single tour

19. A class 2 spelunker who has previously explored cave Ewell may be restricted in choosing a tour by which rule(s)?
(I) Rule(4)
(II) Rule(5)
(III) Rule(6)
(a) I only
(b) II only
(c) I and III only
(d) II and III only
(e) I, II and III
Ans. (c)

20. In how many different ways may a class 1 spelunker who has never explored any of the eightcaves before set up a tour of three caves, if she wishes to explore caves Abbott and Caesar?
(a) 2
(b) 3
(c) 4
(d) 5
(e) 6
Ans. (b)

21. What is the maximum number of caves that a class 3 spelunker who has previously explored only cave Benny may include
in a single tour?
(a) 4
(b) 5
(c) 6
(d) 7
(e) 8
Ans. (b)

22. If x + y = 3 and y/x= 2 then y = ?
(a) 0
(b) 1/2
(c) 1
(d) 3/2
(e) 2
Ans. (e)

23. How many squares with sides 1/2 inch long are needed to cover a rectangle that is 4 ft long and 6 ft wide
(a) 24
(b) 96
(c) 3456
(d) 13824
(e) 14266

24. If a=2/3b , b=2/3c, and c=2/3d what part of d is b/
(a) 8/27
(b) 4/9
(c) 2/3
(d) 75%
(e) 4/3
Ans. (b)

25. Successive discounts of 20% and 15% are equal to a single discount of
(a) 30%
(b) 32%
(c) 34%
(d) 35%
(e) 36
Ans. (b)

26. The petrol tank of an automobile can hold g liters.If a liters was removed when the tank was full, what part of the full tank was removed?
(a)g-a
(b)g/a
(c) a/g
(d) (g-a)/a
(e) (g-a)/g
Ans. (c)

27.If x/y=4 and y is not '0' what % of x is 2x-y

(a)150%
(b)175%
(c)200%
(d)250%

Ans. (b)

28.If 2x-y=4 then 6x-3y=?
(a)15
(b)12
(c)18
(d)10

Ans. (b)

29.Ifx=y=2z and xyz=256 then what is the value of x?

(a)12
(b)8
(c)16
(d)6

Ans. (b)

30. (1/10)18 - (1/10)20 = ?
(a) 99/1020
(b) 99/10
(c) 0.9
(d) none of these
Ans. (a)

31. Pipe A can fill in 20 minutes and Pipe B in 30 mins and Pipe C can empty the same in 40 mins.If all of them work together, find the time taken to fill the tank
(a) 17 1/7 mins
(b) 20 mins
(c) 8 mins
(d) none of these
Ans. (a)

32. Thirty men take 20 days to complete a job working 9 hours a day.How many hour a day should 40 men work to complete the job?
(a) 8 hrs
(b) 7 1/2 hrs
(c) 7 hrs
(d) 9 hrs
Ans. (b)

33. Find the smallest number in a GP whose sum is 38 and product 1728
(a) 12
(b) 20
(c) 8
(d) none of these
Ans. (c)

34. A boat travels 20 kms upstream in 6 hrs and 18 kms downstream in 4 hrs.Find the speed of the boat in still water and the speed of the water current?
(a) 1/2 kmph
(b) 7/12 kmph
(c) 5 kmph
(d) none of these
Ans. (b)

35. A goat is tied to one corner of a square plot of side 12m by a rope 7m long.Find the area it can graze?
(a) 38.5 sq.m
(b) 155 sq.m
(c) 144 sq.m
(d) 19.25 sq.m
Ans. (a)
.***********************
1. Last month of an year
(a) January (b) February (c) December (d) November

2. Select the odd one
(a) January (b) February (c) Wednesday (d) November

3. Select the antonym of capture from the following
(a) attack (b) Release (c) condemn (d) None of the above

4. Find the antonym of autumn
(a) Spring (b) Winter (c) Summer (d) None of the above

5. One skirt requires 3.75 yards of cloth. How many skirts you can make from 45 yards?
Ans: 12 skirts

6. How can you make a square from two triangles?

7. Is the meaning of Client and Customer,
(a) same (b) contradictory (c) no relation

8. Is the meaning of It's and Its,
(a) same (b) contradictory (c) no relation

9. Is the meaning of Canvas and Canvass,
(a) same (b) contradictory (c) no relation

10. Is the meaning of Ingenious and Ingenuous,
(a) same (b) contradictory (c) no relation

11. Is the meaning of Credible and Credulous,
(a) same (b) contradictory (c) no relation

12. Select the odd one out.
(a) 1/4 (b) 1/3 (c) 1/6 (d) 1/18

13. Select the least from the following.
(a) 0.99 (b) 1 (c) 81 (d) 0.333

14. Find the next number in the series. 1, 0.5, 0.25, 0.125
Ans: 0.0625

15. One dollar is saved in one month. Then how much dollar is saved in one day?
Ans: 1/30 =0.0333$

16. Y catches 5 times more fishes than X. If total number of fishes caught by X and Y is 48, then number of fishes caught by X?
Ans: 8

17. Y catches 5 times more fishes than X. If total number of fishes caught by X and Y is 42, then number of fishes caught by X?
Ans: 7

18. If a train covers 600m in 0.5 seconds, how long it will cover in 10 seconds?
Ans: 3000m = 3km

19. The girl's age is twice that of boy, if the boy is four years old. After four years the age
of the girl is
Ans: 12 years

20. Sister's age is twice than that of the brother. If the brother's age is six, what is the sister's age after two years?
Ans: 14 Yrs.

21. Two lemons cost 10 cents. Then one and a half dozen cost
Ans: 90 cents

22. A clock is late by 1 minute 27 seconds in a month. Then how much will it be late in 1 day?
Ans: 2.9 seconds

23. Which of the following figures together will make a triangle?
Ans: a,b,c,d

24. Make a square by drawing only one line
Ans: line 2-5, square 2-3-4-5-2

25. Which of the following is the odd one? crew, constellation, companion, league, participants.
Ans: companion

26. Opposite of Remote?
(a) Far (b) Near (c) Huge (d) Village

27. Statement A: All great men are ridiculous;
Statement B: I am ridiculous ;
Inference : I am a great man;
(a) True (b) False (c) Not clear

28. Statement: Normal children are active;
Inference: All children are active;
(a) True (b) False (c) Uncertain

29. Next number in the series 1, 1/2, 1/4, 1/8 ?
Ans: 1/16

30. In 6 seconds a light flashes once. In one hour how many times it will flash?
Ans: 601 times

31. At 20% discount, a cycle is sold at a selling price of 2500 Rs. What is the actual price?
Ans: Rs. 3125

32. Statement A: A & B have same age;
Statement B: B is younger than C;
Inference : A is younger than C;
(a) True (b) False (c) Uncertain

33. All chickens lay eggs (True/False)
Ans: False

34. A invests $12000, B invests $8000, C invests $6000 and they got a profit of $1200. How much share A got more than B and C?
Ans: 2/13 and 3/13

**************************
VERBAL SECTION
Directions: Give the synonyms for the following words
1. Depreciation: deflation, depression, devaluation, fall, slump
2. Depricate : feel and express disapproval,
3. Incentive : thing one encourages one to do (stimulus)
4. Echelon : level of authority or responsibility
5. Innovation : make changes or introduce new things
6. Intermittent : externally stopping and then starting
7. Detrimental: harmful
8. Conciliation : make less angry or more friendly

9. Orthodox: conventional or traditional, superstitious

10. Fallible : liable to error

11. Volatile : ever changing

12. Manifest: clear and obvious

13. Connotation : suggest or implied meaning of expression

14. Reciprocal: reverse or opposite

15. Agrarian : related to agriculture

16. Vacillate : undecided or dilemma

17. Expedient : fitting proper, desirable

18. Simulate : produce artificially resembling an existing one.

19. Access : to approah

20. Compensation: salary

21. Truncate : shorten by cutting

22. Adherence : stick

23. Heterogenous: non similar things

24. Surplus : excessive

25. Assess : determine the amount or value

26. Congnizance : knowledge

27. Retrospective : review

28. Naive : innocent,rustic

29. Equivocate : tallying on both sides, lie, mislead

30. Postulate : frame a theory

31. Latent : dormant, secret

32. Fluctuation : wavering,

33. Eliminate : to reduce

34. Affinity : strong liking

35. Expedite : hasten

36. Console : to show sympathy

37. Adversary : opposition

38. Affable : lovable or approachable

39. Decomposition : rotten

40. Agregious : apart from the crowd, especially bad

41. Conglomaration: group, collection

42. Aberration: deviation

43. Aurgury : prediction

44. Crediability : ability to common belief, quality of being credible

45. Coincident: incidentally

46. Constituent : accompanying

47. Differential : having or showing or making use of

48. Litigation : engaging in a law suit

49. Maratorium: legally or offficiallly determined period of dealy before
fulfillment of the agreement of paying of debts.

50. Negotiate : discuss or bargain

51. Preparation : act of preparing

52. Preponderant : superiority of power or quality

53. Relevance : quality of being relevant

54. Apparatus : appliances

55. Ignorance : blindness, in experience

56. Obsession: complex enthusiasm

57. precipitate : speed,active
SERIES SECTION
Directions: In the following questions complete the series
NOTE: This section is quite tough and consists of 26 questions to be done in 10 minutes. Please keep track of time.
1. A C B D E F G I - I H K J L
Ans. H


2. A I Z B E Y C I X D I - G E N J W
Ans. W


3. A D G J M P - R W T S
Ans. S


4. A B C E F G I J K - M L O N P
Ans. M

5. A B F G K L P Q - T S V U W
Ans. U


6. J W X U V S T - Q P S E T
Ans. Q


7. A R H X Y T D T W S T - N P T K R
Ans. P


8. F M B I P Z V I E V - I R Y O U


9. N Z I Y C X KW F - J F V M Y
Ans. V


10. A A S A S P A S P K A - R Q T S U
Ans. S


11. A E C P S - T R U E
Ans. U


12. B B P R D D L N F F I K - H Q J I K
Ans. H


13 A Z E X I V M T - R Q N S O
Ans. Q


14. A B D G K P - L I W U X
Ans. U


15. B C D A E G H I F J L M N L K N M O
Ans. K


16. X W E F G V U H I J K - P N S R T
Ans. T


17. O D J T O P Q N O E R T - Q O U V W
Ans. O


18. P R N U U P E J R B B - H V U N E
Ans. E


19.L U L M G M N F N P S - O N Q P S
Ans. P
NUMERICAL ABILITY
1. 420% OF 7.79 = ?
Ans. 32.718

2. 3427 / 16.53 = ?
Ans. 202
3. 10995 /95 = ?
Ans.115.7365
4. 43+557-247 =?
Ans. 353
5. 3107*3.082= ?
Ans. 9591
6. 48.7 + 24.9 - 8.7 = ?
Ans. 64.90
7.525.0/47.8 = ?
Ans. 11
8. (135-30-14)*7 - 6 +2 = ?
Ans. 3
9. 3/8 * 5.04=?
Ans. 1.89
10. 697 /219 = ?
Ans. 3.18
11.8/64 +64/16 =?
Ans. 4.14
12. 298 * 312 / 208 = ?
Ans. 453.54
13. 0.33 *1496 /13 = ?
Ans. 37.98
14.0.26 + 1/8 = ?
Ans. 0.385
15. 66.17+1/3= ?
Ans. 67.03
16. 2.84+1/4= ?
Ans. 3.09
17. 33% OF 450 = ?
Ans. 148.5
18. 907.54 / 0,3073= ?
Ans.3002
19.There are two categories of persons in ratio A:B = 2:3. A type earns 2.5 dollars/hr and
B type 1 dollar/hr total money earned by both is 24dollars. Then what is the total number of persons
Ans. 15
20. Total balls are z, the number of red balls is n and the remaining are blak balls,then the % of black
balls equal to ?
Ans. (z - n) / z*100
21. If A = C, B = 2D what should be done to make the ratio same. i.e.a/b = c/d
Ans. Multiply A by 2
22. If P=Total number of components, Q = number of defective components .What is the % of non defective components?
Ans. (p-q) / p*100
23. If the cost of an article is x , first discount given is y% of cost, second discount given is z% of cost .
The selling price of x is

Ans. x (1-y / 100) (1- z / 100)
24.Which of the following are prime numbers
(a) 119
(b) 115
(c) 127
(d) none
Ans. (c)
25. A / B = C; C > D then
(a) A is always greater than D
(b) C is always greater than D
(c) B is always less than D
(d) None of these
Ans. (a)
26. If B>C and AAns. ( A + B )C
27. If for H hours of work the salary is S and the employee gets x hours of medical leave, then what is the salary/hr ?
Ans. s/H-x
28. ( 1/6 of 596) / (0.695) = ?
Ans. 142
29. 35-30 + 4/7 - 5 + 1 = ?

Ans. 3
30. 10995 + 95 = ?
Ans. 11090
31. If on a salary of Rs."S" per month,one has to pay one tax of x Rs. and a second type of tax of y Rs
then % of salary taken home is?

Ans. s-(x+y)/s * 100
32. B>A then which expression will be highest value
(a) A-B
(b) AB
(c) A+B
(d) Can't Say
Ans. (b)
33. K, L are men who take home a salary of x, y respectively.The total amount taken home is
Ans. Kx + Ly

34. If out of X bulbs y bulbs are broken;The % of non broken bulbs
Ans. (x-y) / x*100

35. If on a salary s per month, a tax of x% of the salary and another of r% of the salary is deducted what
is the income.

Ans. s*(1-(x+r)/100
36. 0.512 * 18902358 =?
Ans. 9678007.296
37. If the % of defective balls is 10% balls,and the number of defective balls is 5.The number of balls is

Ans.50

38. 6.29% of 2.8 =?
Ans. 0.18
39. 0.398 * 456= ?
Ans. 181.49

40. 0 < x < 1 which is greater
(a) 1/x2
(b) 1/x
(c) x
(d) x2

Ans. (a)
41. If c = a/b; a-1 = c, what is the relation between a and b?

Ans. b = a/a-1
42. What is the sum of 7 consecutive odd numbers with 27 as the fourth number
Ans.189

Examples of flow charts asked to be filled :
(1) There are 3 boxes of 3 balls each. you have to select the heaviest among all.
(2) There are red and black balls in a box. You select some balls from the blocks.If the ball chosen is red then you get one point. If the chosen ball is ball black and previous ball is red then you get two points. For winning u have to get seven points. No point for selecting consecutive balls of the same color.
(3)Classify objects in class A, class B and scrap. for classfing you have to do different tests such as weight test, material test etc.
(4)There is production process in which action depends on temperature and pressure and we have some temperature and pressure controls.Draw a flowchart to complete the process.
(5)Find max. and min. of the 12 nos.in an array.Arrang the array in ascending order and find the maximum and
minimum value in the array
(6)Diffrent age group are given and also diffrent salary slabs are given. Depending on the salary group as well as his group you have to classify the group of people in particular class.
******************************
Verbal
ACUMEN
a. exactness
b. potential
c. shrewdness
d. bluntness
e. None of these
BEHEST
a. behavior
b. hold down
c. hold up
d. relieve
e. condemn
DISCRETION
a. prudence
b. consistency
c. precipice
d. disturbance
e. distemper

ORDAIN
a. arrange
b. command
c. contribute
d. establish
e. control

FLORID
a. ornate
b. thriving
c. artistic
d. elegant
e. None of these
PENITENCE
a. liking
b. insightful
c. attractive
d. penetrable
e. compunction

WHET
a. stimulate
b. humorous
c. inculate
d. dampen
e. None of these

INCENTIVE
a. reflex
b. amplitude
c. inflection
d. provocation
e. escutcheon

LATITUDE
a. scope
b. segment
c. globule
d. legislature
e. lamentation
MORTIFY
a. make a cavity
b. displease
c. humiliate
d. relapse
e. murder

ADAGE
a. advice
b. proverb
c. enlargement
d. advantage
e. usage

TO DISPEL
a. to dissipate
b. to dissent
c. to distort
d. to disfigure
e. to dissect
ERRATIC
a. unromantic
b. free
c. popular
d. steady
e. unknown

TO MERIT
a. to embrace
b. to devote
c. to deserve
d. to combine
e. to display

RAPT
a. lively
b. concealed
c. engrossed
d. prototype
e. None of these

TO HEAP
a. to pile
b. to forbid
c. to proceed
d. to share
e. to stoop

CAJOLE
a. coax
b. motivate
c. profound
d. mollify
e. evade
OVULATE
a. penury
b. immunize
c. fertilize
d. reproduce
e. incisions
ABODE
a. clay
b. obstacle
c. dwelling
d. bind
e. to beguile

POTENTIAL
a. latent
b. hysterical
c. conventional
d. symmetrical
e. conscientious

EXTRICATE
a. terminate
b. isolate
c. liberate
d. simplify
e. frustrate

DISPARITY
a. inequality
b. impartiality
c. unfairness
d. twist
e. None of these

TO CONFISCATE
a. to harass
b. to repulse
c. to console
d. to appropriate
e. to congregate
PIOUS
a. historic
b. devout
c. multiple
d. fortunate
e. authoritative

LETHARGY
a. reminiscence
b. category
c. fallacy
d. unanimity
e. stupor

CARGO
a. cabbage
b. camel
c. lance
d. freight
e. flax
OVATION
a. oration
b. gesture
c. emulation
d. applause
e. nourish


Aptitude
1. A family, planning a weekend trip, decides to spend not more than a total of 8 hours driving. By leaving early in the morning, they can average 40 miles per hour on the way to their destination. Due to the heavy Sunday traffic, they can average only 30 miles per hour on the return trip. What is the farthest distance from home they can plan to go?
(a) 120 miles or less (b) Between 120and 140 miles (c) 140 miles
(d) Between 140 and 160 miles (e) 160 miles or more
2. A car is filled with four and half gallons of fuel for a round trip. If the amount of fuel taken while going is 1/4 more than the amount taken for coming, what is the amount of fuel consumed while coming back?
(a) Less than 2 gallons (b) 2 gallons (c) 2 1/2 gallons
(d) 3 gallons (e) More than 3 gallons

3. A 3-gallon mixture contains one part S and two parts R. In order to change it to a mixture containing 25% S, how much R should be added?
(a) 1/2 gallon (b) 2/3 gallon (c) 3/4 gallon (d) 1 gallon (e) 1 1/2 gallon

4. A tree grows only 3/5 as fast as the one beside it. In four years the combined growth of the two trees is eight feet.
How much does the shorter tree grow in two years?
(a) Less than 2 feet (b) 2 feet (c) 2 1/2 feet
(d) 3 feet (e) more than 3 feet.
5. Wind flows at 160 miles in 330 minutes, for traveling 80 miles how much time does it require?
(a) 1 hour 30 minutes (b) 1 hour 45 minutes (c) 2 hours
(d) 2 hours 45 minutes (e) 3 hours

6. A stationary engine has enough fuel to run 12 hours when its tank is 4/5 full. How long will it run when the tank is 1/3 full?
(a) Less than 2 hours (b) 2 hours (c) 3 hours
(d) 4 hours (e) 5 hours
7. If A is traveling at 72 km per hour on a highway. B is traveling at a speed of 25 meters per second on a highway. What is the difference in their speeds in meters per second?
(a) 1/2 m/sec (b) 1 m/sec (c) 1 1/2 m/sec
(d) 2 m/sec (e) 3 m/sec
8. A salesperson by mistake multiplied a number and got the answer as 3, instead of dividing the number by 3. What is the answer he should have actually got?
(a) 0 (b) 1/3 (c) 1 (d) 2 (e) 3
9. If the length of a rectangle is increased by 30% and the width is decreased by 20%, then the area is increased by...
(a) 10% (b) 5% (c) 4% (d) 20% (e) 25%

10. In the class of 40 students, 30 speak Hindi and 20 speak English. What is the lowest possible number of students who speak both the languages?
(a) 5 (b) 20 (c) 15 (d) 10 (e) 30

11. The most economical prices among the following prices is:
(a) 10 kilo for Rs.160 (b) 2 kilo for Rs.30 (c) 4 kilo for Rs.70
(d) 20 kilo for Rs.340 (e) 8 kilo for Rs.130

12. A truck contains 150 small packages, some weighing 1 kg each and some weighing 2 kg each. how many packages weighing 2 kg each are in the truck if the total weight of all the packages is 264 kg?
(a) 36 (b) 52 (c) 88 (d) 124 (e) 114
13. A man was arrested for exceeding the speed limit by 10 miles an hour. A second man was charged with exceeding the same limit by twice as much. The second man was driving 35 miles per hour. What was the speed limit?
(a) 10 miles per hour (b) 15 miles per hour (c) 20 miles per hour
(d) 25 miles per hour (e) 30 miles per hour

14. One year ago Pandit was three times his sister's age. Next year he will be only twice her age. How old will Pandit be after five years?
(a) 8 (b) 12 (c) 11 (d) 13 (e) 15
15. If two pencils cost 8 cents, then how much do 5 pencils cost?
(a) 18 cents (b) 20 cents (c) 22 cents (d) 23 cents (e) 24 cents

Passage
All the questions are of multiple choice type. You have to answer the questions based on the preceding paragraph. All the questions have the same answer choice. The choices are as given below:
(a) True.
(b) False.
(c) Cannot Say.

Consider the following paragraph:
Researchers in Mumbai have found that certain types of gallstones can be dissolved by injecting them with a gasoline additive in the form of ether. The ether is injected through a tube directly into the gall bladder. The one-day treatment works only on cholesterol-based stones, not those composed largely of calcium. However, as the cholesterol stones are by far the most common type, for millions of gallstone sufferers the treatment should offer a welcome alternative to surgery, the commonest option in most hospitals.

# "It takes more than one day for ether to dissolve a calcium-based gallstone".

# "Gallstones can only be dissolved by injections".

# "Gallstones can quickly be cured with surgery".

# "Ether is largely used for dissolving gallstones".
# "Calcium stones can be cured in one day".
# "Hundreds of people contains calcium stones".
Consider the following paragraph:
My father had no brothers, but his three sisters are all married and each has two children. My grandfather has two sons.

# "My father was the only child".

# "I have only one uncle".

# "One of my aunts is a spinster".

# "I have six cousins on my father's side".

# "My grandfather was the only son".
Consider the following paragraph:
In the Totalitarian days, the words have very much devalued. In the present day, they are becoming domestic, that is, the words will be much more devalued. In that days, the words will be very much effected in political area. But at present, the words came very cheap. We can say they come free at cost.

# "In Totalitarian society, words are devalued".

# "Totalitarians will have to come much about words".

# "In the Totalitarian society the words are used for the political speeches".

Consider the following paragraph:
In past helicopters were forced to ground or crash because of the formation of the ice on the rotors and engines. A new electronic device has been developed which can detect the water content in the atmosphere and warns the pilot, if the temperature is below freezing temperature; about the formation of ice on the rotors and wings.

# "The electronic device can avoid the formation of ice on the wings".

# "There will be malfunction of rotor and engine because of formation of ice".

# "The helicopters were to be crashed or grounded".

# "There is only one device that warns about the formation of ice".

Consider the following paragraph:
Human existence is not susceptible of arbitrary division between consciousness and unconsciousness. The conscious world invaders and shapes the activities of the unconscious, while many of the great achievements of humanity's waking hours where wholly or partly inspired by dreams. Even if it could argued that dreams precede experience such a dichotomy could not be drawn, as the influence of the dreaming on the waking state would remain unclear, but as yet no common vocabulary exists to record the substance of prenatal dreaming.

# "Sleep can be a creative state".

Consider the following paragraph:
FLORA 3-piece sofa-set is at the top of our upholstery range. This high-backed quality sofa-set boasts an impressive specification which starts with a hardwood frame in teak and a padded front edge ensuring really deep, long-lasting comfort. Seat cushions are of high resilience foam and back cushions of softest hollow fill. The whole set is carefully upholstered throughout in a choice of superb fabrics ranging from cotton print to velvet.

# "The padding is there to ensure that the furniture will last for a long time".

# "The firm sells other upholstery furniture".
Consider the following paragraph:
Hacking is a crime made possible by a relatively new technology, which is one of the reasons it is often poorly understood and reported. Many computers, but by no means all, are now linked together in networks which allow users on one computer to communicate with others on the same network. If a computer is not networked, no manipulation of its data from another machine possible. So long as users are authorized, networking is just a way of making work easier more productive. Hacking, on the other hand is the unauthorized use of networks or unauthorized entry into the computers themselves. Most people do not break into the networks they use, since they are already accredited users.

# "Hackers do not work for the firms whose networks they break into".
# "Hacking is the only vulnerability of the computers for the usage of the data".
# "Hacking is done mostly due to the lack of computer knowledge".
Consider the following paragraph:
Polycythemia often occurs in people who have chronic lung disease, but can appear spontaneously in healthy individuals. Excessive numbers of red blood cells manufactured by the body and the individual then develops a very healthy-looking, ruddy complexion. The blood becomes thicker and is liable to clot and block major blood vessels. High blood pressure is another frequent complication. Treatment involves venesection, in which a liter or so of blood is removed from the body. Medication may also be given to reduce the numbers of red blood cells manufactured in the body.

# "Lung disease frequently precedes polycythemia".

Consider the following paragraph:
Bindweed is only effectively controlled by applying a solution of brushwood-killer to the growing tips. It is necessary to unwind a suitable length from the host plant before treatment, but this is not so very difficult, and it does not seem essential to find and treat every leader on the same weed. The solution should be made up in a can which is carried in one hand, while the other, in a rubber glove, inserts the leaders in the can. If the the leaders can be laid out on the the ground , they can easily be wetted with a small brush. As long as the weather is calm, there is no real risk of damage to adjacent plants, and in two or three weeks the weeds should have disappeared.

# "Brushwood-killer can pose a threat to other plants in the garden".

Consider the following paragraph:
Senior managers in a leading company said that new Japanese investment in India was transforming the car industry, and warned that jobs were under threat from Japanese competition. They stated that increasing competition would be coupled with an inevitable downturn in the car market and the recent rise interest rates which has already hit demand.

# "The managers issued their warning after a rise in the interest rates".

# "According to the senior managers, the Japanese investment in India will lead to a glut in the car market".

# "Some senior managers said that more people will want to by new cars in the future".

# "The perception of the senior managers is the new Japanese investment in India is leading to more automation of the car industry".

# "The increased rate of interest will mean that Japanese firms will cease to operate in this country".

# "The increase in loan interest will adversely affect car sales".
# "Japanese workers are taking over the jobs of Indian industry".
# "Managers said that interests in car will go down after seeing the raise in interest rates".
# "People are very interested to buy the cars".
Consider the following paragraph:
The new Starfire has an advanced four-cylinder engine with catalytic converter and uses only unleaded petrol. Versatility is a major feature of the range and the 1500 and 1800 models have the same high level of specification inside and out. The only obvious visual difference, internally and externally, is the use of alloy wheels on the 1800 version, together with a discreet change in badging. The StarFire 2000 is distinguished by its tailgate spoiler and the rectangular fog and driving lamps integrated into the front bumper which are also included in the specification.

# "Internally, the Starfire 2000 looks like the 1500 model".

Consider the following paragraph:
Pierre Claude Jean Allouez explored lake superior from 1665 to 1667. At his little mission station near the western end of the lake, he heard from the Indians of a great river to the west. Pierre Jacques Marquette determined to investigate. In 1673, accompanied by Louis Jolliet and five others, he left St. Ignace mission and ascended the fox river, which flows into green bay crossed over to Wisconsin river and followed it to the upper Mississippi. The party then descended the Mississippi to the mouth of Arkansas. These Frenchmen were not first Europeans to sight or travel the Mississippi. De Soto and Moscoso had done so a century and a half before. The report of the exploration was rushed back to Quebec, where, in 1672, Count Frontenac had arrived as Governor of the province. He and his friend, the remarkable La Salle-who earlier may have penetrated the Ohio river valley-listened with deep interest.

# "Allouez explored the western end of lake superior".

# "Marquette and his party were not the first French men to travel the Mississippi river".

# "La Salle listened with deep interest- the report of exploration of De Soto and Moscoso".

# "La Salle explored the Mississippi river valley".
Consider the following paragraph:
Dr. Goddard was the first to fire a rocket that reached a speed faster than the speed of sound. He was the first to develop a gyroscopic steering apparatus for rockets. He was the first to use vanes in the jet stream for rocket stabilization during the initial phase of a rocket flight. And he was the first to patent the idea of step rockets. After proving on paper and in actual tests that a rocket can travel in vacuum, he developed the mathematical theory of rocket propulsion and rocket flight, including basic designs for long-range rockets. All of this information was available to our military men before World War II, but evidently its immediate use didn't seem applicable. Near the end of World War II we started intense work on rocket-powered guided missiles, using the experiments and developments of Dr. Goddard and the American Rocket Society.

# "The stabilization problem of rockets in the initial phase was solved by Dr. Goddard."

# "Rockets can travel faster than sound, thanks to gyroscopic steering."

# "Goddard lived before World War II".

# "After careful mathematical calculations, Dr. Goddard proved that rockets can travel in vacuum".

Consider the following paragraph:
In March 1513, de Leon sailed off confidently from Peuto Rico for the Bahamas. Landing briefly at San Salvador, Bahamas, he wound through unchartered islands until he sighted an extensive coastline. He had no reason to suspect that it is anything more than an island, but he followed the coast for a day without rounding its end or finding a suitable landing place. He named the "island" La Florida. This name came to be applied by the Spanish to the entire Southeastern United States and beyond. Then, near the 30th parallel, de Leon landed at the mouth of the St. Johns river. Determined to be the first to circumnavigate the "island", he turned south, traced the coast around the tip of the peninsula, moved to the west, perhaps reaching Tampa bay. After 7 weeks, he gave up hope of circling the northern tip of this "island"; it was incredibly large and he may have suspected that he had discovered the long sought mainland. If so, it all belonged to his King, for he had earlier planted the Spanish flag and claimed Florida for Ferdinand.

# "de Leon is from Spain, ruled by Ferdinand".

# "de Leon is very patriotic".

# "de Leon discovered part of US during his journey".

Consider the following paragraph:
James Madison understood that interests groups will inevitably develop within a free political system. The problem, as Madison saw it, was to prevent any single interest group from becoming so strong that it was able to dominate the political system. This could be accomplished by legislating restrictions on political behavior, but that solution meant a sacrifice of some of the freedom that Madison prized so highly. A better solution, he thought, was to extend the territorial scope of the government. This would allow for greater diversity of interests in the nation, and a greater number of groups competing for power. Each interest group would thereby find it more difficult to appeal to a majority of the people, and to dominate the political process.

# "The more interest groups there are in a political system, the less freedom there is for everyone."

# "Legislating restrictions on political behavior is sometimes the only method of preserving political freedom."

# "Increasing the territorial scope of a government can help to preserve freedom."

# "According to Madison, in a free political system, interest groups are undesirable."

Consider the following paragraph:
The regulations and expenses to invent, patent and market new ideas and products imposes a heavy burden on inventors. The cost is often absorbed by large corporations with research and development facilities they provide. Corporations also help creative people contribute to society without suffering the loss of income or security of the private inventor. The realities of this arrangement are that many good ideas are never brought into the marketplace and the cost of products on the market is high because of the development cost. However, protection provided by the patents and the safety to the public to avoid placing harmful products on the market is important to maintain. Thus, as is often the case, rules and regulations have their favorable and unfavorable consequences.

# "The regulations and expenses to invent, patent and market new ideas is an expensive proposition to the inventors".

# "Good ideas are never brought into the marketplace because of the costs involved in inventing, patenting marketing them".

# "Corporations steal the individual inventor of their inventions".
Consider the following paragraph:
Being born female and black were two handicaps Gwendolyn Brooks states that she faced from her birth, in 1917, in Kansas. Brooks was determined to succeed. Despite the lack of encouragement she received from her teachers and others, she was determined to write and found the first publisher for one of her poems when she was 11. In 1945, she marketed and sold her first book; national recognition ensued. She applied for and received grants and fellowships from such organizations as the AAAL and the Guggenheim Foundation. Later she received the Pulitzer prize for poetry; she was the first black woman to receive such an honor. Brooks' reaction to fame is atypical. She continues to work and work hard. She writes, travels, and helps many who are interested in writing. Especially important for her is increasing her knowledge of her black heritage and encouraging other people to do the same. She encourages dedication to the art to would-be writers.

# "Brooks' story illustrates the power of strong determination".

# "She became the author of a book in her teens".

# "Gwendolyn received the Pulitzer prize for her first poetry".

Consider the following paragraph:
A cave is a natural opening in the ground extending beyond the zone of light and large enough to permit the entry of man. Occurring in a wide variety of rock types and caused by widely differing geological processes, caves range in size from single small rooms to interconnecting passages many miles long. The scientific study of caves is called speleology. It is a composite science based on geology, hydrology, biology and archeology, and thus holds special interest for earth scientists. Caves have been natural attractions since prehistoric times. Prolific evidence of early man's interest has been discovered in caves scattered throughout the world. Skeletons of some of the earliest manlike creatures (Australopithecines) have been discovered in cave deposit in South Africa, and the first evidence of primitive Neanderthal man was found in Germany. Cro-Magnon man created his remarkable murals on the walls of caves in France.

# "Primitive human form originated in Germany".

# "Study of caves is the study of earth, water, life and early man".

# "Cro-Magnon man was more intelligent than Neanderthal man".

# "Caves are a natural attraction because they reveal information about the early man".

Consider the following paragraph:
Although invaders represent the threat to the conservation of flora and fauna, there are two special cases in which invasion have been deliberately brought about. One is the desire to control pests by natural predators, which may have to be brought from other countries. The second is releasing organisms into the wild (or on to farms, from which they might escape) that are completely novel, because they have been genetically engineered. There is nothing intrinsically sinister about engineered organisms, but any novelty must be regarded as potential invader.

# "Pests are more dangerous than their natural predators".

Consider the following paragraph:
Life in colonial times was harsh, and the refinements of the mother country were ordinarily lacking. The colonists, however, soon began to mold their English culture into the fresh environment of new land. The influence of religion permeated the entire way of life. In most Southern colonies, the Anglican church was the legally established church. In New England, the Puritans were dominant; and in Pennsylvania, the Quakers. Especially in the New England colonies, the local or village church was the hub of community life; the authorities strictly enforced the Sabbath and sometimes banished non-believers. Unfortunately, the same sort of religious intolerance, bigotry and superstition associated with the age of Reformation in Europe also prevailed in some of the colonies, though on a lesser scale. In the last half of the 17th century, during sporadic outbreak of religious fanaticism and hysteria, Connecticut authorities tried and hanged several women as "witches". Early in the 17th century, some other witchcraft persecution occurred in Virginia. As the decades passed, however, religious tolerance developed in colonies.

# "New England was part of the Southern colonies".

# "During the mid 17th century there was significant improvement in religious tolerance and superstition".

# "Life in colonial times was harsh due to the strong influence of religion".

# "The Anglican church used to govern the people in most Southern colonies".
Consider the following paragraph:
Confucius said that to know the future we have to understand the past. in his time, transport, communications and scientific knowledge were less developed than they are today. news took weeks to travel whereas today satellite links connect the continents virtually instantaneously, but our technological advances in the field of communications seem not to improved our capacity to understand one another.

# "We understand each other better now than in Confucius' time because we can travel more quickly".

# "In Confucius' day people were more intelligent".

# "We have made great improvements in transport since Confucius' day".

# "Technological advances in communication and human capacity to understand one another are directly proportional".

# "In Confucius' day time news took months to travel".

# "According to Confucius the past has a linkage to the future".
# "Even with the fast developments of the technology we can't live happily".
Consider the following paragraph:
Every form of art is protected by copyright, upon the expiration of which the property passes to the public domain and becomes freely available to anyone wishing to exploit it commercially. the time has come when all treasures should pass to the control of a trust, and by this be made readily available to anyone on payment of a fee or royalty. The income from the works Van Gogh would alone be enormous. Those who now gain financial benefit from his genius should make some contribution to the welfare of the arts in general.

# "Instead of buying a ticket, museum goers should pay a fee to a trust for the benefit of arts".

# "It is not desirable to pass the control of treasures to a trust".

# "Van Gogh's paintings are not protected by copyright".

# "All artworks must be managed by a trust, so that the income generated can be used for the welfare of the arts".

# "Copyright in art is valid only for a limited period of time".

# "Van Gogh's descendants should be asked to make some contribution to the arts".
# "Van Gogh's works are under this copy right rule".
# "People are free to go to the public because of the copy right rule".
# "People gives to theater and collect the money for development".
# "We have asked the Van Gogh descendants to help for the developments of art".
Consider the following paragraph:
Organizing the home can be perceived as conferring power, so large numbers of women are unwilling to let go of chores, even when they have careers. A survey found that, out of 65 new marriages, not one single wife expected her husband to share work equally. According to the Family Policy Studies Center, 81% of working wives return home to do all the cooking. The average male has nearly half as much as more free time at weekends than his wife, and the typical new father spends just 37 seconds a day talking to his baby.

# "Only career women perceive organizing the home as conferring power".

# "The average wife has half as much free time at weekends as her husband".

# "The family planning studies center shows that 81% working wives do all the cooking at home".

# "19% working wives do not want to do the cooking at home".
# "Housewives want the husbands to take part equally in the household".
# "Wives have half as much leisure time as the husbands have".
# "39% of the men will work equally in the house in cleaning and washing".
Consider the following paragraph:
Statistics show that millions of vehicles have been carried by shuttle over the past 30 years through Alpine tunnels without one ever catching fire. In the Alpine tunnels, drivers and passengers sit in their vehicles on the shuttle trains. Only one vehicle has ever caught fire on the busy French motorail equivalent system. This sort of accident is not possible in a closed shuttle. Assertion that a vehicle fire will lead to catastrophe have no basis. Since the resources exist to detect, control and extinguish a fire, and to remove any persons present safely to an adjoining wagon, leaving any surviving fire facing rapid extinction within a wagon built to contain fire for 30 minutes, catastrophe seems very unlikely.

# " It is theoretically possible for a vehicle to catch fire even in a closed wagon".

# " The French motorail system is inferior to the shuttle train system."
# "No accident can occur in the closed tunnels".
# "Fire is allowed to live for 30 min".
# "All the cars that travel in the tunnels will be carried by rail shutters".
PHYSCOMETRY TEST
Direction:
In this section you will find different questions with the same meaning. In all such questions your answer has to be same. for e.g.:
In being thrown by chance with a stranger, you wait for the person to introduce himself or herself.
(a) Yes (b) No (c) ?

It is difficult for you to chat about things in general with people.
(a) Yes (b) No (c) ?

These two questions have similar meanings. If you answer the first one 'NO' and the second one 'YES', i.e. if you differ in your answers to similar questions you lose marks for every question with the above meaning.

The choices to these questions are:
(a) Yes.
(b) No.
(c) ?

1. You start to work on a project with great deal of enthusiasm.
2. You would rather plan an activity than take part in it.
3. You have more than once taken lead in organizing project or a group of some kind.
4. You like to entertain guests.
5. Your interests change quickly from one thing to another.
6. When you eat a meal with others, you are usually one of the last to finish.
7. You believe in the idea that we should " eat, drink and be merry, for tomorrow we die."
8. When you find that something you have bought is defective, you hesitate to demand an exchange or a
refund.
9. You find it easy to find new acquaintances.
10. You are sometimes bubbling over with energy and sometimes very sluggish.
11. You are happiest when you get involved in some projects that calls for rapid action.
12. Other people think of you as being very serious minded.
13. In being thrown by chance with a stranger, you wait for the person to introduce himself or herself.
14. You like to take part in many social activities.
15. You sometimes feel "just miserable" for no good reason at all.
16. You are often so much " on the go" that sooner or later you may wear yourself out.
17. You like parties you attend to be lively.
18. If you hold an opinion that is radically different that expressed by a lecturer, you are likely to tell the
person about it either during or after the lecture.
19. It is difficult for you to chat about things in general with people.
20. You give little thought to your failures after they are passed.
21. You often wonder where others get all the excess energy they seem to have.
22. You are inclined to stop to think things over before you act.
23. You avoid arguing over a price with a clerk or sales person.
24. You would dislike very much to work alone in some alone place.
25. You often find it difficult to go to sleep at night because you keep thinking of what happened during the
day.
26. You find yourself hurrying to get to places even when there is plenty of time.
27. You like work that requires considerable attention to details.
28. You are satisfied to let some one else take the lead in group activities.
29. You enjoy getting acquainted with people.
30. It takes a lot to get you emotionally stirred up or excited.
31. You work more slowly and deliberately than most people of your sex and age.
32. You are a carefree individual.
33. When people do not play fair you hesitate to say anything about it to them.
34. It bothers you to have people watch you at your work.
35. You have usually been optimistic about your future.
36. You like to have plenty of time to stop and rest.
37. You take life very seriously.
38. You enjoy applying for a job in person.
39. You would like to be a host or hostess for parties at club.
40. You often feel uncomfortable or uneasy.
41. You are the kind of person who is "on the go" all the time.
42. You often crave excitement.
43. The thought of making a speech frightens you.
44. You find it easy to start conversation with strangers.
45. You often feel guilty without a very good reason for it.
46. People think you are a very energetic person.
47. You sometimes make quick decisions that you later wish you hadn't made.
48. You find it difficult to ask people for money or other donations, even for a cause in which you are
interested.
49. You are so naturally friendly that people immediately feel at ease with you.
50. You daydream a great deal.
51. You are quick in your actions.
52. You have a habit of starting things and then losing interest in them.
53. When you were a child many of your playmates naturally expected you to be the leader.
54. You sometimes avoid social contacts for fear of doing or saying the wrong thing.
55. You have frequent ups and downs in mood, sometimes with and sometimes without apparent cause.
56. You always seem to have plenty of vigour and vitality.
57. It is difficult for you to understand people who get very concerned about things.
58. When a clerk in a store waits on others who come after you, you call his or her attention to the fact.
59. You would be very unhappy if you were prevented from making numerous social contacts.
60. There are times when your future looks very dark.
61. You sometimes wish that people would slow down a bit and give you a chance to catch up.
62. Many of your friends think you take your work too seriously.
63. You hesitate to walk into a meeting when you know that everyone's eye will be upon you.
64. You limit your friendships mostly to members of your own sex.
65. You almost always feel well and strong.
66. You seem to lack the drive necessary to get as much as other people do.
67. You make decisions on the spur of the moment.
68. You are rather good at bluffing when you find yourself in difficulty.
69. After being introduced to someone , you just cannot think of things to say to make good conversation.
70. You feel lonesome even when with other people.
71. You are able to work for unusually long hours without feeling tired.
72. You often act on the first thought that comes into your head.
73. At the scene of an accident, you take an active part in helping out.
74. You have difficulty in making new friends.
75. Your mood often changes from happiness to sadness or vice versa without knowing why.
76. You talk more slowly than most people.
77. You like to play practical jokes upon others.
78. You take the lead in putting life into a dull party.
79. You would like to belong to as many clubs and social organizations as possible.
80. There are times when your mind seems to work very slowly and other times when it works very rapidly.
81. You like to do things slowly and deliberately.
82. You are a happy-go-lucky individual.
83. When you are served stale or inferior food in a restaurant, you say nothing about it.
84. You would rather apply for a job by writing a letter than by going through with a personal interview.
85. You are often in low spirits.
86. You are inclined to rush from one activity to another without pausing enough for rest.
87. You are so concerned about the future that you do not get as much fun out of the present as you might.
88. When you are attracted to a person whom you have not met earlier you make an active attempt to get
acquainted even though it may be quite difficult.
89. You are inclined to limit your acquaintances to select few
90. you seldom give your past mistakes a second thought.
91. You are less energetic than many people you know.
92. You often stop to analyzed your thoughts and feelings.
93. You speak out in meetings to oppose those whom you feel sure are wrong.
94. You are so shy it bothers you.
95. You are sometimes bothered by having a useless thought come into your mind over and over.
96. You get things in hurry.
97. It is difficult for you to understand how some people can be so unconcerned about the future.
98. You lie to sell things (i.e. to act as a sales person)
99. You are often "Life of the Party".
100. You find daydreaming very enjoyable.
101. At work or at play other people find it hard to keep up with the pace you set.
102. You can listen to a lecture without feeling restless.
103. You would rather work for a good boss than for yourself.
104. You can express yourself more easily in speech than in writing.
105. You keep in fairly uniform spirits.
106. You dislike to be hurried in your work.
107. You sometimes find yourself "crossing bridges before you come to them".
108. You find it somewhat difficult to say "no" to a sales person who tries to sell you something you do not
really want.
109. There are only a few friends with whom you can relax and have a good time.
110. You usually keep cheerful in spite of trouble.
111. People sometimes tell you to "slow down" or "take it easy".
112. You are one of those who drink or smoke more than they know they should.
113. When you think you recognize people you see in a public place, you ask them whether you have met
them before.
114. You prefer to work alone.
115. Disappointment affect you so little that you seldom think about them twice.
116. You are slow and deliberate in movements.
117. You like wild enthusiasm, sometimes to a point bordering on rowdyism at a football or baseball game.
118. You feel self conscious in the presence of important people.
119. People think of you as being a very social type of person.
120. You have often lost sleep over your worries.
121. You can turn out a large amount of work in a short time.
122. You keep at a task until it is done, even after nearly everyone else has given up.
123. You can think of a good excuse when you need one.
124. Other people say that it is difficult to get to know you well.
125. You daydreams are often about things that can never come true.
126. You often run upstairs taking two steps at a time.
127. You seldom let your responsibility interfere with your having a good time.
128. You like to take on important responsibilities such as organizing a new business.
129. You have hesitated to make or to accept "dates" because of shyness.
130. Your mood is very easily influenced by people around you.
131. Others are often amazed by the amount of work you turn out.
132. You generally feel as though you haven't a care in the world.
133. You find it difficult to get rid of sales person whom you do not care to listen or give your time.
134. You are a listener rather than a talker in a social conversation.
135. You almost always feel that life is very much worth living.
136. It irritates you to have to wait at a crossing for a long freight train to pass.
137. You usually say what you feel like saying at the moment.
138. You like to speak in public.
139. You like to be with people.
140. You generally keep cool and think clearly in exciting situations.
141. Other people regard you as a lively individual.
142. When you get angry, if you let yourself go, you feel better.
143. You seek to avoid all trouble with other people.
144. People seem to enjoy being with you.
145. You sometimes feel listless and tired for no good reason.
146. It is hard to understand why many people are so slow and get so little done.
147. You are fond of betting on horse races and games, whether you can afford it or not.
148. If someone you know has been spreading untrue and bad stories about you, you see the person as
soon as possible and have a talk about it.
149. Shyness keep you from being as popular as you should be.
150. You are generally free from worry about possible misfortunes.
*****************************
SYNONYMS

CENSURE
a. purify
b. approve
c. edit
d. uncertain
ONUS
a. honest
b. inclination
c. responsibility
d. accuse

DIVERGENT
a.
b.
c. deviating
d.

FIDELITY
a. restlessness
b. disloyalty
c. feeble
d. vagueness

SURVEILLANCE
a. inattention
b. visibility
c. census
d. prevention
WHIMSICAL
a. victorious
b. swift
c. fanciful
d. momentary

NASCENT
a. threat
b. purpose
c. quality
d. emerging

BENIGN
a. kindly
b. malignant
c. envy
d. tenfold
ANALOGOUS
a. capable
b. culpable
c. comparable
d. corporeal

ATTENUATE
a. appear
b. weaken
c. testify
d. soothe
EFFUSIVE
a. wise
b. reserved
c. peaceful
d. spontaneous
GREGARIOUS
a. logical
b. helpful
c. solitary
d. noisy

EMPIRICAL
a. theoretical
b. mathematical
c. verbal
d. royal
CITE
a. galvanize
b. quote
c. locate
d. visualize
TENACIOUS
a. intentional
b. obnoxious
c. holding fast
d. collecting

TRANSIENT
a. ephemeral
b. permanent
c. clear
d. emptiness

VOLUBLE
a. worthwhile
b. loquacious
c. circular
d. serious
CANDID
a. vague
b. outspoken
c. experienced
d. anxious
VERACITY
a.
b.
c. truthfullness
d.
STANDING
a. reputation
b. activity
c. long time
d. duration
SACROSANCT
a. too important
b. worship
c. sacrifice
d. best

CLUTCH
a.
b.
c.
d.

AUGMENT
a. decrease
b. belittle
c. simplify
d. magnify

GENERIC
a. external
b. particular
c. personal
d. subdued
DIVULGE
a. look
b. refuse
c. deride
d. reveal
EFFIGY
a. dummy
b. organ
c. charge
d. accordion

PRECARIOUS
a. hazardous
b. priceless
c. premature
d. primitive

SONOROUS
a. reassuring
b. resonant
c. repetitive
d. sisterly
CIRCUITOUS
a. indirect
b. complete
c. obvious
d. aware
PEDIGREE
a. dogs
b. vast
c. courage
d. line of ancestry

ANTONYMS:
EXPEDIENT
a. illiterate
b. delayed
c. mistake
d. impediment
IRRADIATE
a. agreement
b. distance
c. flight
d. clarity

ANOMALY
a. desperation
b. requisition
c. registry
d. regularity

BENIGN
a. peaceful
b. blessed
c. wavering
d. malignant

ANALOGUE
a. same
b. digital
c. lengthy
d. dull

ANALOGOUS
a. not comparable
b. not capable
c. not culpable
d. not congenial

CENSURE
a. process
b. enclose
c. praise
d. penetrate
DIVULGE
a. converge
b. intake
c. involve
d. conceal
SURVEILLANCE
a. inattention
b. visibility
c. census
d. prevention
HAMPER
a.
b.
c.
d.
DANGLE
a. hanging
b. loose
c. secure
d. mingle

SPENDTHRIFT
a. miser
b. savings
c. cautious
d. extravagant
INDIGENOUS
a.
b.
c.
d.
CRYPTIC
a. futile
b. famous
c. candid
d. indifferent
OPTIMUM
a. pessimistic
b. minimum
c. chosen
d. worst

RETROGRADE
a. progressing
b. inclining
c. evaluating
d. concentrating

TRANSIENT
a. carried
b. close
c. permanent
d. certain

VERITY
a. falsehood
b. sanctity
c. rarity
d. household

CENSURE
a. augment
b. eradicate
c. enthrall
d. commend
EMPIRICAL
a. theoretical
b. mathematical
c. verbal
d. royal
AUGMENT
a. keep away
b. be disturbed
c. to increase
d. dig out

BOLSTER
a. defeat
b. to strengthen
c. be angry
d. depth
COMPLIANCE
a. light
b. fresh
c. take away
d. energize
DEBILITATE
a. balmy
b. bedevil
c. animate
d. deaden
DEROGATORY
a. roguish
b. immediate
c. conferred
d. praising

ERRONEOUS
a. accurate
b. dignified
c. curious
d. abrupt

EXONERATE
a. forge
b. accuse
c. record
d. reimburse

GREGARIOUS
a. anticipating
b. glorious
c. antisocial
d. similar

OBJECTIVE
a. indecisive
b. apathetic
c. emotionally involved
d. authoritative
******************************

1. If VXUPLVH is written as SURMISE, what is SHDVD ?
Ans. PEASA (hint: in the first word, the alphabets of the jumbled one is three alphabets after the corresponding alphabet in the word SURMISE. S = V-3, similarly find the one for SHDVD)
2. If DDMUQZM is coded as CENTRAL then RBDJK can be coded as ---------
Ans. QCEIL (hint: Write both the jumbled and the coded word as a table, find the relation between the corresponding words, i.e C= D-1, N=M+1 & so on

3. In the word ECONOMETRICS, if the first and second , third and forth ,forth and fifth, fifth and sixth words are interchanged up to the last letter, what would be the tenth letter from right?
Ans. word is CENOMOTEIRSC tenth word is R
4. Find the result of the following expression if, M denotes modulus operation, R denotes round-off, T denotes truncation: M(373,5)+R(3.4)+T(7.7)+R(5.8)
Ans. 19

5. What is the largest prime number that can be stored in an 8-bit memory?
Ans.

6. Find the physical quantity in units from the equation: (Force*Distance)/(Velocity*Velocity)
Ans. Ns2/m
7. Find the value of @@+25-++@16, where @ denotes "square" and + denotes "square root".
Ans: 621
8. If f(0)=1 and f(n)= f(n-1)*n, find the value of f(4).
Ans: 24
9. Convert the decimal number 310 to the base 6.
Ans: 1234
10. Find the missing number in the series: 2, 5, __ , 19 , 37, 75
Ans: 9
11. In a two-dimensional array, X(9,7), with each element occupying 4 bytes of memory, with the address of the first element X(1,1) is 3000, find the address of X(8,5).
Ans.
12. Find the fourth row, having the bit pattern as an integer in an 8-bit computer, and express the answer in its decimal value.
A 0 0 0 0 1 1 1 1
B 0 0 1 1 0 0 1 1
C 0 1 0 1 0 1 0 1
(AU(B-C)) ?
Ans. 29

13. Complete the series 2, 7, 24, 77,__ (hint: 2*12= 24, 7*11= 77, therefore 24*10= 240)
Ans: 240
14. Consider the following diagram for answering the following questions:
INCLUDEPICTURE "http://freshersworld.com/questions/circles.gif" \* MERGEFORMATINET

A. Find the difference between people playing cricket and tennis alone.
Ans: 4
B. Find the percentage of people playing hockey to that playing both hockey and cricket.
Ans:
C. Find the percentage of people playing all the games to the total number of players.
Ans: 6%
15. One more question of the same type (Same type of diagram; of course in a different set)

1. How many more or less speak English than French?

2. What % people speak all the three languages?

3. What % people speak German but not English?

{In another set cricket, hockey and tennis are changed with the name of some computer languages, such as Java, Cobol, Fortran (may be some other name)}
16. Select the odd one out
a. Oracle b. Linux c. Ingress d. DB2

17. Select the odd one out
a. SMTP b. WAP c. SAP d. ARP

18. Select the odd man out.
a. Java b. Lisp c. Smalltalk d. Eiffel

19. Which of the following are orthogonal pairs?
a. 3i+2j b. i+j c. 2i-3j d. -7i+j

20. Number of faces, vertices and edges of a cube
a. 12,8,6 b. 4,6,8 c. 6,8,12 d. 6,12,8
21. Given a Bar Chart showing the sales of a company. (In Figure) The sales in years as shown in the figure are (in crores) 1998-1999 - 130, 1997-1998 - 90, 1996-1997 - 90, 1995-1996 - 70

1. The highest growth rate was for the year
Ans. 1998-1999

2. The net increase in sales of the company in the year span of 1995-1999
Ans. 60 crores.

3. The lowest growth rate was for the year
Ans. 1997

22. Find the value of the decimal number to the base 7.
Ans. 1436.

23. Complete the series:5,6,7,8,10,11,14,__.
Ans. 15

24. If the vertex (5,7) is placed in the memory. First vertex (1,1) ‘s address is 1245 and then address of (5,7) is ----------
Ans.

25. In which of the system, decimal number 384 is equal to 1234?
Ans.

26. A man, a woman, and a child can do a piece of work in 6 days. Man only can do it in 24 days. Woman can do it in 16 days and in how many days child can do the same work?
Ans.

27. In Madras, temperature at noon varies according to -t^2/2 + 8t + 3, where t is elapsed time. Find how much temperature more or less in 4pm to 9pm.
Ans.

28. The size of the bucket is N kb. The bucket fills at the rate of 0.1 kb per millisecond. A programmer sends a program to receiver. There it waits for 10 milliseconds. And response will be back to programmer in 20 milliseconds. How much time the program takes to get a response back to the programmer, after it is sent?
Ans.

29. The size of a program is N. And the memory occupied by the program is given by M = square root of 100N. If the size of the program is increased by 1% then how much memory now occupied ?
Ans.

30. A power unit is there by the bank of the river of 750 meters width. A cable is made from power unit to power a plant opposite to that of the river and 1500mts away from the power unit. The cost of the cable below water is Rs. 15/- per meter and cost of cable on the bank is Rs.12/- per meter. Find the total of laying the cable.
Ans. Rs. 22,500 (hint: the plant is on the other side of the plant i.e. it is not on the same side as the river)

{There are two questions, both showing a curve. In the first one, you have to identify the curve. In the second one you have to Write the equation of the curve. In }
SECTION II

1. If A can copy 50 pages in 10 hours and A and B together can copy 70 pages in 10 hours, how much time does B takes to copy 26 pages?
a. b. c. d.
2. Match the following:
1. Male - Boy ---> a. A type of
2. Square - Polygon ---> b. A part of
3. Roof - Building ---> c. Not a type of
4. Mushroom - Vegetables ---> d. A superset of
Ans: 1- d, 2- a, 3- b, 4- c
3. Match the following.
1. brother – sister ---> a. Part of
2. Alsatian – dog ---> b. Sibling
3. sentence – paragraph ---> c. Type of
4. car - steering ---> d. Not a type of
Ans. 1-b, 2-c, 3-a, 4-d

Questions 20- 24 are based on the following passage:
The office staff of the XYZ corporation presently consists of three bookkeepers (A, B and C) and five secretaries (D, E, F, G and H). Management is planning to open a new office in another city using three secretaries and two bookkeepers of the current staff. To do so they plan to separate certain individuals who do not function well together. The following guidelines were established to set up the new office:
I. Bookkeepers A and C are constantly finding fault with one another and should not be sent as a team to the new office.
II. C and E function well alone but not as a team. They should be separated.
III. D and G have not been on speaking terms for many months. They should not go together.
IV. Since D and F have been competing for promotion, they should not be a team.
Ans.

4. If A is to be moved as one of the bookkeepers, which of the following cannot be a possible working team?
(a) ABDEH (b) ABDGH (c) ABEFH (d) ABEGH (e) ABFGH

5. If C and F are moved to the new office, how many combinations are possible?
(a) 1 (b) 2 (c) 3 (d) 4 (e) 5

6. If C is sent to the new office, which member of the staff cannot go with C?
(a) B (b) D (c) F (d) G (e) H

7. Under the guidelines developed, which of the following must go to the new office?
(a) B (b) D (c) E (d) G (e) H

8. If D goes to the new office which of the following is (are) true?
I. C cannot go.
II. A cannot go.
III. H must also go.
a. I only. b. II only. c. I and II only. d. I and III only. e. I, II and III.
9. Two stations A & B are 110 km apart. One train starts from A at 7 am, and travels towards B at 20kmph. Another train starts from B at 8 am and travels towards A at 25kmph. At what time will they meet?
a. 9 am b. 10 am c. 11 am d. 10.30 am

10. If a man can swim downstream at 6kmph and upstream at 2kmph, his speed in still water is:
a. 4kmph b. 2kmph c. 3kmph d. 2.5kmph

Answer the following three questions based on the paragraph:
A student applying at a college should take three courses. There are altogether four courses, namely, Science, Maths, Social Sudies and Economics. (The names may vary, but the pattern is the same)
One can take a Science course onty if he has taken a Maths course.
One can take a Maths course only if he has taken a Science course.
One can take a n Economics course only if he has taken a Social Studies course.

11. Which of the following is a possible course?
a. Two Science courses and a Social Studies Course.
b. Two Maths courses and an Economics course.
c. One Mats course, one Science course and a Social Studies course.
d. One Maths course, one Science course and an Economics course.

12. Which of the following courses a student can take?
I. One Science, one Social Studies, one Economics
II. Two Science, one Social Studies
III. Two Science, one Maths.
a. I only b. III only c. I and II only d. II and III only

13. Which of the following is not a possible course?
a. Two Science courses and a Maths course. b. Two Maths course and a Science course.
c. One Maths course, one Science course and a Social Studies Course.
d. One Maths course, one Science course and an Economics course.
******************************
Q1. Mr. Shah decided to walk down the escalator of a tube station. He found
that if he walks down 26 steps, he requires 30 seconds to reach the bottom.
However, if he steps down 34 stairs he would only require 18 seconds to
get to the bottom. If the time is measured from the moment the top step begins
to descend to the time he steps off the last step at the bottom, find out
the height of the stair way in steps?
Ans.46 steps.
Q2. The average age of 10 members of a committee is the same as it was 4
years ago, because an old member has been replaced by a young member. Find
how much younger is the new member ?
Ans.40 years.
Q3. Three containers A, B and C have volumes a, b, and c respectively; and
container A is full of water while the other two are empty. If from container
A water is poured into container B which becomes 1/3 full, and into
container C which becomes 1/2 full, how much water is left in container A?
Q4. ABCE is an isosceles trapezoid and ACDE is a rectangle. AB = 10 and EC = 20.
What is the length of AE?
Ans. AE = 10.
Q5. In the given figure, PA and PB are tangents to the circle at A and B respectively and
the chord BC is parallel to tangent PA. If AC = 6 cm, and length of the tangent AP
is 9 cm, then what is the length of the chord BC?
Ans. BC = 4 cm.

Q6. Three cards are drawn at random from an ordinary pack of cards. Find
the probability that they will consist of a king, a queen and an ace.
Ans. 64/2210.
Q7. A number of cats got together and decided to kill between them 999919
mice. Every cat killed an equal number of mice. Each cat killed more mice
than there were cats. How many cats do you think there were ?
Ans. 991.
Q8. If Log2 x - 5 Log x + 6 = 0, then what would the value / values of x
be?
Ans. x = e2 or e3.
Q9. The square of a two digit number is divided by half the number. After
36 is added to the quotient, this sum is then divided by 2. The digits of the
resulting number are the same as those in the original number, but they
are in reverse order. The ten's place of the original number is equal to twice
the difference between its digits. What is the number?
Ans. 46
Q10. Can you tender a one rupee note in such a manner that there shall be
total 50 coins but none of them would be 2 paise coins.?
Ans. 45 one paisa coins, 2 five paise coins, 2 ten paise coins, and 1 twenty-five paise coins.
Q11. A monkey starts climbing up a tree 20ft. tall. Each hour, it hops 3ft.
and slips back 2ft. How much time would it take the monkey to reach the
top?
Ans.18 hours.
Q12. What is the missing number in this series?
8 2 14 6 11 ? 14 6 18 12
Ans. 9
Q13. A certain type of mixture is prepared by mixing brand A at Rs.9 a kg.
with brand B at Rs.4 a kg. If the mixture is worth Rs.7 a kg., how many
kgs. of brand A are needed to make 40kgs. of the mixture?
Ans. Brand A needed is 24kgs.

Q14. A wizard named Nepo says "I am only three times my son's age. My father
is 40 years more than twice my age. Together the three of us are a mere 1240
years old." How old is Nepo?
Ans. 360 years old.
Q15. One dog tells the other that there are two dogs in front of me. The
other one also shouts that he too had two behind him. How many are they?
Ans. Three.
Q16. A man ate 100 bananas in five days, each day eating 6 more than the
previous day. How many bananas did he eat on the first day?
Ans. Eight.
Q17. If it takes five minutes to boil one egg, how long will it take to boil
four eggs?
Ans. Five minutes.
Q18. The minute hand of a clock overtakes the hour hand at intervals of 64
minutes of correct time. How much a day does the clock gain or lose?
Ans.32 8/11 minutes.
Q19. Solve for x and y:
1/x - 1/y = 1/3, 1/x2 + 1/y2 = 5/9.
Ans. x = 3/2 or -3 and y = 3 or -3/2.
Q20. Daal is now being sold at Rs. 20 a kg. During last month its rate was
Rs. 16 per kg. By how much percent should a family reduce its consumption so
as to keep the expenditure fixed?
Ans. 20 %.
Q21. Find the least value of 3x + 4y if x2y3 = 6.
Ans. 10.
Q23. Can you find out what day of the week was January 12, 1979?
Ans. Friday.
Q24. A garrison of 3300 men has provisions for 32 days, when given at a rate
of 850 grams per head. At the end of 7 days a reinforcement arrives and it was
found that now the provisions will last 8 days less, when given at the rate
of 825 grams per head. How, many more men can it feed?
Ans. 1700 men.
Q25. From 5 different green balls, four different blue balls and three
different red balls, how many combinations of balls can be chosen taking at least
one green and one blue ball?
Ans. 3720.
Q26. Three pipes, A, B, & C are attached to a tank. A & B can fill it in 20
& 30 minutes respectively while C can empty it in 15 minutes. If A, B & C
are kept open successively for 1 minute each, how soon will the tank be filled?
Ans. 167 minutes.
Q27. A person walking 5/6 of his usual rate is 40 minutes late. What is his
usual time?

Ans. 3 hours 20 minutes.

***********************
The questions 41-46 are based on the following pattern.The problems below contain a question
and two statements giving certain data. You have to decide whether the data given in the
statements are sufficient for answering the questions.The correct answer is
(A) If statement (I) alone is sufficient but statement (II) alone is not sufficient.
(B) If statement(II) alone is sufficient but statement(I) alone is not sufficient.
(C) If both statements together are sufficient but neither of statements alone is sufficient.
(D) If both together are not sufficient.
41. What is John's age?
(I) In 15 years John will be twice as old as Dias would be
(II) Dias was born 5 years ago
Ans. (C)
42. What is the distance from city A to city C in kms?
(I) City A is 90 kms from City B
(II) City B is 30 kms from City C
Ans. (D)

43.Is A=C ? A,B,C are real numbers
(I) A-B=B-C
(II) A-2C = C-2B
Ans. (C)

44. What is the 30th term of a given sequence ?
(I) The first two terms of the sequence are 1,1/2
(II) The common difference is -1/2
Ans. (A)

45.Was Avinash early, on time or late for work?
(I) He thought his watch was 10 minutes fast
(II) Actually his watch was 5 minutes slow
Ans. (D)

46. What is the value of A if A is an integer?
(I) A4 = 1
(II) A3 + 1 = 0
Ans. (B)

47. A person travels 12 km in the southward direction and then travels 5km to the right and then travels 15km toward the right and finally travels 5km towards the east, how far is he from his starting place?
(a) 5.5 kms
(b) 3 km
(c) 13 km
(d) 6.4 km
Ans. (b)

48. X's father's wife's father's granddaughter uncle will be related to X as
(a) Son
(b) Nephew
(c) Uncle
(d) Grandfather
Ans. (c)

49. Find the next number in the series 1, 3 ,7 ,13 ,21 ,31
(a) 43
(b) 33
(c) 41
(d) 45
Ans. (a)

50. If in a certain code "RANGE" is coded as 12345 and "RANDOM" is coded as 123678.
Then the code for the word "MANGO" would be
(a) 82357
(b) 89343
(c) 84629
(d) 82347
Ans. (d)

51. If "PROMPT" is coded as QSPLOS ,then "PLAYER" should be
(a) QMBZFS
(b) QWMFDW
(c) QUREXM
(d) URESTI
Ans. (a)

The questions 52-53 are based on the following data
6 people A,B,C,D,E and F sit around a table for dinner.Since A does not like C, he doesn't sit either opposite or beside C.B and F always like to sit opposite each other.
52. If A is beside F then who is are the two neighbours of B?
(a) D and C
(b) E and C
(c) D and E
(d) Either (a) or (b)
Ans. (c)

53. If D is adjacent to F then who is adjacent to C?
(a) E and B
(b) D and A
(c) D and B
(d) either (a) or (c)
Ans.(d)

54. Complete the sequence A, E ,I ,M ,Q ,U , _ , _
(a) B, F
(b) Y, C
(c) G, I
(d) K, O
Ans.(b)

55. A person travels 6km towards west, then travels 5km towards north ,then finally travels
6km towards west. Where is he with respect to his starting position?
(a) 13km east
(b) 13km northeast
(c) 13km northwest
(d) 13km west
Ans. (c)

*******************************
Select the alternative that logically follows from the two given statements.
38.
• Some forms are books
• All books are made of paper
(a) Some forms are made of paper
(b) Some forms are not made of paper
(c) No forms are made of paper
(d) None of the above
Ans. (a)

39.
• All toffees are chocolates
• Some toffees are not good for health
(a) Some chocolates are not good for health
(b) Some toffees are good for health
(c) No toffees are good for health
(d) Both (a) and (b)
Ans. (a)

The questions 40-46 are based on the following pattern.The problems below contain a question and two statements giving certain data. You have to decide whether the data given in the statements are sufficient for answering the questions.The correct answer is
(A) If statement (I) alone is sufficient but statement (II) alone is not sufficient.
(B) If statement(II) alone is sufficient but statement(I) alone is not sufficient.
(C) If both statements together are sufficient but neither of statements alone is sufficient.
(D) If both together are not sufficient.
(E) If statements (I) and (II) are not sufficient

40. What is the volume of a cubical box in cubic centimetres?
(I) One face of the box has an area of 49 sq.cms.
(II) The longest diagonal of the box is 20 cms.
Ans. D

41. Is z positive?
(I) y+z is positive
(II) y-z is positive
Ans. E

42. Is x>y ? x, y are real numbers?
(I) 8x = 6y
(II) x = y + 4
Ans. B

43. If a ground is rectangular, what is its width?
(I) The ratio of its length to its breadth is 7:2
(II) Perimeter of the playground is 396 mts.
Ans. C

44. If the present age of my father is 39 yrs and my present age is x yrs, what is x?
(I) Next year my mother will be four times as old as i would be.
(II) My brother is 2 years older than I and my father is 4 years older than my mother.
Ans. C

45. How many brothers and sisters are there in the family of seven children?
(I) Each boy in the family has as many sisters as brothers
(II) Each of the girl in the family has twice as many brothers as sisters
Ans. D

46. x is not equal to 0, is x + y = 0?
(I) x is the reciprocal of y
(II) x is not equal to 1
Ans. A

Following questions are based on letter's analogy.First pair of letters should have the same relationship as the second pair of letters or vice versa.
47. ? : BGLQ : : YDIN : VAFK
(a) EKNS
(b) DKMT
(c) DLMS
(d) EJOT
Ans. (d)

48. NLO : RPS : : ? : ZXA
(a) VUW
(b) VTR
(c) VTW
(d) TRP
Ans. (c)

49. If "segment" is coded as rffndou, then "ritual" is coded as
(a) shutbm
(b) qjutbk
(c) qhutbk
(d) qhubtk
Ans. (c)

50. If "football" is "cricket" ,"cricket" is "basketball" ,"basketball" is "volleyball","volleyball" is "khokho" and "khokho" is cricket, which is not a ball game?
(a) cricket
(b) football
(c) khokho
(d) basketball
Ans. (a)
Q9. A, B ans C are three speakers. They have to speak randomly along with another 5 speakers in a function.
A has to speak before B and B has to speak before C. What is the probability.

Ans. 1/6
******************************
1) it has 20 mixutre conatins mil and water in the ratio 3:5,replace 4 litres of mixture with 4 litres of water what is the final ratio of milk and water.
2) + means * and * means / and / means % what is the value of these question
2+3*5/7 it was two question of these type.
3) The equivalent compound ratio of 5:6::7:10::6:5 ( question of this type this is not
exact question).
3) work can be done by 8 men and 10 women in 25 days, the same work can be done
by 10 children and 5 women . in how many days 2 children and 3 men
(similar to this)
4) one man or two women or three boys can do a work in 44 days then one man, one women and one boy together can fininsh the same work in ---- dyas
5) (998-1)(998-2)(998-3)…………..(998-n)=------- when n>1000 ans is zero
6) in how many ways can a lock be opened if that lock has three digit number lock if
i) the last digit is 9
ii) and sum of the first two digits is less than or equal to the last digit.
numbers are from 0-9
7)if a man reduces the selling price of a fan from 400 to 380 his loss increases by 20% .cost price of fan is.
8) there are 76 persons. 53 can read hindu,46 can read times,39 can read deccan
and 15 can read all.if 22 can read hindu and deccan and 23 can read deccan and times
then what is the number of persons who read only times and hindu………ans 18

9) in pure milk if 20% replaced by water and in this again 20% is replaced by water and again 20% is replaced by water then what is the praportion of milk in that mixture

10) after 10 years A will be twice the age of B before 10 years.and now if the difference is 9 years between them then what is the age of B after 10 years ans 49
11)races and games ---- 2 questions from this chapter like (A beats B by 10 meters and B beats C by 15 metres the A beats C by )
12)in the year 1990 there are 5000 men 3000 women 2000 boys .in 1994 men are increased by 20% women are increased by ratio of boys and women (this type of question but some what difficult I mean it takes too much time to solve)
************************
Directions for question 1-10: Each question comprises four scattered segments of a sentence. Identify from among the four choices the sequences that correctly assembles the segments and completes the sentence.

A. Anniversaries can be dicey things.
B. As long as the dead are being commemorated on a particular day, it is fine.
C. The opposition could keep its gun powder dry and ready as well.
D. But when it comes to a government celebrating a year or two in office, there can be trouble.
(a) DACB
(b) CADB
(c) ABDC
(d) BCDA

A. It is less concerned with telling a tale.
B. As with so much Huxley's later fiction, one is not sure whether or not to call this book a true novel.
C. It is also weak on characterization but strong on talk.
D. Than with presenting an attitude of life.
(a) CBDA
(b) BADC
(c) ABDC
(d) DCBA

A. While the above is true for private sector companies, it is not so in the public limited companies.
B. But with the removal of control over premia, the premia at which issues are marked has gone up quite sharply.
C. So the cost of capital at even a lower debt equity ratio comes out lower.
D. Traditionally, the cost of equity is higher than the cost of debt.
(a) DBCA
(b) BADC
(c) ACDB
(d) CDAB

A. Compiling and debugging.
B. Testing.
C. Writing the code.
D. Thinking of the algorithm.
E. Understanding the problem
(a) DBECA
(b) CDABE
(c) ACDBE
(d) EDCAB

A. Such alliances are shaky from the start.
B. In this manner parties which are not able to get a mandate from the electorate are able to come to power.
C. We have seen the unique spectacle of political forming alliance just to form a government.
D. Indian democracy continues to amaze.

(a) ADBC
(b) BADC
(c) DCBA
(d) BDCA

A. They are the three faces of dysphoria - bad feeling.
B. Anxiety, Depression and Anger.
C. When the three combine and get out of control, we get what is called mental illness.
D. All of us experience three emotions almost daily.
(a) ACDB
(b) CBDA
(c) CADB
(d) DBAC

A. This is the key to tap the creative power inside us.
B. It is difficult to control our thinking and feelings.
C. That is, unless we work at it conciously and persistently.
D. We are influenced and limited by attitudes, prejudices by other individuals and by external conditions to such an extent that few can control mental and emotional processes.
(a) ACDB
(b) BDCA
(c) CADB
(d) DBAC

A. Two vital facts must be understood.
B. The subconscious mind has the power to create.
C. The second is that it obeys the orders given to it by the conscious mind.
D. Its function is to bring to full expression whatever is desired by the conscious mind.
(a)ABCD
(b) CBDA
(c) ADBC
(d) DBAC

A. But what is not often understood is that this flash is outcome of long periods of incubation.
B. The layman thinks that it is a spell of divine flash which illuminates the dark and the hidden.
C. True, it does.
D. Inspiration is much misunderstood term.

(a) DBAC
(b) ACBD
(c) DBCA
(d) CADB

A. As a result, the world has undergone a transformation.
B. Science and civilization have made rapid strides especially in recent times.
C. This constitutes the basic factor in the use of productive resources.
D. But behind all the progress of mankind is the human factor which is invaluable and irreplaceable.

(a) ABDC
(b) BADC
(c) DCAB
(d) CBDA

Directions for questions 11-20: Each problem contains a question and two statements giving certain data. You have to select the correct answer from (a) to (d) depending on the sufficiency of the data given in the statements to answer the question. Mark your answer as
(a) If statement (I) alone is sufficient.
(b) If statement (II) alone is sufficient.
(c) If both (I) and (II) together are sufficient but neither of statements alone is sufficient.
(d) Either of the statements (I) and (II) is sufficient.
(e) If statements (I) and (II) together are not sufficient.


What is the distance from City A to City C in kms?
(I) City A is 90 kms from City B.
(II) City B is 30 kms from City C.

Is z less than w? z and w are real numbers.
(I) z2 = 25
(II) w = 9

The value of an estate in January 1905 started gradually declining in such a way that at the end of each year it was worth only x times its value at the beginning of the year. What was its worth in end December 1910 ?
(I) It was worth Rs.10,109 in the end of December 1906.
(II)It was worth Rs.12,345 in the beginning of January 1905.

In an election, 3 candidates A,B and C were representing for a membership of parliament. How many votes did each receive?
(I) A received 1006 votes more than B and 1213 more votes than C.
(II) Total votes cast were 15,414.

John studies Chinese in a school. Which school does he attend?
(I) All students in Jefferson High school take French.
(II) Maysville High School offers only Chinese.

How many girls passed the entrance exam this year?
(I) Last year 560 girls passed
(II) This year there was a 10% decrease over last year in the number of failures.

What is Raju's age?
(I) Raju, Vimala and Kishore are all of the same age.
(II) Total age of Vimala, Kishore and Abishek is 32 and Abishek is as old as Vimala and Kishore together.

Is Sreedhar eligible for an entry pass to the company premisers?
(I) The company does not allow strangers to enter the company.
(II)All employees are elgible to get a pass.

Among five friends who is the tallest?
(I) D is taller than A and C.
(II)B is shorter than E but taller than D.

Can a democratic system operate without effective opposition?
(I) The opposition is indispensable.
(II) A good statesman always learns more from his opponents than from
his fervent supporters.

Directions for question 21-22 : Answer the questions based on the passage below them

A temple has 3 gateways, each of them is leading you into the temple, and at the end of each gateway there is an idol and as a devotee passes through the gateway with some flowers the number of flowers double. Ram enters the 1st gateway with some flowers and he puts same number of flowers at each idol and the end he is left with none.

How many flowers did Ram start with?
(a) 4
(b) 5
(c) 3
(d) 7

How many flowers does he put at each idol?
(a) 10
(b) 8
(c) 6
(d) 5

Directions for question 23-25 : Answer the questions based on the passage below them
Liz, Jenni, Jolie and Rick have an English final on Friday and they all would like to study together at least once before the test.
Liz can study only on Monday, Tuesday and Wednesday nights and Thursday afternoon and night.
Jenni can study only on Monday, Wednesday and Thursday nights and Tuesday afternoon and night.
Jolie can study only on Wednesday and Thursday nights, Tuesday afternoon and Monday afternoon and night.
Rick can study the afternoons and nights of Tuesday, Wednesday and Thursday, and on Monday afternoon.


If the group is to study twice, then the days could be
(a) Monday and Wednesday
(b) Tuesday and Thursday
(c) Wednesday and Thursday
(d) Monday and Friday
(e) Tuesday and Wednesday


If three of them tried to study together when all four couldn't
(a) this would be possible twice
(b) it would have to be on Wednesday night
(c) Rick could not attend the three person groups
(d) This could be accomplised on Monday and Tuesday only
(e) This would not be possible


If Liz decided to study every night,
(a) she would never be able to study with Rick
(b) she would never be able to study with Jolie
(c) she would have at least two study partners each night
(d) she would have to study alone on Monday night
(e) she would study with only Jenni on Thursday night


*********************
0.03 of 0.05 is what
a. 15%
b. .15%
c. 0.015%
d. 0.0015%

xw which of the follwing is always true
a. xb. yc. x>w
d. y>w

12 men can do a job in 4 hours .in wat time the same job can be done by 15 men ( I did not do)
a. 3 hrs
b. 3 hrs 24 min
c. 2 hrs
d. 3 hrs 30 min

30 socks r there in a basket.60% are red and rest are blue. How many draws should be taken from the basket to make sure that u have 2 blue and 1 red
a. 2
b. 3
c. 14
d. 20

When operator * is applied to a number the result is 10 subtracted from the twice of the original number ,so wat is *(*9)

A husband and wife has 6 sons and each has 5 children each, How many are there in the family altogether

3x-2y=8 so what is 4y-6x
a. –16
b. 16
c. data not sufficient

From chennai to trichy it is 250 miles and from Chennai to pondicherry it is 120 miles.wat percentage of mile from Chennai to trichy is Chennai to pondi
a. 34
b. 20
c. 36

City B is 8 miles east of City A.City C is 6 miles north of City B. City D is 16 miles east of city C. City E is north of City D by 12 miles .what is the shortest distance from City A to City E

An employer pays X,Y,Z a weekly wage of total 610. X gets 120% of Y, and X gets 80% of Z wat is the weekly wage of X I think answer is 200.

There are 50 employees of a company .21 were in training for both economics and science training.11 were in 2 different training programs .find how many of them do not attend any training programfew questions were given with some relationship like:
1 head is to cap as finger is to
Ring
Nail
Thumb

In a certain code language RANGER was written as REGNAR .with the same code how is TABLE written Elbat(reverse the word)

In a certain code if READ is written as SEADR then how is SING written with that code SINGS

If kanchan is son of Sunil’s son’s son then how is Sunil related to Kanchan- grandson

Find the odd one out
a. Caution
b. Tresspasers will b prosecuted
c. Only one way
d. Keep left

One more odd man out was asked few questions on profit and loss were asked
****************************
Verbal
ACUMEN
a. exactness
b. potential
c. shrewdness
d. bluntness
e. None of these
BEHEST
a. behavior
b. hold down
c. hold up
d. relieve
e. condemn
DISCRETION
a. prudence
b. consistency
c. precipice
d. disturbance
e. distemper

ORDAIN
a. arrange
b. command
c. contribute
d. establish
e. control

FLORID
a. ornate
b. thriving
c. artistic
d. elegant
e. None of these
PENITENCE
a. liking
b. insightful
c. attractive
d. penetrable
e. compunction

WHET
a. stimulate
b. humorous
c. inculate
d. dampen
e. None of these

INCENTIVE
a. reflex
b. amplitude
c. inflection
d. provocation
e. escutcheon

LATITUDE
a. scope
b. segment
c. globule
d. legislature
e. lamentation
MORTIFY
a. make a cavity
b. displease
c. humiliate
d. relapse
e. murder

ADAGE
a. advice
b. proverb
c. enlargement
d. advantage
e. usage

TO DISPEL
a. to dissipate
b. to dissent
c. to distort
d. to disfigure
e. to dissect
ERRATIC
a. unromantic
b. free
c. popular
d. steady
e. unknown

TO MERIT
a. to embrace
b. to devote
c. to deserve
d. to combine
e. to display

RAPT
a. lively
b. concealed
c. engrossed
d. prototype
e. None of these

TO HEAP
a. to pile
b. to forbid
c. to proceed
d. to share
e. to stoop

CAJOLE
a. coax
b. motivate
c. profound
d. mollify
e. evade
OVULATE
a. penury
b. immunize
c. fertilize
d. reproduce
e. incisions
ABODE
a. clay
b. obstacle
c. dwelling
d. bind
e. to beguile

POTENTIAL
a. latent
b. hysterical
c. conventional
d. symmetrical
e. conscientious

EXTRICATE
a. terminate
b. isolate
c. liberate
d. simplify
e. frustrate

DISPARITY
a. inequality
b. impartiality
c. unfairness
d. twist
e. None of these

TO CONFISCATE
a. to harass
b. to repulse
c. to console
d. to appropriate
e. to congregate
PIOUS
a. historic
b. devout
c. multiple
d. fortunate
e. authoritative

LETHARGY
a. reminiscence
b. category
c. fallacy
d. unanimity
e. stupor

CARGO
a. cabbage
b. camel
c. lance
d. freight
e. flax
OVATION
a. oration
b. gesture
c. emulation
d. applause
e. nourish


Aptitude
1. A family, planning a weekend trip, decides to spend not more than a total of 8 hours driving. By leaving early in the morning, they can average 40 miles per hour on the way to their destination. Due to the heavy Sunday traffic, they can average only 30 miles per hour on the return trip. What is the farthest distance from home they can plan to go?
(a) 120 miles or less (b) Between 120and 140 miles (c) 140 miles
(d) Between 140 and 160 miles (e) 160 miles or more
2. A car is filled with four and half gallons of fuel for a round trip. If the amount of fuel taken while going is 1/4 more than the amount taken for coming, what is the amount of fuel consumed while coming back?
(a) Less than 2 gallons (b) 2 gallons (c) 2 1/2 gallons
(d) 3 gallons (e) More than 3 gallons

3. A 3-gallon mixture contains one part S and two parts R. In order to change it to a mixture containing 25% S, how much R should be added?
(a) 1/2 gallon (b) 2/3 gallon (c) 3/4 gallon (d) 1 gallon (e) 1 1/2 gallon

4. A tree grows only 3/5 as fast as the one beside it. In four years the combined growth of the two trees is eight feet.
How much does the shorter tree grow in two years?
(a) Less than 2 feet (b) 2 feet (c) 2 1/2 feet
(d) 3 feet (e) more than 3 feet.
5. Wind flows at 160 miles in 330 minutes, for traveling 80 miles how much time does it require?
(a) 1 hour 30 minutes (b) 1 hour 45 minutes (c) 2 hours
(d) 2 hours 45 minutes (e) 3 hours

6. A stationary engine has enough fuel to run 12 hours when its tank is 4/5 full. How long will it run when the tank is 1/3 full?
(a) Less than 2 hours (b) 2 hours (c) 3 hours
(d) 4 hours (e) 5 hours
7. If A is traveling at 72 km per hour on a highway. B is traveling at a speed of 25 meters per second on a highway. What is the difference in their speeds in meters per second?
(a) 1/2 m/sec (b) 1 m/sec (c) 1 1/2 m/sec
(d) 2 m/sec (e) 3 m/sec
8. A salesperson by mistake multiplied a number and got the answer as 3, instead of dividing the number by 3. What is the answer he should have actually got?
(a) 0 (b) 1/3 (c) 1 (d) 2 (e) 3
9. If the length of a rectangle is increased by 30% and the width is decreased by 20%, then the area is increased by...
(a) 10% (b) 5% (c) 4% (d) 20% (e) 25%

10. In the class of 40 students, 30 speak Hindi and 20 speak English. What is the lowest possible number of students who speak both the languages?
(a) 5 (b) 20 (c) 15 (d) 10 (e) 30

11. The most economical prices among the following prices is:
(a) 10 kilo for Rs.160 (b) 2 kilo for Rs.30 (c) 4 kilo for Rs.70
(d) 20 kilo for Rs.340 (e) 8 kilo for Rs.130

12. A truck contains 150 small packages, some weighing 1 kg each and some weighing 2 kg each. how many packages weighing 2 kg each are in the truck if the total weight of all the packages is 264 kg?
(a) 36 (b) 52 (c) 88 (d) 124 (e) 114
13. A man was arrested for exceeding the speed limit by 10 miles an hour. A second man was charged with exceeding the same limit by twice as much. The second man was driving 35 miles per hour. What was the speed limit?
(a) 10 miles per hour (b) 15 miles per hour (c) 20 miles per hour
(d) 25 miles per hour (e) 30 miles per hour

14. One year ago Pandit was three times his sister's age. Next year he will be only twice her age. How old will Pandit be after five years?
(a) 8 (b) 12 (c) 11 (d) 13 (e) 15
15. If two pencils cost 8 cents, then how much do 5 pencils cost?
(a) 18 cents (b) 20 cents (c) 22 cents (d) 23 cents (e) 24 cents
**********************************
Sample Test Paper

One of the following is my secret word: AIM DUE MOD OAT TIE. With the list in front of you, if Iwere to tell you any one of my secret word, then you would be able to tell me the number of vowels in my secret word. Which is my secret word?
Ans. TIE

In the following figure: A B C
D
E F G
H
I
Each of the digits 1, 2, 3, 4, 5, 6, 7, 8, and 9 is:
a) Represented by a different letter in the figure above.
b) Positioned in the figure above so that each of A + B + C, C + D +E, E + F + G, and G + H + I is equal to 13. Which digit does E represent? Ans. E is 4

One of Mr. Horton, his wife, their son, and Mr. Horton's mother is a doctor and another is a lawyer.
a) If the doctor is younger than the lawyer, then the doctor and the lawyer are not blood relatives.
b) If the doctor is a woman, then the doctor and the lawyer are blood relatives.
c) If the lawyer is a man, then the doctor is a man.Whose occupation you know?
Ans. Mr. Horton: he is the doctor.

Here is a picture of two cubes:

a) The two cubes are exactly alike.
b) The hidden faces indicated by the dots have the same alphabet on them.Which alphabet-q, r, w, or k is on the faces indicated by the dots? Ans. q

In the following figure:
A D
B G E
C F
Each of the seven digits from 0, 1, 2, 3, 4, 5, 6, 7, 8, and 9 is:
a) Represented by a different letter in the figure above.
b) Positioned in the figure above so that A*B*C, B*G*E, and D*E*F are equal.Which digit does G represent? Ans. G represents the digit 2.

Mr. and Mrs. Aye and Mr. and Mrs. Bee competed in a chess tournament. Of the three games played:
a) In only the first game were the two players married to each other.
b) The men won two games and the women won one game.
c) The Ayes won more games than the Bees.
d) Anyone who lost game did not play the subsequent game. Who did not lose a game?
Ans. Mrs. Bee did not lose a game.

Three piles of chips--pile I consists one chip, pile II consists of chips, and pile III consists of three chips- are to be used in game played by Anita and Brinda. The game requires:
a) That each player in turn take only one chip or all chips from just one pile.
b) That the player who has to take the last chip loses.
c) That Anita now have her turn.
From which pile should Anita draw in order to win?
Ans. Pile II

Of Abdul, Binoy, and Chandini:
a) Each member belongs to the Tee family whose members always tell the truth or to the El family whose members always lie.
b) Abdul says ''Either I belong or Binoy belongs to a different family from the other two."Whose family name do you know? Ans. Binoy's family--El.

****************************
1. Complete the series 1 6 3 7 5 8 7 ?
ans: 9

2. If circle is one octagon is 2 4 6 8 or 1

3. pick the odd one out bend shave chop whittle shear
ans bend.

4. 2 persons start from a pt. and go in opposite directions. After going 3 km they turn left
and walk 4 km .how far r they now? Ans: 10 km

5. There was a ques on work something like 2 typists type 2 papers in 4 hrs then in how much
time will 5 typists take or something like that

6. There were 2 ques on proverbs u were supposed to tell the meaning.they were easy.

7. A person runs 6 ft in 1/4 sec. how much it will run in 10 sec. ans 240

8. Find odd one cat ,dog,rabbit,hamster,elk.

9. In a party man & his wife ,and there r 2 son with their wives, and every son has 4 children

in his family .how manyperson r in party.

10. Unsreambled the word P U T E C A S A H (like that) paraschute

11. Add a word in starting of SCAPE and at the end of Grass, u will get two different word
ans:- land grassland landscape

13. All men of this party are republican . mohan is member of this party. mohan is republican is

true or false or camn't say?

14. There r 1200 elephant .some have blue and pink strips.some have pink & green strip. 400 ele.
have only pink strip. how many have only blue strip Ans:. 400
b. can't determined. c.none

15. Day light Day break Day time After clearing aptitude there was technical paper. It had 2
sections.In section A there were 4 subsections out of which 2 were to be attempted. the
first subsection was on c and unix, second on c++,third on vb/asp/com and fourth on rdbms. In
section B general computer awareness. In section A I did c and c++. Some ques were
as follows

If m-4 is 7 more than n then m+9 is how much more than n+2

If 10x coins are added to the no. of original coins he has (5y + 1) times more coins. Find out how many coins he
had originally in terms of x and y

Find out the Simple Interest paid for a sum of $4000 at the rate of 8% per annum for 3 months.

If it takes 10 technicians working 6 hours to build a server. They start woring at 11 AM and 1 technician is
added per hour starting at 5 PM. At what time they will finish the server?

A chart was there giving the speed of train after some minutes like
0 30 60 90 120 150 180
40 45 50 55 60 65 70

Find the % increase in the speed of train?

If it is extrapolated further what will be the speed after 5 hrs.

If the data is for 10 bogies and for every 2 bogies added speed decreases by 5% what will be the speed after 5
hrs.

The type of questions where Both stmt are necessary First one is sufficient in itself…..… What is the perimeter of the quadrilateral?

The quadrilateral is rectangle

The area of the quadrilateral is 36 What is a*b

6a+4b=43

3a+12b=63

The house is big and lovely but since nobody uses it it is pretty much a white Elephant What does White Elephant mean?

Sometime they irritate me because they are up with the lark whereas I like to lie in What does Up with Lark
mean?

If [x] indicates integral of x i.e is the largest integer less than x and |x| indicates absolute value of x then what is the maximum value of [x]/|x|.
A. 1 B. 0 C.-1 D. None of these Ans: A

In the above question what is the minimum value of [x]/|x|.
A. 1 B. 0 C.-1 D. None of these Ans: D
(3-6) If the clock(Conventional clock with numbers from 1 to 12 in order) is cut into 3 pieces such that the sum of numbers on each piece are in Arithemetic Progression(A.P) with a common difference of 1.

What is the sum of even numbers in the group where 5 is present?
A. 4 B. 10 C. 12 D. 14 Ans: B


What is the product of all numbers in the group in which 12 is present
A. 212 B. 252 C. 244 D. None of these


What is the count of numbers in each piece.
A. 2,2,5 B. 5,5,2 C. 3,4,5 D. 6,4,2 Ans: C


What is the count of numbers in each piece.
A. 2,2,5 B. 5,5,2 C. 3,4,5 D. 6,4,2 Ans: C

Avinash takes 15 days to complete a work and Bada takes 12 days to complete the same work. If they work in alternate days, In how many days they finish the work.
A. 13 days B. 13 1/4 days C. 6 1/4 days D. None Ans:

There is a circular track of length 400 mts. If A and B Starts at the same point but in opposite direction with a speeds of 8 m/sec and 12 m/s respectively.Then at what time after the begining they will meet for the second time.
A. 1hr 40 sec B. 20 sec C. 40sec D. 3hr 20 sec Ans: C

In the above question when will they meet for the first time at the starting point.
A. 1hr 40 sec B. 20 sec C. 40sec D. 3hr 20 sec Ans: A

If the vertices of the triangle are A(1,2), B(-2,-3) and C(2,3) then which is the largest angle?
A. Angle(ABC) B.Angle(BAC) C.Angle(ACB) D.None Ans:B

If (-1,0), (0,-1) and (-1,-1) are three vertices of a square then what is the 4th vertex.


If [x] indicates integral of x i.e is the largest integer less than x and |x| indicates absolute value of x then find the value of
[1.99]+[-2.99]+[1.03]+[2.50]
A. 2 B.1 C.-2 D. -5
Ans: A
Step1: Add all the numbers
Step2: If it is less than 10 STOP, else go to Step1.

If X=6724 then what is the end result after applying the above algorithm.
A. 19 B.10 C.1 D. None Ans:C


If the 4 numbers are arranged in all possible orders then how many solutions are possible.
A. ONE B. TWO 3.THREE 4. NONE Ans A


A trader frauds by 10% while buying and 10% while selling the same. What is the total gain he obtained during the transaction?
A. 13 B.221/4 C.20 D.None of these Ans.

There are three cylinders with same height and surface area. If a new cylinder is created by melting these three with the same height as before what is the surface area of the new cylinder when compared to that of the previous.
A. 25% more B.50% more C.100% more D.None

If x=a then y=b except when x=b and y=a. If x=a, then p,q,r,s but when x not equal to a then p,q,r,s=e,f,g,h. If x=m or n then both charecters preeciding it and following it also equals the same with the precedence to the preceding charecter.
There r 5 questions based on the above.

I have 20 rupees. I bought 1, 2, 5 rupee stamps. They are different in numbers by the reason of no change, the shop keeper gives 3 one rupee stamps. So how many stamp(s) I have. Ans: 10

An Engine length 1000 m moving at 10 m/s. A bird is flying from engine to end with x kmph and coming back at 2x. Take total time of bird traveling as 187.5 s. Find x and 2x.

All persons know either swimming or rowing. There are 14 persons who know only swimming and 14 persons who know only rowing. 8 tickets sold for rowing. How many people are there for swimming?

Which polygon has no. of sides = diagonal (Eg. Pentagon)

One Cigar can be made from 7 bits. If there are 16 Cigars then how many bits collected? Ans: 4

A, B, C, D went to a hotel and planned to share the bill equally. But afterwards they changed their plan and to pay proportional to consumption A paid 240, B & C paid equally, D paid only half the amount that he should have paid based on the first plan. What is the amount paid by B?

There is a point P on the circle. A and B started running in two constant different speeds. A in Clockwise and B in Anti-clockwise. First time 500 m in Clockwise from P then 400 Anti-clockwise. If B is yet to complete one round, What is the circumference of the circle?

There are 5 Sub with equal high marks. Mark scored by a boy is 3:5:6:7:8 (Not sure). If his total aggregate if 3/5 of the total of the highest score, in how many subjects has he got more than 60%?

There are 11 lines in plane. How many intersections (Maximum) can be made?

There are 3 Sections with 5 Qns each. If three Qns are selected from each section, the chance of getting different Qns is________

There is a 20 X 20 array. In Each row , the tallest person is called and among them, the tallest person is A. In Each column, the shortest person is called and among them, the shortest person is B. Who is taller?

P # Q = (P-Q)(Q-P) = - 1. Then Which is true?
P = 3, Q = 2 P = 2,Q = 3 P = -1,Q = 1 P = 1, Q = -1 Ans: I & II only

7 Pink, 5 Black, 11 Yellow balls are there. Minimum no. atleast to get one black and yellow ball Ans: 17

A, B, C and D are four people. There are four houses Red, Yellow, Blue, White. P, Q, R and S are four sections not in same order Conditions like Three are sisters B comes from Red C comes from Blue Qns were asked based on that

A Father is willing his estates like this. If a boy is born, wife has 1/3 part and remaining for boy. If a girl is born, Wife has 2/3 and remaining for the girl. But twins (Boy + Girl) are born. What is the share that the daughter would get?

MBA, GRE prob from Barrons GRE (Don know whether it is there in all GRE Editions). Ans: GMAT, CAT (Sure)

If ü - Married
û - Not Married and
M-ü N-û
N-ü L-û
L-û M-ü
Who is married? Ans: N
**************************
Aptitude.



a=2, b=3, c=6 Find the value of c/(a+b)-(a+b)/c
Ans. 11/30

What does the hexa number E78 in radix 7.
(a) 12455
(b) 14153
(c) 14256
(d) 13541
(e) 131112
Ans. (d)

What does the hexa number E78 in radix 7.
(a) 12455
(b) 14153
(c) 14256
(d) 13541
(e) 131112
Ans. (d)

Q is not equal to zero and k = (Q x n - s)/2.What is n?
(a) (2 x k + s)/Q
(b) (2 x s x k)/Q
(c) (2 x k - s)/Q
(d) (2 x k + s x Q)/Q
(e) (k + s)/Q

From the following statements determing the order of ranking
M has double the amount as D Y has 3 rupess more than half the amount of D
Ans. Data insuffiecient

Questions 6 - 10 are to be answered on the following data
A causes B or C, but not both
F occurs only if B occurs
D occurs if B or C occurs
E occurs only if C occurs
J occurs only if E or F occurs
D causes G,H or both
H occurs if E occurs
G occurs if F occurs

If A occurs which of the following must occurs
I. F and G
II. E and H
III. D
(a) I only
(b) II only
(c) III only
(d) I,II, & III
(e) I & II (or) II & III but not both
Ans. (e)

If B occurs which must occur
(a) D
(b) D and G
(c) G and H
(d) F and G
(e) J
Ans. (a)

If J occurs which must have occured
(a) E
(b) either B or C
(c) both E & F
(d) B
(e) both B & C
Ans. (b)

Which may occurs as a result of cause not mentioned
I. D
II. A
III. F
(a) I only
(b) II only
(c) I & II
(d) II & III
(e) I,II & III
Ans. (c)

E occurs which one cannot occurs
(a) A
(b) F
(c) D
(d) C
(e) J
Ans. (b)

A 5 litre jug contains 4 litres of a salt water solution that is 15 percent salt. If 1.5 litres of the solution spills out of the jug, and the jug is then filled to capacity with water,approximately what percent of the resulting solution in the jug is salt?
(A)7.5% (B)9.5% (C) 10.5% (D)12% (E)15%

Working independently, Tina can do a certain job in 12 hours. Working independently, Ann can do the same job in 9 hours. If Tina Works independently at the job for 8 hours and then Ann works independently, how many hours will it take Ann to complete the remainder of the jobs?
(A)2/3 (B)3/4 (C)1 (D)2 (E)3
Answer :E)3

In a murder case there are four suspects P,Q,R,S. Each of them makes a statement. They are p: "I had gone to the theatre with S at the time of the murder".q: "I was playing cards with P at the time of the murder".r: "Q didn't commit the murder".s: "R is not the murderer".Assuming the only one of the above statement is false and that one of them is the murderer, who is the murderer?
a) P
b) Q
c) R
d) Cann't be concluded
e) S
Ans: E

Mohan earned twice as much as Deep. Yogesh earned rs.3/- more than half as much as deep. If the amounts earned by Mohan,Deep,Yogesh are M,D,Y respectively, Which of the following is the correct ordering of these amounts?
a) M < D < Y
b) M< Y < D
c) D< M < Y
d) It cann't be determined from the information given
e) D< Y < M

Statistics indicate that men drivers are involved in more accidents than women drivers. Hence it may be concluded that
a) sufficiently information is not there to conclude anything
b) Men are actually better drivers but drive more frequently
c) Women Certainly drive more cautiously than Men
d) Men chauvinists are wrong about women's abilties.
e) Statistics sometimes present a wrong picture of things

Given that A,B,C,D,E each represent one of the digits between 1 and 9 and that the following multiplication holds:
A B C D E
X 4
--------------
E D C B A
-------------- what digit does E represent ?
a) 4
b) 6
c) 8
d) 7
Ans: c

HCL prototyping machine can make 10 copies every 4 seconds. At this rate, How many copies can the machine make in 6 min.?
a) 900
b) 600
c) 360
d) 240
e) 150
Ans: a

If a=2,b=4,c=5 then
a+b c
- ---- =
c a+b
a) 1
b) 11/30
c) 0
d) -11/30
e) -1
Ans: b

10^2(10^8+10^8) =-------------- 10^4
a) 2(10)^4
b) 2(10)^6
c) 10^8
d) 2(10)^8
e) 10^10
Ans: b

Worker W produces n units in 5 hours. Workers V and W, workers independently but at the same time, produce n units in 2 hours.how long would it take V alone to produce n units?
a) 1 hr 26 min
b) 1 hr 53 min
c) 2 hr 30 min
d) 3 hr 30 min
e) 3 hr 20 min
Ans: d

Six knights - P,Q,R,S,T and U - assemble for a long journey in Two ravelling parties. For security, each travellingparty Consists of at least two knights. The two parties travel by separate routes, northern and southern. After one month, the routes of the northern and southern groups converge for a brief time and at that point the knights can, if they wish, rearrange their travelling parties before continuing, again in two parties along separate northern and southern routes. Throughout the entire trip, the composition of traveling parties must be in accord with the following conditions P and R are deadly enemies and, although they may meet briefly,can never travel together. p must trave in the same party with sQ cann't travel by the southern route U cann't change

If one of the two parties of knights consists of P and U and two other knights and travels by the southern route,the other members of this party besides P and U must be
a) Q and S
b) Q and T
c) R and S
d) R and T
e) S and T
Ans: e

If each of the two parties of knights consists of exactly three members, which of the following is not a possible travelling party and route?
a) P,S,U by the northern route
b) P,S,T by the northern route
c) P,S,T by the southern route
d) P,S,U by the southern route
e) Q,R,T by the southern route
Ans: b

If one of the two parties of knights consists of U and two other knights and travels by the northern route, the other memnbers of this party besides U must be
a) P and S
b) P and T
c) Q and R
d) Q and T
e) R and T
Ans: c

If each of the two parties of knights consists of exactly three members of different parties, and R travels by the northern route,then T must travel by the
a) southern route with P and S
b) southern route with Q and R
c) southern route with R and U
d) northern route with Q and R
e) northern route with R and U
Ans: a

If, when the two parties of knights encounter one another after a month, exactly one knight changes from one travelling party to the other travelling party, that knight must be
a) P
b) Q
c) R
d) S
e) T
Ans: e

A gambler bets on the team of seven players ABCDEFG whose winning a-4 to 1 against b-4 to 1 against c-4 to 1 against d-4 to 1 against e-5 to 1 against f-6 to 1 against g. how should he bet on g to set 20% profit.

If a person buy radio worth Rs 2468 and pay 7% sales .how much price of radio should reduce to pay only Rs 2468

What is vasu salary if salary of vasu is more than rajan salary working in same company
i)vasu salary is 100 more than rajan salary.
ii)rajan found 2000 allowns which is 50 less than vasu.
(iii)basic salry of rajan is 1000.
(i)only i is required
(ii)i & ii is required
(iii)i& iii is required
(iv)i&ii&iii is required
(v)none of these

If in 100 miles race 8 person is running winner take 9.8sec and fifth man takes 10.4 sec the time of 8 man is in AP if in 4*100 meters realy of onside is 1,4,5,8 position then win by.
a).3 sec b).1 sec c).7 sec d).5 sec e)none

How many sons X have qwe based on relation
i)
ii)
iii)
ans(data i,ii,iii is insufficient)

A sink has 12 lits of water some quantity of water is taken out. if the remainng water is 6 litres less then thewater taken out then quantity of water taken out is.
a.3
b.6
c.9
d.1

which is the 4 digit number whose second digit is thrice the first digit and 3'rd digit is sum of 1'st and 2'nd and last digit is twice the second digit.
1.2674
2.1349.
3.3343
4.3678

In a straight highway 2 cars starts from the same point in opposite directions each travels for 8 Kms and take left turn then travel for 6 Kms what is the distance between them now.
1.16
2.20
3.25
4.10
*******************************
APTITUDE SECTION

Directions for questions 1-6: Find the correct meaning of the following phrases

A man of letters

A man of straw

To be in the air

To bite the dust

Man of few words

Penny wise pound foolish

Find the reminder when 333666777888999 divided by 3 or 9 or 11 ?

Which is the biggest perfect square amongst the following
15129, 12348, 23716, 20736

The greatest area of the following
(a) The radius of circle is 4
(b) The square of diagonal is 4
(c) The square of side is 4

The area of the maximum size of the circle described from the 10 square inch square?

In the series 0, 3, 8, 15,__ What is the next number?

X < 0, Y <> 0 then what is the possibility that the result is always positive?
Ans. xy

3 red and 4 blue balls are in a basket. A member of PPTeam is drawing balls from the basket. What is the
probablity of getting the 3 red balls simultaneously?

Let ax2 + bx + c = 0
If the sum of the equal roots is equal to the product of the same roots.Then which of the following hold true
(a) a + b = 0
(b) a = 0
(c) c = 0
(d) a + c = 0

A fold density is 19 times greater than the water and for copper it is 9 times.At what ratio you can mix gold and
copper to get 15 times denser than water.
Ans. 3 : 2

Find the value of (1.99)2
Ans. 3.9601

There is a room with 6' x 8'. A 1' tile is fixed along the 4 walls in one row. How many 1" tiles require to
finish the work.
Ans. 24

2 persons can finish a job in 8 days. First person alone can finish the work in 24 days. How many days
does the second person take to finish the job?
Ans. 12 days

A 4" cube is painted in all its faces and then it is cut down into 1" blocks. How many 1" blocks are there
even without a single face being painted?
Ans. 8

A cylinder is inserted in a sphere d/h = 2/3. Find the surface area of the cylinder ?

In a car wheel, two spokes cover 15 degree. Then for the entire car,how many spokes are there?
Ans. 24.

What is the angle of degree suspended when two hands of clock showing the time 2.30.
Ans. 105 degrees

he age difference between two brothers is 3 years. After 6 years the ratio between the age is 9:8
What are their ages?
Ans. 21 and 18

A person's salary is getting reduced by 20%. What percentage should be added to get back his
original salary?
Ans. 25%

Two persons start at the same point, walk in opposite directions with 5km/hr and 5.5km/hr respectively.
What is the distance separated after 2 and half hrs?
Ans. 26.25 (approx)

A person starts walking at a speed of 5km/hr through half the distance, rest of the distance he covers with a
speed 4km/hr. Total time of travel is 9 hours. What is the maximum distance he can cover?
Ans. 40km.

Initially two cups of same volume are present with milk filled upto 3/5th and 4/5th of their volumes.Water is then
filled. Then two mixtures are mixed. Find the ratio of water to milk in the mixture.
Ans. 3 : 7

16 grams of radioactive material decays into 8 grams in 10 years. How long will it take to decay to 1 gram ?
Ans. 70 yrs.

In a rectangle the length is increased by of the original length . By what proportion should the
width be reduced so that the area will be the same?
Ans. 33

Find the nth number in the series is 1, -3, 5, -7.___
Ans. (-1)*(2n-1)

If a square is formed by the diagonal of the square as an edge, what is the ratio between the area?
Ans. 2

The perimeter of a rhombus is 52 units. One of its diagonal is 24 units.What is its second diagonals length?
Ans. 10

A cubical rectangular bar has the dimensions with the ratio 5 : 4 : 3. Its volume is 7500. What is the
surface area of the bar?
Ans. 2350

In a class total 34 students, 16 are have a brother, 15 are have sisters, 9 students don't have either
brothers or sisters.Find the number of students having both brother and sisters.
Ans. 6

A batsman scored 18 runs in his 18th innings and that makes his average 18. Find his average upto the
17th innings?
Ans. 19

6 women can do 75 units of work in 8 days by working 5hrs/day. In how many days can 4 women do 30 units
of work by working 8hrs/day ?

A persons salary iis decreased by steps of 20%, 15% and 10%. What will be the percentage decrease, if the
salary is decreased in a single shot?

The ratio of the length : breadth : height of a cuboid is 5 : 4: 3, and the volume is 7500. What will be its surface
area ?

If the circumference of a circle is 100 units, Then what will the length of the arc described by an angle of 20
degree ?

3 persons started placementpapers with a capital of Rs.3000 . B invest Rs.600 less than A, C invest Rs.300 less
than B. Then what is the share amount of B in a profit of Rs.886 ?
Directions for 41-50: Which of the following is the correct spelling for the word

supercede and supersede

recommend and reccomend

superitendent and superitendant

separate and seperate

succeed and suceed

coolly and cooly

despair and dispair

ridiculous and rediculous

indespensible and indepensable

tranquility or tranquillity

In a city, 80% speaks english, 70% speaks hindi and 10% do not speak both. It was found that 162 people talk both. How many are there in the city?

Find the missing number: 0 6 24 X 120

There are 26 pearls in a bag. One among them is less in weight. You have been given 2 pan weight machine. In how many trials, you will be able to find the defected one?

There are 30 socks in a bag. 30% is in blue color. What is the probability to take two blue socks?

In a party, every man has his dog with him. There are 22 heads and 72 legs all together. How many men & dogs are there?

Find the missing number: 0, 0.577, 1, 1.732, ?
a. 0.656 b.2 c.2.743 d.none

Two different types of tea are mixed, at 6 Kg of type 1 and 4 Kg of type 2. One Kg of type 1 is Rs. 6 and that of type 2 is Rs. 7. The seller get 10% profit, by this action. Find at what price, he 'd have sold the mixture/Kg?

There are 10 people in a party. "How many other people, you met?" is the question asked to everyone. First person says 1, Second says 2, Third says 3..........ninth says 9. Now what tenth person 'd have answered?

.The Distance between A to B is 1000 miles... A person has 3000 Apples ..He has to deliver to the market in B , he is carring Maximum of 1000 apples by the camel.. For every mile the camel eats a Apple ...Like that then u have to find out the number of apples that he delivered to the market?

In the old era humans used --------- to prevent themself from the attack of dianosur?(Easy one)

Mr x & MRS x and Mr.y and MRS y are playing a chess ..like that then the chance of winning? ans (MR.y)








2 pipes A and B can fill a tank in 30minutes and 20 minutes respectively. B filled ¾ of the tank. Then for the rest both A and B filled the tank. How much time it took to fill the entire tank.

5 monkeys eat 5 bananas in 5 minutes. Then how many monkeys can eat 20 bananas in 20 minutes.

A does a work in 20 days. A did for 28 days. Then b came. They together take 76 days to complete the 7 job. Find howmany days B complete a work.

My house number is
if not multiple by 3 it is with in 50-59
if not multiple by 5 it is with in 60-69
if not multiple by 10 it is with in 70-79
find house number.

After 6 years the ratio of age of X and Y will be 5:6. Before 6 years it was 3:4. Find the age of younger one.

The arithmetic mean of two numbers is 10 and the geometrid mean is 8. Find the two numbers.

3 consecutive non zero number which of these will result a positive odd integer.
1. x+y+z
2. xy-z
3. x-y-z

420 km distance. It goes at a speed of 10km/hr less than it took 1.4 hour. Find the initial speed.

Tem, Harry and Dick are three friends. From these two are musicians.
Tem----Harry
Shorter one is older musician.
Harry----Dick
Shorter one is younger musician.
Dick-----Tem
Taller is the younger musician.
Find the two musician.

A’s mother is only daughter of B’s mother. B is A’s what.

In a 20 liter mixture of milk and water, the ratio of milk and water is 3:1. How much milk is to be mixed with the mixture so as to make the ratio 4:1.


For a party 240 guests were invited. Contractor have gained 12.5% profit. But 30 guests were absent. He charged the same amount for a meal and lost 100 rupees. How much a meal costs.

Out of three A,B,C one is thief. All give two statements. One is false another is true. Find the thief.
A-i did not do, B did not do.
B-i did not do, C did not do.
C-i did not do, I donot know who did it.

The sides of an isocles tringale are given. The perpendicular distance is given. Find the base of the tringale.(Numerical values are given, but I donot remember)

In a tringale length of 3 sides are given. It is given to find the highest length of scale which can be used to measure all the sides.(To find gcf).

There is a metallic cylinder of radius r and height h is melted. If the radius of smaller cylinder is r/10 and height is h/10, then howmany small cylinders are made from the molten.

*************** the first paper ***********
a,b,c,d and e 5 different quantities are given:
-> if and only if a=c then b is not = e;
-> a-b=b-c;
-> a>b;
-> a>b and c>d; write in descending order.

3 men a,b,c went in 3 direction and had stolen a mule,a horse and
a camel.they were caught by cbi.during there interrogation they said:
a: b had stolen a horse.
b: a and c are both lying and i had stolen nothing.
c: a is lying and b had stolen mule.
one who had stolen camel is lying and who has stolen horse is telling
truth.among a,b,c who had stolen what.

amarpur is north of landgang and west of chutpur.basti is north of
amarpur and west of fulganj.dama is south and east of
amarpur.landganj is north of fulganj and east of dania.fulganj is north of dania and west of amarpur.chutpur is south of fulganj and west of dania.a.which of
the towns mentioned is farthest to the north west.
b.which one must be bothnorth and east of fulganj.

c. which one must be situated both south and west of at least one
another town.
d.which of the statements if true would make information in the
numbered statements more specific.
(there are two lost statements here - cpc)
->amarp is east of basti
->chutp is east of fulg
->basti is north of ful.

ther are 4 women and in quest. the condition about what they are
selling and there last name are given.from this u have to find what
they are selling and their lastname.

*************** the second paper ***************
two boys riding on the bicycle in opposite directions at 10 km/hr
start at same instant . a fly on one bicycle flies to another and after
reaching there returns to the original cyclist. it does this upto the time the two cyclist meet. how much distance has the fly travelled. speed of the fly is
15 km/hr and total dist between cyclists is 20 km.

B Jones sells 1/4th of the total paper +1, A smith sells 1/4th of
the remaining +1, C Jones sells 1/4th of the remaining +1, B smith sells
1/4th of the remaining +1. Up to this Jones has sold 100 more papers than
smith. C smith sells the remaining papers . how much more have the smith sold than the Jones ?

a clock takes 6 seconds to strike 5 , 9 sec to strike 6 . how much
would it take throughout a day ?

an old lady have age between 50 & 70 her each son has same no of
brothers as his own sons. what is the age of the lady ?. age of the lady is
equal to the total members in the family excluding the lady .

george, michael & harry each give one false and 2 true statements.
find
their respective ages.
1). i'm 22 years old
2). harry is a year older than me
3). michael is 2 years younger than me.

********** the third paper ************
when a man is going on a cycle with wind it takes him 3 minutes to
cover one mile. when he is going against wind it takes him 4 minutes for 1
mile. Then how much time will it take to cover the same distance without
wind .

i am having a watch. the hour hand and minute hand are coming
together every 65 minutes. whether my watch is gaining or loosing time, and
how much per hour. ( puzzles to puzzel u - shakuntala devi q no 3D63 )

barrorns gre 12th ed page496 q 8-12

a person had spent 1/6 th of his total life as a child, 1/12th as
young and 1/7th as bachelor. after 5 years of his present age he becomes the
father of a son. his son dies 4 years before the man becomes half of his age.
then what is the total age of the person .

there r some numbers such as alpha, beta, gamma, epsilon & zeta.
Few conditions r given u have to find the smallest one.

******** the fourth paper ***********

employe 1.-you told me ,"two years ago, you
have worked 3 times as much
as me.
employe2.-yes, isaid that now i have worked twice as much as you.
how many years both of them have worked.-----

mr X says that the 2 dial of the watch concide every 65 min. is
the or gaining the time, and by how much in a hour.----


a person leaves the city at const speed. on his way he saw a mile-
stone,a2 digit no. after going for 1 hour , he sees another , milestone having the same number in reverse order still further 1 hour he again sees the mile stone showing the two number but a zero in between. what is the velocity of x

MOON
SOON
NOON
-------
JUNE FIND JUNE?

a,b,C,D HAS 4 CARDS IN FRONT OF THEM. and the colour of the cards on 2 faces are diffrent. 2 red 2green, 2 blue.

a says- yellow or blue

b says- niether yellow nor blue

c says blue or yellow

d says- blue or green

if the open face colour is the order
red, green,red,green.

find the hidden colour, if 2 of them says lie.-----------

red green red green
yellow blue yellow blue

A's wife died .he is living with his niece.B is a widow an dliving
with her daughter and C is living with hr wife.C's wife suggested that they should live together.each member of the group should contribute 25/- and the rest should be distributed equally .if the monthly expense was rs 92/- and each got whole sum of money(no fraction).after cutting their expenses what is amount each will get?

a man leaves home for his office in time according to his watch but
in the office he finds that he is not on time and checks his watch.he finds that his watch shows one hour when actually 65 min has been passed say whether his watch loses or gain time and by what amount per hour?

three people A,B,C live in a place , their occupations are policeman ,fireman , teacher. out of the five conditions , only two are true
(a) A is the neighbour of teacher.
(b) B is the neighbour of techer.
(c) policeman and fireman are neighbour of C.
(d) all of them are neighbour of each other.
(e) B is the neighbour of fireman.
FIND THEIR OCCUPATION OF A,B,C.

**********************************************************************

actual question paper which i wrote:
1.there are 100 balls;20 white,20 black,20 blue,20 green and 20
yellow balls.find the least no. of balls to be drawn so that u have a
pair of balls of same colour?
ans:6
2.there was question regarding dimensions?
ans:14*7 inches
3.quantities alpha,beta,gamma,epsilon and theta were to be compared?
ans:alpha,
beta=epsilon,
gammma
theta.
4.birth year of field marshall roy?
ans:1911
5.some time related question was asked?
ans:supposed to be 9 am.
6.distance problem?
ans:126 miles.

rest of the questions were analyticals.
7.questions related with semesters,papers and teachers.
ans:a,d,d

*****************************
1. A man rides a bicycle and travels a distance of 1 mile in 3 minutes with
the wind, and comes back in 4 minutes agaist the direction of wind. what will
be the time taken for him to travel a mile without wind

ans: 24/7 minutes marks (3)
_____________________________________________________________
2. Arrange 5 numbers in descending order given 3 conditions

ans : alpha, beta, epsilon, deta, gamma. (check not sure) marks(3)

____________________________________________________________
3. SEND Find the values of M,O,N,E,Y
+MORE
----- ans: M=1; O=0; N=5; E=4; marks(4)
MONEY
-----
_____________________________________________________________
4. An observer sees that in his watc 'minute' and 'hour' hand meets every 65
minutes. Does the watch loses or gains and by how much for one hour.

ans: Refer Shakuntala Devi, 'Puzzles to Puzzle you' marks(5)
_____________________________________________________________
5. Difficult Question: Matching 'First' name and 'Surname' of five different
name and the products they sell

ans: Refer 'George Summers' marks(5)
_____________________________________________________________
6. A thief enters a jewellery shop and steals half of the diamond present
there, then he took two more diamonds second thief stole half of the remaining and two more, third thief and fourth thief did so. when the 5th thief arrived there were no diamonds present. what was the total no. of diamonds present initially?
ans: 60 diamonds marks(6)
_____________________________________________________________
7. A,B,C stole three animals(mule, horse, camel, each one) 3 statements were given(some true, some fals). A person who stole camel was telling the truth and the one who stole horse was lying. marks(4)
_____________________________________________________________
8. A man spends 1/6th of his total life in child hoos, 1/7th as youth, 1/12th
as bachelor, five years after marriage a son was born. the son died four
years before his father, at an age of half of his father's final age.

ans: 84 days marks(4)

10. Some (7) townswere given and their geographical location with respect to the other towns. Locate them. four question based on it marks(8)

Total Marks --> 50
_______________________________________________________
4)a very simple problem (GRE pattern) 4 names were given,their
proffesion,their dance.some conditios for each of them were given.
ans: direct relation very very simple try to attend this first.(8 marks)

5)GRE pattern qs.
A is in the north of B
D west of C
C Southwest of E

we couldn't remember more than this.very simple problem
draw the location as per the given conditions don't assume anything.
four qs
from this section.

7)three men A,B,C each tell one statement of which one may be true/false

depending on conditions find the animals they steal.
ans: A steals camel(check very simple problem)

***************************
1) There are two balls touching each other circumferencically.
The radius of the big ball is 4 times the diameter of the small
ball.The outer small ball rotates in anticlockwise direction
circumferencically over the bigger one at the rate of 16 rev/sec.
The bigger wheel also rotates anticlockwise at Nrev/sec. what is
'N' for the horizontal line from the centre of small wheel always
is horizontal.

2) 1 2 3 4
+ 3 4 5 5
----------
4 6 8 9
- 2 3 4 5
----------
2 3 4 4
+ 1 2 5 4
------------
3 6 9 8

Q) Strike off any digit from each number in seven rows (need not
be at same place) and combine the same operations with 3 digit numbers
to get the same addition. After this strike off another digit from all
and add all the No.s to get the same 2 digit No. perform the same
process again with 1 digit No.s. Give the ' no.s in 7 rows at
each stage.

3) there is a safe with a 5 digit No. The 4th digit is 4 greater than
second digit, while 3rd digit is 3 less than 2nd digit. The 1st digit
is thrice the last digit. There are 3 pairs whose sum is 11. Find
the number. Ans) 65292.

4) there are 2 guards Bal and Pal walking on the side of a wall of a
wearhouse(12m X 11m) in opposite directions. They meet at a point and
Bal says to Pal " See you again in the other side". After a few moments
of walking Bal decides to go back for a smoke but he changes his
direction again to his previous one after 10 minutes of walking in
the other(opposite) direction remembering that Pal will be waiting
for to meet.If Bal and Pal walk 8 and 11 feet respectively, how
much distance they would have travelled before meeting again.

5) xxx)xxxxx(xxx
3xx
-------
xxx
x3x
------
xxx
3xx
------

Q) Find the 5 digit No.
Hint: 5 is used atleast once in the calculation.
6) Afly is there 1 feet below the ceiling right across a wall length
is 30m at equal distance from both the ends. There is a spider 1 feet
above floor right across the long wall eqidistant from both the ends.
If the width of the room is 12m and 12m, what distance is to be
travelled by the spider to catch the fly? if it takes the shortest
path.

7) Ramesh sit around a round table with some other men. He has one
rupee more than his right person and this person in turn has 1 rupee
more than the person to his right and so on, Ramesh decided to give
1 rupee to his right & he in turn 2 rupees to his right and 3 rupees
to his right & so on. This process went on till a person has
'no money' to give to his right. At this time he has 4 times the
money to his right person. How many men are there along with Ramesh
and what is the money with poorest fellow.


8)Question related to probabilities of removing the red ball from a
basket,given that two balls are removed from the basket and the other
ball is red. The basket contains blue,red,yellow balls.

9)Venkat has 1boy&2daughters.The product of these children age is 72.
The sum of their ages give the door numberof Venkat.Boy is elder of
three.Can you tell the ages of all the three.

ANALYTICAL
----------
1)L:says all of my other 4 friends have money
M:says that P said that exact one has money
N:says that L said that precisely two have money
O:says that M said that 3 of others have money.
P:Land N said that they have money.
all are liers.Who has money&who doesn't have?

2)A hotel has two,the east wing and the west wing.some east wing rooms
but not all have an ocean view(OV).All WW have a harbour view(HV).The
charge for all rooms is identical, except as follows
* Extra charge for all HV rooms on or above the 3rd floor
* Extra charge for all OV rooms except those without balcony
* Extra charge for some HV rooms on the first two floor&some EW rooms
without OV but having kitchen facilities. (GRE modrl Test 3-question
1J-22)
3)Post man has a data of name surname door no.pet name of 4 families.
But only one is correct for each family.There are a set of statements
&questions.
4)4 couples have a party.Depending on the set of statements,find who
insulted whom and who is the host of the party.
5)5 women given some of their heights(tall,medium,short)Hair( long,
plainted),stards(Black or Brown), sari,2 medium,2-short.Tall->no
sari.Plainted->medium.Answer the combinations.


1) A person has to go both Northwards&Southwards in search of a job.
He decides to go by the first train he encounters.There are trains for
every 15 min both southwards and northwards.First train towards south
is at 6:00 A.M. and that towards North is at 6:10 .If the person arrives
at any random time,what is the probability that he gets into a train
towards North.
2) A person has his own coach&whenever he goes to railway station he
takes his coach.One day he was supposed to reach the railway station
at 5 O'clock.But he finished his work early and reached at 3 O'clock.
Then he rung up his residence and asked to send the coach immediately.
He came to know that the coach has left just now to tje railway station.
He thought that the coach has left just now to the railway station.He
thought that he should not waste his time and started moving towards
his residence at the speed of 3mi/hr.On the way,he gets the coach and
reaches home at 6 o'clock.How far is his residence from railway
station.

3)Radha,Geeta&Revathi went for a picnic.After a few days they forgot the
date,day and month on which they went to picnic.Radha said that it was
onThursday,May 8 and Geeta said that it was Thursday May 10.Revathi
said Friday Jun 8.Now one of them told all things wrongly,others one
thing wrong and the last two things wrongly.If April 1st is tuesday
what is the right day,date and month?

*********************************88
1. A person was collecting end pieces of Cigarettes. He was managed to collect 49 pieces. We need 7 such pieces to make new one. So how many Cigarettes he can make? (2 marks)
2. A person moving from a place(camp)., towards East one mile, then towards North half mile, towards West 1/4 the mile, towards South 1/8th mile and again
towards East 1/16th mile and so on. That is the distance between him and the
starting point. (3 marks)
3. 500 men were arranged in 10 rows and 50 columns. They picked the oldest person from each row and the tallest of these persons is 'A'. They replace them in their places and again picked tallest person from each column and the shortest of these persons is 'B'. Assume A & B are different people. Who is the
tallest person among A & B? (5 marks)
4. The product of two nonzero numbers is 1,000,000,000. What are the
numbers ? (5 marks)
5. There is Mathematician. He met his friend and asked about his children.His friend told in the form of a problem that he has three children. The
product of their ages is 36. The addition of their ages is the door number
of his left side house. Mathematician went and checked the door number
He told that the clues are not sufficient. He gave another clue that
is his younger daughter is clearly younger. What are their ages ? (6 marks)
6. There is a light glows in the interval of 13 sec. It glows first at one
hour 54 minutes 50 seconds and the last glow is at 3 hour 17 minute and 49
seconds. How many times is that light glow, with in these two times(2 marks)
7. After spending 1/3 of money and then 1/4th of what remained and finally 1/5th of what remained, I found that I had Rs.100/- left. How much money I had
at first ? (3 marks)
8. A Black Smith was given a chain torn into equal sections of 3 links
each and asked to fix it. How many links(minimum) would he has to open
up and reforge ? (1 mark)
9. There are six boxes, of which some boxes contain only red balls and some other boxes contain only Black balls. The number of balls in six differnet boxes are 5,6,12,14,23,and 29. If he sells
Another Set:
1. 1/3rd liquid in a container evaporates in first day. In the second day 3/4th remaining avaporates. How much is left over at the end of second day ?
2. first person : I am sure you are at least 40 years old and I am 5 years
younger than you.
second person : I am 35 and you are atleast 5 years elder than me. None
of these persons spoke truth. What are their ages ?
3. Orange cup has Orange Juice. White cup has apple Juice. %0 ml of
Orange Juice is taken and mixed with Apple Juice. From that mixture
50 ml is taken and poured into Orange cup. Now whether apple Juice
in Orange Cupis more or Orange Juice in White cup is more and by what amount?
4. Ms. Sheela goes her home by car from Station. Her driver comes and
picks her up daily at 5.00 p.m. One day Sheela arrives at Station one
hour earlier and starts walking towards home. On the way driver
picked her up. By this they reached home 30 minutes earlier. For how
long she was walking ?
5. Some students went on a trip to Goa in holidays. Unfortunately it
rained on some days. In a surprising manner if it rained in the
morning, they had a good afternoon and vice versa. They had 11 morning
visits and 12 afternoon visits. Altogether it rained for 13 days
during their stays. What is the duration of Holidays ?
6. A survey was conducted for 100 people by Door Darshan
1. 44 people watched channel I
2. 43 " " " II
3. 27 " " " III
4. 17 " " " I & II
5. 14 " " " I &III
6. 13 " " " II & III
7. 23 watched none.
How many watched I II & III ?
7. Two Swimmers at different rate but at constant Speed were swimming.
They met at 18 meters from deep end. Both swimmers took rest for
4.5 seconds. During their return they crossed at 10 meters from
ashlor end. What is the length of the pool ?
8. Mr. Raj goes to Office by Train. First train in Main line Starts at
5:02:0. In Harbour Line it starts at 5:10:0. Every Ten minutes there
is one train. What is the probability that Raj travels in harbour line
at a random time of Driving the Station?
PART 1
1). A beggr collects cigarette stubs and makes one ful cigarette
with every 7 stubs. Once he gets 49 stubs . How many cigarettes
can he smoke totally. Ans. 8
2). A soldiar looses his way in a thick jungle at random walks
from his camp but mathematically in an interestingg fashion.
First he walks one mile east then half mile to north. Then 1/4
mile to west, then 1/8 mile to south and so on making a loop.
Finally hoe far he is from his camp and in which direction.
Ans: in north and south directions
1/2 - 1/8 + 1/32 - 1/128 + 1/512 - and so on
= 1/2/((1-(-1/4))
similarly in east and west directions
1- 1/4 + 1/16 - 1/64 + 1/256 - and so on
= 1/(( 1- ( - 1/4))
add both the answers
3). hoe 1000000000 can be written as a product of two factors
neither of them containing zerosAns 2 power 9 x 5 ppower 9 ( check the answer
4). Conversation between two mathematcians:
first : I have three childern. Thew pproduct of their ages is 36.
If you sum their ages . it is exactly same as my neighbour's
door number on my left. The sacond mathematiciaan verfies the
door number and says that the not sufficient . Then the first
says " o.k one more clue is that my youngest is the youngest"
Immmediately the second mathematician answers . Can you aanswer
the questoion asked by the first mathematician?
What are the childeren ages? ans 2 and 3 and 6
5). Light glows for every 13 seconds . How many times did it
between 1:57:58 and 3:20:47 am ans : 383 + 1 = 384
6). 500 men are arranged in an array of 10 rows and 50 columns .
ALL tallest among each row aare asked to fall out . And the
shortest among THEM is A. Similarly after resuming that to their
originaal podsitions that the shorteest among each column are
asked to fall out. And the longest among them is B . Now who is
taller among A and B ? ans A
7). A person spending out 1/3 for cloths , 1/5 of the remsaining
for food and 1/4 of the remaining for travelles is left with
Rs 100/- . How he had in the begining ?
ans RS 250/-
8). there are six boxes containing 5 , 7 , 14 , 16 , 18 , 29
balls of either red or blue in colour. Some boxes contain only
red balls and others contain only blue . One sales man sold one
box out of them and then he says " I have the same number of red
balls left out as that of blue ". Which box is the one he solds
out ?
Ans : total no of balls = 89 and (89-29 /2 = 60/2 = 30
and also 14 + 16 = 5 + 7 + 18 = 30
9). A chain is broken into three pieces of equal lenths
conttaining 3 links each. It is taken to a backsmith to join into
a single continuous one . How many links are to tobe opened to
make it ?
Ans : 2.
10). Grass in lawn grows equally thickand in a uniform rate. It
takes 24 days for 70 cows and 60 for 30 cows . How many cows can
eat away the same in 96 days.?
Ans : 18 or 19
11). There is a certain four digit number whose fourth digit is
twise the first digit.
Third digit is three more than second digit.
Sum of the first and fourth digits twise the third number.
What was that number ? Ans : 2034 and 4368
If you qualify in the first part then you have to appear for
the second i.e the following part.
Part 2.
1. From a vessel on the first day, 1/3rd of the liquid
evaporates. On the second day 3/4th of the remaining liquid
evaporates. what fraction of the volume is present at the end of
the II day.
2. an orange galss has orange juice. and white glass has apple
juice. Bothe equal volume 50ml of the orange juice is taken and
poured into the apple juice. 50ml from the white glass is poured
into the orange glass. Of the two quantities, the amount of
apple juice in the orange glass and the amount of orange juice in
the white glass, which one is greater and by how much?
3. there is a 4 inch cube painted on all sides. this is cut
into no of 1 inch cubes. what is the no of cubes which have no
pointed sides.
4. sam and mala have a conversation. sam says i am vertainly not
over 40. mala says i am 38 and you are atleast 5 years older
than me. Now sam says you are atleast 39. all the sattements by
the two are false. How hold are they realy.
5. ram singh goes to his office in the city, every day from his
suburbun house. his driver mangaram drops him at the railway
station in the morning and picks him up in the evening. Every
evening ram singh reaches the station at 5 o'clock. mangaram
also reaches at the same time. one day ramsingh started early
from his office and came to the station at 4 o'clock. not
wanting to wait for the car he starts walking home. Mangaram
starts at normal time, picks him up on the way and takes him back
house, half an hour early. how much time did ram singh walk.
6. in a railway station, there are tow trains going. One in the
harbour line and one in the main line, each having a frequency of
10 minutes. the main line service starts at 5 o'clock. the
harbour line starts at 5.02a.m. a man goes to the station every
day to catch the first train. what is the probability of man
catchinhg the first train
7. some people went for vaction. unfortunately it rained for 13
days when they were there. but whenever it rained in the
morning, they had clean afternood and vice versa. In all they
enjoyed 11 morning and 12 afternoons. how many days did they
stay there totally
8. exalator problem repeat
9. a survey was taken among 100 people to firn their preference
of watching t.v. programmes. there are 3 channels. given no of
people who watch
at least channel 1
" " 2
" " 3
no channels at all
atleast channels 1and 3
" " 1 and 2
" " 2 and 3
find the no of people who watched all three.
10. albert and fernandes they have two leg swimming race. both
start from opposite and of the pool. On the first leg, the boys
pass each other at 18 mt from the deep end of the pool. during
the II leg they pass at 10 mt from the shallow end of the pool.
Both go at const speed. but one of them is faster. each boy
rests for 4 sec to see at the end of the i leg. what is the
length of the pool.
11. T H I S Each alphabet stands for one
I S digit, what is the maximum value T
-------------- can take
X F X X
X X U X
--------------
X X N X X
--------------
1. an escalator is descending at constant speed. A walks down
and takes 50 steps to reach the bottom. B runs down and takes 90
steps in the same time as A takes 10 steps. how many steps are
visible when the escalator is not operating.
2. evvery day a cyclist meets a train at a particular crossing.
the road is straignt before the crossing and both are travelling
in the same direction. cyclist travels with a speed of 10 Kmph.
One day the cyclist comes late by 25 min. and meets the train 5km
before the crossing. what is the seppd of the train.
3. five persons muckerjee, misra, iyer, patil and sharma, all
take then first or middle names in the full names. There are 4
persons having I or middle name of kumar, 3 persons with mohan, 2
persons withdev and 1 anil.
--Either mukherjee and patil have a I or middle name of dev or
misra and iyer have their I or middle name of dev
--of mukherkjee and misre, either both of them have a first or
middle name of mohan or neither have a first or middle name of
mohan
--either iyer of sharma has a I or middle name of kumar hut not
both.
who has the I or middle name of anil
4. reading conprehension
5. a bird keeper has got Ppigeon, M mynas and S sparrows. the
keeper goes for lunch leaving his assistant to watch the birds.
a. suppose p=10, m=5, s=8 when the bird keeper comes back, the
assistant informs the x birds have escaped. the bird keeper
exclaims oh no! all my sparrows are gone. how many birds flew
away.
b. when the bird keeper come back, the assistand told him that
x birds have escaped. the keeper realised that atleast2 sparrows
have escaped. what is minimum no of birds that can escape.
6. select from the five alternatives A,B,C,D,E
AT THE end of each question ,two conditions will be given.
the choices are to filled at follows.
a. if a definete conclusion can be drawn from condition 1
b. if a definete conclusion can be drawn from condition 2
c. if a definete conclusion can be drawn from condition 1 and 2
d. if a definete conclusion can be drawn from condition 1 or 2
e. no conclusion can be drawn using both conditions
1. person 1 says N<5
person says n>5
person 3 says 3N>20
person 4 says 3n>10
person 5 says N<8
whaT IS value of N
a) 1. no of persons who speak false being less than no of persons
who tells the truth.
2. person 2 is telling the truth.
b) 1. no of persong telling the truth is greater than no of
persons telling lies
2. person 5 is telling the truth.
7. there are N coins on a table. there are two players A&B.
you can take 1 or 2 coins at a time. the person who takes the
last coin is the loser. a always starts first
--1. if N=7
a) A can always win by taking two coins in his first chanse
b) B can win only if A takes two coins in his first chance.
c) B can always win by proper play
d) none of the above
--2. A can win by proper play if N is equal to
a) 13 b) 37 c) 22 d) 34 e) 48 ans. e.
--3. B can win by proper play if N is equal to
a) 25 b)26 c) 32 d) 41 e) none
--4. if N<4, can A win by proper play always

8. Two turns have vertain pecular charcteristics. One of them
always lies on Monday, Wednesday, Friday. \the other always lies
on Tuesdays, thursdays and saturdays. On the other days they tel
the truth. You are given a conversation.
person A-- today is sunday my name is anil
person B-- today is tuesday, my name is bill
answers for selected questions
2. equal 1. 150
3. 8 2. 60 kmph
4. 37(M),41(S) 3. Mukherjee
5. 45 min. 8. today is tuesday
6. 0.8
7. 18
11. T max value = 4

3. Anil kumar Mukherjee
Kumar Misra dev
Mohan iyer dev
kumar patil mohan
mohan sharma kumar
************************
There are 6 people A B C D E F.each of them have 10 marbles with them.Then A and B take 4 marbles each from D and E.then C gives 3 marbles to F.then gives away the marbles to C back and two more statements 3 questions on this.

How many marbles E had

Who had one more marble than F

The average of scores of 40 innings of A is 50. The highest score is 178 more than the least score .If those two are removed then the average of the remaining 38 innings is 40 Then the value of the least score is .numbers r not same but the model is

The sum of the ages of sohan sunil and sanjay is 60 yrs .6 yrs before sohans age was two thirds of sunil and half of sanjay.find the age of sanjay

Some amount of money has to be shared between A B C .the amount is 1500(not sure).A share is two ninghts of that of B and C .B share is three elevenths that of A and c togheter find the shares of A B C Ans first option 280 330

There is a building of height 500m. The angles of depressionn of the top of the building with the top and bottom of the tower are 30 and 45..Find the height of the tower ans 200( don t remember)

There are two persons standing in front of tower of height 300m..the angles of elevation are 30 and 60 find the distance between the two people Ans 21.96

matrix was given
x+2 x+3 x+4
x+3 x+5 x+11
x+7 x+12 x+14 find the determinant of the matrix

A B C 6

* F E

B C D G

A F H H

_ _ _ _ 8 the letters are not exactly the same but the numbers 6 and 8 are correct 4 auestions based on this


What is the least number when subtraced frm 23 51 37 29 gives the result as 0


Some question was given on stocks and shares..it was very big.and then 4 questions based on this ..


One more on trignometry consiting of some tan@


One more on complex numbers consisting of some alpha4beta4+alpha beta ..find the complex root


If X4+3X3+X2+1=0 then find the value of x3+2x2-1 question not exactly same but model is same


Two points were given (1,0,0) and (1,1,0). Find the perpendicular distance


Find the length of the wire required to tie around a square 15 times whose area is 30 hectares


Find the amount of water that can be filled in a pipe of diameter 8 cm and length some value


There are 101 natural numbers find the ratio of the sum of even numbers to the sum of odd numbers ans (51:50)


There are 500 students out of which 320 eat fish and 250 eat eggs..and 150 eat both..find the number of students who neither eat fish nor eat eggs


There are 6 boys and 4 girls .A committee of 5 is to be formed ..find the no of ways in which it can be formed in such a way that it includes at least 2 girls


Series -------4, 9, 25,_,169, 361

a .49 b.81 c.121 d e none
The letters from A-Z are written cyclically in the reverse orderin such a way that A=Z, B=Y, C=X, , Z=A.


What is the code for MANAGEMENT.


What is the code for( some word is given) we need to find the answer be careful the answer here is not INTERNET ..the last letter is different


What is the code for ISRO


If P* Q denotes P is father of Q .If P=Q denotes P is the sister of Q and 3 more conditions like this and 4 questions on this


A person starts walking to his home..first he walks 5 m northeast an then turns right n walks 5 m south east and then turns right and walks 5 m southwest and then turns right and finally walks 5 km north west..how far is he from starting point.


0. Classic: If a bear walks one mile south, turns left and walks one mile to the east and then turns left again and walks one mile north and arrives at its original position, what is the color of the bear.

ANS. The color of the bear is trivial. The possible solutions to it are interesting. In addition to the trivial north pole, there are additional circles near south pole. Think it out.

* 1. Given a rectangular (cuboidal for the puritans) cake with a rectangular piece removed (any size or orientation), how would you cut the remainder of the cake into two equal halves with one straight cut of a knife?

ANS. Join the centers of the original and the removed rectangle. It works for cuboids too! BTW, I have been getting many questions asking why a horizontal slice across the middle will not do. Please note the "any size or orientation" in the question! Don't get boxed in by the way you cut your birthday cake :) Think out of the box.

2. There are 3 baskets. one of them have apples, one has oranges only and the other has mixture of apples and oranges. The labels on their baskets always lie. (i.e. if the label says oranges, you are sure that it doesn't have oranges only,it could be a mixture) The task is to pick one basket and pick only one fruit from it and then correctly label all the three baskets.

HINT. There are only two combinations of distributions in which ALL the baskets have wrong labels. By picking a fruit from the one labeled MIXTURE, it is possible to tell what the other two baskets have.

3. You have 8 balls. One of them is defective and weighs less than others. You have a balance to measure balls against each other. In 2 weighings how do you find the defective one?

4. Why is a manhole cover round?

HINT. The diagonal of a square hole is larger than the side of a cover!

Alternate answers: 1. Round covers can be transported by one person, because they can be rolled on their edge. 2. A round cover doesn't need to be rotated to fit over a hole.

5. How many cars are there in the USA?

6. You've got someone working for you for seven days and a gold bar to pay them. The gold bar is segmented into seven connected pieces. You must give them a piece of gold at the end of every day. If you are only allowed to make two breaks in the gold bar, how do you pay your worker?

7. One train leaves Los Angeles at 15mph heading for New York. Another train leaves from New York at 20mph heading for Los Angeles on the same track. If a bird, flying at 25mph, leaves from Los Angeles at the same time as the train and flies back and forth between the two trains until they collide, how far will the bird have traveled?

HINT. Think relative speed of the trains.

8. You have two jars, 50 red marbles and 50 blue marbles. A jar will be picked at random, and then a marble will be picked from the jar. Placing all of the marbles in the jars, how can you maximize the chances of a red marble being picked? What are the exact odds of getting a red marble using your scheme?

9. Imagine you are standing in front of a mirror, facing it. Raise your left hand. Raise your right hand. Look at your reflection. When you raise your left hand your reflection raises what appears to be his right hand. But when you tilt your head up, your reflection does too, and does not appear to tilt his/her head down. Why is it that the mirror appears to reverse left and right, but not up and down?

10. You have 5 jars of pills. Each pill weighs 10 gram, except for contaminated pills contained in one jar, where each pill weighs 9 gm. Given a scale, how could you tell which jar had the contaminated pills in just one measurement?

ANS. 1. Mark the jars with numbers 1, 2, 3, 4, and 5.
2. Take 1 pill from jar 1, take 2 pills from jar 2, take 3 pills from jar 3, take 4 pills from jar 4 and take 5 pills from jar 5.
3. Put all of them on the scale at once and take the measurement.
4. Now, subtract the measurement from 150 ( 1*10 + 2*10 + 3*10 + 4*10 + 5*10)
5. The result will give you the jar number which has contaminated pill.

11. If you had an infinite supply of water and a 5 quart and 3 quart pail, how would you measure exactly 4 quarts?

12. You have a bucket of jelly beans. Some are red, some are blue, and some green. With your eyes closed, pick out 2 of a like color. How many do you have to grab to be sure you have 2 of the same?

13. Which way should the key turn in a car door to unlock it?

14. If you could remove any of the 50 states, which state would it be and why?

15. There are four dogs/ants/people at four corners of a square of unit distance. At the same instant all of them start running with unit speed towards the person on their clockwise direction and will always run towards that target. How long does it take for them to meet and where?

HINT. They will meet in the center and the distance covered by them is independent of the path they actually take (a spiral).

16. (from Tara Hovel) A helicopter drops two trains, each on a parachute, onto a straight infinite railway line. There is an undefined distance between the two trains. Each faces the same direction, and upon landing, the parachute attached to each train falls to the ground next to the train and detaches. Each train has a microchip that controls its motion. The chips are identical. There is no way for the trains to know where they are. You need to write the code in the chip to make the trains bump into each other. Each line of code takes a single clock cycle to execute.
You can use the following commands (and only these);
MF - moves the train forward
MB - moves the train backward
IF (P) - conditional that's satisfied if the train is next to a parachute. There is no "then" to this IF statement.
GOTO


ANS.
A: MF
IF (P)
GOTO B
GOTO A
-----
B: MF
GOTO B
Explanation: The first line simply gets them off the parachutes. You need to get the trains off their parachutes so the back train can find the front train's parachute, creating a special condition that will allow it to break out of the code they both have to follow initially. They both loop through A: until the back train finds the front train's parachute, at which point it goes to B: and gets stuck in that loop. The front train still hasn't found a parachute, so it keeps in the A loop. Because each line of code takes a "clock cycle" to execute, it takes longer to execute the A loop than the B loop, therefore the back train (running in the B loop) will catch up to the front train.
Personality














1)find the next number in the sequence
1,2,3,2,5,6,7,8,3
2)4 coins arrange them in 2 lines ,such that each line contains 3 coins


3)Find the odd one out

rest,held,deaf,back
4)Find the odd one out

alpha,beta,sigma,omega

5)sun,ury,nus,----,ars,ter,---,une,uto

Answer: rth,nus

6)There are few ropes,which vary in size,length and thickness and all its characteristics.Each rope takes 12 minutes to burn irrespective of all its characteristics.
Find the number of ropes required,if the elapsed time is 9 minutes.

7)1,2,25,50,75,100 .With these numbers evaluate 81,each number can be used only once.

8)there are few huge,large and small boxes.Out of the 11 boxes I randomly select few of them,and then put in each of them 8 large boxes and select randomly few of these large boxes and put 8 small boxes in each of the selected large boxex.If 102 boxes are left empty on the table.Find the total number of boxes in the table.

************************************
1) There are two balls touching each other circumferencically. The radius of the big ball is 4 times the diameter of the small ball.The outer small ball rotates in anticlockwise direction circumferencically over the bigger one at the rate of 16 rev/sec. The bigger wheel also rotates anticlockwise at Nrev/sec. what is 'N' for the horizontal line from the centre of small wheel always is horizontal.
2) 1 2 3 4 + 3 4 5 5 ---------- 4 6 8 9 - 2 3 4 5 ---------- 2 3 4 4 + 1 2 5 4 ------------ 3 6 9 8
Q) Strike off any digit from each number in seven rows (need not be at same place) and combine the same operations with 3 digit numbers to get the same addition. After this strike off another digit from all and add all the No.s to get the same 2 digit No. perform the same process again with 1 digit No.s. Give the ' no.s in 7 rows at each stage.
3) there is a safe with a 5 digit No. The 4th digit is 4 greater than second digit, while 3rd digit is 3 less than 2nd digit. The 1st digit is thrice the last digit. There are 3 pairs whose sum is 11. Find the number. Ans) 65292.
4) there are 2 guards Bal and Pal walking on the side of a wall of a wearhouse(12m X 11m) in opposite directions. They meet at a point and Bal says to Pal " See you again in the other side". After a few moments of walking Bal decides to go back for a smoke but he changes his direction again to his previous one after 10 minutes of walking in the other(opposite) direction remembering that Pal will be waiting for to meet.If Bal and Pal walk 8 and 11 feet respectively, how much distance they would have travelled before meeting again. 5) xxx)xxxxx(xxx 3xx ------- xxx x3x ------ xxx 3xx ------ Q) Find the 5 digit No. Hint: 5 is used atleast once in the calculation.
6) Afly is there 1 feet below the ceiling right across a wall length is 30m at equal distance from both the ends. There is a spider 1 feet above floor right across the long wall eqidistant from both the ends. If the width of the room is 12m and 12m, what distance is to be travelled by the spider to catch the fly? if it takes the shortest path.
7) Ramesh sit around a round table with some other men. He has one rupee more than his right person and this person in turn has 1 rupee more than the person to his right and so on, Ramesh decided to give 1 rupee to his right & he in turn 2 rupees to his right and 3 rupees to his right & so on. This process went on till a person has 'no money' to give to his right. At this time he has 4 times the money to his right person. How many men are there along with Ramesh and what is the money with poorest fellow.
8)Question related to probabilities of removing the red ball from a basket,given that two balls are removed from the basket and the other ball is red. The basket contains blue,red,yellow balls.
9)Venkat has 1boy&2daughters.The product of these children age is 72. The sum of their ages give the door numberof Venkat.Boy is elder of three.Can you tell the ages of all the three.
ANALYTICAL ----------
1)L:says all of my other 4 friends have money M:says that P said that exact one has money N:says that L said that precisely two have money O:says that M said that 3 of others have money. P:Land N said that they have money. all are liers.Who has money&who doesn't have?
2)A hotel has two,the east wing and the west wing.some east wing rooms but not all have an ocean view(OV).All WW have a harbour view(HV).The charge for all rooms is identical, except as follows * Extra charge for all HV rooms on or above the 3rd floor * Extra charge for all OV rooms except those without balcony * Extra charge for some HV rooms on the first two floor&some EW rooms without OV but having kitchen facilities. (GRE modrl Test 3-question 1J-22)
3)Post man has a data of name surname door no.pet name of 4 families. But only one is correct for each family.There are a set of statements &questions.
4)4 couples have a party.Depending on the set of statements,find who insulted whom and who is the host of the party.
5)5 women given some of their heights(tall,medium,short)Hair( long, plainted),stards(Black or Brown), sari,2 medium,2-short.Tall-no sari.Plainted-medium.Answer the combinations.
1) A person has to go both Northwards&Southwards in search of a job. He decides to go by the first train he encounters.There are trains for every 15 min both southwards and northwards.First train towards south is at 6:00 A.M. and that towards North is at 6:10 .If the person arrives at any random time,what is the probability that he gets into a train towards North.
2) A person has his own coach&whenever he goes to railway station he takes his coach.One day he was supposed to reach the railway station at 5 O'clock.But he finished his work early and reached at 3 O'clock. Then he rung up his residence and asked to send the coach immediately. He came to know that the coach has left just now to tje railway station. He thought that the coach has left just now to the railway station.He thought that he should not waste his time and started moving towards his residence at the speed of 3mi/hr.On the way,he gets the coach and reaches home at 6 o'clock.How far is his residence from railway station.
3)Radha,Geeta&Revathi went for a picnic.After a few days they forgot the date,day and month on which they went to picnic.Radha said that it was onThursday,May 8 and Geeta said that it was Thursday May 10.Revathi said Friday Jun 8.Now one of them told all things wrongly,others one thing wrong and the last two things wrongly.If April 1st is tuesday what is the right day,date and month? ------------------------------------------------------------------------- I am sending mainly c paper and some questions.Rao also will send somethig.There are 15 c q's all are discriptive. int a=2; f1(a++); } f1(int c) { printf("%d", c); }
1)fallacy f() { int a; void c;f2(&c,&a);
2)a=0; b=(a=0)?2:3;
a) What will be the value of b? why
b) If in 1st stmt a=0 is replaced by -1, b=?
c) If in second stmt a=0 is replaced by -1, b=?
3)char *a[2] int const *p; int *const p; struct new { int a;int b; *var[5] (struct new)
4)f() { int a=2; f1(a++); } f1(int c) { printf("%d", c); } c=?
5)f1() { f(3);} f(int t) { switch(t); { case 2: c=3; case 3: c=4; case 4: c=5; case 5: c=6; default: c=0;} value of c? 6)Fallacy int *f1() { int a=5; return &a; } f() int *b=f1() int c=*b; }
7) a)Function returning an int pointer
b)Function ptr returning an int ptr
c)Function ptr returning an array of integers
d)array of function ptr returning an array of integers (See Scham series book)
8)fallacy int a; short b; b=a;
9)Define function ?Explain about arguments?
10)C passes By value or By reference?
11)Post processed code for abc=1; b=abc1; (1 or 2 blank lines are given) strcpy(s,"abc"); z=abc;
12)difference between my-strcpy and strcpy ?check
13)f() { int *b; *b=2; }
14)Function which gives a pointer to a binary trees const an integer value at each code, return function of all the nodes in binary tree.(Study)Check
15)Calling refernce draw the diagram of function stack illustrating the variables in the -----then were pushed on the stack at the point when function f2 has been introduced type def struct { double x,double y} point; main( int argc, char *arg[3]) {double a; int b,c; f1(a,b);} f1(double x, int y) { point p; stack int n; f2(p,x,y)} f2(point p, double angle) { int i,j,k,int max) } _____________________________________________________________
1)Least no. when divide by [7 gives remainder 6,6gives 5,5 gives 4 and soon ans;419
2)What compilation do (ans source code to obj)
3)Artficial language is provided which of the language (Lisp) check
4)241 change its equivalent octal ?
5)for cube and sphere 3 views are similarly draw one such figure?
6)Write a program to exchange two variaables without temp
7)Fortran cannot have value by reference
8)4,6,8,
__
9)success is to failure, joy is to
10)MEANING OF JOLLY?
11)opposite to essential?
12)"Raw" means
13)To be good "Wrestler " one should have?
14)"Command" opposite?
15)genuine opposite?
16)Two proverbs are goven
17)Sum of two consecutive nos is 55, larger one is?
18)A person goes 4/5 of his usual speed reaches 10min lateto his destinaton, time taken?
19)80% pass in english, 70%pass in maths , 10%fail in both , 144 pass in both . How many all appeared to the test?
20)To get a parabola if you cut a section of?
21)Bird is flying 120km/hr b/w B to R. two trians at B to R at 60 kmph The distance trvelled by the bird before it is killed.Ans.120
22)meaning of inert If any are there rao will send you. Prepare well for the interview. Mostly on graphics , geometry .Prepare questions like (for interview)Prove some of the angles in a triangle are 180.Angle in a half circle is 90.How will you measure hight of building when you are at the top of the building and if you have stone with you.






******** Thank You *********
















16 comments:

Unknown said...

www.sangambayard-c-m.com

Anonymous said...

Who can skilled in and tell what goods are sold on this plat: [url=http://hydsevehar.chez.com]here[/url]
Thanks representing waiting!

Anonymous said...

Who can recognize and circa what goods are sold on this locale: [url=http://terchieheart.0catch.com]here[/url]
Thanks for waiting!

Anonymous said...

For latest information you have to go to see internet and on web I found this
website as a best site for most recent updates.
My web page ; piano lessons

Anonymous said...

Your style is so unique compared to other folks I've read stuff from. I appreciate you for posting when you've got
the opportunity, Guess I'll just book mark this web site.
Also visit my web page ... boligityrkia.net

Anonymous said...

Deb… The actual gambling My spouse and i spotted not too long ago were being a little
different… (can’t remember in which My spouse and i noticed these.
)
My web blog : People Records

Anonymous said...

Hi, after reading this awesome post i am also happy to share
my experience here with colleagues.
installing hardwood floors

Review my webpage: hardwood floor refinishing

Anonymous said...

Nice post. I learn something totally new and challenging on blogs I stumbleupon
on a daily basis. It will always be interesting to read
through content from other authors and use something from their websites.


My web blog: affordable hardwood flooring

Anonymous said...

Hey I am so excited I found your blog page, I really found
you by accident, while I was browsing on Google for something else, Anyways I am here now and would just
like to say kudos for a tremendous post and
a all round exciting blog (I also love the theme/design),
I don’t have time to browse it all at the moment but I have bookmarked it and also added your RSS feeds, so when I have time I will
be back to read more, Please do keep up the superb b.
hardwood floor

Also visit my web site; engineered hardwood floors

Anonymous said...

whoah this weblog is wonderful i like reading your articles.
Stay up the good work! You recognize, lots of persons are looking around for this information,
you can aid them greatly.

Review my page hardwood flooring

Anonymous said...

My developer is trying to convince me to move to .net from PHP.
I have always disliked the idea because of the costs.
But he's tryiong none the less. I've been using WordPress on a number of websites for about a year and am worried about
switching to another platform. I have heard very good things about blogengine.
net. Is there a way I can import all my wordpress posts into it?
Any kind of help would be really appreciated!



my blog ... hardwood floors

Anonymous said...

Stunning quest there. What occurred after?

Good luck!

my webpage: toenail fungus treatment

Anonymous said...

I've been exploring for a little for any high quality articles or blog posts in this sort of house . Exploring in Yahoo I eventually stumbled upon this web site. Reading this info So i am happy to convey that I have a very excellent uncanny feeling I found out just what I needed. I so much undoubtedly will make certain to don?t omit this website and provides it a glance on a continuing basis.

My web-site nail fungus treatment

Anonymous said...



Feel free to visit my web blog: ophiocordyceps-sinensis.com

Anonymous said...

I really like what you guys tend to be up too. This kind of clever work and reporting!
Keep up the terrific works guys I've you guys to our blogroll.

Look at my blog individual reputation management

pxljobs said...

Thank you for your valuable stuff this is helpful for all the competitive examinations...

Graphic designer jobs in bangalore | UI designer jobs in bangalore